Sei sulla pagina 1di 1040

Westinghouse 4 loop Braidwood1 Braidwood 2 Byron 1

Byron 2 Callaway Catawba 1* Catawba 2* Comanche Peak 1

Comanche Peak 2 D.C.Cook 1 D.C.Cook 2 Diablo Canyon 1 Diablo Canyon 2

Indian Point 2 Indian Point 3 McGuire 1* McGuire 2 Millstone 3

Salem 1 Salem 2 Seabrook 1 Sequoyah 1 Sequoyah 2

South Texas 1 South Texas 2 Vogtle 1* Vogtle 2* Watts Bar 1

Wolf Creek 1

Quest No: RO SRO: 1 Both System/Evolution Name: Reactor Trip

TIER: 1

GROUP: Topic No: KA No: RO: SRO: Cog Level: 1 000007 EK1.04 3.6 3.9 High Category Statement: Knowledge of the operational implications of the following concepts as they apply to the reactor trip:

KA Statement: Decrease in reactor power following reactor trip (prompt drop and subsequent decay) UserID: Question Stem: Topic

Unit 1 was operating at 100% power. At Time=0 an inadvertent turbine trip occurs. All systems respond as designed. At Time=2 minutes, IR Nuclear Instrument N35 reads 5E-2% power. Which of the following is the MINIMUM additional time expected for the POWER ABOVE PERMISSIVE P6 lite to GO OUT on the IR NI N35 Drawer on 1PM07J?

A B C D

6 minutes

12 minutes

15 minutes

18 minutes
Question Source: S.NI2-08-B, A.NT7New Medium Question Difficulty

Answer: Task No: B Obj No:

Ti e: m Cr R : oss ef 1 I1-NT-XL-07, Nuclear Theory Chapter 7, Neutron Kinetics, pg 33, 46, 60 Reference: ILT Systems: I1-NI-XL-01BY, Gamma-Matric Source and Intermediate Range Nuclear Instrumentation, pg 11, 21 22, 28.

A prompt drop and rapid power level decay occurs over the first 2-4 minutes following a reactor trip. The reactor then reaches a Explanation: stable -1/3 DPM startup rate due to the dominant decay of the long-lived fission product precursers. This stable period is sustained well into the source range. From the 5E-2% power to 5E-6% (Setpoint for the POWER ABOVE PERMISSIVE P6 lite) is 4 decades and will take ~12 minutes. MJJ 2/27/06 Date Written: 2/27/2006 Author: M. Jorgensen App. Ref:

Quest No: RO SRO: TIER: GROUP: Topic No: KA No: 2 Both 1 1 000008 82.1.30 System/Evolution Name: Category Statement: Pressurizer (PZR) Vapor Space Accident Conduct of Operations (Relief Valve Stuck Open) KA Statement: Ability to locate and operate components, including local controls. UserID: Question Stem: Topic

RO: 3.9

SRO: 3.4

Cog Level: High

Unit-1 is at 100% power. PZR PORV DSCH TEMP HIGH (1-12-C6) is in alarm. PORV 1RY456 indicates partially OPEN after C/S was placed in CLOSE 1RY8000B, PORV Isolation Valve, C/S was taken to CLOSE but continues to show dual indication. Charging Header flow has risen ~ 5gpm and stablized PZR level on program. PZR Pressure is stable at 2218 psig The crew has entered 1BOA PRI-1, Excessive Primary Plant Leakage. What is/are the NEXT required action(s) based on these indications?

A B

Trip the Reactor and manually actuate Safety Injection.

Locally CLOSE 1RY8000B at MCC 131X2 by resetting the breaker and reCLOSING the breaker.

Dispatch an operator with RP to enter Containment and manually CLOSE 1RY8000B.

Locally CLOSE 1RY8000B at MCC 132X2 after placing the LOCAL/REMOTE switch at the breaker in LOCAL.
Answer: Task No: D Time: Obj No: T.OA12-02 Cross Ref: Reference: Question Source: New Medium Question Difficulty

1BOA PRI-1, Excessive Primary Plant Leakage, Step 3 RNO I1-OA-XL-12, 1BOA PRI-1, Excessive Primary Plant Leakage

A. is incorrect because the system has stablized within normal charging capacity and does not warrant a Reactor trip or SI. Explanation: B. is incorrect because this is the wrong MCC and there is no indication that the breaker has tripped requiring a reset. C. is incorrect because this is not a procedural option nor would this occur at 100% power. D is the correct action as specified for this condition in 1BOA PRI-1.

Date Written: 2/28/2006 Author: M. Jorgensen App. Ref: Quest No: RO SRO: TIER: GROUP: Topic No: KA No: RO: SRO: Cog Level: 3 Both 1 1 000009 EA2.24 2.6 2.9 High System/Evolution Name: Category Statement: Small Break LOCA Ability to determine and interpret the following as they apply to a small break LOCA: KA Statement: RCP temperature setpoints UserID: Question Stem: Topic

A Small Break LOCA has just occurred on Unit 1 and Safety Injection has been actuated. All systems responded as designed. Without any additional operator action, which of the following RCP temperatures would RISE significantly during the next 10 minutes of RCP operation?

A B C D

Thrust Bearing

Motor Lower Radial Bearing

Motor Stator Winding

Pump Lower Radial Bearing


Question Source: New Medium Question Difficulty

Answer: Task No: C Obj No: S.RC2-08-C

Ti e: m Cr R : oss ef 1 I1-RC-XL-02, Ch 13, Reactor Coolant Pump

Reference:

C. is correct because the SI and Phase A actuations cause RCFCs to run in Low Speed without Chilled Water for cooling (SX only) and Explanation: Post-LOCA environment will cause higher temperatures in Cnmt. The Motor Windings are air cooled from the Cnmt atmosphere. A and B are incorrect because CCW flow actually increases post SI (2 CC pumps running) and adds additional cooling for the oil coolers causing temperature to be stable or drop. D. is incorrect since the additional seal injection flow rate post SI aids in cooling and temperature will be stable or drop. Date Written: 3/2/2006 Author: M. Jorgensen App. Ref: Quest No: RO SRO: TIER: GROUP: Topic No: KA No: 4 Both 1 1 000022 AA2.04 System/Evolution Name: Category Statement:

RO: 2.9

SRO: 3.8

Cog Level: High

Loss of Reactor Coolant Makeup

Ability to determine and interpret the following as they apply to the Loss of Reactor Coolant Pump Makeup:

KA Statement: How long PZR level can be maintained within limits UserID: Question Stem: Topic

Unit-1 is at 20% and stable Tave is on program PZR level control is in AUTO Letdown flow is 75 gpm Total RCP Leakoff flow is 16 gpm The running CV pump impeller degrades resulting in a Charging Header flow stablizing at 59 gpm With continued operation and no operator action, when would Pressurizer level BEGIN to RISE? ASSUME 130 gallons per % in the PZR.

A B C D

30 -35 minutes

40 -45 minutes

50 - 55 minutes

60 - 65 minutes
Question Source: S.RY1-03/20 New Medium Question Difficulty

Answer: Task No: D Obj No:

Ti e: m Cr R : oss ef 1 I1-RY-XL-01, Ch 14, Pressurizer Reference:

D. is correct because at 20% power, PZR level program is 32%. Level will RISE in the PZR when Letdown Auto isolates at 17%. Explanation: 32%-17% is 15% level drop. PZR has 130 gal/% ~ 1950 gallons. Level is dropping at 16 (75-59) gpm + 16 gpm RCP leakoff which = 32 gpm. At 32 gpm, 1950 gal will take between 60 and 65 minutes to isolate letdown, then charging will cause PZR level to RISE.

Date Written:

2/28/2006 Author: M. Jorgensen

App. Ref:

Quest No: RO SRO: TIER: GROUP: Topic No: KA No: RO: SRO: Cog Level: 5 Both 1 1 000026 AK3.04 3.5 3.7 Low System/Evolution Name: Category Statement: Loss of Component Cooling Water (CCW) Knowledge of the reasons for the following responses as they apply to the Loss of Component Cooling Water: KA Statement: Effect on the CCW flow header of a loss of CCW UserID: Question Stem: Topic

Component Cooling Water (CCW) is in a normal at power lineup with Unit-1 CCW providing cooling for the inservice Spent Fuel Pool heat exchanger. A tube leak develops in the Spent Fuel Pool heat exchanger. Unit-1 CCW Surge Tank level will___(1)____ and CCW pump discharge header flowrate will ___(2)____.

(1)

(2)

A B C D

rise

drop

rise

rise

drop drop

drop rise
Question Source: S.CC1-18 New Medium Question Difficulty

Answer: Task No: D Obj No:

Time: Cross Ref: 1 I1-CC-XL-01, Ch 19, Component Cooling Water System, Pgs 12, 30 1BOA PRI-6, Component Cooling Water Malfunction

Reference:

CCW is at a higher pressure in the SFP heat exchanger, therefore Surge Tank level will drop and the increased system demand will Explanation: cause pump discharge flowrate to rise.

Date Written:

2/28/2006 Author: M. Jorgensen

App. Ref:

Quest No: RO SRO: TIER: 6 Both 1 System/Evolution Name: Anticipated Transient Without Scram (ATWS)

GROUP: Topic No: KA No: 1 000029 2.2.25 Category Statement: Equipment Control

RO: 2.5

SRO: 3.7

Cog Level: Low

KA Statement: Knowledge of bases in technical specifications for limiting conditions for operations and safety limits. UserID: Question Stem: Topic

10 CFR 50.62 established requirements for reducing the risk of an Anticipated Transient Without Scram (ATWS) in Light-Water-Cooled Nuclear Power Plants. The ATWS Rule, when applied, is based on reducing the risk of which of the following?

A B C D

Exceeding DNBR limit

Excessive RCS overpressure

Exceeding Kw/ft Fuel limit

Excessive RCS cooldown


Question Source: S.RP3-01/04 New Medium Question Difficulty

Answer: Task No: B Obj No:

Time: Cross Ref: 1 Byron UFSAR Ch 15.8 Reference: Ch 60c, I1-RP-XL-03, ATWS Mitigation System

B. is correct because AMS provides a diverse start signal for AFW and a Turbine Trip to maintain the SGs as a Heat Sink to minimize Explanation: the RCS overpressure transient that occurs with no Reactor Trip. A. is incorrect since pressure rise precludes reaching DNB. C. is incorrect since temperature rise adds negative reactivity (Doppler) and limits linear heat rate. D. is incorrect since a heatup is the concern, cooldown would limit the overpressurization. Date Written: 2/28/2006 Author: M. Jorgensen App. Ref:

Quest No: RO SRO: TIER: GROUP: Topic No: KA No: RO: SRO: Cog Level: 7 Both 1 1 000038 EA2.01 4.1 4.7 High System/Evolution Name: Category Statement: Steam Generator Tube Rupture (SGTR) Ability to determine and interpret the following as they apply to a SGTR:

KA Statement: When to isolate one or more S/Gs UserID: Question Stem: Topic

An accident is in progress on Unit 1. The following plant conditions exist: Containment Pressure = 3 psig (slowly dropping) SG levels (NR): (All are slowly rising) 1A: 5% 1B: 7% 1C: 8% 1D: 5% Main Steamline 1B radiation ALERT alarm is lit. In accordance with 1BEP-3, Steam Generator Tube Rupture, the operator is directed to:

A B

immediately manually CLOSE the 1B AF isolation valves, 1AF013B and F.

maintain feed to the 1B SG until narrow range level is 10%, then manually isolate AF to the 1B SG. maintain feed to all SGs until all narrow range levels are at or above 10%, then manually isolate AF to the 1B SG. maintain feed to 1B SG until narrow range level is 31%, then manually isolate AF to the 1B SG.
Question Source: Byron 2000 NRC exam Medium Question Difficulty

Answer: Task No: B Obj No: T.EP04-08

Ti e: m Cr R : oss ef 1 1/2BEP-3, Steam Generator Tube Rupture, Step 4 I1-EP-XL-04, BEP-3 series, Pg 15

Reference:

The ruptured SG level must be > 10% to ensure an adequate thermal layer exists prior to isolation of Aux Feed. This will insulate the Explanation: steam space in the SG from the cooler RCS water as the RCS is cooled down to allow depressurization of the RCS to stop the leakage to the SG without losing subcooling, if the SG were to depressurize while trying to equalize RCS to SG pressure. A. is incorrect since the >10% is required prior to isolation. B. is correct as described above. C. is incorrect since the requirement for isolation only applies to the ruptured SG. The other, intact, SGs can be throttled for heat removal control once at least one of them is >10%. D. is incorrect since this is the ADVERSE CNMT value which is not used when CNMT pressure is < 5 psig for these conditions. Date Written: 2/27/2006 Author: M. Jorgensen App. Ref:

Quest No: RO SRO: 8 Both System/Evolution Name: Steam Line Rupture

TIER: 1

GROUP: Topic No: KA No: RO: SRO: Cog Level: 1 000040 AK2.02 2.6 2.6 High Category Statement: Knowledge of the interrelations between the Steam Line Rupture and the following:

KA Statement: Sensors and detectors UserID: Question Stem: Topic

Unit 1 tripped from 100% power due to a steamline break inside of Containment. Shortly after the trip, the following parameters were recorded: PZR pressure is 1750 psig and stable PZR level is 22% and stable CNMT pressure has reached 7.8 psig S/G NR levels; 1A 31%, 1B 30%, 1C 25%, 1D 34% S/G pressures; 1A 760 psig, 1B 775 psig, 1C 660 psig, 1D 800 psig An automatic steamline isolation occurred due to _____ .

A B C D

low S/G pressure

low S/G level

containment pressure circuit

low PZR pressure SI


Question Source: 2000 Byron NRC exam Low Question Difficulty

Answer: Task No: A Obj No: S.MS1-07-D

Time: Cross Ref: 1 I1-MS-XL-01, Ch 23, Main Steam System Reference:

A. is correct since a steamline break would cause steamline pressure to drop rapidly. The low pressure setpoint is 640 psig, but is rate Explanation: sensitive and could have occurred without actually going below 640 psig. B. is incorrect because the SG level circuit does not input to the steamline isolation logic. C. is incorrect since 8.2 psig in CNMT has not been reached. D. is incorrect since low PZR pressure SI does not cause a main steamline isolation to occur. Date Written: 3/2/2006 Author: M. Jorgensen App. Ref:

Quest No: RO SRO: TIER: 9 Both 1 System/Evolution Name: Loss of Main Feedwater (MFW)

GROUP: Topic No: KA No: RO: SRO: Cog Level: 1 000054 AA1.03 3.5 3.7 High Category Statement: Ability to operate and/or monitor the following as they apply to the Loss of Main Feedwater (MFW):

KA Statement: AFW auxiliaries, including oil cooling water supply UserID: Question Stem: Topic

Unit 1 was at 100% power. Unit 2 is in MODE 6. 2B SX pump is OOS for replacement A spurious Feed Water Isolation occurred on Unit 1. The crew manually tripped the Unit 1 Reactor. When the Unit 1 Main Generator output breakers opened, Off-Site power was lost to BOTH Units resulting in the following condition: 1A DG started and loaded. 1A SX pump tripped after starting. 1B DG did not start 2A DG is running, but the output breaker will NOT CLOSE. 2B DG started and loaded What is the IMPACT on CONTINUED operation of the Unit 1 Aux Feed Pumps? (Ignore any fuel consumption concerns)

A B C D

Both pumps can continue to run as long as Fuel Oil is available.

1A only requires 1A DG running; 1B will trip on High Bearing temperature.

1B will trip on High Jacket Water temperature; 1A can continue to run.

1B can continue to run as long as Fuel Oil is available; 1A should be stopped to prevent tripping.
Question Source: S.AF1-04/15 New Medium Question Difficulty

Answer: Task No: D Obj No:

Ti e: m Cr R : oss ef 1 I1-AF-XL-01, Ch 26, Auxiliary Feedwater System

Reference:

A. is incorrect because 1A requires an SX pump running for oil cooling and will rapidly overheat bearings and most likely trip the Explanation: supply breaker. B. is incorrect because of the reasoning in A. above. It may run until the EDG overheats and stops, but will probably have already tripped. 1B is designed to run without an SX pump running. It has it's own shaft driven SX pump. C. is incorrect for the same reasons described in B. above. D. is correct because 1B has it's own shaft driven SX pump and will supply cooling as long as an SX reservoir is maintained while 1A requires an SX pump running to provide oil cooling and should be stopped when the no SX condition is recognized to prevent damage beyond repair. Date Written: 3/2/2006 Author: M. Jorgensen App. Ref:

Quest No: RO SRO: TIER: GROUP: Topic No: KA No: RO: SRO: Cog Level: 10 Both 1 1 000055 EK1.01 3.3 3.7 Low System/Evolution Name: Category Statement: Loss of Offsite and Onsite Power (Station Knowledge of the operational implications of the following concepts as they apply Blackout) to the Station Blackout: KA Statement: Effect of battery discharge rates on capacity UserID: Question Stem: Topic

Unit 1 is at 100% power - At 0800 a Loss of all AC power occurs on Unit 1. - DC Bus 111 and 112 is being supplied by their respective Batteries. With no operator action, the MINIMUM DESIGN DC Bus voltage will only be available on Unit 1 DC Buses until ?

A B C D

0830

0900

1000

1600
Question Source: S.DC1-04-B/05-C New Medium Question Difficulty

Answer: Task No: B Obj No:

Time: Cross Ref: 1 I1-DC-XL-01, Ch 8a, 125 VDC Power Systems Byron UFSAR, CH 8.3 1BOA ELEC-3, Loss of 4KV ESF Bus

Reference:

The design of the 1E 125 VDC Batteries is Blackout with LOCA and only one DG (no AC to Battery Charger for 1 hour) and Explanation: minimum voltage of 105 VDC. 1BOA ELEC-3 states that if a battery charger cannot be restored and the opposite unit charger is energized, then crosstie within 1 hour per BOP DC-7. A. is incorrect but a plausible action time. B. is correct as described above. C. is incorrect but plausible action time used in Tech Specs for electrical problems. D. is incorrect but plausible action time used in Tech Specs for electrical problems. Date Written: Quest No: RO SRO: 11 Both 3/2/2006 Author: M. Jorgensen TIER: GROUP: Topic No: 1 1 000056 App. Ref: KA No: AK1.04

RO: 3.1

SRO: 3.2

Cog Level: High

System/Evolution Name: Loss of Offsite Power

Category Statement: Knowledge of the operational implications of the following concepts as they apply to Loss of Offsite Power:

KA Statement: Definition of saturation conditions, implication for the systems UserID: Question Stem: Topic

A Reactor Trip with a Loss of Off-Site Power has occurred. The following conditions exist: CETCs = 582F and stable All Hot Leg Temperatures = 580F and stable All Cold leg Tempertures = 550F and stable Pressurizer Pressure = 2085 psig and stable It is desired to reduce RCS pressure but maintain 50F subcooling. The MINIMUM pressure that will maintain 50F RCS subcooling as indicated on SPDS is ____.

A B C D

1916 psig

1928 psig

1933 psig

1948 psig
Question Source: T.EP01-06-D Byron 2001 Cert exam Medium Question Difficulty

Answer: Task No: C Obj No:

Ti e: m Cr R : oss ef 1 1BEP 0.2, Natural Circulation Cooldown, Fig 1BEP 0.2-2 and ATT A Reference : I1-EP-XL-01, 1BEP-0, Reactor Trip or Safety Injection - series pgs 95, 100 Steam Tables

Add 50F to 582F (highest temp) = 632F; look up Sat Press for 632F in Steam Tables = 1947 psia; subtract 15 psi = 1932 psig Explanation: A. Is incorrect but plausible if wrong temp used and subtracted 15psi instead of adding for steam table use. B. is incorrect but plausible if 15 psi was not added in for steam table use. C. is correct as described above. D. is incorrect but plausible if 15 psi was not subtracted back out of the steam table result to get psig. Date Written: 3/2/2006 Author: M. Jorgensen App. Ref:

Quest No: RO SRO: TIER: GROUP: Topic No: KA No: RO: SRO: Cog Level: 12 Both 1 1 000057 AK3.01 4.1 4.4 High System/Evolution Name: Category Statement: Loss of Vital AC Electrical Instrument Bus Knowledge of the reasons for the following responses as they apply to the Loss of Vital AC Instrument Bus: KA Statement: Actions contained in EOP for loss of vital ac electrical instrument bus UserID: Question Stem: Topic

Unit 1 was at 100% power when a Reactor Trip and Safety Injection occurred due to a SGTR. 2 Minutes later, Off-site power was lost. Both 1A and 1B DG started and closed on to their respective buses. All B Train loads are running. A Train loads are being manually started. When the 1A AF pump is started, the operator notices that the flowrates to each SG from the 1A AF pump rapidly LOWER from ~ 180 gpm to 0 gpm. Of the following, the manual load start and the resultant 1A AF flow response occurred because of loss of .

A B C D

125 VDC Bus 111

120 VAC Bus 111

125 VDC Bus 113

120 VAC Bus 113


Answer: Task No: B Obj No: S.AP1-14-B Question Source: New Medium Question Difficulty

Ti e: m Cr R : oss ef 1 1BOA ELEC-2, Loss Of Instrument Bus Unit 1 I1-AP-XL-01, AC Electrical Power Systems

Reference:

Sequencer for A Train equipment and to control setpoint signal for the A Train 1AF005 valves are powered by Explanation: 120 VAC Bus 111 requiring manual sequencing of loads and causes the flow control setpoint for the 1AF005 valves to position for a zero flow setpoint. A. is incorrect because this would not allow breaker operation of A Train loads from the MCR. B. is correct as stated above. C. is incorrect because this would have no impact on sequencing of loads or AF valve control. D. is incorrect because this would also have no impact on sequencing of loads or AF valve control. Date Written: 3/3/2006 Author: M. Jorgensen App. Ref:

Quest No: RO SRO: 13 Both System/Evolution Name: Loss of DC Power

TIER: 1

GROUP: Topic No: KA No: RO: SRO: Cog Level: 1 000058 AA1.01 3.4 3.5 High Category Statement: Ability to operate and/or monitor the following as they apply to the Loss of DC Power:

KA Statement: Cross-tie of the affected dc bus with the alternate supply UserID: Question Stem: Topic

At 0800: - 125 VDC Bus 111 is being supplied by Battery 111 due to failure of the Battery Charger. - 125 VDC Bus 111 voltage is 122 VDC. - 125 VDC Bus 211 is being supplied by it's Battery Charger and voltage is steady at 128 VDC. At 0805: - 125 VDC Bus 111 voltage 120 VDC If the rate of usage has been STEADY from Time = 0, what is the LATEST time BEFORE it becomes UNACCEPTABLE to X-tie with a load on 125 VDC Bus 111?

A B C D

0816

0822

0830

0835
Question Source: New Medium Question Difficulty

Answer: Task No: D Obj No: S.DC1-05-D

Time: Cross Ref: 1 BOP DC-7, 125 VDC ESF Crosstie/Restoration I1-DC-XL-01, Ch 8a, 125 VDC Power Systems

Reference:

BOP DC-7 states in a Precaution that X-tie to a loaded DC Bus should not occur with > 20 volts differential. At 2 VDC usage every 5 Explanation: minutes, it will take 20 minutes additional time to reach 108 VDC on Bus 111, so at 21 minutes the 20 VDC differential limit will be exceeded. A. is incorrect since Bus 111 voltage will be between 112 and 114 and it is still aceptable to crosstie. B. is incorrect since Bus 111 voltage will be between 111 and 112 and it is still aceptable to crosstie. C. is incorrect since Bus 111 voltage will be 110 and it is still aceptable to crosstie D. is correct since Bus 111 voltage will be at 108 and any more time will be > 20 volts difference. Date Written: 3/4/2006 Author: M. Jorgensen App. Ref:

Quest No: RO SRO: TIER: 14 Both 1 System/Evolution Name: Loss of Nuclear Service Water

GROUP: Topic No: KA No: RO: SRO: Cog Level: 000062 AK3.04 3.5 3.7 High Category Statement: Knowledge of the reasons for the following responses as they apply to the Loss of Nuclear Service Water:

KA Statement: Effect on the nuclear service water discharge flow header of a loss of CCW UserID: Question Stem: Topic

Unit 1 and Unit 2 are at 100% power in a normal at power lineup with the following conditions: - 2B SX pump is running. - 2A CC pump has tripped on overcurrent. - 2B CC pump has auto started. - A large tube leak develops in the U-2 CC Heat Exchanger. - U-2 CC Surge Tank level is dropping rapidly with full make-up capacity. Considering the impact on 2B SX pump alone, with no operator action, what would be the response of 2B SX pump discharge pressure, AFTER the U-2 CC Surge Tank level drops to 13%, and why?

A B

Discharge pressure will initially drop due to the 2B CC pump trip, then rise and stablize below the original value. Discharge pressure will rise immediately due to the loss of the large heat load and remain above the original value. Discharge pressure will drop and stablize at a lower value due to the loss of the large heat load.

Discharge pressure will rise due to the 2B CC pump trip, then drop due to the loss of the large heat load and should stablize close to the original value.
Answer: Task No: A Obj No: S.SX1-15, S.CC1-18 Question Source: New Medium Question Difficulty

Time: Cross Ref: 1 I1-SX-XL-01, Ch 20, Essential Service Water System, Reference: I1-CCXL-01, Ch 19, Component Cooling Water System, Pg 4, 12, 26

CC is at a higher pressure than SX. The SX pump discharge pressure is normally ~ 90 psig. The CCW pump discharge pressure is ~ Explanation: 130 psig with system reliefs are set at 150 psig or higher. When the 2B CC pump trips, the SX pump discharge will drop initially while the CC system is refilled by SX. When CC is refilled, a very small demand will continue through the Surge tank vent. CCW heat load on SX is minimal with both Units at 100% power. A. is correct because pressure drops originally due to the increase demand on the system to refill CCW. Once CCW is full, the increased demand will be small through the surge tank vent, thus the pressure will recover, but remain below the original value. B is incorrect because SX flow goes up, thus pressure drops. C. is incorrect because the completion of CCW refill will cause pressure to rise. D. is incorrect because the increased SX flow will cause pressure to drop.

Date Written:

3/12/2006 Author: M. Jorgensen

App. Ref:

Quest No: RO SRO: 15 Both System/Evolution Name: Loss of Instrument Air

TIER: 1

GROUP: Topic No: KA No: RO: SRO: Cog Level: 1 000065 AA1.02 2.6 2.8 High Category Statement: Ability to operate and/or monitor the following as they apply to the Loss of Instrument Air:

KA Statement: Components served by instrument air to minimize drain on system UserID: Question Stem: Topic

Unit 2 has completed refueling and is in the process of plant heatup with the following conditions: - 2B RH is in shutdown cooling mode. - RCS temperature is 300F. - RCS pressure is 340 psig. - 2B CV pump is in operation. - PZR bubble has just been established. - A loss of instrument air has just occurred. Which ONE of the following describes the INITIAL response and WHY, if NO operator action is taken?

A B C D

RCS will cooldown due to the RISE in RH flow through the 2B RH Hx.

PZR level DROPS due to the RISING RH letdown flow.

2B CV pump suction pressure will DROP due to the RISE in charging flow.

RCS will heatup due to the DROP in RH flow through the 2B RH Hx.
Question Source: S.RH1-11, T.OA39-03 Byron NRC exam bank (1996) Medium Question Difficulty

Answer: Task No: A Obj No:

Ti e m: Cr sR : os ef 1 Ch 18, Residual Heat Removal, Pg 12 Reference: 1BOA SEC-4, Loss of Instrument Air, Table A, Pg 12 I1OA-XL-39, Loss of Instrument Air, Pg 31

B Train RHR Hx outlet valve, 1RH607, fails open and the Hx bypass, 1RH619, fails closed forcing total flow through the RHR Hx. Explanation: CCW flow through the Hx remains unchanged, therefore an RCS cooldown will occur. A. is correct because of the rise in RH flow through the Hx. B. is incorrect because RH letdown flow stops due control valve 1CV-131 failing closed and PZR level will go up with charging to the RCP seals maintained. C. is incorrect because total charging flow will drop and CV pump suction pressure will rise or stay about the same. D. is incorrect because a cooldown will occur due to valve fail positions in th RH system. Date Written: 3/5/2006 Author: M. Jorgensen App. Ref:

Quest No: RO SRO: 16 Both System/Evolution Name: LOCA Outside Containment

TIER: 1

GROUP: Topic No: KA No: RO: SRO: Cog Level: 1 00WE04 EK2.2 3.8 4.0 Low Category Statement: Knowledge of the interrelations between the LOCA Outside Containment and the following:

KA Statement: Facility's heat removal systems, including primary coolant, emergency coolant, the decay heat removal systems, and relations between the proper operation of these systems to the operation of the facility UserID: Topic Question Stem:

A small break LOCA has occurred on Unit 1 outside containment. Actions of 1BCA-1.2, LOCA Outside Containment, have been completed. - RCS pressure continues to DROP. A transition was made to 1BCA-1.1, Loss of Emergency Coolant Recirculation. Which of the following is the reason a transition was made to 1BCA-1.1?

A B C D

To recover after the break was isolated.

To terminate off-site dose release.

To reverify that all automatic actions have been completed.

To take compensatory actions for lack of inventory in the contaiment sump.


Question Source: Byron NRC bank (2000) Medium Question Difficulty

Answer: Task No: D Obj No: T.CA2-05

Time: Cross Ref: 1 1BCA-1.1, Loss of Emergency Coolant Recircualtion Reference: 1BCA-1.2, LOCA Outside of Containment I1-CA-XL-02, BCA 1.1, 1.2 Contingency Action, Pg 1

This procedure is used when recirculation cannot be accomplished due to alignment problems or lack of inventory in the CNMT sump Explanation: and tries to delay depletion of the RWST by stopping unnecessary ECCS and CS pumps and reducing RCS pressure to slow or stop the leakage. A. is incorrect since RWST inventory and leakage would no longer be a concern. B. is incorrect since this BCA is not intended to isolate a barrier. C. is incorrect since these actions would have been addressed in other procedures and this BCA is strictly buying time for inventory depletion. D. is correct as described above. Date Written: 3/5/2006 Author: M. Jorgensen App. Ref:

Quest No: RO SRO: TIER: GROUP: Topic No: KA No: RO: SRO: Cog Level: 17 Both 1 1 00WE11 EK2.1 3.6 3.9 High System/Evolution Name: Category Statement: Loss of Emergency Coolant Recirculation Knowledge of the interrelations between the Loss of Emergency Coolant Recirculation and the following: KA Statement: Components, and functions of control and safety systems, including instrumentation, signals, interlocks, failure modes, and automatic and manual features UserID: Topic Question Stem:

Given the following conditions on Unit 1: - Reactor Trip and SI occurred at 0700. - RH system problems resulted in a loss of recirculation capability. - Current time is 1340 - RCS subcooling is 10F - 1A and 1B CV pumps are running - Hi head SI flow is 350 gpm (Assume equal flow from each CV pump) - 1A SI pump discharge flow is 20 gpm - 1B SI pump discharge flow is 50 gpm In order to meet the MINIMUM ECCS flow for decay heat removal, the ONLY ECCS pump(s) that should be RUNNING will be _____ . (Figure 1BCA 1.1-1 is attached)

A B C D

BOTH SI pumps

BOTH CV pumps

1A CV pump and 1A SI pump

1A CV pump and 1B SI pump


Question Source: T.CA2-05 Byron Cert bank (2001) Low Question Difficulty

Answer: Task No: D Obj No:

Time: Cross Ref: 1 1BCA 1.1, Loss of Emergency Coolant Recirculation, Step 15 RNO I1-CA-XL-02, BCA 1.1, 1.2 Contingency Action, Pg 17

Reference:

Per Figure BCA 1.1-1, the required flow for 400 minutes (6 hours, 40minutes) is ~ 220 gpm. To accomplish this and delay RWST Explanation: level drop, 1 CV pump with 175 gpm/pump and the 1B SI pump with 50 gpm would be the closest combination given to provide adequate decay heat removal and minimize the rate of RWST level drop. A. is incorrect since required flow rate is at least 220 gpm and this would only provide 70 gpm. B. is incorrect since 350 gpm will provide adequate decay heat removal, but excessive depletion of the RWST. C. is incorrect since the flow rate of 195 gpm is not adequate for the ~ 220 gpm required for decay heat removal. Date Written: 3/5/2006 Author: M. Jorgensen App. Ref: Figure 1BCA-1.1-1

Quest No: RO SRO: 18 Both System/Evolution Name: Loss of Secondary Heat Sink

TIER: 1

GROUP: Topic No: KA No: 1 00WE05 2.1.27 Category Statement: Conduct of Operations

RO: 2.8

SRO: 2.9

Cog Level: Low

KA Statement: Knowledge of system purpose and or function. UserID: Question Stem: Topic

The first step of 1BFR-H.1, Response to Loss of Secondary Heat Sink, is to check if RCS pressure is greater than any intact SG pressure. If NOT, the crew is sent to 1BEP-1 or 1BEP ES-1.3. Why is 1BFR-H.1 NOT performed under these conditions?

A B C D

ECCS injection is NOT providing adequate cooling.

The SGs are NOT needed as a sink.

Higher priority RED paths are anticipated due to the indicated large break LOCA.

Cold leg recirculation, re-establishes thermal coupling with the SGs.


Question Source: T.FR03-03 Ginna NRC bank (1996) Medium Question Difficulty

Answer: Task No: B Obj No:

Time: Cross Ref: 1 1BFR-H.1, Response to Loss of Secondary Heat Sink. Reference: I1-FR-XL-03, BFRs H.1-H.5, Pg 5

With the RCS depressurized below the SGs, the break size is large enough for ECCS to provide adequate heat removal. Also, when this Explanation: occurs, the SGs are no longer thermodynamically coupled and they become a heat source instead of a heat sink, therefore, actions to restore them as a heat sink are not required and not necessary. A. is not correct for all LOCAs B. is correct for this assessment. C. is not correct, assumed ECCS is adequate. Higher order RED paths should NOT be anticipated with adequate ECCS flow. D. is incorrect since ECCS flow should be adequate before cold leg recirc is required. Date Written: 3/5/2006 Author: M. Jorgensen App. Ref:

Quest No: RO SRO: 19 Both System/Evolution Name: Emergency Boration

TIER: 1

GROUP: Topic No: KA No: RO: SRO: Cog Level: 000024 AK3.02 4.2 4.4 High Category Statement: Knowledge of the reasons for the following responses as they apply to the Emergency Boration:

KA Statement: Actions contained in EOP for emergency boration UserID: Question Stem: Topic

Given the following Unit 1 plant conditions: - Reactor Tripped 20 minutes ago due to a partially stuck open SG safety valve. - 2 RCCAs from Shutdown Bank B stuck in the mid-out position. - A loss of off-site power occurred while completing Step 3 of 1BEP-0, REACTOR TRIP OR SAFETY INJECTION - 1B DG is OOS - 1A DG is operating as expected. - RCS temperature is currently 548F. - The stuck open SG safety is causing a cooldown rate of 15F/Hour. Which of the following is/are required operator action(s) and why?

A
due to

Emergency borate using 1B CV pump from the RWST and MAXIMIZE CV pump flow the 2 RCCAs NOT fully inserted.

B
RCCAs

Emergency borate using the Boric Acid Transfer pump due to the cooldown and the 2 NOT fully inserted.

C
inserted.

No action is required because the Loss of Offsite Power ensures ALL Rods are fully

D
flow due

Emergency borate using the 1A CV pump from the RWST and MAXIMIZE charging to the cooldown and 2 RCCAs NOT fully inserted.
Answer: Task No: D Obj No: T.EP01-06-C, Question Source: Byron NRC exam bank (2000) Medium Question Difficulty

Time: Cross Ref: 1 1BEP ES-0.1, Reactor Trip or Safety Injection Reference: I1-EP-XL-01, BEP-0, Reactor Trip or Safety Injection, Pgs 57, 60 BOA PRI-2, Emergency Boration I1-OA-XL-13, BOA PRI-2, Emergency Boration, Pg 1

BEP ES-0.1 requires emergency boration for RCS temperature < 557F and for each rod not fully inserted if more than 1 is not fully Explanation: Date Written: 3/5/2006 Author: M. Jorgensen App. Ref:

inserted. Emergency boration can be accomplished from the BATs via Boric Acid Transfer pump to the CV pump suction or from the RWST via the CV pump. For this case, only 1A CV pump is available. Boric Acid Transfer pump is non ESF power. A. is incorrect since 1B CV pump has no power. B. is incorrect since the flowpath must include a CV pump (1A). C. is incorrect since Stuck RCCAs are still Stuck and the cooldown is still in progress. D. is correct as described.

Quest No: RO SRO: TIER: 20 Both 1 System/Evolution Name: Loss of Source Range Nuclear Instrumentation

GROUP: Topic No: KA No: 2 000032 2.2.22 Category Statement: Equipment Control

RO: 3.4

SRO: 4.1

Cog Level: High

KA Statement: Knowledge of limiting conditions for operations and safety limits. UserID: Question Stem: Topic

The following conditions exist on Unit 1: - A reactor startup is in progress. - Source range counts are 6.5E4 cps on N-31 and 6.95E4 on N-32. - Intermediate range power is 3.2E-5% on N-35 and 4.1E-5% on N-36. - Control Bank D is at 136 steps. Just prior to blocking SR NI's, N-31 fails low. Which of the following identifies the power limits due to the SR N-31 failure?

A B C D

Power must be immediately reduced to < P-6.

NO further positive reactivity additions can occur.

Reactor startup may proceed.

Power may be raised, but a MODE change SHALL NOT occur.


Question Source: T.OA10-06/12, S.NI1Byron 2001 Cert exam Medium Question Difficulty

Answer: Task No: C Obj No:

Ti e m: Cr sR : os ef 1 BOA INST-1, Nuclear Instrument Malfunction Reference: I1-OA-XL-10, BOA INST-1, NI Malfunction, Pg 13 Tech Spec/Bases 3.3.1, RTS Instrumentation I1-NI-XL-01, Ch 31, Source Range Nuclear Instrumentation, Pg 23

A trip will not occur with the SR channel failing low. The Tech Spec applies in MODE 2 with power < P-6. The condition given is > Explanation: P-6 and adequate IR overlap is given, therefore Blocking SR would be appropriate and power ascension may continue. A. is incorrect since this is not required action. B. is incorrect since power is > P-6, if below P-6, this would be required. C. is correct as described above. D. is incorrect, this is not a MODE change requirement. Date Written: 3/5/2006 Author: M. Jorgensen App. Ref:

Quest No: RO SRO: TIER: GROUP: Topic No: KA No: RO: SRO: Cog Level: 21 Both 1 2 000059 AK2.01 2.7 2.8 Low System/Evolution Name: Category Statement: Accidental Liquid Radwaste Release Knowledge of the interrelations between the Accidental Liquid Radwaste Release and the following: KA Statement: Radioactive-liquid monitors UserID: Question Stem: Topic

What action occurs when 0RE-PR16J, 0A Blowdown After Filter Outlet Radiation Monitor, detects a high radiation condition?

A B

The inlet valve to the Blowdown Monitor tank CLOSES, then re-opens automatically when the high radiation condition clears. The isolation valve to the main condenser or CST CLOSES, then re-opens automatically when the high radiation condition clears. The inlet valve to the Blowdown Monitor tank OPENS, then must be manually closed when the high radiation condition clears. The isolation valve to the main condenser or CST OPENS, then must be manually closed when the high radiation condition clears.
Answer: Task No: C Obj No: S.AR1-04-B-04 Question Source: Byron NRC exam bank (2000) - Modified Medium Question Difficulty

Time: Cross Ref: 1 I1-AR-XL-01, Ch 49, Radiation Monitors, Pgs 28 BAR RM-11 for 0RE-PR16J

Reference:

When 0RE-PR16J generates a HIGH alarm the auto action will close isolation valve to the condenser or CST after opening the inlet Explanation: to the Blowdown Monitor Tank. This allows collection of radioactive drains in a tank that can be aligned for processing in radwaste. The interlock is only auto in this direction and requires manual realignment after the condition clears. This will prevent a monitor malfunction from causing a realignment resulting in a release to the environment. A. is incorrect since this valve is opened and no auto realignment takes place. B. is incorrect since no auto realigment occurs. C. is correct as described above. D. is incorrect since this valve is closed, not opened. Date Written: 3/5/2006 Author: M. Jorgensen App. Ref:

Quest No: RO SRO: TIER: GROUP: Topic No: KA No: RO: SRO: Cog Level: 22 Both 1 2 000061 AK1.01 2.5 2.9 Low System/Evolution Name: Category Statement: Area Radiation Monitoring (ARM) System Knowledge of the operational implications of the following concepts as they apply Alarms to Area Radiation Monitoring (ARM) System Alarms: KA Statement: Detector limitations UserID: Question Stem: Topic

Which of the following transients will cause Main Steam Line Radiation Monitors, _AR22A/B/C/D, to indicate ERRONEOUSLY?

A B C D

LOCA inside Containment

Steam Generator Tube Rupture

Main Steam Line break inside Containment.

Feed Line break in the Safety Valve Enclosure.


Question Source: A.PF3-03, S,AR1-02New Medium Question Difficulty

Answer: Task No: D Obj No:

Time: Cross Ref: 1 I1-PF-XL-03, I&C Ch 3, Radiation Detection and Measurement Reference: I1-AR-XL-01, Ch 49, Radiation Monitors, Pgs 7 BAR RM11-1-1AR22J

These monitors use GM detectors and are placed in close proximity to each of the Main Steam lines in the Safety Valve Enclosure. Explanation: Since they are proximity devices, they have very little shielding around them from the room environment and, as stated in the BAR, are succeptible to alarming due to high temperature in the area. A. is incorrect since the harsh environment is in CNMT and not in the vicinity of the detectors. B. is incorrect since the detectors should work as designed for this condition, i.e. no change to their environment. C. is incorrect since the harsh environment is in CNMT and not in the vicinity of the detectors. D. is correct since this is the same enclosure where the detectors are located and high temperatures would result. Date Written: 2/27/2006 Author: M. Jorgensen App. Ref:

Quest No: RO SRO: TIER: 23 Both 1 System/Evolution Name: Control Room Evacuation

GROUP: Topic No: KA No: RO: SRO: Cog Level: 2 000068 AA2.11 4.3 4.4 High Category Statement: Ability to determine and interpret the following as they apply to the Control Room Evacuation:

KA Statement: Indications of natural circulation UserID: Question Stem: Topic

Given the following conditions on Unit 1: - Reactor tripped due to a fire in the Upper Cable Spreading Room. - All RCPs are stopped - The crew is in 1BOA PRI-5, Control Room Inaccessability Unit 1, with a cooldown established from the Remote Shutdown Panel. Which of the following indicates core heat removal by natural circulation is DEGRADING? (Consider each condition independently)

A B C D

SG pressures have dropped from 700 psig to 600 psig and are now starting to drop at a slower rate. PZR level is 15% and slowly dropping while indicated subcooling is 80F and rising.

Thot is 520F and Tcold is 460F and the Delta T between them is rising.

Thot is 490F and dropping slowly and indicated CETCs are dropping slowly.
Question Source: Byron bank Medium Question Difficulty

Answer: Task No: C Obj No: A.HT5-03/04

Ti e m: Cr sR : os ef 1 I1-HT-XL-01, Ch 5, Natural Circulation, Pg 11 Reference: BOA PRI-5, Control Room Inaccessability, Step 18 RNO

Items verified for adequate Nat Circ flow are subcooling, CETCs stable or dropping, Thot stable or dropping and consistent with Explanation: CETCs (CETCs will normally be slightly higher), Tcold ~ Tsat for SG pressure, and SG pressure stable or dropping. One other consideration is the design Nat Circ power-to-flow ratio is such that Thot-Tcold (delta-T) should never be > full power delta-T (60F). A. is incorrect since this meets the Nat Circ verification criteria. B. is incorrect since subcooling is adequate and PZR level response is not a criteria. C. is correct since the delta-T is 60F and rising. This denotes the driving head of Tcold is not adequate to provide sufficient flow through the core. D. is incorrect since this is an adequate response to verify Nat Circ is occurring. Date Written: 3/6/2006 Author: M. Jorgensen App. Ref:

Quest No: RO SRO: TIER: 24 Both 1 System/Evolution Name: High Containment Pressure

GROUP: Topic No: KA No: RO: SRO: Cog Level: 2 00WE14 EA2.2 3.3 3.8 Low Category Statement: Ability to determine and interpret the following as they apply to the High Containment Pressure:

KA Statement: Adherence to appropriate procedures and operation within the limitations in the facility's license and amendments UserID: Question Stem: Topic

Given the following conditions on Unit 1: - A LOCA has occurred. - Transition has been made to 1BEP ES-1.3, Transfer to Cold Leg Recirculation. - Containment Spray (CS) actuated as designed. - Spray Add Tank LO-2 lights are LIT - All systems are operating as expected. When and how is Containment Spray terminated?

A B

ONE CS pump is stopped when containment pressure is < 15 psig. The other CS pump is stopped when RWST LO-3 level is reached. ONE CS pump is stopped when containment pressure is < 20 psig. The other CS pump is stopped after it has operated for at least 2 hours. BOTH CS pumps are stopped when containment pressure is < 15 psig and have operated for at least 2 hours. BOTH CS pumps are stopped when containment pressure is < 20 psig and RWST LO-3 level is reached.
Question Source: S.CS1-12 Byron NRC exam bank (1998) Medium Question Difficulty

Answer: Task No: C Obj No:

Time: Cross Ref: 1 Ch 59, Containment Spray System, Pg 29 Reference: 1BEP-1, Loss of Reactor or Secondary Coolant, step 7

Following a LOCA it is part of the design criteria to terminate CS when containment pressure has dropped below 15 psig and the Explanation: Spray Add tank contents have been injected into the CNMT atmosphere and sump and recirculatedfor 2 hours for iodine removal and maintenance. A. is incorrect since no CS pumps are required to be run once CNMT pressure falls below 15 psig, however, 2 hours is required. If necessary, at Lo-3 in the RWST, suction is switched to the CNMT sump. B. is incorrect since the criteria is < 15 psig and both pumps are stopped. C. is corrrect as described above. D. is incorrect since the criteria is < 15 psig and 2 hours. If necessary, at Lo-3, suction is switched to the CNMT sump. Date Written: 3/6/2006 Author: M. Jorgensen App. Ref:

Quest No: RO SRO: 25 Both System/Evolution Name: Degraded Core Cooling

TIER: 1

GROUP: Topic No: KA No: RO: SRO: Cog Level: 2 00WE06 EA1.3 3.7 4.0 Low Category Statement: Ability to operate and/or monitor the following as they apply to the Degraded Core Cooling:

KA Statement: Desired operating results during abnormal and emergency situations UserID: Question Stem: Topic

A CAUTION in 1BFR-C.2, Response to Degraded Core Cooling, states that an SI accumulator injection may cause a Red path condition in INTEGRITY and that 1BFR-C.2 should be completed before transition to 1BFR-P.1, Response to Imminent Pressurized Thermal Shock Condition. The CAUTION applies during depressurization, prior to transitioning to 1BFR-P.1. What is the reason for this CAUTION?

A B

Responding to the INTEGRITY Red path at this time could result in a CORE COOLING Red path. The INTEGRITY Red path only protects the RCS barrier and the continued actions in 1BFRC.2 will protect both the fuel clad and the RCS barriers. Responding to the INTEGRITY Red path at this time could result in an INVENTORY Red path.

The INTEGRITY Red path will be corrected by continuing the actions of 1BFR-C.2.
Question Source: Byron NRC bank (2000) Medium Question Difficulty

Answer: Task No: A Obj No: T.FR02-02

Time: Cross Ref: 1 I1-FR-XL-02, BFR-C.1, C.2, C.3, Pg 54 Reference: BFR C.2, Response to Degraded Core Cooling, step 10

SI Accumulator injection will cause a significant temperature drop and rapid cooldown and may cause a Red path on INTEGRITY, Explanation: however, exiting before completing BFR C.2 would result in only a short term cooldown resulting in a Red path on CORE COOLING, since the first thing BFR-P.1 would have you do is stop the cooldown. A. is correct as described above. B. is incorrect since INTEGRITY is directly concerned with the RCS barrier and CORE COOLING action here is trying to cool the fuel to minimize/prevent fuel barrier damage and is not concerned at this point in protecting the RCS barrier. C. is incorrect since CORE COOLING is a higher priority and INVENTORY at worst can be a Yellow path. D. is incorrect since this is not necessarily true, but for barrier protection of the fuel, it is more important to gain control of core cooling, then deal with INTEGRITY.

Date Written:

3/6/2006 Author: M. Jorgensen

App. Ref:

Quest No: RO SRO: TIER: 26 Both 1 System/Evolution Name: LOCA Cooldown and Depressurization

GROUP: Topic No: KA No: RO: SRO: Cog Level: 2 00WE03 EK3.1 3.3 3.7 Low Category Statement: Knowledge of the reasons for the following responses as they apply to the LOCA Cooldown and Depressurization:

KA Statement: Facility operating characteristics during transient conditions, including coolant chemistry and the effects of temperature, pressure, and reactivity changes and operating limitations and reasons for these operating characteristics UserID: Topic Question Stem:

A reactor trip and SI have occurred on Unit 2. - Control room operators are responding to a small break LOCA. - All RCPs are stopped. - Containment pressure is normal. - The crew has transitioned to 2BEP ES-1.2, Post-LOCA Cooldown and Depressurization. - A PZR PORV is being used to depressurize the RCS until PZR level is > 25%. In addition to ensuring that RCS conditions are under adequate operator control, what is the basis for establishing this PZR level?

A
started.

Ensures sufficient inventory to prevent a low PZR level condition when a RCP is

B C D
start.

Ensures that a reduction in subcooling does not occur when SI flow is reduced.

Ensures that letdown can be established prior to starting a RCP.

Ensures adequate PZR steam space to absorb pressure fluctuations during an RCP

Answer: Task No: A Obj No: T.EP02-01-D

Question Source: Byron NRC bank (1996)

Question Difficulty Medium

Time: Cross Ref: 1 I1-EP-XL-02, Loss of Reactor or Secondary Coolant, Pgs 86, 87. Reference: 1/2BEP ES-1.2, Post-LOCA Cooldown and Depressurization

This level is established to start one RCP. It is assumed the level will drop when the RCP is started and allows adequate level to Explanation: maintain PZR level and pressure control to maintain the RCP running as the possible head void is collapsed. A. is correct as described above. B. is incorrect since this is not part of the ECCS flow reduction segment. C. is incorrect since letdown is not a concern at this point. Forced flow for cooldown and pressure control is. D. is incorrect since the concern is adequate inventory to support the RCP start to ensure it continues to run. There is plenty Date Written: 3/6/2006 Author: M. Jorgensen App. Ref: of steam space and the subsequent start will cause pressure to drop, not rise.

Quest No: RO SRO: TIER: GROUP: Topic No: KA No: RO: SRO: Cog Level: 27 Both 1 2 00WE10 EK2.1 3.3 3.5 Low System/Evolution Name: Category Statement: Natural Circulation with Steam Void in Vessel Knowledge of the interrelations between the Natural Circulation with Steam Void with/without RVLIS in Vessel with/without RVLIS and the following: KA Statement: Components, and functions of control and safety systems, including instrumentation, signals, interlocks, failure modes, and automatic and manual features UserID: Topic Question Stem:

Given the following Unit 1 conditions: - The crew is in 1BEP ES-0.2, Natural Circulation Cooldown. - PZR pressure is being controlled using Aux. Spray and PZR heaters. - Charging and letdown are in manual and are balanced. As pressure is being lowered through 1300 psig, a rapid rise is observed in PZR level. What action is required to be taken?

A B C D

Repressurize the RCS.

Isolate the SI accumulators.

Raise the RCS cooldown rate.

Place excess letdown in service.


Question Source: T.EP01-06-D Byron NRC bank (1998) Medium Question Difficulty

Answer: Task No: A Obj No:

Ti e m: Cr sR : os ef 1 1BEP ES-0.2, Natural Circulation Cooldown step 15 RNO I1-EP-XL-01, Reactor Trip or SI, Pg 102

Reference:

With vessel head cooling available, this procedure assumes no head voiding will occur. At step 15, any substantial rise in PZR level or Explanation: RVLIS indicating head voiding is occurring requires immediate repressurization of the RCS to collapse the void. A. is correct as described above. B. is incorrect since this is done at RCS pressure < 1000 psig as in any normal cooldown and at the pressure in question, SI accumulators would not be the reason for the rapid PZR level rise. C. is incorrect since the occurrence of head voiding is not diminished by raising the cooldown rate of the RCS. The metal mass in the head has very minimal cooling from the RCS fluid in Nat Circ. D. is incorrect since additional letdown will remove inventory, but the inventory has not risen, only displaced by an additional steam bubble. Date Written: 3/6/2006 Author: M. Jorgensen App. Ref:

Quest No: RO SRO: TIER: 28 Both 2 System/Evolution Name: Reactor Coolant Pump System (RCPS)

GROUP: Topic No: KA No: 003000 2.1.33 Category Statement: Conduct of Operations

RO: 3.4

SRO: 4.0

Cog Level: High

KA Statement: Ability to recognize indications for system operating parameters which are entry-level conditions for technical specifications. UserID: Question Stem: Topic

Given the following Unit 1 conditions: - RCS temperature (Average CETCs) = 340F - All SG pressures = 100 psig - RCS pressure = 390 psig - RCPs 1B and 1D running - RCPs 1A and 1C have breakers tagged OOS - RHR loops 1A and 1B are aligned for ECCS RCP 1D has just tripped on breaker overcurrent. What is/are the required action(s)?

A
operation.

Immediately take actions to place the unit in MODE 5 with either RHR Train in

B C D

Immediately place one train of RHR in service for shutdown cooling.

Restore 1A or 1C RCP to OPERABLE within 1 hour.

Immediately restore 1A or 1C RCP to service and make 1A or 1C RCP OPERABLE.


Question Source: S.RC1-12 Byron NRC exam bank (1998) Medium Question Difficulty

Answer: Task No: D Obj No:

Ti e m: Cr sR : os ef 1 Byron ITS Section 3.4.6 Reference: I1-RC-XL-01, Ch 12, Reactor Coolant System

The Tech Spec requires 2 loops of 4(2 RCS + 2 RHR) be OPERABLE and at least ONE in operation in MODE 4. The action Explanation: requirement (Cond B) states that for one required loop inoperable to initiate action to restore a second loop to operable Date Written: status immediately. 3/6/2006 Author: M. Jorgensen App. Ref:

A. is incorrect since a RHR loop is unavailable for SDC in this condition and is required to attain MODE 5. B. is incorrect since RCS pressure is too high to place RHR in service (<337 psig). C. is incorrect since the Tech Spec requires immediate action to restore one loop to operable, not 1 hour. D. is correct as described above.

Quest No: RO SRO: TIER: 29 Both 2 System/Evolution Name: Reactor Coolant Pump System (RCPS)

GROUP: Topic No: KA No: RO: SRO: Cog Level: 1 003000 K5.05 2.8 3.0 High Category Statement: Knowledge of the operational implications of the following concepts as they apply to the RCPS:

KA Statement: The dependency of RCS flow rates upon the number of operating RCPs UserID: Question Stem: Topic

Given Unit 1 at 25% power with the following plant conditions: - Steam Dumps are in TaveMode. - Rod Control is in Manual - 1A RCP trips With NO operator action, what is the response of SG pressures in the OPERATING loops one minute after the RCP tripped?

A B C D

Higher due to higher SG temperature.

Lower due to reactor trip on LO-2 SG level.

No change due to a constant steam demand.

Lower due to higher steam flow.


Question Source: Byron exam bank Medium Question Difficulty

Answer: Task No: D Obj No: A.HT7-09-A

Ti e m: Cr sR : os ef 1 I1-HT-XL-07, Ch 7, Steady State, Normal and Abnormal, Pgs 28-31

Reference:

This creates less total system flow; affected loop SG Tsat=Tcold from the other loops due to reverse flow resulting in essentially no Explanation: steam flow from this SG, core flow is less, core delta-T rises, SGs with RCP steam more for same steam demand, thus SG pressure drops with same steam demanded and same core power produced. A. is incorrect since just the opposite occurs. B. is incorrect since maintaining SG level will not be a result of the RCP trip. C. is incorrect since pressure must drop with same steam demanded from only 3 of the 4 SGs. D. is correct as described above. Date Written: 3/6/2006 Author: M. Jorgensen App. Ref:

Quest No: RO SRO: TIER: GROUP: Topic No: KA No: RO: SRO: 30 Both 2 1 004000 A4.04 3.2 3.6 System/Evolution Name: Category Statement: Chemical and Volume Control System Ability to manually operate and/or monitor in the control room: (CVCS) KA Statement: Calculation of boron concentration changes UserID: Question Stem: Topic

Cog Level: High

Unit 1 is operating at 50% power with Rod Control in Manual. A special RCS Chemistry procedure requires RAISING Tave 6F, with no change in rod position or plant power, by changing boron concentration ONLY. Given the following parameters: - Initial RCS boron concentration = 600 ppm - Moderator Temperature Coefficient = -15 pcm/F - Differential boron worth = -10 pcm/ppm Which of the following is the final RCS boron concentration needed to RAISE Tave 6F?

A B C D

591 ppm

596 ppm

604 ppm

609 ppm
Question Source: A.RT5-07 Byron Cert exam bank (2001) Medium Question Difficulty

Answer: Task No: A Obj No:

Ti e m: Cr sR : os ef 1 Reactor Theory, Ch 5, I1-RT-XL-05, Chemical Shim Control, Pgs 21-23, 30 Reference:

Raising temperature at constant power and a negative MTC means that negative reactivity needs to be removed (add + reactivity) to Explanation: offset the change, therfore boron must be removed to raise temperature. The negative reactivity from the moderator temperature rise requested is -15 pcm/F X 6F = -90 pcm to offset. -90 pcm divided by -10 pcm/ppm boron worth = 9 ppm reduction in boron concentration. 600 ppm - 9 ppm = 591 ppm. A. is correct as calculated above. B. is incorrect and could have resulted from taking 6F X -10 pcm/ppm, then divide by -15 pcm/F = 4 and subtract from 600 to get 596. Misconception error. C. is incorrect and could have happened with the misconception in B. above, then add it to 600. D. is incorrect and could have resulted from the misconception of reactivity added being + or sign mistake and misconception. Date Written: 3/6/2006 Author: M. Jorgensen App. Ref:

Quest No: RO SRO: TIER: GROUP: Topic No: KA No: RO: SRO: Cog Level: 31 Both 2 1 004000 K4.16 2.6 3.0 Low System/Evolution Name: Category Statement: Chemical and Volume Control System Knowledge of CVCS design feature(s) and/or interlock(s) which provide for the (CVCS) following: KA Statement: Temperature at which the temperature control valve automatically diverts flow from the demineralizer to the VCT; reason for this diversion UserID: Topic Question Stem:

As CVCS Letdown temperature reaches ___(1)___, Letdown flow automatically bypasses the demineralizers to ____(2)______.

(1)

(2) protect the demin resin beads from decomposing and releasing impurities into the RCS. maintain the VCT at the proper temperature for Hydrogen addition.

A B C

133F

125F

133F

prevent reverse ion exchange from occurring resulting in RCS dilution. prevent excessive channeling through the resin bed reducing ion exchange efficiency.
Question Source: New Low Question Difficulty

125F

Answer: Task No: A Obj No: S.CV1-05-D

Time: Cross Ref: 1 BAR 1-9-E2 Reference: GFE Fundamentals, Sect 1, Ch 7 I1-CV-XL-01, Ch 15a, CVCS, Pg 15, 16

CV129, Letdown Demin High Temp Divert Valve, will divert flow directly to the VCT, bypassing the demins, at 133F. The high Explanation: temperature alarm is also at 133F on 1-9-E2. At > 140F, the resin will overheat resulting in degradation. This may lead to release of ions exchanged as well as the resin matrix itself. These constituants are not compatible with RCS chemistry control and once activated can cause coolant activity levels to be elevated and cause pH problems. A. is correct as described. B. is incorrect since the actuation is at 133F and diversion has nothing to do with H2 addition. C. is incorrect since reverse ion exchange is not the concern. Some additonal boron is initially released as temperature rises, but this is not due to reverse ion exchange. D. is incorrect since the setpoint is 133F and high flow would cause channeling to occur, not high temperature.

Date Written:

3/6/2006 Author: M. Jorgensen

App. Ref:

Quest No: RO SRO: TIER: GROUP: Topic No: KA No: RO: SRO: Cog Level: 32 Both 2 1 005000 K5.02 3.4 3.5 Low System/Evolution Name: Category Statement: Residual Heat Removal System (RHRS) Knowledge of the operational implications of the following concepts as they apply to the RHRS: KA Statement: Need for adequate subcooling UserID: Question Stem: Topic

A LIMITATION in BOP RH-6, Operation of the RH System in Shutdown Cooling, states that "Switchover from the shutdown cooling to ECCS alignment should not be performed when the RCS Hot Leg temperature is > 260F". What is the reason for this LIMITATION?

A B

Realignment could overpressurize the RWST.

Realignment may cause steam binding at the RH pump suction to be aligned for ECCS injection. Realignment may result in inadequate subcooling at the RH pump suction that is being aligned to provide shutdown cooling. Realignment may cause RWST temperature limit to be exceeded.
Question Source: New Medium Question Difficulty

Answer: Task No: B Obj No: S.RH1-12

Time: Cross Ref: 1 BOP RH-6, Operation of the RH System in Shutdown Cooling. Reference: I1RH-XL-01, Ch 18, Residual Heat Removal System, Pg 23

When an RHR train is in the SDC lineup and temperature at the suction is > 260F and subsequently realigned for ECCS injection, the Explanation: suction header change in pressure from the RCS to the RWST will cause voiding (flashing) at the pump suction or lack of subcooling for a pump start. This makes the train inoperable for ECCS injection and, until MODE 5 is entered, 1 Train is required to be in the ECCS injection lineup. A. is incorrect since the RWST is adequately vented, overpressure would not be a concern. B. is correct as described above. C. is incorrect since the concern is the RHR train being realigned for ECCS injection. The train taking suction on the RCS will be well above saturation on the pump suction. D. is incorrect since the RWST volume is so large, there would be very little temperature rise. Date Written: 3/6/2006 Author: M. Jorgensen App. Ref:

Quest No: RO SRO: TIER: 33 Both 2 System/Evolution Name: Residual Heat Removal System (RHRS)

GROUP: Topic No: KA No: RO: SRO: Cog Level: 1 005000 K6.03 2.5 2.6 High Category Statement: Knowledge of the effect of a loss or malfunction of the following will have on the RHRS:

KA Statement: RHR heat exchanger UserID: Question Stem: Topic

Unit 1 is shutdown with the following conditions: - 1B Train RHR providing shutdown cooling. - RCS pressure = 350 psig - RCS temperature = 330F - RCS cooldown rate = 30F/hour - RHR total flow = 3300 gpm - 1RH607, RH HX 1B Outlet Flow Control Vlv, throttled at 52% open (1500 gpm) Flow transmitter 1FT619, 1B RH Discharge Flow, fails LOW with flow controller for 1RH619, RH HX Bypass valve, in automatic. What will the operator observe due to this failure?

A B C D

The RCS cooldown rate will NOT change.

The RCS cooldown rate will RISE.

The RCS cooldown rate will DROP.

RCS pressure would rapidly DROP.


Question Source: S.RH1-11 Byron Cert exam bank (2001) - modified Medium Question Difficulty

Answer: Task No: C Obj No:

Ti e m: Cr sR : os ef 1 I1-RH-XL-01, Ch 18, Residual Heat Removal System, Pg 12, 46, 52, 59

Reference:

1RH619 controller is set at 3300 gpm in auto to maintain total RH pump flow at that value. The valve modulates to maintain that Explanation: flow as sensed by 1FT619. With 1FT619 failing low, the 1RH619 valve will open to raise the flow sensed by 1FT619. As it opens, less RH pump discharge is through the heat exchanger and more is bypassed through the 1RH619 valve, therefore the RCS cooldown rate will drop. A. is incorrect since less flow will be through the heat exchanger. B. is incorrect since the cooldown rate will drop as more flow bypasses the heat exchanger. C. is correct as described above. D. is incorrect since the total flow will rise, but returns to the RCS, no RCS inventory is lost and a bubble exists in the PZR. Date Written: 3/6/2006 Author: M. Jorgensen App. Ref:

Quest No: RO SRO: TIER: GROUP: Topic No: KA No: RO: SRO: Cog Level: 34 Both 2 1 006000 K6.03 3.6 3.9 High System/Evolution Name: Category Statement: Emergency Core Cooling System (ECCS) Knowledge of the effect of a loss or malfunction of the following will have on the ECCS: KA Statement: Safety Injection Pumps UserID: Question Stem: Topic

Given the following plant conditions on Unit 1: - A LOCA has occurred - 1B SI pump tripped - Transfer to Cold Leg Recirculation is required. - RCS pressure is ~ 50 psig. What is the approximate total SI pump flow indicated on the main control board and how will this value change following transfer of BOTH trains of ECCS to Cold Leg Recirculation? (Flow) (Change) lower

A B C D

605 gpm

605 gpm

higher

1210 gpm

lower

1210 gpm

higher
Question Source: Question Difficulty Medium

Answer: Task No: B Obj No: S.EC1-03-B

Byron Cert exam bank (2001)

Ti e m: Cr sR : os ef 1 I1-EC-XL-01, Ch 58, ECCS, Pgs 9, 10

Reference:

Maximum flowrate for each SI pumps is ~ 650 gpm with 800 psig in the RCS. Below 800 psig, flow is restricted to this value. The Explanation: total flow includes ~ 45 gpm recirc flow, so the actual injection will be read on the MCR indicator as ~ 605 gpm. Since the pumps create this flowrate taking suction from the RWST and creates flow based on DP across the pump, the flowrate will rise when the suction pressure rises in cold leg recirc because the RH pumps are now aligned to provide suction pressure to the SI pump. The actual suction pressure changes from ~ 20 psig to as much as 200 psig. Also, part of the alignment shift to Cold Leg Recirc closes the SI mini-flow valves, thus the 45 gpm recirc will now go into the RCS and be seen on the MCB meter. And in this case only one SI pump is running. A. is incorrect since flow will rise due to significantly higher suction pressure to the pump. B. is correct as described above. C. is incorrect since only one SI pump is running and flow would be higher. D. is incorrect since only one SI pump is running. Date Written: 3/6/2006 Author: M. Jorgensen App. Ref:

Quest No: RO SRO: TIER: GROUP: Topic No: KA No: RO: SRO: Cog Level: 35 Both 2 1 007000 A2.06 2.6 2.8 High System/Evolution Name: Category Statement: Pressurizer Relief Tank/Quench Tank System Ability to (a) predict the impacts of the following malfunctions or operations on (PRTS) the PRTS and (b) based on those predictions, use procedures to correct, control, or mitigate the consequences of those malfunctions or operations: KA Statement: Bubble formation in PZR UserID: Question Stem: Topic

Unit 1 is in MODE 5 preparing to draw a bubble in the PZR with the following initial conditions in the PRT:

- PRT level = 71% - PRT pressure = 4.5 psig - PRT temperature = 85F Following venting the PZR solid: - PRT level = 77% and stable - PRT pressure = 7.2 psig and slowly rising - PRT temperature = 92F and stable What caused this response and what is the required action?

A B C D

PZR PORV did NOT Close; CLOSE the PZR PORV Block valve.

Gaseous Waste isolation valve did NOT close; CLOSE the PRT to GW isolation valve.

Nitrogen Regulator has failed; CLOSE N2 Supply to PRT isolation valve.

RCP Seal Leakoff Relief valve is lifting; VERIFY Seal Leakoff lineup to the VCT.
Question Source: S.RY1-13/14/15 New Medium Question Difficulty

Answer: Task No: C Obj No:

Time: Cross Ref: 1 BAR 1-12-B7, PRT PRESS HIGH Reference: I1-RY-XL-01, Ch 14, Pressurizer, Pgs 14, 15, 22

At 6 psig, RY469 auto closes to GW from the PRT. The BAR says probable cause is (1) Valve leakoff or relief valve flow, (2) PORV Explanation: or Safety valve lifted, (3) Filling PRT, and (4) N2 Regulator failure. With no additional level or temperature rise, the N2 regulator would be the appropriate selection for cause. Subsequent action closes PW to PRT CNMT isolation (this would not fix the problem since there is no indication it is leaking by, i.e. no level rising) and closes N2 supply to the PRT. This would be correct action for the indicated pressure rise. A. is incorrect because temperature and level are stable. B. is incorrect because pressure would only be sensed from the PRT to the GW header, check valve would prevent reverse. C. is correct by process of elimination as described above. D. is incorrect since level is stable. Date Written: 3/7/2006 Author: M. Jorgensen App. Ref:

Quest No: RO SRO: TIER: GROUP: Topic No: KA No: RO: SRO: Cog Level: 36 Both 2 1 008000 K3.01 3.4 3.5 High System/Evolution Name: Category Statement: Component Cooling Water System (CCWS) Knowledge of the effect that a loss or malfunction of the CCWS will have on the following: KA Statement: Loads cooled by CCWS UserID: Question Stem: Topic

Unit 1 is operating with the following conditions: - Cooldown rate = 20F/hour. - 1A RH Train is in service, 1B RH Train is in standby. - 1A RCP is running. - 1B, 1C, and 1D RCPs are stopped. - RCS pressure = 225 psig with a bubble in the PZR (Aux Spray valve 1CV8145 is open). - RCS Hot Leg temperature = 230F. - Feeding and steaming SGs has been secured. - Indicated letdown flow = 75 gpm. What would be the effect of 1CC130A, Letdown HX Outlet Temperature Control Valve, failing OPEN? (Assume no operator action and neglible change in 1A RH HX return temperature to the RCS)

A B C D

RCS temperature will RISE.

RCS pressure will DROP.

RCS pressure will RISE.

Neglible effect on RCS conditions.


Question Source: S.CV1-05-A, S.CV1New Medium Question Difficulty

Answer: Task No: B Obj No:

Ti e m: Cr sR : os ef 1 I1-CV-XL-01, Ch 15a, CVCS, Pg 91

Reference:

This failure would put maximum CC flow through the letdown Hx dropping letdown temperature. This would then cool the VCT. Explanation: Cooler water is then returned to the RCS via RCP seals, normal charging, and Aux spray to the PZR. With no operator action, RCS pressure would drop requiring manual action to restore pressure. A. is incorrect; even if some flow is diverted from the RH Hx to provide more for the L/D Hx, this would only slow the cooldown rate. The major impact would be charging to the PZR. B. is correct as discussed above. C. is incorrect since overall a temperature drop would occur, thus pressure, would drop. D. is incorrect since the colder water to the PZR would cause a significant drop in pressure. Date Written: 3/7/2006 Author: M. Jorgensen App. Ref:

Quest No: RO SRO: TIER: GROUP: Topic No: KA No: RO: SRO: Cog Level: 37 Both 2 1 010000 K1.03 3.6 3.7 High System/Evolution Name: Category Statement: Pressurizer Pressure Control System (PZR Knowledge of the physical connections and/or cause-effect relationships between PCS) the PZR PCS and the following systems: KA Statement: RCS UserID: Question Stem: Topic

Given the following conditions on Unit 1: - Reactor power is steady at 100%. - All systems normally aligned. - Tave is steady on program. - PZR level is on program and stable. - PZR pressure is 2230 psig and begins dropping slowly.

Which of the following has occurred?

A B C D

1LK-459, PZR Level Controller, has failed HIGH.

1RY456, PZR PORV, is full OPEN

1PT-458, PZR pressure transmitter, has failed HIGH.

1RY455B, PZR Spray valve, has failed to 20% OPEN.


Question Source: Byron NRC exam bank (2000) Medium Question Difficulty

Answer: Task No: D Obj No: S.RY1-25

Ti e m: Cr sR : os ef 1 I1-RY-XL-01, Ch 14, Pressurizer, Pgs 39, 60, 73

Reference:

Pressure drops from the condensing action of a large amount of spray flow with no change in PZR level. Explanation: A. is incorrect because this failure will cause both level and pressure to rise. This will call for max charging flow. B. is incorrect since this would cause pressure to drop rapidly, not slowly. C. is incorrect since this failure will not open a PORV. It only disables PORV reset at 2185 psig, if it were to open inadvertently. D. is correct as described above. Date Written: Quest No: RO SRO: 38 Both 3/7/2006 Author: M. Jorgensen TIER: GROUP: Topic No: 2 1 012000 App. Ref: KA No: K3.01

RO: 3.9

SRO: 4.0

Cog Level: High

System/Evolution Name: Reactor Protection System

Category Statement: Knowledge of the effect that a loss or malfunction of the RPS will have on the following:

KA Statement: CRDS UserID: Question Stem: Topic

Given the following Unit 1 conditions: - Reactor power is at 100%. - Reactor Trip Bypass breaker A (BYA) is racked in and closed for testing. - Both Reactor Trip breakers (RTA and RTB) are closed. What would occur if a single 15 VDC power supply failed in the 1B Train SSPS Logic cabinet?

A B C D

The redundant power supply will maintain normal SSPS Train B conditions and only a Safeguards Test Cabinet Power Failure alarm is generated. The reactor trips when BOTH the UV and Shunt trip coils are actuated for BYA and RTB.

Plant conditions remain unchanged with a General Warning alarm lit for 1B train.

The reactor trips when BOTH the UV and Shunt trip coils are actuated for RTA and RTB.
Question Source: S.RP1-06/09/11 Byron NRC exam bank (2000) - modified High Question Difficulty

Answer: Task No: D Obj No:

Time: Cross Ref: 1 I1-RP-XL-01, Ch 60a, SSPS, Pgs 16-18

Reference:

The condition of Train A has generated a General Warning, since BYA is racked in and closed. The loss of the 15 VDC on Train B Explanation: generated another General Warning. With 2 General Warnings an automatic Rx Trip is generated. An automatic Rx Trip will deenergize the UV coils on BYA, RTA, and RTB, but only energize the shunt trip coils on RTA and RTB. A manual Rx Trip actuation would also energize the shunt trip coil on BYA. A. is incorrect since a Rx trip is generated with 2 general warnings and the alarm is activated when a 120 VAC power source to SSPS is lost. B. is incorrect because the shunt trip coil is not actuated for BYA on an automatic Rx Trip signal. C. is incorrect because a Rx Trip will occur since Train A already had a general warning in from BYA racked in and closed. D. is correct as described above. Date Written: 3/7/2006 Author: M. Jorgensen App. Ref:

Quest No: RO SRO: TIER: GROUP: Topic No: KA No: RO: SRO: 39 Both 2 1 013000 A4.01 4.5 4.8 System/Evolution Name: Category Statement: Engineered Safety Features Actuation System Ability to manually operate and/or monitor in the control room: (ESFAS) KA Statement: ESFAS-initiated equipment which fails to actuate UserID: Question Stem: Topic

Cog Level: High

Given the following indications on Unit 1: - Reactor power was at 100% when a spurious SI signal was generated. - Reactor trip breaker B failed to OPEN. - The spurious SI signal has cleared - Both SI reset pushbuttons have been depressed. - The RH pumps, SI pumps, and the 1A CV pump have been stopped. Then a small break LOCA occurs. What will occur as containment pressure RISES to 10 psig? (Assume no operator action is taken)

A B C
CLOSE.

ONLY the MSIVs and MSIV Bypass valves CLOSE.

1B and 1C MSIVs CLOSE, 1A and 1D MSIVs remain OPEN

1A RH, 1A SI, and 1A CV pumps START; all MSIVs and MSIV Bypass valves

1B RH and 1B SI pumps START; all MSIVs and MSIV Bypass valves CLOSE.
Question Source: S.MS1-07-D, S.EC1Byron NRC exam bank (2000) Medium Question Difficulty

Answer: Task No: D Obj No:

Time: Cross Ref: 1 I1-MS-XL-01, Ch 23, Main Steam System, Pgs 10, 11 Reference: I1-EC-XL-01, Ch 58, Emergency Core Cooling System, Pg 25 I1-EFXL-01, Ch 61, ESFAS, Pgs 17, 18

with the B Rx trip breaker open, B train SI reset has no seal in and therefore any additional SI actuation setpoint exceeded will cause a Explanation: B train SI. A Train SI is reset and sealed in until A Rx trip breaker is cycled. There has been no change in the inputs for Main Steam Isolation Actuation. Now, when pressure rises in CNMT, B train SI will actuate at 3.4 psig and MSIVs and Bypass valves will receive a close signal at 8.2 psig (both Trains). Since offsite power is supplying the ESF buses and 1B CV pump was already running, it will continue to run, therefore only the 1B RH and 1B SI pump will start. A train ECCS will not auto start. A. is incorrect since B train SI pumps will also start. B. is incorrect since all MSIVs and Bypasses close and B train SI pumps start. C. is incorrect since A train SI will not occur. Date Written: 3/7/2006 Author: M. Jorgensen D. is correct as described above. App. Ref:

Quest No: RO SRO: TIER: GROUP: Topic No: KA No: RO: 40 Both 2 1 013000 K2.01 3.6 System/Evolution Name: Category Statement: Engineered Safety Features Actuation System Knowledge of bus power supplies to the following: (ESFAS) KA Statement: ESFAS/safeguards equipment control UserID: Question Stem: Topic

SRO: 3.8

Cog Level: High

Unit 2 Reactor trip and SI have occurred. All systems responded as designed. 1 minute into the event the following indications occur: - 2B DG indicates tripped. - 2B Train ESF pumps show running amps, but all RUN lights are NOT LIT. What could have caused these indications to occur?

A B C D

SAT Feed Breaker 242-2 has tripped

120 VAC Bus 212 has deenergized

480 VAC MCC 232X has tripped

125 VDC Bus 212 has deenergized


Question Source: S.EF1-08, S.DC1-06 New Medium Question Difficulty

Answer: Task No: D Obj No:

Ti e m: Cr sR : os ef 1 I1-EF-XL-01, Ch 61, ESFAS Reference: I1-DC-XL-01, Ch 8a, 125 VDC Power Systems

Loss of DC power to a running DG will cause the DG to shutdown (fuel racks close), but in the control room the annunciators will Explanation: indicate a trip condition. Run lights are also DC powered, as is breaker indications of position, but the amp indications are AC powered, therefore with running pumps, amps will continue to display, but run lights go out. A. is incorrect since this would have caused the DGs to close onto their respective buses and load. B. is incorrect since this would not cause either indication to occur. C. is incorrect since loss of any single MCC would not have caused these indications to occur. D. is correct as described above. Date Written: 3/7/2006 Author: M. Jorgensen App. Ref: none

Quest No: RO SRO: TIER: 41 Both 2 System/Evolution Name: Containment Cooling System (CCS)

GROUP: Topic No: KA No: RO: SRO: Cog Level: 1 022000 A1.02 3.6 3.8 High Category Statement: Ability to predict and/or monitor changes in parameters (to prevent exceeding design limits) associated with operating the CCS controls including:

KA Statement: Containment pressure UserID: Question Stem: Topic

The following conditions exist on Unit 1: - LOCA is in progress. - Containment Spray has actuated. - Containment pressure is currently 18 psig. - Containment Spray signal has been reset. - All actions of 1BEP ES-1.3, Transfer to Cold Leg Recirculation, have been completed. Off-site power has been lost and the DG output breakers have just CLOSED on the ESF buses. How will the Containment Spray Pumps respond?

A B C

Pumps will auto start 15 seconds later.

Pumps will auto start 40 seconds later.

If the operator immediately places BOTH CS & PHASE B ISOL switches to ACTUATE, pumps will start immediately. If the operator immediately places BOTH CS & PHASE B ISOL switches to ACTUATE, pumps will auto start 15 seconds later.
Question Source: S.CS1-0/16 Byron NRC exam bank (1998) - modified Medium Question Difficulty

Answer: Task No: D Obj No:

Ti e m: Cr sR : os ef 1 I1-CS-XL-01, Ch 59, Containment Spray System, Pgs 13-15

Reference:

CS pumps will sequence with a DG output breaker closure at 15-18 seconds or after 40 seconds depending on whether 2/4 logic is Explanation: satisfied by CNMT pressure at or above 20 psig. Reset of CS & Phase B wil block auto restart of the CS pumps. For the loss of offsite power, the loads on the 4KV buses will load shed, then sequence. At 15 seconds the CS pump will restart only if the operator manually reactivates both CS & Phase B relay switches for the start logic to see an active start permissive. However, the DG sequencer will still control the actual start time of the CS pumps. A. is incorrect since the CS pump circuit will not see HI-3 Cnmt pressure in this condition. B. is incorrect since the CS pump circuit will not see HI-3 Cnmt pressure and would have started at 15 seconds if it had. C. is incorrect since manual actuation is blocked by the sequencer to prevent a DG overload during sequencing. D. is correct as described above.

Date Written:

3/7/2006 Author: M. Jorgensen

App. Ref:

Quest No: RO SRO: TIER: 42 Both 2 System/Evolution Name: Containment Spray System (CSS)

GROUP: Topic No: KA No: RO: SRO: 1 026000 A3.01 4.3 4.5 Category Statement: Ability to monitor automatic operation of the CSS, including:

Cog Level: High

KA Statement: Pump starts and correct MOV positioning UserID: Question Stem: Topic

Given the following conditions on Unit 1: - The unit is operating at 100% power. - 1CS007A, 1A CS Pump Header Isolation valve, is OOS and CLOSED for breaker inspection. - A spurious SI signal has actuated. - SI is reset and ECCS has been terminated. Five minutes later, a steamline break occurs. - Containment pressure RISES to 25 psig. - Main steamline pressure DROPS to 600 psig. With no operator action, the 1A CS pump will ___(1)___ and the 1B CS pump will ___(2)___.

(1)

(2)

A B C D

START

NOT START

START

START

NOT START START

NOT START

NOT START
Question Source: Question Difficulty Medium

Answer: Task No: B Obj No:

S.CS1-08-C, S.CS1-09 Byron Cert exam bank (2001)

Ti e m: Cr sR : os ef 1 I1-CS-XL-01, Ch 59, Containment Spray System, Pgs 13-17

Reference:

CS pumps will auto start with 2/4 CNMT pressures at or above 20 psig (HI-3) with power to the breaker. 1CS007A/B receive an open Explanation: signal, but are not interlocked to be open for a CS pump to start. The 1A CS pump will not provide any flow to CNMT with 1CS007A closed, but the pump will start as long as 1CS019A, Eductor Suction Isolation valve, from the Spray Additive Tank is open. A. is incorrect since nothing is identified on Train B that would prevent an auto start of 1B CS pump. B. is correct as described above. C. is incorrect since 1CS007A closed will not prevent an auto start of 1A CS pump. D. is incorrect since both CS pumps would receive an auto start signal and have all required interlocks satisfied to start. Date Written: 3/7/2006 Author: M. Jorgensen App. Ref:

Quest No: RO SRO: TIER: GROUP: Topic No: KA No: RO: SRO: 43 Both 2 1 039000 A4.01 2.9 2.8 System/Evolution Name: Category Statement: Main and Reheat Steam System (MRSS) Ability to manually operate and/or monitor in the control room:

Cog Level: Low

KA Statement: Main steam supply valves UserID: Question Stem: Topic

Unit 1 is at 3% power preparing for power ascension. Reheater Temperature Control System (RTC) is in AUTO with a Cold Start in progress. When should OPEN indication for the 1MS009A/B/C/D, MSR MS Shutoff valves, be observed?

A B C D

Main Turbine reaches 600 rpm.

Main Turbine load reaches 35%.

Main Turbine reaches 1800 rpm.

Main Turbine Generator reaches 250 Mwe.


Answer: Task No: B Obj No: S.MT1-08 Question Source: New Medium Question Difficulty

Time: Cross Ref: 1 I1-MT-XL-01, Ch 35, Main Turbine and Reheaters

Reference:

A. is incorrect - 600 rpm is true for a Hot Start. Explanation: B. is correct - RTC in Auto and Cold Start opens valves at 35% as sensed off 1st stage HP turbine. C. is incorrect - 1800 rpm is turbine rated speed but provides NO feedback to RTC. D is incorrect - Mwe does not feedback to RTC. Date Written: 3/8/2006 Author: M. Jorgensen App. Ref:

Quest No: RO SRO: TIER: GROUP: Topic No: KA No: RO: SRO: Cog Level: 44 Both 2 1 039000 K5.05 2.7 3.1 Low System/Evolution Name: Category Statement: Main and Reheat Steam System (MRSS) Knowledge of the operational implications of the following concepts as they apply to the MRSS: KA Statement: Bases for RCS cooldown limits UserID: Question Stem: Topic

Unit 1 reactor has tripped. If all systems respond as expected, the 1MS009A-D, MSR MS Reheater Shutoff valves, 1MS067A-D, S/U Purge valves, and 1MS147A-D and 1MS010A-D, RHTR Temperature Control valves, will automatically CLOSE. In 1BEP ES-0.1, Reactor Trip Response, at Step 2 the Reheater Shutoff and S/U Purge valves may need to be VERIFIED CLOSED. What is the concern if these valves are still OPEN?

A B
turbine

MSR reliefs may be challenged causing internal damage to the MSRs.

Excessive heating may occur with potential damage to the turbine LP sections during coastdown.

Excessive RCS cooldown may occur resulting in uncontrolled positive reactivity addition and subsequent challenge to RCS INTEGRITY.

D
thus

Excessive loading may be placed on the Main Condenser resulting in overpressure, limiting use of Steam Dumps for decay heat removal.

Answer: Task No: C Obj No: S.MT1-08, S.EP01-

Question Source: New

Question Difficulty Medium

Time: Cross Ref: 1 I1-MT-XL-01, Ch 35, Main Turbine and Reheaters Reference: I1-EP-XL-01, 1BEP-0 (- ES-0.2), Reactor Trip or SI, Pg 56

Reactivity management is the first issue with temperature in the RCS < 557F. These valves are addressed in the RNO for step 2 of Explanation: BEP ES-0.1 to ensure excess steam demand is not causing an uncontrolled cooldown adding positive reactivity and reducing SDM. A continued excessive cooldown rate could eventually challenge RCS INTEGRITY, therefore the RNO action of ensuring these valves are closed is necessary. The valves open will continue to draw main steam through the MSR tube bundle to the condenser. This could add undue stress to the tube bundle, however, all drains should be open to the condenser. Even though conditions could be created to speculate damage to components, the primary reason for the action is the continued main steam draw creating impacts on the RCS, not what may occur to secondary components. A. is incorrect since with proper Main Turbine Trip actuation, this pressure is dumped rapidly to the condenser. B. is incorrect since this steam or the environment that might be heated up will be dissipated to the condenser. The LP segments Date Written: 3/8/2006 Author: M. Jorgensen App. Ref: are also protected from overpressure, as well as the condenser, by blowout rupture discs on the LP casings.

C. is correct as described above. D. is incorrect since overpressure of the condenser is protected by blowout rupture discs on the LP casings.

Quest No: RO SRO: TIER: 45 SRO 2 System/Evolution Name: Main Feedwater (MFW) System

GROUP: Topic No: KA No: RO: SRO: Cog Level: 1 059000 A1.03 2.7 2.9 High Category Statement: Ability to predict and/or monitor changes in parameters (to prevent exceeding design limits) associated with operating the MFW System controls including:

KA Statement: Power level restrictions for operation of MFW pumps and valves UserID: Question Stem: Topic

Unit 2 is operating at 95% power. - Low Pressure Feedwater Heater Bypass Valve receives an OPEN signal due to a circuit problem. Plant cycle efficiency will _____________________________ as a result of this valve opening.

A B

lower, driving Reactor Power up, since more energy from the reactor is required due to feedwater entering the SGs at a lower temperature rise, driving reactor power down, due to less work required to pump the water through the low pressure feedwater heaters lower, driving Reactor Power up, since mass flowrate entering the SGs will be higher

C D

rise, driving Reactor Power down, due to the rise in feedwater temperature resulting in raising the efficiency of the high pressure feedwater heaters
Question Source: Millstone NRC exam bank (1997) Medium Question Difficulty

Answer: Task No: A Obj No: A.HT7-02

Time: Cross Ref: 1 I1-HT-XL-07, Ch 7, Steady State Operation, Normal and Abnormal, Pgs 8, 14 Reference:

This actuation results in a drop in feedwater reheating for the return feed to the steam generators. Efficiency is basically Work of the Explanation: turbine less work of the FW, CD/CB pumps divided by Q reactor. Overall, for a constant load on the main turbine, the Net work will be relatively unchanged, however, the Q of the reactor will have to be higher with the lower feedwater temperature to maintain the same turbine work. This will result in lowering plant efficiency. A. is correct as described above. B. is incorrect since efficiency is net work divided by Q reactor. The Net work change is minimal and the steam flow through the turbine to maintain MWs out will have to rise, since Steam pressure will drop slightly since Tave will drop with no operator action to maintain Tave the same. C. is incorrect since mass flowrate change in the secondary will have much less effect than the Q reactor change assuming MWs out remain unchanged. D. is incorrect since just the opposite will occur as described above.

Date Written:

3/7/2006 Author: M. Jorgensen

App. Ref:

Quest No: RO SRO: TIER: GROUP: Topic No: KA No: 46 Both 2 1 061000 2.1.28 System/Evolution Name: Category Statement: Auxiliary / Emergency Feedwater (AFW) Conduct of Operations System KA Statement: Knowledge of the purpose and function of major system components and controls. UserID: Question Stem: Topic

RO: 3.2

SRO: 3.3

Cog Level: Low

The 1B AF pump can be started locally at the 364' level in the Aux Building near the CC Heat Exchangers. One of the switch positions is "START WITH BYPASS". Which of the following trip signals does this switch position bypass?

A B C D

All trips are bypassed.

Low lube oil pressure.

Low suction pressure.

High Jacket Water temperature.


Question Source: S.AF1-05/15 Byron Cert exam bank (2001) Medium Question Difficulty

Answer: Task No: C Obj No:

Ti e m: Cr sR : os ef 1 I1-AF-XL-01, Ch 26, Auxiliary Feedwater System, Pg 10

Reference:

This Switch position can only be used if there is a Lo-Lo suction trip and a fire is present in the AEER or Aux building elevation 383', Explanation: 401' or 426'. The switch also has a NORMAL position that allows starts from normal locations and a START position to allow a remote start with suction pressure > Lo-Lo suction trip. A. is incorrect since it will bypass the Lo-Lo suction trip only. The other trips are still in the circuit but not as likely impacted by a fire. B. is incorrect since this is never bypassed (except a time delay on start for pressure to buildup). C. is correct as described above. D. is incorrect since this is never bypassed. Date Written: 3/8/2006 Author: M. Jorgensen App. Ref:

Quest No: RO SRO: TIER: 47 Both 2 System/Evolution Name: Auxiliary / Emergency Feedwater (AFW) System KA Statement: AFW recirculation UserID: Question Stem:

GROUP: Topic No: KA No: RO: SRO: Cog Level: 061000 K4.08 2.7 2.9 Low Category Statement: Knowledge of AFW System design feature(s) and/or interlock(s) which provide for the following:

Topic

Which of the following describes how Auxiliary Feedwater Pumps are protected from damage when running with feed flow to the SGs isolated?

A B

Recirculation is controlled at ~100 gpm back to the suction source by an air operated control valve that senses downstream flowrate. Recirculation is limited to ~100 gpm back to the suction source by an in-line orifice and is isolated to the CST with an air operated isolation valve that fails open on loss of air pressure.

At least 85 gpm recirculation flow is maintained by a manual throttle valve with a motor operated isolation valve that automatically closes when the SX suction valves open. A maximum recirculation flow of 85 gpm is controlled by an in-line orifice with an air operated isolation valve to the CST that automatically closes when the SX recirculation valve opens.
Question Source: S.AF1-14 New Medium Question Difficulty

Answer: Task No: B Obj No:

Time: Cross Ref: 1 I1-AF-XL-01, Ch 26, Auxiliary Feedwater System

Reference:

A is incorrect because flow is not regulated by a control valve. Explanation: B. is correct because this is how the sytem is designed. C. is incorrect because 100 gpm is the design to ensure at least 85 gpm is met and an orifice is used with an AOV for isolation. D. is incorrect because 100 gpm is the design to ensure at least 85 gpm is met and the AOV closes if BOTH respective SX valves are NOT fully closed. Date Written: 3/8/2006 Author: M. Jorgensen App. Ref:

Quest No: RO SRO: TIER: 48 Both 2 System/Evolution Name: A.C. Electrical Distribution System

GROUP: Topic No: KA No: RO: 1 062000 K2.01 3.3 Category Statement: Knowledge of bus power supplies to the following:

SRO: 3.4

Cog Level: Low

KA Statement: Major system loads UserID: Question Stem: Topic

A reactor trip has just occurred on Unit 1. The Automatic Bus Transfer (ABT) failed to operate for Bus 157. Which of the following loads is UNAVAILABLE?

A B C D

1A Main Feed pump

1B Reactor Coolant pump

1C Heater Drain pump

1D Condensate/Condensate Booster pump


Question Source: S.AP1-12-A Byron Cert exam bank (2001) Low Question Difficulty

Answer: Task No: C Obj No:

Time: Cross Ref: 1 I1-AP-XL-01, Ch 4, AC Electrical Power Systems, Pg 34

Reference:

Bus 157 is a 6.9KV bus that provides breakers to 1A RCP, 1A and 1C HD pumps, and the RSH transformers. Explanation: A. is incorrect because it's power source is 6.9KV bus 156. B. is incorrect because it's power source is 6.9KV bus 156. C is correct D is incorrect because it's power by 6.9KV bus 158 Date Written:
Quest No: RO SRO: 49 Both

3/8/2006 Author: M. Jorgensen


TIER: 2 GROUP: Topic No: 1 063000

App. Ref:
KA No: K1.02 RO: 2.7 SRO: 3.2 Cog Level: High

System/Evolution Name: D.C. Electrical Distribution System

Category Statement: Knowledge of the physical connections and/or cause-effect relationships between the D.C. Electrical System and the following systems:

KA Statement: AC electrical system UserID: Question Stem: Topic

Unit 2 was at 100% power in a normal at power lineup. - Power is lost to DC Bus 212 - 5 minutes later, SAT 242-1 develops a fault What is the status of ESF Bus 242 immediately following the SAT fault?

A B C D

Energized from 2B DG.

Energized from SAT 242-2.

Deenergized with all feed breakers tripped.

Deenergized with ACB 2422 closed.


Question Source: Byron NRC exam bank (2000) Medium Question Difficulty

Answer: Task No: D Obj No: S.DC1-09

Ti e m: Cr sR : os ef 1 I1-DC-XL-01, Ch 8a, 125 VDC Power Systems, pgs 19-20

Reference:

A. B. C. D.

is incorrect since 2B DG will not start due to no DC power. Explanation: is incorrect since SAT 242-2 will also be lost from the tripping of th SAT's feed breaker from the switchyard. is incorrect since all feed breakers will still be closed without DC to open them. is correct since there will be no power source to the bus and 2422 will not have opened since DC was lost.

Date Written: 3/8/2006 Author: M. Jorgensen App. Ref: Quest No: RO SRO: TIER: GROUP: Topic No: KA No: 50 Both 2 1 064000 2.1.2 System/Evolution Name: Category Statement: Emergency Diesel Generator (ED/G) System Conduct of Operations

RO: 3.0

SRO: 4.0

Cog Level: Low

KA Statement: Knowledge of operator responsibilities during all modes of plant operation. UserID: Question Stem: Topic

The crew is performing 1BEP ES-1.2, Post LOCA Cooldown and Depressurization. - ESF busses are supplied by the diesel generators (DG). - The DGs have been continuously loaded to 6000 KW at 1050 amps for 1 hour. By design, how much longer can the DGs remain running at this present load?

A B C D

DGs must be secured immediately.

1 hour.

1999 hours.

Indefinitely.
Question Source: S.DG1-01 Byron NRC exam bank (2000) Medium Question Difficulty

Answer: Task No: B Obj No:

Ti e m: Cr sR : os ef 1 I1-DG-XL-01, Ch 9, Diesel Generator and Auxiliary System, Pg 32

Reference:

The DGs are rated for 5500 Kw @954 amps continuous; 5500-5935 Kw @1030 amps for 2000 hours; 5935-6050 Kw @1050 amps Explanation: for 2 hours. A. is incorrect since this load is allowed for 2 hours. B. is correct as stated above. C. is incorrect since this load is allowed for 2 hours. D. is incorrect since this load is allowed for 2 hours. Date Written: 3/8/2006 Author: M. Jorgensen App. Ref: Quest No: RO SRO: TIER: GROUP: Topic No: KA No: RO: SRO: Cog Level: 51 Both 2 1 064000 K4.10 3.5 4.0 Low System/Evolution Name: Category Statement: Emergency Diesel Generator (ED/G) System Knowledge of ED/G System design feature(s) and/or interlock(s) which provide for the following:

KA Statement: Automatic load sequencer: blackout UserID: Question Stem: Topic

Given the following conditions on Unit 2: - The reactor is at 100% power. - A grid problem has just LOWERED voltage to 3700 volts on Buses 241 and 242. With no operator action and voltage sustained at 3700 volts, what is the SEQUENCE of events for this condition?

A B C D

2A and 2B DGs start after ~310 seconds, then both ESF buses deenergize.

2A and 2B DGs start immediately, then both ESF buses deenergize.

Both ESF buses deenergize after ~310 seconds, then 2A and 2B DGs start.

Both ESF buses deenergize immediately, then 2A and 2B DGs start.


Question Source: S.AP1-10-A Byron Cert exam bank (2001) Medium Question Difficulty

Answer: Task No: C Obj No:

Ti e m: Cr sR : os ef 1 I1-AP-XL-01, Ch 4, AC Electrical Power System, Pg 40

Reference:

With voltage degraded to < 3847.5 volts but > 2870 volts, a 310 second time delay is actuated and must time out to cause the feeder Explanation: breakers for the ESF buses to trip. Once tripped, the DGs see an UV and receive a start signal. A. is incorrect because the sequence is not correct. B. is incorrect because both the time frame and sequence are not correct. C. is correct D. is incorrect because the timer must time out first. Date Written: 3/8/2006 Author: M. Jorgensen App. Ref:

Quest No: RO SRO: TIER: GROUP: Topic No: KA No: RO: SRO: Cog Level: 52 Both 2 1 073000 A2.02 2.7 3.2 High System/Evolution Name: Category Statement: Process Radiation Monitoring (PRM) System Ability to (a) predict the impacts of the following malfunctions or operations on the PRM System and (b) based on those predictions, use procedures to correct, control, or mitigate the consequences of those malfunctions or operations: KA Statement: Detector failure UserID: Question Stem: Topic

A liquid release is in progress from Release Tank 0WX01T. - RM-11 alarm is received and acknowledged to be 0PR01J, Liquid Radwaste Effluent Rad Monitor, with Color Code: BLUE. What is the impact of this alarm and appropriate action(s), if any?

Detector RM-80 micro-processor has lost communication with the RM-11; VERIFY with RW operator that release flow rate has NOT changed; NOTIFY Chemistry to sample now and each 4 hours until communications is reestablished. Detector may have failed; VERIFY with RW operator that release flow rate has NOT raised; NOTIFY Chemistry to recalculate and verify release rate and continue to sample each 4 hours until the detector is restored. OPERATE FAILURE is indicated; VERIFY with RW operator that the release has TERMINATED; NOTIFY Chemistry to calculate new release limits; initiate LCOAR for 0PR01J and reestablish conditions for a new release. OPERATE FAILURE is indicated; VERIFY 0PR10J, Station Blowdown monitor, is in service and below the ALERT limit; If YES, the release may continue; initiate LCOAR for 0PR01J.
Question Source: S.AR1-04-B-01/09/18, New Medium Question Difficulty

Answer: Task No: C Obj No:

Time: Cross Ref: 1 I1-AR-XL-01, Ch 49, Radiation Monitors Reference: I1-WX-XL-01, Ch 48, Liquid Radwaste RM-11 BAR for 0PR01J BCP-400-TWX01, Liquid Radwaste Release Form For Realease Tank 0WX01T

The BLUE Code on the RM-11 means an OPERATE FAILURE has occurred, which could be several things from detector failure to Explanation: loss of sample flow, but the exact cause will not be readily apparent at the RM-11. Since this is the case, the output will be generated to any auto actuation components as if the monitor is in a HIGH alarm condition. For 0PR01J, this will close the liquid release isolation valve, stopping the release. This is the BAR action for this condition. This detector INOPERABLE requires T ech Spec/TRM action, thus LOCAR initiation. Release may be reestablished and resumed without this monitor using alternate procedure steps. A. is incorrect because this Color Code is Magenta and does not constitute action for release effects. 0PR01J is still operating and OPERABLE. B. is incorrect, this reason is plausible, but isolation of the release path is expected to occur. C. is correct per the RM-11 BAR D. is incorrect, this reason is plausible, but isolation of the release path is expected to occur. Date Written: 3/8/2006 Author: M. Jorgensen App. Ref:

Quest No: RO SRO: TIER: 53 Both 2 System/Evolution Name: Service Water System (SWS)

GROUP: Topic No: KA No: RO: SRO: 1 076000 A3.02 3.7 3.7 Category Statement: Ability to monitor automatic operation of the SWS, including:

Cog Level: High

KA Statement: Emergency heat loads UserID: Question Stem: Topic

Unit 1 was being synchronized to the grid when a steamline break occurred in containment - 2 minutes later, a switchyard fault caused both Unit 1 SATs to deenergize. - Containment pressure peaked at 16.5 psig. When would the 1A SX pump re-start?

A B C D

5 seconds after the 1A CS pump.

Between the start of 1A CV pump and 1A RH pump.

10 seconds before 1A AF pump.

Coincident with the start of 1A and 1C RCFCs.


Question Source: S.DG1-07-C, S.SX1Byron NRC exam bank (1998) - modified Medium Question Difficulty

Answer: Task No: C Obj No:

Time: Cross Ref: 1 I1-DG-XL-01, Ch 9, Diesel Generators and Auxiliaries, pgs 36-38 Reference: I1-SX-XL-01, Ch 20, Essential Service Water System, pgs 48-49

1A SX pump will always sequence on the DG at 25 seconds. The 1A CS pump sequences at 15-18 seconds or after 40 seconds if an Explanation: auto actuation is present (at or above 20 psig in containment). The SI pump starts between the CV and RH pump. The RCFC start at time 0 in the sequence. Regardless of whether an SI occurred or just loss of offsite power the SX pump always starts 10 seconds before the 1A AF pump, which will auto sequence at the same time for either event. A. is incorrect because the 1A CC pump would be 5 seconds after 1A CS may have received it's 1st start permissive. B. is incorrect since the 1A SI pump would sequence at 10 seconds which is between these two pumps. C. is correct since the 1A AF pump would start at 35 seconds and the 1A SX pump always sequences at 25 seconds. D. is incorrect because the RCFCs will start at time 0 in this sequence being supplied by 480 VAC MCCs that are powered as soon as the DG output breaker closes.

Date Written:

3/8/2006 Author: M. Jorgensen

App. Ref:

Quest No: RO SRO: TIER: 54 Both 2 System/Evolution Name: Instrument Air System (IAS)

GROUP: Topic No: KA No: RO: SRO: Cog Level: 1 078000 K3.02 3.4 3.6 High Category Statement: Knowledge of the effect that a loss or malfunction of the IAS will have on the following:

KA Statement: Systems having pneumatic valves and controls UserID: Question Stem: Topic

Unit 2 is at 100% power with all systems in normal lineup, when Instrument air is lost to the containment.

Complete the following statement. With no operator action for the next 30 minutes, Pressurizer (PZR) pressure will RISE __ .

A B C D

until the PZR PORVs will cycle to control pressure.

initially, but will control at 2235 psig due to PZR Sprays opening.

causing backup heaters to deenergize with the variable heaters remaining full on.

continuously to the PZR pressure Reactor trip setpoint due to loss of letdown and the rise in charging flow.
Answer: Task No: A Obj No: S.RY1-05/25A/B, Question Source: Byron Cert exam bank (2001) Medium Question Difficulty

Time: Cross Ref: 1 I1-RY-XL-01, Ch 14, Pressurizer, Pgs 33 Reference: I1-CV-XL-01, Ch 15a, CVCS, Pgs 47 1/2BOA SEC-4, Loss of IA, steps 4, 5 and Table A

This is an integrated plant question. CV letdown and normal charging fail closed. The seal injection flow is maintained resulting in a Explanation: net rise in RCS inventory. This causes PZR level to rise, thus PZR pressure will rise as the bubble is compressed. The loss of air has caused the normal PZR spray valves to fail closed, Aux spray is isolated, thus pressure will rise to the PORV's lift setpoint and lift because the PORVs have a reservoir (accumulator) that maintains operation of the PORVs with no IA available. A. is correct as described above. B. is incorrect since the normal spray valves require IA to open. C. is incorrect since the insurge will cause B/U heaters to energize, the variable heaters will be off with the higher pressure. D. is incorrect since PORV's will still function to keep pressure below the reactor trip setpoint. Date Written: 3/8/2006 Author: M.Jorgensen App. Ref:

Quest No: RO SRO: 55 Both System/Evolution Name: Containment System

TIER: 2

GROUP: Topic No: KA No: RO: SRO: Cog Level: 103000 A2.03 3.5 3.8 High Category Statement: Ability to (a) predict the impacts of the following malfunctions or operations on the Containment System and (b) based on those predictions, use procedures to correct, control, or mitigate the consequences of those malfunctions or operations:

KA Statement: Phase A and B isolation UserID: Question Stem: Topic

A LOCA has occurred on Unit 1. - Immediate actions in 1BEP-0, Reactor Trip or Safety Injection, are complete - Containment pressure is 24 psig The following Group Monitor lights are NOT LIT: - 1CV8152, Letdown Line Containment Isolation Valve - 1CC685, CC From RCPs Thermal Barrier Isolation Valve - 1CS019B, Eductor 1B Spray Add Valve What are the FIRST actions required in 1BEP-0, Reactor Trip or Safety Injection, for this indication?

A B

Manually actuate Phase A, CS & Phase B.

VERIFY position of each valve; Manually position each valve using the control switch as required. Manually actuate SI, CS & Phase B.

C D

VERIFY associated opposite Train valve CLOSED, then continue in 1BEP-0.


Question Source: New Medium Question Difficulty

Answer: Task No: A Obj No: T.EP01-05

Time: Cross Ref: 1 1BEP-0, Reactor Trip or SI, Steps 8 and 14 Reference: I1-EP-XL-01, 1BEP-0, Reactor Trip or SI, pgs 12, 14

BEP-0 RNO action for each of these group lights NOT LIT specifically states to Manually Actuate Phase A for 1CV8152, and Explanation: Manually Actuate CS&Phase B (2 of 2 switches) for 1CC685 and/or 1CS019B. A. is correct as stated in the procedure. B. is incorrect since this is followup action if the Manual actuations were not successful. C. is incorrect since SI has already been manually actuated by procedure as a backup to auto actuation early in the event. D. is incorrect since this is also a followup action after Manual actuation has been attempted. Date Written: 3/9/2006 Author: M. Jorgensen App. Ref:

Quest No: RO SRO: 56 Both System/Evolution Name: Control Rod Drive System

TIER: 2

GROUP: Topic No: KA No: RO: 2 001000 K2.01 3.5 Category Statement: Knowledge of bus power supplies to the following:

SRO: 3.6

Cog Level: High

KA Statement: One-line diagram of power supply to M/G sets UserID: Question Stem: Topic

Unit 1 is operating at 50% power in a normal at power lineup. Which of the following would cause Alarm 1-10-D8, ROD DRIVE M/G SET TROUBLE, to alarm?

A B C D

Loss of 4KV Bus 141

Loss of 4KV Bus 142

Loss of 4KV Bus 143

Loss of 120 VAC Bus 113


Question Source: S.RD1-11-A New Medium Question Difficulty

Answer: Task No: C Obj No:

Time: Cross Ref: 1 BAR 1-10-D8, Rod Drive M/G Set Trouble Reference: I1RD-XL-01, Ch 28, Rod Control System, Pgs 20, 53

The power supplies to the MG sets are 480 VAC MCCs 133Y and 134Y, which are fed by 4KV buses 143 and 144. If power is lost to Explanation: a running MG set with the other MG set running, the breakers for the deenergized MG set will trip on reverse power, which is one of the inputs to the alarm. Other alarm inputs are associated with other running breaker faults (i.e. OC, OV, Grnd). A. is incorrect since this is an ESF power supply that could be x-tied to supply bus 143, but that is NOT a normal lineup. B. is incorrect since this is an ESF power supply that could be x-tied to supply bus 144, but that is NOT a normal lineup. C. is correct since this would cause 1A RD MG set to lose power (i.e. 133Y will deenergize) causing a reverse power trip from the other energized MG set. This will cause the alarm. D. is incorrect since there is NO interface or impact to the MG sets from a loss of this bus. Date Written: 3/9/2006 Author: M. Jorgensen App. Ref:

Quest No: RO SRO: TIER: 57 Both 2 System/Evolution Name: Reactor Coolant System (RCS)

GROUP: Topic No: KA No: 2 002000 2.1.32 Category Statement: Conduct of Operations

RO: 3.4

SRO: 3.8

Cog Level: Low

KA Statement: Ability to explain and apply all system limits and precautions. UserID: Question Stem: Topic

Unit 1 is in MODE 4 during a plant heatup with the following conditions: - RCS temperature = 300F - RCS pressure = 400 psig - PZR level = 33% - Preparations are in progress to start the FIRST RCP. Complete the following statement of applicability. Per BOP RC-1, Startup of a Reactor Coolant Pump, the requirement of having < 50F difference between S/G temperature and the associated RCS loop temperature ________

A B C D

is NOT applicable since this is the first RCP to be started.

ensures RCP seal parameters remain within normal operating range.

prevents an overpressure event in the RCS.

provides adequate NPSH at the suction of the selected RCP.


Question Source: S.RC2-09-C Byron NRC exam bank (1998) Low Question Difficulty

Answer: Task No: C Obj No:

Time: Cross Ref: 1 I1-RC-XL-02, Ch 13, Reactor Coolant Pump, Pg 31 Tech Spec 3.4.6 and 3.4.7

Reference:

This is the Tech Spec requirement, thus preventing a challenge to LTOP, due to the rapid distribution of the higher energy fluid when Explanation: an RCP is initially started. The limit is the analyzed range boundary. A. is incorrect since this is a Tech Spec limit and it always applies. B. is incorrect since the seal parameters are not impacted by this requirement. C. is correct as part of the LTOP analysis to prevent overpressurization of the RCS on an RCP start. D. is incorrect since this is established strictly by the P/T limits in th RCS alone. Date Written: 3/9/2006 Author: M. Jorgensen App. Ref:

Quest No: RO SRO: TIER: GROUP: Topic No: KA No: RO: SRO: Cog Level: 58 Both 2 2 011000 K3.03 3.2 3.7 High System/Evolution Name: Category Statement: Pressurizer Level Control System (PZR LCS) Knowledge of the effect that a loss or malfunction of the PZR LCS will have on the following: KA Statement: PZR PCS UserID: Question Stem: Topic

Unit 1 is at 50% power with all systems in Auto, except Rod Control which is in Manual. - Letdown flow is 75 gpm. - RCS Loop 1C Tave channel fails HIGH. What is the response of PZR Pressure Control AFTER 5 minutes with no operator action?

A B C D

One PORV is cycling, Sprays FULL OPEN.

Spray valves CLOSED, Backup heaters ON, Variable heaters full ON.

Spray valves Throttled OPEN, Backup heaters OFF, Variable heaters OFF.

Spray valves Throttled OPEN, Backup heaters ON, Variable heaters OFF.
Question Source: S.RY1-21I New Medium Question Difficulty

Answer: Task No: C Obj No:

Ti e m: Cr sR : os ef 1 I1-RY-XL-01, Ch 14, Pressurizer, Pgs 25, 26, 28

Reference:

This failure inputs to PZR level Control Program level setpoint now calling for 100% power PZR level. Charging flow rate will Explanation: immediately rise, raising PZR level. As PZR level rises, pressure rises causing variable heaters to go to minimum (OFF) and sprays to open to maintain pressure control. Sprays have more than enough capacity to prevent reaching the PORV lift setpoint on this transient. Backup heaters should already be OFF and remain OFF. They will reenergize on a > 5% level deviation above program normally expected on an insurge, but in this case the deviation is the other direction and they won't reenergize. A is incorrect because the sprays will prevent reaching the PORV setpoint. B. is incorrect because this response would be to a drop in PZR level. C. is correct as described above. D. is incorrect because sprays will be on, variable heaters will be off, but backup heaters will be off also.

Date Written: Quest No: RO SRO: 59 Both

3/9/2006 Author: M. Jorgensen TIER: GROUP: Topic No: 2 2 014000

App. Ref: KA No: K4.03

RO: 3.2

SRO: 3.4

Cog Level: Low

System/Evolution Name: Rod Position Indication System (RPIS)

Category Statement: Knowledge of RPIS design feature(s) and/or interlock(s) which provide for the following:

KA Statement: Rod bottom lights UserID: Question Stem: Topic

Which of the following will cause the Rod Bottom LED to FLASH?

A B C D

DRPI General Warning

DRPI Data A OR B failure

An Ejected rod

A Dropped rod from 9 steps during withdrawal


Question Source: S.PI1-04/06 New Medium Question Difficulty

Answer: Task No: C Obj No:

Ti e m: Cr sR : os ef 1 I1-PI-XL-01, Ch 29, Pgs 9-11, 18Reference:

Several failures will cause the Rod Bottom lights to flash including DRPI Data A AND Data B failures coincident with one another, Explanation: Data A and B differ by more than 1 bit, sum of Data A and B exceeds 38 bits, and an ejected rod. A. is incorrect since this is caused by Data A or Data B failure, but not both at the same time or an urgent alarm is present for one particular rod or rods. B. is incorrect since this brings in a DRPI General Warning. C. is correct as described above. D. is incorrect since this would bring in the light solid. Date Written: 3/9/2006 Author: M. Jorgensen App. Ref: Quest No: RO SRO: TIER: GROUP: Topic No: KA No: RO: SRO: Cog Level: 60 Both 2 2 016000 K1.12 3.5 3.5 High System/Evolution Name: Category Statement: Non-Nuclear Instrumentation System (NNIS) Knowledge of the physical connections and/or cause-effect relationships between

the NNIS and the following systems: KA Statement: S/G UserID: Question Stem: Topic

Unit 2 is at 35% power with all systems in normal lineup. What failure will cause an INITIAL DROP in feedwater flow to ALL SGs?

A B C D

PT-505, Turbine First Stage Impulse Pressure, fails LOW.

PT-506, Turbine First Stage Impulse Pressure, fails LOW

PT-507, Main Steamline Pressure, fails LOW.

PT-508, Main Feedwater Header Pressure, fails LOW.


Question Source: Byron NRC exam bank (1998) Medium Question Difficulty

Answer: Task No: C Obj No: S.FW2-16

Ti e m: Cr sR : os ef 1 I1-FW-XL-01, Ch 27, SG Water Level Control System, Pg 20

Reference:

PT-506 is input to FWP turbine speed control for maintaining delta-P program across the FRVs. A low failure would denote a large Explanation: delta-P and the FWP speed will slow down to lower the delta-P, thus reducing FW flow to the SGs. The same effect would occur if PT-508 failed high. A. is incorrect since this PT does not input to the SGWLC program. B. is incorrect since this PT does not input to the SGWLC program. C. is correct as described above. D. is incorrect since this input will cause the SG's level to go up, a failure high would produce the same response as B. Date Written: 3/9/2006 Author: M. Jorgensen App. Ref:

Quest No: RO SRO: TIER: GROUP: Topic No: KA No: RO: SRO: Cog Level: 61 Both 2 2 017000 K5.01 3.1 3.9 Low System/Evolution Name: Category Statement: In-Core Temperature Monitor (ITM) System Knowledge of the operational implications of the following concepts as they apply to the ITM System: KA Statement: Temperature at which cladding and fuel melt UserID: Question Stem: Topic

Implementation of 2BFR C.1 is safety significant, when CETCs are > 1200F, because additional operator action is required to _________ .

A B C D

Prevent core uncovery.

Provide core cooling to stop the hydrogen generation from the zircaloy-water reaction.

Limit containment pressure to less than the design pressure.

Provide core cooling to prevent exceeding peak clad temperature limit.


Answer: Task No: D Obj No: S.FR02-01 Question Source: Byron exam bank Medium Question Difficulty

Time: Cross Ref: 1 I1-FR-XL-02, BFR-C.1, C.2, C.3, Pg 2, 3 Reference:

This is the express purpose of the BFR C-series, protect the first barrier (clad). Explanation: A. is incorrect because the fact is the core uncovery has already occurred at this temperature. B. is incorrect because this is a by-product of clad degradation and a process of embrittlement. Clad breach is the concern. C. is incorrect because this barrier is addressed in BFR Z-series. D.is correct as described above. Date Written: 3/9/2006 Author: M. Jorgensen App. Ref:

Quest No: RO SRO: TIER: 62 Both 2 System/Evolution Name: Containment Purge System (CPS)

GROUP: Topic No: KA No: RO: SRO: Cog Level: 029000 A3.01 3.8 4.0 Low Category Statement: Ability to monitor automatic operation of the Containment Purge System, including:

KA Statement: CPS isolation UserID: Question Stem: Topic

Which Containment Radiation Monitor provides a signal to automatically actuate a Containment Vent Isolation?

A B C D

AR011, Containment Fuel Handling Incident monitor.

PR011, Containment Atmosphere monitor.

AR014, Containment General Area monitor.

PR001, Containment Purge Effluent monitor.


Question Source: S.AR1-04-A-02 Byron Cert exam bank (2001) Medium Question Difficulty

Answer: Task No: A Obj No:

Ti e m: Cr sR : os ef 1 I1-AR-XL-01, Ch 49, Radiation Monitors, Pg 30

Reference:

AR011 and AR012 respectively isolate Train A and Train B Primary (Not used term at Byron) CNMT (Vent at Byron) Purge (VQ) Explanation: valves. A. is correct as described above. B. is incorrect because this monitor in alarm isolates the CNMT Air Sampling Panel and monitor. C. is incorrect- this monitor provides indication and alarm only. D. is incorrect- this monitor provides indication and alarm only. Date Written: 3/9/2006 Author: M. Jorgensen App. Ref:

Quest No: RO SRO: TIER: GROUP: Topic No: KA No: RO: SRO: Cog Level: 63 Both 2 2 034000 K6.02 2.6 3.3 Low System/Evolution Name: Category Statement: Fuel Handling Equipment System (FHES) Knowledge of the effect of a loss or malfunction of the following will have on the Fuel Handling System: KA Statement: Radiation monitoring systems UserID: Question Stem: Topic

While using the Spent Fuel Pool Crane to move new fuel into the Spent Fuel Pool, radiation monitor 0RE- AR039, Fuel Handling Building Crane Monitor, alarms HIGH. What ACTION for the Fuel Handling Building Crane is affected?

A B C D

Traverse of the bridge and trolley.

Both lowering and raising the hoist.

Both Traverse of the trolley and raising the hoist.

Raising the hoist only.


Question Source: S.AR1-04-A-03 Byron NRC exam bank (1998) Medium Question Difficulty

Answer: Task No: D Obj No:

Time: Cross Ref: 1 I1-AR-XL-01, Ch 49, Radiation Monitors, Pg 27

Reference:

Raising the hoist is the only movement that is inhibited. Explanation: A. is incorrect since this movement is not inhibited. B. is incorrect since lowering the hoist is not inhibited. C. is incorrect since trolley movement is not inhibited. D. is correct as stated above. Date Written: 3/9/2006 Author: M. Jorgensen App. Ref:

Quest No: RO SRO: TIER: 64 Both 2 System/Evolution Name: Steam Dump System (SDS) and Turbine Bypass Control KA Statement: Steam pressure UserID: Question Stem:

GROUP: Topic No: KA No: RO: SRO: Cog Level: 041000 A1.02 3.1 3.2 High Category Statement: Ability to predict and/or monitor changes in parameters (to prevent exceeding design limits) associated with operating the SDS controls including:

Topic

During a Unit 2 plant cooldown, the following conditions exist: - RCS loop Tave: 550F Loop 1 is lowering 548F Loop 2 is lowering 551F Loop 3 is lowering 548F Loop 4 is lowering

- Steam header pressure is 1030 psig and lowering. - Steam Dump Mode Selector switch is in STM PRESS MODE. - Steam Dump Controller is in MAN, set at 30% demand. The operator momentarily places the Train A and Train B Steam Dump Bypass Interlock switches to BYPASS and then releases them. What is the Steam Dump valve status following this action?

A B C D

All valves are fully CLOSED.

Three valves in groups 1, 2, and 3 will OPEN.(9 total)

Three valves in group 1 only will OPEN.

One valve in groups 1, 2, and 3 will fully OPEN.(3 total)


Question Source: S.DU1-04-C/07-B Byron NRC exam bank (2000) Medium Question Difficulty

Answer: Task No: C Obj No:

Ti e m: Cr sR : os ef 1 I1-DU-XL-01, Ch 24, Steam Dumps, Pg 12 Reference:

If temperature on 2/4 Tave channels is below 550F, all valves close and when BYPASS INTERLOCK is selected on both trains, the Explanation: steam dumps will reopen on demand, however, only group 1 valves can open below 550F, without jumpering the control circuit. A is incorrect since this action will allow group 1 valves to open with a demand on the controller. B. is incorrect since groups 2 and 3 valves are blocked from opening below 550F. C. is correct as described above. D. is incorrect since 3 total valves is correct, but they are all in group 1; groups 2 and 3 are blocked below 550F. Date Written: 3/10/2006 Author: M. Jorgensen App. Ref:

Quest No: RO SRO: TIER: 65 Both 2 System/Evolution Name: Waste Gas Disposal System (WGDS)

GROUP: Topic No: KA No: RO: SRO: 2 071000 A4.24 2.9 3.4 Category Statement: Ability to manually operate and/or monitor in the control room:

Cog Level: Low

KA Statement: The double verification required before waste gas release UserID: Question Stem: Topic

While preparing to perform BCP 400-TWASTE GAS, Gaseous Effluent Release Form: Waste Gas Decay Tank, you discover that 0PR02J, Gas Decay Tank Effluent, Radiation Monitor is INOPERABLE. It is still desired to perform the Gaseous Release. Which of the following does NOT REQUIRE an independent verification for the release to proceed? A Lifting Leads BOP GW-13, 0PR02J Interlock Function Defeat.

Placing the Gas Decay Tank, that will be released, in Storage alignment per BOP GW-6, Realignment of Gas Decay Tanks. Sampling the Gas Decay Tank that will be released for activity.

C D

Calculating the release rate of the Gas Decay Tank that will be released.
Question Source: New Medium Question Difficulty

Answer: Task No: B Obj No: S.GW1-12

Ti e m: Cr sR : os ef 1 BCP 400-TWASTE GAS, Gaseous Effluent Release Form: Waste Gas Decay Tank. BOP GW-13, 0PR02J Interlock Defeat Function. BOP GW-6, Realignment of Gas Decay Tanks I1-GW-XL-01, Ch 46, Gaseous Radwaste System. Pgs 14, 20, 21 TRM 3.11.b, Radioactive Gaseous Effluent Monitoring Instrumentation

Reference:

BCP 400-TWASTE GAS requires an independent verification (also TRM 3.11.b) of tank activity and release rate. Also the procedure Explanation: requires BOP GW-13 be performed to defeat the interlock function of the gas release islolation valve, 0GW014. This involves lifting leads and always requires an independent verification. The realignment of the Gas Decay tanks is a normal function for operations and does not require an independent verification. A. is incorrect since this involves lifting leadss, which always reqires IV. B. is correct since this is a normal operator evolution and does NOT require IV. C. is incorrect since the procedure and the TRM requires this action for 0PR02J inoperable. D. is incorrect since the procedure and the TRM requires this action for 0PR02J inoperable. Date Written: 3/16/2006 Author: M. Jorgensen App. Ref:

Quest No: RO SRO: 66 Both System/Evolution Name: Generic

TIER: 3

GROUP: Topic No: KA No: 194001 2.1.12 Category Statement: Conduct of Operations

RO: 2.9

SRO: 4.0

Cog Level: High

KA Statement: Ability to apply technical specifications for a system. UserID: Question Stem: Topic

Unit 2 is in MODE 1. Due to a turbine malfunction and a trip of the running containment chiller, the following conditions exist: - Current time = 1000. - RCS Tave = 549F. - PZR pressure = 2202 psig. - Containment pressure = 1.1 psig. - Containment temperature (Ave of running RCFCs) = 122F Which Technical Specification LCO parameter MUST BE RESTORED by 1030 to allow continued operation in MODE 1? (Assume turbine problem has been corrected)

A B C D

RCS Tave.

PZR pressure.

Containment pressure.

Containment temperature.
Question Source: S.RC1-12, S.PC1-08 Byron Cert exam bank (2001) Medium Question Difficulty

Answer: Task No: A Obj No:

Te m i : Cos e: r sR f 1 Tech Specs 3.4.1, 3.4.2, 3.6.4 and 3.6.5 Reference: I1-RC-XL-01, Ch 12, Reactor Coolant System, Pg 41 I1PC-XL-01, Ch 40, Primary Containment, Pgs 25, 26

RCS Tave is required to be at or above 550F when critical per Tech Spec 3.4.2. Explanation: A. is correct as stated above. B. is incorrect since PZR pressure is required to be restored at or above 2209 psig, per Tech Spec 3.4.1, within 2 hours. C. is incorrect since CNMT Pressure is required to be within limits (-.1 to +1.0 psig) within 1 hour per Tech Spec 3.6.4. D is incorrect since CNMT Temperature is required to be at or below 120F within 8 hours per Tech Spec 3.6.5. Date Written: 3/10/2006 Author: M. Jorgensen App. Ref:

Quest No: RO SRO: 67 Both System/Evolution Name: Generic

TIER: 3

GROUP: Topic No: KA No: 194001 2.1.27 Category Statement: Conduct of Operations

RO: 2.8

SRO: 2.9

Cog Level: Low

KA Statement: Knowledge of system purpose and or function. UserID: Question Stem: Topic

In accordance with the Operating Department Standards found in BAP 300-1, Byron Addendum to Conduct of Operations Manual, which of the following would meet the "Operator's judgement" call for taking a controller from AUTO to MANUAL?

A B C D

To establish the control output to desired setpoint faster during a load ramp.

Periodically to check if MANUAL is tracking AUTO.

Periodically to RESET the integral during a power ramp.

Automatic response is NOT consistent with changing plant conditions.


Question Source: New Low Question Difficulty

Answer: Task No: D Obj No: T.AM03-29

Time: Cross Ref: 1 BAP 300-1, OP-AA-100, Conduct of Operations Manual, Byron Addendum Reference: I1-AM-XL-70, BAP 300-1, OP-AA-101-101, Conduct of Operations Maunual, Byron Addendum, Pg

The statement in BAP 300-1, C.1.h. says " the operator may place a controller in the manual mode from the automatic mode Explanation: whenever, in the operator's judgement, continued automatic operation is unsafe or whenever it may cause any unnecessary transients. This should only be done when conditions are "stable and under control", or when it is apparent that continued operation would aggravate or worsen the plant conditions". A. is incorrect since it does not meet the intent of this standard. B. is incorrect since it does not meet the intent of this standard. C. is incorrect since it does not meet the intent of this standard. D. is correct as described above.

Date Written:

3/16/2006 Author: M. Jorgensen

App. Ref:

Quest No: RO SRO: 68 Both System/Evolution Name: Generic

TIER: 3

GROUP: Topic No: KA No: 194001 2.2.22 Category Statement: Equipment Control

RO: 3.4

SRO: 4.1

Cog Level: Low

KA Statement: Knowledge of limiting conditions for operations and safety limits. UserID: Question Stem: Topic

An overpressure event caused the Technical Specification Safety Limit for RCS Pressure to be exceeded in MODE 1 at 10:00, by what time must the Unit be in HOT STANDBY with RCS Pressure within limits?

A B C D

10:05

10:15

10:30

11:00
Question Source: S.RC1-12 Byron Cert exam bank (2001) Medium Question Difficulty

Answer: Task No: D Obj No:

Time: Cross Ref: 1 I1-RC-XL-01, Ch 12, Reactor Coolant System, Pg 35 Reference: Tech Spec 2.1.2

Tech Spec 2.1.2 actions in MODE 1or 2 require reducing pressure to 2735 psig or below and being in MODE 3 within 1 hour. Explanation: A.is incorrect since this is the time required to restore pressure to within limits if the Unit is in MODEs 3, 4, or 5. B. is incorrect because although plauseable, not correct. C. is incorrect because although plauseable, not correct. D.is correct as stated above. Date Written: 3/10/2006 Author: M. Jorgensen App. Ref:

Quest No: RO SRO: 69 Both System/Evolution Name: Generic

TIER: 3

GROUP: Topic No: KA No: 194001 2.2.25 Category Statement: Equipment Control

RO: 2.5

SRO: 3.7

Cog Level: Low

KA Statement: Knowledge of bases in technical specifications for limiting conditions for operations and safety limits. UserID: Question Stem: Topic

Limits on RCS activity provided in Technical Specifications are based on the dose that would be received at the site boundary in a SGTR accident that begins with a steady-state primary-tosecondary leakage of 1 gpm. Maintaining those limits ensures that the 2-hour dose at the site boundary during a SGTR will NOT exceed _______________.

A B C D

10 CFR 20, Standards for Protection Against Radiation, limits.

10 CFR 100, Reactor Site Criteria, limits.

EPA Protective Action Guideline thresholds.

5 Rem TEDE for the general public.


Question Source: S.RC1-14, S.BZ1-03 Byron NRC exam bank (2000) Medium Question Difficulty

Answer: Task No: B Obj No:

Te m i : Cos e: r sR f 1 I1-BZ-XL-01, Ch 2, Site and Buildings, Pgs 5, 6. Reference: I1-RC-XL-01, Ch 12, Reactor Coolant System, Pg 51. Tech Spec 3.4.16, RCS Activity, and 3.4.13, RCS Operational Leakage, bases I1-SG-XL-01, Ch 22, Steam Generators, Pg 19 This is the bases statement for the RCS Activity limit in Tech Spec 3.4.16 and the RCS Operational Leakage Limit bases in Tech Explanation: Spec 3.4.13. It is also taught as the document used to establish site boundary guidelines in Site and Building lesson plan. A. is incorrect, but plausible since most controlled release limits are found in this document. B. is correct as stated above. C. is incorrect, but plausible since evacuation actions are based on this guidline. D.is incorrect, but plausible since this is the federal limit for rad workers. Date Written: 3/10/2006 Author: M. Jorgensen App. Ref:

Quest No: RO SRO: 70 Both System/Evolution Name: Generic

TIER: 3

GROUP: Topic No: KA No: 194001 2.2.34 Category Statement: Equipment Control

RO: 2.8

SRO: 3.2

Cog Level: High

KA Statement: Knowledge of the process for determining the internal and external effects on core reactivity. UserID: Question Stem: Topic

A Unit 2 reactor startup is in progress following a refueling outage. - Moderator Temperature Coefficient (MTC) is slightly positive at +.5 pcm/F - With critical data just obtained, rods are withdrawn to establish a +.1 DPM steady-state startup rate. With no additional operator action, reactor power will RISE until which of the following occurs?

A B C D

The RCS heatup is sufficient to negate the reactivity added by the rods.

Fuel temperature rises sufficiently to negate the reactivity added by the rods.

Automatic steam dump response lowers RCS temperature.

An automatic reactor trip occurs.


Question Source: A.RT3-08/15 Byron Cert exam bank (2001) Medium Question Difficulty

Answer: Task No: B Obj No:

Ti e m: Cr sR : os ef 1 I1-RT-XL-03, Ch 3, Reactor Theory, MTC and Total Power Defect, Pgs 9, 19, 33.

Reference:

MTC is + which means the point of adding heat will then add additional + reactivity as the coolant temperature rises. However, FTC Explanation: is always negative and fuel temperature rise must occur to raise RCS temperature. The magnitude of negative reactivity added by the fuel temperature rise will rapidly offset the positive reactivity added by the moderator temperature rise. FTC is ~ 3 times as large in magnitude as MTC. A is incorrect since RCS heatup alone would continue to add + reactivity and power would continue to rise. B. is correct as described above. C. is incorrect since this would raise steam demand, raising power, holding temperature, not cooling down. Actual stabilization from FTC feedback to hold power. D. i s i n c o r r e c t s i n c e p o w e r w i l l t u r n a n d s t a b i l i z e l o w i n t o t h e p o w e r r a n g e . Critical data is taken well above P-6 and stablization will be well below P-10. .

Date Written:

3/10/2006 Author: M. Jorgensen

App. Ref:

Quest No: RO SRO: 71 Both System/Evolution Name: Generic

TIER: 3

GROUP: Topic No: KA No: 194001 2.3.2 Category Statement: Radiological Controls

RO: 2.5

SRO: 2.9

Cog Level: Low

KA Statement: Knowledge of facility ALARA program. UserID: Question Stem: Topic

Given the following conditions on Unit 2: - Operators are in the process of removing an OOS on a valve located in a high radiation area. - Rad Protection estimates that performing an independent verification will cause an individual to receive 26 mrem. What is the requirement for independent verification during this evolution?

A B C

Must be performed unless waived by the Operations Manager.

Operator self-check is substituted whenever an accumulated dose > 20 mrem is involved.

Is NOT required with Shift Manager approval. However, alternate verification techniques shall be considered. The verifier should position themselves, allowing the best view but lowest dose, and observe the positioning and self-check of the first operator.
Question Source: Byron Cert exam bank (2001) Medium Question Difficulty

Answer: Task No: HU-002 C Obj No:

Time: Cross Ref: 1 Conduct of Operations, BAP 300 Reference: HU-AA-101, section 4.3.1.1

The independent verification requirements are given in HU-AA-101 and this issue is specifically addressed and states that in this Explanation: circumstance, the Shift Manager can make a determination for ALARA concerns to NOT perform a hands-on independent verification, but shall consider alternate methods of verification, such as flow, pressure, indicating lights, etc. A. is incorrect since the procedure specifies the Shift Manager. B. is incorrect since this is not left for the operator to decide. C. is correct as described above. D. is incorrect since this does NOT constitute an independent verification. This is close to concurrent verification. Date Written: 3/10/2006 Author: M. Jorgensen App. Ref:

Quest No: RO SRO: TIER: 72 Both 3 System/Evolution Name: Generic

GROUP: Topic No: KA No: 194001 2.3.4 Category Statement: Radiological Controls

RO: 2.5

SRO: 3.1

Cog Level: High

KA Statement: Knowledge of radiation exposure limits and contamination control, including permissible levels in excess of those authorized. UserID: Question Stem: Topic

A reactor operator has a Deep Dose Equivalent (DDE) of 1.3 Rem and a Committed Effective Dose Equivalent (CEDE) of 0.6 Rem to date. With management approval, what is the maximum additional CEDE dose allowed during this year for this reactor operator? (Assume NO change in DDE dose for the remainder of the year)

A B C D

1.1 Rem.

3.1 Rem

3.7 Rem

4.4 Rem
Answer: Task No: B Obj No: 6, 7, 15, 16 Question Source: Prairie Island NRC exam bank (1996) Medium Question Difficulty

Te m i : Cos e: r sR f 1 RP-AA-203, section 4.1.1 Reference: RWT, Exelon Radiation Worker Training, Pgs 5, 1, 12

DDE+CEDE=TEDE and the federal limit is 5 Rem/yr. 1.3 Rem + .6 Rem = 1.9 Rem. The Admin limit is 2 Rem, but with Explanation: management approval dose to the federal limit can be authorized. Therefore, 1.9 Rem + 3.1 Rem = 5 Rem, so 3.1 Rem is correct. A.is incorrect since this would = 3 Rem total, which used to be an old quarterly limit. B. is correct as described above. C. is incorrect, but plausible if it was thought DDE was the only dose that was considered part of TEDE. D.is incorrect, but plausible if it was thought CEDE was the only dose that was considered part of TEDE. Date Written: 3/10/2006 Author: M. Jorgensen App. Ref:

Quest No: RO SRO: 73 Both System/Evolution Name: Generic

TIER: 3

GROUP: Topic No: KA No: 194001 2.4.17 Category Statement: Emergency Procedures/Plan

RO: 3.1

SRO: 3.8

Cog Level: Low

KA Statement: Knowledge of EOP terms and definitions. UserID: Question Stem: Topic

A LOCA has occurred on Unit 2 with the following Containment conditions: TIME (1)-1000 (2)-1005 (3)-1010 (4)-1015 (5)-1020 CNMT Pressure 3.4 psig 4.8 psig 8.2 psig 4.9 psig 2.0 psig CNMT Radiation 5.5 E4 R/hr 1.1 E5 R/hr 6.2 E5 R/hr 5.0 E5 R/hr 4.5 E4 R/hr

Based on the above parameters being addressed at the designated times during this event, When did Containment FIRST go ADVERSE and when can NORMAL values be used?

FIRST GO ADVERSE

NORMAL values can be used

A B C D
Answer: Task No: A

2 1 3 1, 5

4, 5

1, 4, 5
Question Source: New Medium Question Difficulty

Obj No: T.EP01-04

Time: Cross Ref: 1 Procedure use and Adherence Reference: I1-EP-XL-01, Reactor Trip or Safety Injection, Pg 7

Adverse CNMT conditions are defined as pressure above 5 psig or rad levels > 1E5 R/hr. The pressure component portion will not Explanation: permanently impact the instrumentation and normal values may resume when pressure drops below 5 psig. The radiation component may cause permanent damage to the instrumentation and requires an engineering evaluation before normal values can be resumed regardless of how low the rad levels drop after reaching the adverse limit. A. is correct since rad levels are first > 1E5 R/hr and time 1 is the only time that allowed use of normal values. B. is incorrect since rad levels were above the limit at time 2 and 5 would not be allowed since rad levels had exceeded 1E5 R/hr. C. is incorrect since rad levels were above the limit at time 2 and 4 or 5 do not allow normal use after rad levels were > 1E5 R/hr. D. is incorrect since time 1 is the only time normal values could have been used.

Date Written:

3/10/2006 Author: M. Jorgensen

App. Ref: None

Quest No: RO SRO: 74 Both System/Evolution Name: Generic

TIER: 3

GROUP: Topic No: KA No: 194001 2.4.22 Category Statement: Emergency Procedures/Plan

RO: 3.0

SRO: 4.0

Cog Level: Low

KA Statement: Knowledge of the bases for prioritizing safety functions during abnormal/emergency operations. UserID: Question Stem: Topic

What are the priorities of the Byron Status Trees based on?

A
are intact

Ensures symptoms are addressed so as many fission product barriers as possible at all times to insure health and safety of the public.

B
analyzed

Ensures critical safety functions are addressed in the same sequence as the events in the UFSAR and Byron Emergency Procedures.

C
RCS

Ensures the hierarchy is sequenced to protect plant personnel by addressing the boundary, then the fuel boundary, then the Containment boundary.

Ensures events are addressed in a sequence of MOST likely barrier to be lost to LEAST likely barrier to be lost to protect public health and safety.
Answer: Task No: A Obj No: T.FR7-03 Question Source: New Medium Time: Cross Ref: 1 I1-FR-XL-07, Status Trees, Pgs 6, 7 Question Difficulty

Reference:

As stated in the background documents and the Lesson Plan, BSTs are symptom based, not event based and prioritized to ensure as Explanation: many barriers as possible at all times to protect public health and safety. The hierarchy is sequenced to address fuel first, RCS second and containment third. A. is correct as stated above. B. is incorrect because the analysis sequence has nothing to do with the bases and the bases is not event based. C. is incorrect because bases is to protect public first and the barrier sequence is not correct. D. is incorrect since this is not how BSTs are sequenced and they are symptom based, not event based.

Date Written:

3/16/2006 Author: M. Jorgensen

App. Ref:

Quest No: RO SRO: TIER: 75 Both 3 System/Evolution Name: Generic

GROUP: Topic No: KA No: 4 94001 2.4.39 Category Statement: Emergency Procedures/Plan

RO: 3.3

SRO: 3.1

Cog Level: Low

KA Statement: Knowledge of the RO's responsibilities in emergency plan implementation. UserID: Question Stem: Topic

What is the MAXIMUM time that may elapse, after emergency event classification, before State and County agencies must be notified?

A B C D

15 minutes.

30 minutes.

1 hour.

4 hours.
Question Source: Kewaunee NRC exam bank (1996) Medium Question Difficulty

Answer: Task No: A Obj No: G-5-8/9

Time: Cross Ref: 1 Emergency Preparedness, G-5, Classification, Notification, and PARs, Pgs 17, 18 EP-AA-1002, Byron Annex for Radiological Emergency Plan, pg 3-10

Reference:

Per the Byron Annex, and reportability manual, any classification exceeding EALs requires that State and Local authorities be Explanation: notified within 15 minutes of the declaration to activate these outside agencies for event support to protect public health and safety. A is correct as stated above. B. is incorrect since this is not one of the notification times, but seems plausible. C. is incorrect since this is the required time to notify the NRC of the event. D. is incorrect since this would be a somewhat standard NRC notification time for many plant events that don't exceed EALs. Date Written: 3/10/2006 Author: M. Jorgensen App. Ref:

Quest No: RO SRO: TIER: GROUP: Topic No: KA No: 76 SRO 1 1 000008 2.1.32 System/Evolution Name: Category Statement: Pressurizer (PZR) Vapor Space Accident Conduct of Operations (Relief Valve Stuck Open) KA Statement: Ability to explain and apply all system limits and precautions. UserID: Question Stem: Topic

RO: 3.4

SRO: 3.8

Cog Level: High

Unit 2 is at 80% power. - PZR PORV 2RY456 lifted and stuck partially open causing an RCS pressure transient. - Operators were unable to position the PORV manually and closed 2RY8000B, PORV Block valve. - RCS pressure dropped to 2215 psig and is slowly trending to normal. Which of the following is the NEXT required action and explains why the action is taken?

Remove power from 2RY8000B within 1 hour to ensure positive control of the relief path while efforts to restore 2RY456 to OPERABLE status are in progress.

Maintain 2RY8000B CLOSED and ENERGIZED to ensure the relief path is available with manual actions if needed. Place the control switch for 2RY456 in the CLOSED position within 1 hour to preclude automatic opening for an overpressure event at a time that the block valve is CLOSED. Place the unit in MODE 3 within 7 hours since Technical Specifications do not address the operability of a PZR PORV that is STUCK partially open.
Question Source: S.RY1-26/28 New Medium Question Difficulty

Answer: Task No: A Obj No:

Time: Cross Ref: 1 Tech Spec 3.4.11 and Bases, PZR PORVs Reference: I1-RY-XL-01, Ch 14, Pressurizer, Pgs 40, 41 43(b) (2)

Per Tech Spec 3.4.11, if a PORV is inoperable, the PORV Block valve must be closed and deenergized within 1 hour. This ensures Explanation: positive control of the relief path per Tech Spec Bases and affords isolation for repair efforts to restore operability within 72 hours. A. B. C. D. is correct as described above. is incorrect since this action is done if the PORV is still capable of manual operation. is incorrect since this is not the required action and it would have no affect on a stuck valve. is incorrect since this is Tech Spec 3.0.3 action and does not apply. The inoperability is addressed by Tech Spec 3.4.11. 3/17/2006 Author: L. Wehner App. Ref:

Date Written:

Quest No: RO SRO: TIER: 77 SRO 1 System/Evolution Name: Large Break LOCA

GROUP: Topic No: KA No: 1 000011 2.4.4 Category Statement: Emergency Procedures/Plan

RO: 4.0

SRO: 4.3

Cog Level: High

KA Statement: Ability to recognize abnormal indications for system operating parameters which are entry-level conditions for emergency and abnormal operating procedures. UserID: Topic Question Stem:

Given the following conditions on Unit 2: - An SI has occurred. - RCS pressure = 1200 psig and lowering. - PZR level is off-scale low. - CETC's = 535F. - Containment pressure = 9 psig and rising. - Containment radiation monitors are in ALARM. - All SG pressures = 775 psig and lowering slowly. With these conditions, what event is occurring and what procedure would be entered after applicable steps of 2BEP-0, Reactor Trip or Safety Injection Unit 2, have been completed?

A B

A LOCA has occurred on an RCS loop cold leg, enter 2BEP-1, Loss of Reactor or Secondary Coolant Unit 2. ONE SG has a steamline break inside of containment, enter 2BEP-2, Faulted Steam Generator Isolation Unit 2 A PZR PORV has fully opened and it's block valve is open, enter 2BEP-1, Loss of Reactor or Secondary Coolant Unit 2. A feedwater line break to ONE SG and MSIVs failed to close, enter 2BEP-2, Faulted Steam Generator Isolation Unit 2.
Answer: Task No: A Obj No: T.EP01-06-A, Question Source: Byron Cert exam bank (2001) Medium Question Difficulty

Time: Cross Ref: 1 I1-EP-XL-01, 1/2BEP-0, Reactor Trip or Safety Injection, Pg 28 Reference: I1-EP-XL-02, Loss of Reactor or Secondary Coolant, Pg 7 43(b) (5)

The parameter that absolutely determines RCS vs steam/feed line is containment rad monitors. RCS cold leg vs PZR PORV will be Explanation: seen in PZR level (low=loop break, high indicates PORV or Safety open). The determination in 2BEP-0 at steps 27 and 29 will direct entering 2 BEP-1 based on these symptoms. A. is correct based on the diagnostic in step 27 and 29 of 2BEP-0, low RCS press, high cnmt press and radiation, and low PZR level. B. is incorrect since high rad exists in the cnmt and MSIVs would be closed which would identify a faulted SG by seeing a lower pressure in one SG after isloation. C. is incorrect since this would have been looked at in step 24 of 2BEP-0. It would be considered a small break LOCA and with low PZR press and level and a large RCS cooldown would not be indicative of a stuck open PORV. D. is incorrect for similar reasons described in B. above.

Date Written:

3/11/2006 Author: M. Jorgensen

App. Ref:

Quest No: RO SRO: TIER: GROUP: Topic No: KA No: 78 SRO 1 1 000015 2.1.32 System/Evolution Name: Category Statement: Reactor Coolant Pump (RCP) Malfunctions Conduct of Operations

RO: 3.4

SRO: 3.8

Cog Level: High

KA Statement: Ability to explain and apply all system limits and precautions. UserID: Question Stem: Topic

All RCP's were manually tripped at 1425 psig during an emergency procedure implementation. During the recovery, the TSC has directed the shift to start one RCP per BOP RC-1, STARTUP OF A REACTOR COOLANT PUMP. Given the following information: (Assume other plant conditions are satisfactory to support RCP start) A RCP B RCP C RCP D RCP - Seal injection flow 8 gpm 10 gpm 9 gpm 10 gpm - Seal leakoff flow 1.2 gpm .9 gpm .4 gpm 1.7 gpm - #1 Seal D/P 330 psid 325 psid 198 psid 265 psid - Motor Upper Radial Bearing 144F 145F 132F 147F Temp Lower Radial Bearing - Motor 198F 153F 149F 144F Temp Upper Thrust Bearing - Motor 147F 142F 151F 146F Temp Lower Thrust Bearing - Motor 112F 108F 107F 106F Temp Which RCP should the Unit Supervisor direct the NSO to start? (BOP RC-1A1, RCP NO 1 SEAL LEAKOFF NORMAL OPERATING RANGE, is provided)

A B C D

RCP A

RCP B

RCP C

RCP D
Question Source: S.RC2-09-A/B Byron LORT exam bank - Used last 2001 cycle 6-6a quiz. Medium Question Difficulty

Answer: Task No: B Obj No:

Time: Cross Ref: 1 BOP RC-1, Startup of a Reactor Coolant Pump Reference: BOP RC-1A1, RCP NO 1 Seal Leakoff Normal Operating Range I1-RC-XL-02, Ch 13, Reactor Coolant Pump, Pgs 30-32 43(b)(5)

D RCP seal leakoff is > 1.4 gpm, and C RCP #1 Seal D/P is < 200 psid, leaves B or A RCPs. A RCP exceeds the 195F limit for a Explanation: motor bearing. The order of preferrence is D, C, B, A.

Date Written:

3/27/2006 Author: J. Heaton

App. Ref: BOP RC-1A1

Quest No: RO SRO: TIER: GROUP: Topic No: KA No: RO: SRO: Cog Level: 79 SRO 1 1 000025 AA2.07 3.4 3.7 High System/Evolution Name: Category Statement: Loss of Residual Heat Removal System Ability to determine and interpret the following as they apply to the Loss of (RHRS) Residual Heat Removal System: KA Statement: Pump cavitation UserID: Question Stem: Topic

Unit 2 has just completed refueling. - Preparations are in progress to install the reactor vessel head. - 2B RH pump is providing shutdown cooling with ~ 3300 gpm flow rate. - 2A RH pump is running transferring refueling cavity water to the RWST per BOP RH-9, Pumpdown of the Refueling Cavity to the RWST. - 2A RH pump has just been reduced to ~490 gpm, by throttling 2RH618, RH HX 2A Bypass Flow Control Vlv, with cavity level at ~ 403' elevation. Flow and motor amps ocillations are reported on the 2B RH train. What is the NEXT required action the Unit Supervisor will direct?

A B

Stop draining the refueling cavity by closing 2RH618 and TRIP the 2B RH pump, then enter 2BOA PRI-10, Loss of RH Cooling Unit 2. Stop draining the refueling cavity by closing 2RH618 and reduce flow through the 2B RH train in an attempt to stablize flow and amps per BOP RH-9. Trip BOTH 2A and 2B RH pumps and enter 2BOA S/D-2, Shutdown LOCA.

C D

Stop draining the refueling cavity by closing 2RH618 and trip the 2B RH pump, then place the 2A RH train in the shutdown cooling mode using BOP RH-6, Operations of RH System in Shutdown Cooling.
Question Source: New Medium Question Difficulty

Answer: Task No: B Obj No: S.RH1-09-C

Time: Cross Ref: 1 I1-RH-XL-01, Residual Heat Removal System, Pgs 29-32 BOP RH-9, Pumpdown of the Refueling Cavity 43(b) (5)

Reference:

A CAUTION in BOP RH-9, prior to step F.1, states that if the above condition exist, stop draining immediately, reduce RH flow in Explanation: the shutdown cooling train to stablize and initiate BOP RH-8 as necessary to establish adequate level for suction on the RH train in SDC. If the RH train in SDC will not stablize with the reduced flow and RH pump trip is required or the RH pump trips, go to 2BOA PRI-10. A. is incorrect since tripping the 2B RH pump is not directed first. Attempt to stablize flow by reducing flow first. B. is correct as stated in BOP RH-9. C. is incorrect since tripping both pumps is not the immediate action and if you did, 2BOA S/D-2 is not correct. D. is incorrect since this is action that may be directed in 2BOA PRI-10, but not in the current procedures. Date Written: 3/17/2006 Author: L. Wehner App. Ref:

Quest No: RO SRO: TIER: 80 SRO 1 System/Evolution Name: Pressurizer Pressure Control (PZR PCS) Malfunction KA Statement: Tech-Spec limits for RCS pressure UserID: Question Stem:

GROUP: Topic No: KA No: RO: SRO: Cog Level: 000027 AA2.04 3.7 4.3 Low Category Statement: Ability to determine and interpret the following as they apply to the Pressurizer Pressure Control Malfunctions:

Topic

Following a refueling outage, Unit 2 is at 40% power with a power ascension at maximum preconditioning rate in progress. The Master PZR Pressure Controller, 2PK-455A, was discovered failing high. When the operator placed the controller in MANUAL, the following conditions were present: - RCS Tave = 567F. - PZR pressure = 2175 psig. - PZR level = 33%. - CVCS letdown is isolated. - Excess letdown is in service. How will the RCS DNB limits be addressed under these conditions?

A B C D

PZR pressure must be raised to at least 2209 psig within 2 hours.

PZR level must be restored to within 5% of program within the next 2 hours.

NO action is required since PZR pressure limit is NOT challenged at this power level.

NO action is required since RCS temperature limit is NOT exceeded.


Question Source: S.RC1-12 Byron NRC exam bank (1998) Medium Question Difficulty

Answer: Task No: A Obj No:

Time: Cross Ref: 1 Tech Spec 3.4.1. RCS Pressure, Temperature, and Flow DNB Limits. And COLR I1-RC-XL-01.Reactor Coolant System, Pg 41 NF-AP-440, Fuel Preconditioning Limits 43(b)(2)

Reference:

Above 40% power, fuel preconditioning limits apply and the ramp rate is limited to much < 5%/minute, therefore, the PZR pressure Explanation: below 2209 psig Tech Spec action to restore within 2 hours applies. DNB limit for Tave is 593.1F, therefore 567F is well below that limit. PZR level at 33% is well below program level of ~ 41%, but there is no 2 hour requirement to restore within 5% of program. This would be done in response to a PZR level deviation alarm that would be in. Which letdown system is in service has no bearing on the isssue here. A. is correct as explained. B. is incorrect as explained. C. is incorrect since this is applicable when the ramp is < 5%/minute. D. is incorrect for the PZR pressure, even though true if only based on RCS temperature. Date Written: 3/11/2006 Author: M. Jorgensen App. Ref:

Quest No: RO SRO: TIER: GROUP: Topic No: KA No: RO: SRO: Cog Level: 81 SRO 1 1 00WE12 EA2.1 3.2 4.0 High System/Evolution Name: Category Statement: Uncontrolled Depressurization of all Steam Ability to determine and interpret the following as they apply to the Uncontrolled Generators Depressurization of all Steam Generators: KA Statement: Facility conditions and selection of appropriate procedures during abnormal and emergency operations UserID: Question Stem: Topic

Unit 1 was at 100% power when the following events occurred: - ALL SGs are faulted into containment. - While performing steps in 1BCA-2.1, Uncontrolled Depressurization of All Steam Generators, a RED path is noted on the containment critical safety function. - Actions of 1BFR-Z.1, Response to High Containment Pressure, are performed. - Auxiliary Feedwater has been throttled to 45 gpm to each SG. - When directed by 1BFR-Z.1 to return to procedure and step in effect, the following is noted on the CSF status trees: Subcriticality - Green Core Cooling - Green Heat Sink - Red Integrity - Orange Containment - Red Inventory - Yellow It is required to ENTER AND PERFORM STEPS in which of the following procedures NEXT?

A B C D

1BEP-2, Faulted Steam Generator Isolation.

1BFR-H.1, Response to Loss of Secondary Heat Sink.

1BFR-P.1, Response to Imminent Pressurized Thermal Shock.

1BFR-Z.1, Response to High Containment Pressure.


Question Source: T.FR7-03/07, Byron NRC exam bank (2000) - modified Medium Question Difficulty

Answer: Task No: C Obj No:

Time: Cross Ref: 1 I1-FR-XL-07, Status Trees, Pg 9, Reference: I1-FR-XL-03, BFR H.1-H.5, Pg 5 43(b)(5)

Once 45 gpm has been established to the faulted SGs in 1BCA-2.1, the CAUTION (If total feed flow is < 500 gpm due to operator Explanation: action, this procedure should NOT be performed) prior to the first step in 1BFR-H.1 will apply and the SRO will proceed to the next CSF in the hierachy, INTEGRITY, since CONTAINMENT was just addressed and INTEGRITY is orange.

Date Written:

3/11/2006 Author: M. Jorgensen

App. Ref:

Quest No: RO SRO: TIER: 82 SRO 1 System/Evolution Name: Loss of Containment Integrity

GROUP: Topic No: KA No: RO: SRO: Cog Level: 2 000069 AA2.01 3.7 4.3 High Category Statement: Ability to determine and interpret the following as they apply to the Loss of Containment Integrity:

KA Statement: Loss of containment integrity UserID: Question Stem: Topic

A small break LOCA has occurred on Unit 2. - Manual reactor trip and SI was actuated. - All systems responded as expected. - Estimated leakrate is ~ 400 gpm. - RCS pressure is 2000 psig and slowly rising. - PZR level is 22% and rising. - Containment radiation is 1.5 R/hour and stable. - Containment pressure reached 9.0 psig, then rapidly dropped to 0.5 psig. What is the current classification for this event based on these indications? (Byron Annex Attached)

A B C D

Unusual Event - FU1

Unusual Event - MU8

Alert - FA1

Site Area Emergency - FS1


Answer: Task No: D Obj No: T.ZP1-16A, G-6-8 Question Source: New Medium Question Difficulty

Time: Cross Ref: 1 EP-AA-1002, Byron Annex for Radiological Emergency Plan Reference: Emergency Preparedness, G-6, Emergency Action Levels 43(b)(5)

Applying the indications above will lead you to addressing the fission product barriers. RCS should be identified as potentially lost, due Explanation: to leakrate > 1 CV pump in normal operation, and that alone would be classified as an ALERT-FA1. The containment should be identified as lost due to the rapid, unexplainable, pressure drop. This alone would be an Unusual Event - FU1. If leakage is thought to be within the capacity of 1 CV pump in normal lineup, then an Unusual Event - MU8 may be declared. The combination of the RCS leakage and the containment rapid depressurization together would be a Site Area Emergency - FS1. This would be correct for the event conditions. A. is incorrect since more than just the containment barrier is lost or potentially lost. B. is incorrect since the leakrate applies, but it is large enough to constitute potential failure of the RCS. C. is incorrect since more than the RCS barrier is lost or potentially lost. D. is correct as described above. Date Written: 3/19/2006 Author: M. Jorgensen App. Ref: Byron Annex for Emergency Rad Plan

Quest No: RO SRO: TIER: 83 SRO 1 System/Evolution Name: High Reactor Coolant Activity

GROUP: Topic No: KA No: RO: SRO: Cog Level: 000076 AA2.01 2.7 3.2 High Category Statement: Ability to determine and interpret the following as they apply to the High Reactor Coolant Activity:

KA Statement: Location or process point that is causing an alarm UserID: Question Stem: Topic

At 0800 - Unit 2 was performing a power ascension from 75% to 90% after a Refueling Outage. - Excessive Iodine Spiking is reported by Chemistry. Which Radiation Monitor would first detect the Iodine Spiking and what action would be taken?

A B

VCT Cubicle Monitor; direct RP to survey the Auxiliary Building and establish required area postings. CNMT ATMOS Monitor; verify CNMT Ventilation Isolation and consider placing a CNMT Charcoal filter train in service. MAIN STEAMLINE Monitor; verify a single Charging pump is adequate to maintain PZR level > 17%. GROSS FAIL FUEL Monitor; contact Chemistry to calculate mixed bed demin decontamination factor.
Answer: Task No: D Obj No: T.OA15-03/05 Medium Question Source: Question Difficulty

Time: Cross Ref: 1 I1-OA-XL-15, BOA PRI-4, Abnormal Primary Chemistry, Pg 5 Reference: 1/2BOA PRI-4, Abnormal Primary Chemistry, step 7 43(b) (5)

It is not unusual for some Iodine Spiking to occur on a rapid power change and the activity will usually return to normal levels after a Explanation: few hours of operation. One BOA PRI-4 entry condition is High alarm on the Gross Failed Fuel monitor.The first action to be taken is to have Chemistry calculate the Demin DF to ensure the activity can be removed.. A. is incorrect since this is a High range monitor that may trend up, but is only expected to alrm with very high activity in the VCT and very low level in the VCT. B. is incorrect since there is nothing in the stem to lead one to believe a leak to atmosphere in Cnmt exists. C. is incorrect since there is nothing in the stem to lead one to believe a SG tube leak exists. D. is correct as described above. Date Written: 3/20/2006 Author: M. Jorgensen App. Ref:

Quest No: RO SRO: TIER: 84 SRO 1 System/Evolution Name: SI Termination

GROUP: Topic No: KA No: 2 00WE02 2.2.22 Category Statement: Equipment Control

RO: 3.4

SRO: 4.1

Cog Level: High

KA Statement: Knowledge of limiting conditions for operations and safety limits. UserID: Question Stem: Topic

A small break LOCA has occurred on Unit 1 and transition to 1BEP-1, Loss of Reactor or Secondary Coolant, has been made. The crew is at step 6 which determines if ECCS flow can be reduced. The following conditions exist: - Containment pressure = 6.4 psig - PZR level = 14% - NR SG levels: 1A = 28%; 1B = 29%; 1C = 33%; 1D = 32% - AF flow to SGs = 590 gpm - RCS pressure = 1482 psig and stable - CETCs = 523F - All RCPs are running - RWST level = 49% Based on these conditions, ECCS _____________________ .

A B C

can be terminated, transition to 1BEP ES-1.1, SI Termination.

can not be terminated due to inadequate PZR level, continue in 1BEP-1.

can not be reduced due to RWST level, transition to 1BEP ES-1.3, Transfer to Cold Leg Recirculation. can not be reduced due to RCS subcooling, transition to 1BEP ES-1.2, Post LOCA Cooldown and Depressurization.
Answer: Task No: B Obj No: T.EP02-01-F Question Source: Byron Cert exam bank (2001) Medium Question Difficulty

Time: Cross Ref: 1 1BEP-1, Loss of Reactor or Secondary Coolant, Step 6 criteria Reference: I1-EP-XL-02, Loss of Reactor or Secondary Coolant, Pg 8 1BEP-0, Reactor Trip or Safety Injection, CAUTION prior to step 1 43b(5)

Usage requirements and first CAUTION in 1/2BEP-0 for ADVERSE conditions (> 5 psig in containment) requires use in the Explanation: conditions stated. PZR level must be > 28% for SI termination. All other conditions are met in step 6 of 1BEP-1. A. is incorrect since PZR level (Adverse CNMT) is not adequate. B. is correct as stated above. C. is incorrect since RWST level is not a termination criteria and is above the LO-2 setpoint for transition to 1BEP ES-1.3. D. is incorrect since subcooling is addequate. Date Written: 3/11/2006 Author: M. Jorgensen App. Ref:

Quest No: RO SRO: TIER: 85 SRO 1 System/Evolution Name: Natural Circulation Operations

GROUP: Topic No: KA No: 2 00WE09 2.4.30 Category Statement: Emergency Procedures/Plan

RO: 2.2

SRO: 3.6

Cog Level: High

KA Statement: Knowledge of which events related to system operations/status should be reported to outside agencies. UserID: Question Stem: Topic

Which of the following events would require notification of the NRC within 1 hour?

A B C D

Two vehicles crash in the Byron Station parking lot with minor injuries; no one hospitalized.

Off-site power is lost to Unit 2 at 18% power resulting in a reactor trip.

SAT feeder breaker to Bus 141 trips and 1A DG fails to start with Unit 1 at 100% power.

A tornado is sighted within 5 miles of Byron Station.


Question Source: G-6_8, G-5-8/9 New Medium Question Difficulty

Answer: Task No: B Obj No:

Time: Cross Ref: 1 Emergency Preparedness, G-6, Emergency Action Levels, Pg 15 Reference: Emergency Preparedness, G-5, Classification, Notification, and PARs, Pgs 17, 18 EP-AA-1002, Byron Annex For Radiological Emergency plan, Pg 3-10 43(b) (5)

Per the Byron Annex, the loss of a single ESF Bus nor the reactor trip alone exceed EALs, (SM could declare an NUE, but not Explanation: required). This is reportable to the NRC, but not within 1 hour. However, the loss of both SATs places Unit 2 in a natural circ condition and means the SATs are not available to power the Unit 2 ESF buses. This meets EAL MU1 and any classification requires 15 minute notification of State and Local authorities and notification of the NRC within 1 hour. A. is incorrect since this event does not exceed an EAL, crash inside of the protected area or swithyard into structures containing systems for sfe shutdown would exceed the EAL and require notification. B. is correct as described above. C. is incorrect since this would be reportable, but not within 1 hour and does not exceed an EAL. D. is incorrect since a tornado strike within the protected area or switchyard is required to exceed an EAL. Date Written: 3/22/2006 Author: M. Jorgensen App. Ref: PE-AA-1002, Byron Annex

Quest No: RO SRO: TIER: 86 SRO 2 System/Evolution Name: Reactor Protection System

GROUP: Topic No: KA No: RO: SRO: Cog Level: 1 012000 A2.06 4.4 4.7 Low Category Statement: Ability to (a) predict the impacts of the following malfunctions or operations on the RPS and (b) based on those predictions, use procedures to correct, control, or mitigate the consequences of those malfunctions or operations:

KA Statement: Failure of RPS signal to trip the reactor UserID: Question Stem: Topic

Unit 2 operators are performing steps in 2BFR S.1, Response to Nuclear Power Generation/ATWS Unit 2, and have just isolated steam dumps when the following conditions are noted: - Unit 2 reactor power = 19%. - PZR pressure is 1830 psig and trending down. - Reactor trip breakers are closed. - Safety injection just Actuated. What actions will you direct for these conditions?

A B

Continue attempts to trip the reactor and transition to 2BEP-0, Reactor Trip or Safety Injection, for SI Verification. Verify SI is actuated per the Operator Action Summary page in 2BFR S.1 while continuing with the steps in 2BFR S.1. Continue in 2BFR S.1 and after the reactor is tripped, immediately transition to 2BEP-0.

C D

Verify SI is properly actuated using the verification steps in 2BEP-0 and then continue with the steps of 2BFR S.1.
Question Source: T.FR1-07 Byron LORT Bank. (2002) - Used last in 2004, cycle 6-8 SRO annual. Question Difficulty Medium

Answer: Task No: B Obj No:

Time: Cross Ref: 1 1BFR S.1, Response to Nuclear Power Generation/ATWS Unit 2 Reference: I1-FR-XL-01, BFR S Series Subcriticality, Pg 21 43(b)(5)

Since 2BFR S.1 could have been entered at step 1 of 2BEP-0 and SI may have occurred and not been verifified, the contingency is Explanation: covered separately in 2BFR S.1 after step 5 in a CAUTION stating "If SI actuates, proper ESF actuations should be verified as time permits per the OAS page. A. is incorrect since transition out of this procedure is not done after step 6 until the procedure is completed. Step 7 has just been completed by the crew. B is correct as stated above. C. is incorrect for essentially the same reason as A. above. D. is incorrect since the OAS page of 2BFR S.1 contains the verification steps and 2BEP-0 is not required. Date Written: 3/22/2006 Author: G. Wolfe App. Ref:

Quest No: RO SRO: TIER: 87 SRO 2 System/Evolution Name: Containment Spray System (CSS)

GROUP: Topic No: KA No: 1 026000 2.2.22 Category Statement: Equipment Control

RO: 3.4

SRO: 4.1

Cog Level: Low

KA Statement: Knowledge of limiting conditions for operations and safety limits. UserID: Question Stem: Topic

What is the safety analysis basis for the minimum OPERABILITY requirements for the Spray Additive System? The Design Basis Accident analyses assumes that _________________ .

ONE train is OPERABLE and the volume of NaOH added to the spray in the time the RWST reaches the Lo-2 setpoint is adequate to ensure a minimum 8.0 pH in the containment recirculation sump to reduce stress corrosion of mechanical components. ONE train is OPERABLE and the volume of NaOH added to the spray in the time the RWST reaches Lo-3 setpoint is adequate to remove iodine from the containment atmosphere and maintain it in solution in the recirculation sump. TWO trains are OPERABLE and the volume of NaOH added to the spray in the time the RWST reaches the Lo-3 setpoint is adequate to ensure a minimum 8.0 pH in the containment recirculation sump to reduce stress corrosion of mechanical components. TWO trains are OPERABLE and the volume of NaOH added to the spray in the time the RWST reaches the Lo-2 setpoint is adequate to remove iodine from the containment atmosphere and maintain it in solution in the recirculation sump.
Question Source: S.CS1.03/15 Byron Cert exam bank (2001) Medium Question Difficulty

Answer: Task No: B Obj No:

Time: Cross Ref: 1 Tech Spec 3.6.7 bases, Pgs B3.6.7-2/3 Reference: I1-CS-XL-01, Ch 59, Containment Spray System, Pgs 2, 3, 23, 24 43(b) (3)

As stated in the Tech Spec bases; only ONE train required until CS is switched to the recirculation sump when the RWST is empty Explanation: (Lo-3) and removes iodine and maintains it in solution with the pH adjustment. This, in turn, minimizes corrosion of components. A. is incorrect since the DBA assumes CS runs at least to the Lo-3 setpoint and the premise for pH is iodine removal. B. is correct as stated above. C. is incorrect since the design assumes only a single train operates and the pH reasoning is not complete. D. is incorrect since the design assumes only one train operates to Lo-3 in the RWST. Date Written: 3/11/2006 Author: M. Jorgensen App. Ref:

Quest No: RO SRO: TIER: 88 SRO 2 System/Evolution Name: Main Feedwater (MFW) System

GROUP: Topic No: KA No: 1 059000 2.2.22 Category Statement: Equipment Control

RO: 3.4

SRO: 4.1

Cog Level: Low

KA Statement: Knowledge of limiting conditions for operations and safety limits. UserID: Question Stem: Topic

The following conditions exist on Unit 2: - MODE 3 with reactor startup planned. An NLO reports the following FW valves packing was adjusted due to excessive leakage: - 2FW002C, 2C FW PP MOV DISCH - 2FW035A, S/G 2A FW TEMPERING ISOL VALVE - 2FW043D, S/G 2D FWIV BYPASS ISOL VALVE Which valves will you assign to have valve stroke time tests completed for Tech Spec 3.6.3, Containment Isolation Valves Operability?

A B C D

2FW002C and 2FW043D only.

All three valves.

2FW002C and 2FW035A only.

2FW035A and 2FW043D only.


Answer: Task No: D Obj No: S,CD1-021/022 Question Source: New Medium Question Difficulty

Te m i : Cos e: r sR f 1 Tech Spec 3.6.3 Bases, Containment Isolation Valves, Table B. 3.6.3-1 (page 3 of 9) I1-CD-XL-01, Condensate and Feedwater System, Pg 63 43(b) (2)

Reference:

Tech Spec 3.6.3, Containment Isolation Valves applies to 2FW035A and 2FW043D only. 2FW002C will also require testing since Explanation: auto closure signals are sent to the valve, but it is not a containment isolation valve. A.is incorrect since 2FW002C is not required in Tech Spec 3.6.3, it is not a cnmt isol valve. B. is incorrect for the same reason as A. Date Written: 3/22/2006 Author: G. Wolfe App. Ref: C. is incorrect for the same reason as A.

D.is correct as described above.

Quest No: RO SRO: TIER: 89 SRO 2 System/Evolution Name: A.C. Electrical Distribution System

GROUP: Topic No: KA No: RO: SRO: Cog Level: 1 062000 A2.03 2.9 3.4 High Category Statement: Ability to (a) predict the impacts of the following malfunctions or operations on the A.C. Distribution System and (b) based on those predictions, use procedures to correct, control, or mitigate the consequences of those malfunctions or operations:

KA Statement: Consequences of improper sequencing when transferring to or from an inverter UserID: Question Stem: Topic

Unit 2 was at 100% power when the following occurred: - The Inverter for Instrument Bus 211 failed resulting in loss of Bus 211. - Operators responded per 2BOA ELEC-2, Loss of Instrument Bus Unit 2 and reenergized Bus 211 via the Constant Voltage Transformer (CVT). - PR channel N44 is currently in a tripped condition due to detector failure requiring a shut down to repair - 2 days later, maintenance has repaired the inverter and is ready to place it back in service. What actions need to be directed BEFORE restoring Instrument Bus 211 to the Inverter?

Leave PR channel N44 as is and restore power to the Instrument bus from the Inverter to the CVT per 2BOA ELEC-2.

B
the

Place PR channel N44 in Bypass first, then restore power to the Instrument bus from Inverter per BOP IP-1, Instrument Bus Inverter Startup.

C
Instrument

Place PR channel N41 on its alternate power supply, then restore power to the bus from the Inverter per BOP IP-1, Instrument Bus Inverter Startup.

D
the

Place PR channel N41 in Bypass first, then restore power to the Instrument bus from Inverter per 2BOA ELEC-2.

Answer: Task No: B Obj No: S.AP1-14-B

Question Source: New

Question Difficulty Medium

Ti e m: Cr sR : os ef 1 I1-AP-XL-01, Ch 4, AC Electrical Power Systems, Pgs 52, 53, 63, 64 Reference : BOP IP-1, Instrument Inverter Startup, Precautions 43(b)(5)

BOP IP-1 has a precaution that reminds the operator that when transferring power from the reserve power to the inverter, the Explanation: Date Written: 3/22/2006 Author: M. Jorgensen App. Ref:

circuit is a break before make, resulting in a momentary loss of power, which may cause a reactor trip. So, if another instrument bus channels' instrumentation has a standing trip condition, this momentary loss of this channel may satisfy reactor trip logic. A. is incorrect since this may result in a trip during transfer when power is momentarily lost to N41. B. is correct since this is a trip condition on NI channel 4(N44) and if power is momentarilly lost to NI channel 1(N41) during the transfer back to the Inverter, this could result in a reactor trip(2/4 logic), if N44 is not taken to Bypass first. C. is incorrect since N44 does not have a backup power supply like many of the SSPS instruments do. D. is incorrect since this action alone could result in a reactor trip with N44 already in trip.

Quest No: RO SRO: TIER: GROUP: Topic No: KA No: RO: SRO: Cog Level: 90 SRO 2 1 073000 A2.01 2.5 2.9 High System/Evolution Name: Category Statement: Process Radiation Monitoring (PRM) System Ability to (a) predict the impacts of the following malfunctions or operations on the PRM System and (b) based on those predictions, use procedures to correct, control, or mitigate the consequences of those malfunctions or operations: KA Statement: Erratic or failed power supply UserID: Question Stem: Topic

Unit 1 is at 100% power with the following conditions: - 0100 on 6/5 - 1FS-RF008, Containment Floor Drain Sump Flow monitor, failed. - Parts have been ordered for replacement. - 1300 on 6/30 - 1PR011J, CNMT ATMOS Unit 1, monitor has just alarmed indicating an OPERATE FAILURE. What actions are/were directed as a result of these conditions?

A B
failure.

Enter LCOAR for 1FS-RF008 failure; enter LCO 3.0.3 for 1PR011J failure.

Place 1FS-RF008 in the Degraded Equipment Log; enter LCOAR for 1PR011J

C D

Enter LCOAR for 1FS-RF008 failure; enter separate LCOAR for 1PR011J failure.

Place 1FS-RF008 and 1PR011J in the Degraded Equipment Log; verify alternate instumentation is OPERABLE.
Answer: Task No: B Obj No: Question Source: S.RC1-12, S.AR1-06/15 New Medium Question Difficulty

Time: Cross Ref: 1 I1-RC-XL-01, Ch 12, Reactor Coolant System, Pg 50 Reference: I1-AR-XL-01, Ch 49, Radiation Monitoring, Pg 35 Tech Spec 3.4.15, RCS Leakage Detection Instrumentation BAP 1400-6, Tech Spec LCOARs 43(b) (5) Tech Spec 3.4.15 requires FS-RF008 or PC002 or PC003 AND FS-RF010, AND 1PR011J (A-Particulate channel) to be OPERABLE Explanation: for RCS Leak Detection. With FS-RF008 OOS, the LCO is still met and, since FS-RF008 is part of the operability requirement, BAP 14006 requires that it be entered in the Degraded Equipment Log for tracking with no LCOAR required unless additional sump flow or level instrumentation fails. However, 1PR011J is required to be OPERABLE to meet the LCO, therefore, a LCOAR is required for its failure. This is a 30 day action requirement with additional sampling and surveillance required due to its failure. LCO 3.0.3 would be entered if 1PR011J and less than required sump flow/level indications were available. A. is incorrect since no LCOAR is required for 1FS-RF008 alone failed, therefore LCO 3.0.3 would not apply. B. is correct as described above. C. is incorrect since no LCOAR is required for 1FS-RF008 alone failed and this Tech Spec does not allow separate entries for clock starts on LCOAR actions. Date Written: 3/27/2006 Author: M. Jorgensen App. Ref: D. is incorrect since only 1FS-RF008 is required in the DEL and LCOAR is required for 1PR011J failure.

Quest No: RO SRO: TIER: 91 SRO 2 System/Evolution Name: Fuel Handling Equipment System (FHES)

GROUP: Topic No: KA No: RO: SRO: Cog Level: 034000 K1.04 2.6 3.5 Low Category Statement: Knowledge of the physical connections and/or cause-effect relationships between the Fuel Handling System and the following systems:

KA Statement: NIS UserID: Question Stem: Topic

The Fuel Handling Supervisor has just notified the Control Room that audible counts in CNMT ceased as they were lifting a fuel assembly from the upender. The NSO also reports that audible counts in the Control Room have been lost. The fuel handlers have suspended core alterations. What is the requirement prior to resuming core alterations?

A B

Set the audible count rate selector switch to the other channel, VERIFY audible count rate to the Control Room and CNMT with ONE SR channel OPERABLE. Initiate emergency boration until it can be VERIFIED that all filled portions of the RCS are at least 2300 ppm boron concentration. Set the audible count rate selector switch to the other channel, VERIFY audible count rate to the Control Room and CNMT with BOTH SR channels OPERABLE. Set the audible count rate selector switch to the other channel, VERIFY audible count rate to the Control Room with ONE SR channel OPERABLE.
Question Source: Byron NRC exam bank (1996) Medium Question Difficulty

Answer: Task No: C Obj No: T.OA10-03

Ti e m: Cr sR : os ef 1 1/2BOA INST-1, Nuclear Instrumentation Malfunction, step 3 and 6 for SR channels Reference : Tech Spec 3.9.3 requires at least 2 operable for core alterations. 43(b) (7)

At least 2 operable channels are required for core alterations per Tech Spec 3.9.3. Also _BOA INST-1 action is to select the other Explanation: channel and ensure that at least 2 channels are operable to perform core alterations or any + reactivity additions.

Date Written:

3/11/2006 Author: M. Jorgensen

App. Ref:

Quest No: RO SRO: TIER: GROUP: Topic No: KA No: 92 SRO 2 2 045000 2.2.22 System/Evolution Name: Category Statement: Main Turbine Generator (MT/G) System Equipment Control

RO: 3.4

SRO: 4.1

Cog Level: High

KA Statement: Knowledge of limiting conditions for operations and safety limits. UserID: Question Stem: Topic

Unit 2 is at 100% power with the following condition: - NLO reports from the field that 2ES017A, Extraction Steam Nonreturn Check Valve, from the 24A Feedwater Heater, did not move during performance of the monthly surveillance. What are the required actions for continued operations and why? (BOP HD-6T1, Turbine Operations Limitation Table Concerning the Isolation of Various Strings of Feedwater Heaters, is provided)

Reduce turbine load to 820 Mwe, then open the Low Pressure Heater String Bypass Valve and isolate the 21A, 22A, 23A, and 24A Feedwater Heater string to prevent possible turbine overspeed on a turbine trip. Reduce turbine load to 1173 Mwe, then isolate extraction steam to the 24A Feedwater Heater to prevent possible turbine overspeed on a turbine trip.

Reduce turbine load to 1173 Mwe, then isolate the MOVs in the extraction steam lines to the 24A/B/C Feedwater Heaters to preclude water induction into the turbine on a High level in the associated heaters. Reduce turbine load to 937, then isolate the MOVs in the extraction steam lines to the 22A, 23A, and 24A Feedwater Heaters to prevent possible turbine overspeed on a turbine trip.
Question Source: S.ES1-04/11 New Medium Question Difficulty

Answer: Task No: B Obj No:

Time: Cross Ref: 1 I1-ES-XL-01, Ch 36, Extraction Steam, Feedwater Heater Vents and Drains, Pgs 19, 20, 24, 32, 35 Reference: I1-MT-XL-01, Main Turbine and Reheat Steam TRM 3.3.g BOP HD-6T1, Turbine Operations Limitation Table Concerning the Isolation of Various Strings of Feedwater Heaters BOP HD-16, Isolating and Return to Service LP FW Heaters _4A/B/C Shell Side and Restoration Following HI-2 Level Isolation 43(b) (2) TRM 3.3.g, Condition D applies. Referring to Table 3.3.g-2, the action is to isolate the steam supply to the turbine within 6 hours. In Explanation: the case of extraction steam, this is referring to the MOV (for the valve in question since no manual valve exists for this valve) in the extraction steam line, this isolates return steam to the turbine on a turbine trip, which then prevents the possibility of overspeed. Per BOP HD-6T1, power reduction to 1173 Mwe is required to isolate this nonreturn valve. Only this line is required to be isolated and operations can continue per BOP HD-16. The string or any other heater is not required to be isolated with it. A. is incorrect since this is the required power reduction for removing the entire string from service, but this is not required. B. is correct as described above. C. is incorrect since all three heaters are not required to be isolated. D is incorrect since the power reduction required is 1173. This would be required to remove the 3 heaters in the string, but that is not required.

Date Written:

3/24/2006 Author: M. Jorgensen

App. Ref: BOP HD-6T1

Quest No: RO SRO: TIER: 93 SRO 2 System/Evolution Name: Circulating Water System

GROUP: Topic No: KA No: RO: SRO: Cog Level: 2 075000 A2.01 3.0 3.2 High Category Statement: Ability to (a) predict the impacts of the following malfunctions or operations on the Circulating Water System and (b) based on those predictions, use procedures to correct, control, or mitigate the consequences of those malfunctions or operations:

KA Statement: Loss of intake structure UserID: Question Stem: Topic

With both Units at 100% power, the control room operators have identified a loss of flow condition on CW Makeup flow indicator, 0FT-CW040, in response to INTAKE BAY LEVEL LOW alarm. If 0AOV-CW220, CW Makeup Flow Control Valve, does NOT open, the operators are procedurally directed to conserve flume level. Which of the following actions would the operators be directed to perform? - 1.Reduce reactor power and shutdown a CW pump on each unit. - 2.Secure CW blowdown. - 3.Secure CW makeup to SX. - 4.Reduce reactor power on each unit.

A B C D

2 and 4

2 and 3

1 and 3

1 and 2
Answer: Task No: A Obj No: T.OA43B-03 Question Source: Byron LORT bank - Used last 2006, cycle 1-4. Medium Question Difficulty

Time: Cross Ref: 1 0BOA SEC-11, Inadequate Ciculating Water Makeup Reference: I1OA-XL-43, Inadequate Circulating Water Makeup, Pgs 3, 4 43(b)(5)

OBOA SEC-11 actions for makeup failure is to stop CW blowdown and reduce power on both units to conserve flume level. Explanation: A. is correct as stated above. B. is incorrect since SX makeup is not a viable option, since SX is required for safe shutdown. C. is incorrect since shutdown of CW pumps would not conserve flume level, SX actually takes priority. D. is incorrect since shutdown of CW pumps would not conserve flume level.

Date Written:

3/23/2006 Author: G. Wolfe

App. Ref:

Quest No: RO SRO: TIER: 94 SRO 3 System/Evolution Name: Generic

GROUP: Topic No: KA No: 194001 2.1.7 Category Statement: Conduct of Operations

RO: 3.7

SRO: 4.4

Cog Level: High

KA Statement: Ability to evaluate plant performance and make operational judgments based on operating characteristics, reactor behavior, and instrument interpretation. UserID: Topic Question Stem:

Given the following conditions on Unit 2: - Reactor power - 31% and rising. - RCS Tave = 556F and slowly lowering. - PZR pressure = 2175 psig and slowly lowering. - PZR level = 29% and slowly lowering. - Turbine load is stable. - SG pressures = 940 psig and lowering. - SG PORVs indicate closed. - Steam Dumps indicate closed. - Unit 1 Turbine Bldg Rounds calls the Control Room and reports steam on the 451' level in the Turbine building. What action should be directed for these conditions? Immediately trip the ___________ .

A B C D

turbine and close the MSIVs.

reactor and close the MSIVs.

turbine and initiate safety injection.

reactor and initiate safety injection.


Answer: Task No: B Obj No: Question Source: Byron Cert exam bank (2001) Medium Question Difficulty

Time: Cross Ref: 1 OP-AA-101-111-1001, Operations Philosophy Handbook Conditions of an Operating License, 10CFR50 43(b)(5)

Reference:

The call here is really a condition of license. It is expected that an SRO will have public health and safety foremost in his mind, then Explanation: comes personnel safety on-site. This is clearly an example were the plant is degrading and needs to be placed in a safe condition and then take action to protect plant personnel. The indication of a steam leak in the turbine building is the key to why the plant conditions are trending the way they are. Also, protecting plant personnel requires isolation of the steam source. The proper action is to trip the reactor to place it in a safe condition. This will also generate a turbine trip in all cases. Then close the MSIVs to protect plant personnel and stop the plant transient at the same time. A turbine trip first is inappropriate, since this is then relying on the turbine tripping to generate a reactor trip, which should occur at this power level, but should not be relied on and closing the MSIVs will also stop steam to the turbine. A. is incorrect since the reactor trip will generate a turbine trip and ensuring the reactor is shutdown takes priority. B. is correct as described above. C. is incorrect since this may trip the reactor, it will not islate the steamline break. D. is incorrect since the reactor trip first is OK, but the SI will not stop the steam leak to protect personnel.

Date Written:

3/11/2006 Author: M. Jorgensen

App. Ref:

Quest No: RO SRO: 95 SRO System/Evolution Name: Generic

TIER: 3

GROUP: Topic No: KA No: 194001 2.1.32 Category Statement: Conduct of Operations

RO: 3.4

SRO: 3.8

Cog Level: Low

KA Statement: Ability to explain and apply all system limits and precautions. UserID: Question Stem: Topic

LCO 3.0.6 requires a Loss of Safety Function Evaluation (LOSF) be performed when an inoperabilty of a support system renders a supported system inoperable. During a situation involving two supported systems inoperable due to the same support system without a LOSF, to ensure further LOSF evaluations on the supported systems to be properly performed, per BAP 1400-6, "TECHNICAL SPECIFICATION LIMITING CONDITIONS FOR OPERATION ACTION REQUIREMENTS (LCOAR)", the inoperable supported systems are REQUIRED to be logged in the .

A B C D

Degraded Equipment Log (DEL)

Shift Managers Log

supported systems LCOAR paperwork and the associated Unit Log

associated supported systems LCOAR


Question Source: S.TS1-05-B, T.AM13- New Medium Question Difficulty

Answer: Task No: A Obj No:

Time: Cross Ref: 1 I1-TS-XL-01, Ch 3, Introduction to Technical Specifications, Pg 22 Reference: Selected Administrative Procedures, BAP 1400-6, Pg 49 BAP 1400-6, Tech Spec Limiting Conditions for Operation Action Requirement LCOAR) procedure, Pg 8 43(b)(4)

This is as stated on Page 8 of BAP 1400-6. The LOSF is referred to as the SFD (Safety function Determination). This section states Explanation: that If any supported system LCO entries are precluded by 3.0.6 based on a review of the BOL, these supported systems shall be documented on the DEL to ensure any future inoperabilities are properly evaluated on the SFD. A. is correct as described above. B. is incorrect for this information. It can be noted in the log, but required in the DEL. C. is incorrect since this is required to be documented separately as a reminder and not duplicate LCOAR generation. D. is incorrect for basically the same reason as C. above. Date Written: 3/24/2006 Author: G. Wolfe App. Ref:

Quest No: RO SRO: 96 SRO System/Evolution Name: Generic

TIER: 3

GROUP: Topic No: KA No: 194001 2.2.3 Category Statement: Equipment Control

RO: 3.1

SRO: 3.3

Cog Level: Low

KA Statement: (multi-unit) Knowledge of the design, procedural, and operational differences between units. UserID: Question Stem: Topic

Compare the expected response between Unit 1 and Unit 2 for establishing feedwater flow to a Dry Steam Generator after initiation of Feed and Bleed as part of Attachment B in 1/2BFR-H.1, Response to Loss of Secondary Heat Sink. A Dry SG is any SG with Wide Range level < 10% (27% ADVERSE CNMT) on ___(1)___ and, if feedwater flow has been stopped for > 75 minutes with SG level < 45% Narrow Range, the feed line is considered voided on ____(2)____, and the feedwater initial flowrate is determined by whether the tempering line is at least 75F subcooled on ___(3)____.

(1)

(2)

(3)

A B C D

both Units Unit 2

Unit 1

Unit 1

Unit1

Unit 1

both Units Unit 1

Unit 2

Unit 2

Unit 2

Unit 2
Question Source: New Medium Question Difficulty

Answer: Task No: C Obj No: T.FR3-04-A/E

Time: Cross Ref: 1 1/2BFR-H.1, Attachment B, Response to Loss of Secondary Heat Sink, Pg 53 Reference: I1-FR-XL-03, FRPs-BFR H.1-H.5, Pg 54 - 56 43(b) (1)

With the different types of SGs in the two Units, Unit 1 SGs have a large, single feedline with a loop seal. Unit 2 has a very small Explanation: upper feedline that accomodates ~10% of full flow at 100% power and all flow when the Unit is shutdown. Dispite the different SGs, both use the same Dry SG level criteria. Unit 1 is concerned about voiding in the feed ring and engineering evaluation determined that after 75 minutes without feed and NR SG level < 45%, the feedline may be void (full of steam), so this became the criteria for reducing feed flow when recomencing feed flow to prevent damage to the feed ring. Unit 2 has a much smaller feedline and the concern is thermal shocking of the feed nozzle and is precluded by throttling feed flow if the tempering line is not > 75F subcooled. These differences are proceduralized to prevent damage to either Unit by applying one criteria to the other. A. is incorrect since the application of voiding is Unit 1and subcooling is Unit 2. B. is incorrect since the application of voiding is Unit 1but subcooling is Unit 2. C. is correct as described above. D. is incorrect since the application of voiding is Unit 1.

Date Written:

3/11/2006 Author: M. Jorgensen

App. Ref:

Quest No: RO SRO: TIER: 97 SRO 3 System/Evolution Name: Generic

GROUP: Topic No: KA No: 194001 2.2.3 Category Statement: Radiological Controls

RO: 2.1

SRO: 3.1

Cog Level: High

KA Statement: Knowledge of the requirements for reviewing and approving release permits. UserID: Question Stem: Topic

A gas release from Containment is pending on Unit 2 at 100% power using Containment Mini Purge System with the following progression: - 0200 on 6/19 - Containment sample analyzed. - 2RE-PR011B, CNMT ATMOS UNIT 2, is in service. - 0500 on 6/19 - Release package requested from Radiation Protection (RP). - 1200 on 6/19 - Release was approved by Lead RP. - 1500 on 6/19 - Release was approved by the shift SRO.

What is the expiration time/date for this release package per BCP 400TCNMTROUTINE, Gaseous Effluent Release Form?

A B C D

0000 on 6/20

0200 on 6/20

0800 on 6/20

1500 on 6/20
Answer: Task No: GW-001, S-HP-002, C Obj No: Question Source: New Medium Question Difficulty

Time: Cross Ref: 1 BCP 400-TCNMTROUTINE, Gaseous Effluent Release Form Reference: Byron SRO Certification Guide (OJT), Pg 22, 33 43(b) (6)

A NOTE on the first page of the release form states that analyzed samples are only good for 30 hours provided 2RE-PR011B has Explanation: remained stable. The SRO approval records the expiration time/date based on 30 hours after the sample was obtained by RP. A. is incorrect but plausible since some release durations are only good for the current day. B. is incorrect but plausible since this would be a 24 hour duration from sampling, which is a common sample frequency when a release monitor is OOS. C. is correct as described above. D. is incorrect but plausible since this would be 24 hours from SRO approval, which is a common sample frequency for a release monitor being OOS. Date Written: 3/25/2006 Author: M. Jorgensen App. Ref:

Quest No: RO SRO: TIER: 98 SRO 3 System/Evolution Name: Generic

GROUP: Topic No: KA No: 194001 2.3.10 Category Statement: Radiological Controls

RO: 2.9

SRO: 3.3

Cog Level: Low

KA Statement: Ability to perform procedures to reduce excessive levels of radiation and guard against personnel exposure. UserID: Question Stem: Topic

Unit 2 is in MODE 6 with refueling activities in progress when the following occurs: - A fuel assembly is dropped during removal from the core. - Bubbling is observed from the core region. What is your FIRST required action as the SRO in CNMT for Fuel Handling operations?

A B C D

Direct operations to establish CNMT closure.

Direct the Control Room to start CNMT Charcoal Filter units.

Direct the Control Room to announce, "ALL personnel to evacuate Unit 2 CNMT".

Direct the Fuel Handlers to PLACE any fuel assembly in the transfer device into the change fixture.
Answer: Task No: C Obj No: S.OA29-03 Question Source: Byron NRC exam bank (1994) Medium Question Difficulty

Time: Cross Ref: 1 2BOA REFUEL-1, Fuel Handling Emergency. Pg 1, 2 entry conditions and step 1 I1-OA-XL-29, BOA REFUEL-1, Fuel Handling Emergency, Pg 4 43(b)4),(7)

Reference:

This is actually a Radiological Fundamental and is the first step in the BOA. Explanation: A. is incorrect since this would have been established prior to moving fuel. B. is incorrect since this is not the first action, but will be considered as followup action. C. is correct as described above. D. is incorrect since this is not an option immediately, and additonal movement must be evaluated first. Date Written: 3/11/2006 Author: M. Jorgensen App. Ref:

Quest No: RO SRO: TIER: 99 SRO 3 System/Evolution Name: Generic

GROUP: Topic No: KA No: 194001 2.4.34 Category Statement: Emergency Procedures/Plan

RO: 3.8

SRO: 3.6

Cog Level: High

KA Statement: Knowledge of RO tasks performed outside the main control room during emergency operations including system geography and system implications. UserID: Topic Question Stem: nit 1 control room has been evacuated and 1BOA PRI-5, Control Room

Inaccessability, has been implemented due to a fire in the Upper Cable Spreading Room. The Remote Shutdown Panel (RSP) is being activated. - The NSO has aligned the RSP and 1PL05JA per Attachment A of 1BOA PRI-5. - The NSO reports he has NO indication of 1D SG level or pressure at the RSP. What action(s) is/are directed for this report?

A
consistent

If MSIVs are open, use any other SG for pressure, balance feedwater flow to be with the other 3 SGs.

B
PORV is

If MSIVs are closed, stop feeding the 1D SG, stop the 1D RCP, and verify 1D SG closed.

C
with the

Dispatch an operator to the Unit 1 Fire Hazards Panel to establish communications NSO at the RSP and align 1D SG level and pressure indication.

Verify 1D SG MSIV is closed, isolate AF Flow to 1D SG, initiate a cooldown to < 550F with the other 3 SGs and verify 1D SG PORV remains closed.
Answer: Task No: C Obj No: T.OA16-03/05/08 Question Source: New Medium Time: Cross Ref: 1 1BOA PRI-5, Control Room Inaccessability, step 8 Reference: I1-OA-XL-16, BOA PRI-5, Control Room Inaccessability, Pgs 16 43(b) (5) RMC Question Difficulty

Although the suggested alternatives seem plausible, the proper direction in the BOA is to dispatch an operator to the Fire Hazards Explanation: Date Written: 3/27/2006 Author: M. Jorgensen App. Ref:

Panel to place only the required indications in local that can not be obtained at the RSP. Only 1A and 1D SG parameters are available at the Fire Hazards Communications will need to be established since the controls for the SG are still at the RSP. A. is incorrect, although a plausible alternative, not the procedural action. B. is incorrect, again, plausible, not procedural. C. is correct as directed in the procedure. D. is incorrect, again plausible, not procedural.

Quest No: RO SRO: TIER: 100 SRO 3 System/Evolution Name: Generic

GROUP: Topic No: KA No: 194001 2.4.36 Category Statement: Emergency Procedures/Plan

RO: 2.0

SRO: 2.8

Cog Level: Low

KA Statement: Knowledge of chemistry / health physics tasks during emergency operations. UserID: Question Stem: Topic

Unit 2 Reactor has been manually tripped due to a steamline break on the 2B SG in the Safety Valve room. The crew has transitioned to 2BEP-2, Faulted Steam Generator Isolation, and is currently performing Step 6, Check Secondary Radiation. What direction is given to the Chemistry Department prior to exiting 2BEP-2?

A B C D

Sample ALL SGs for activity.

Sample ALL INTACT SGs for activity.

Continuously sample the FAULTED SG for activity.

Sample the RCS for boron concentration


Question Source: Obj No: T.EP3-03 Cross Ref: Faulted Steam Generator Isolation, Pg 9 Reference New Low Question Difficulty

Answer: Task No: A Time: 1 2BEP-2,

: I1-EP-XL-03, BEP-2, Faulted Steam Generator Isolation, Pg 9, 10 43(b)(4)

This is an important direction since a SGTR could also be in progress or could occur with a substantial delta-p across the tube sheet in Explanation: the faulted SG with it depressurized and also provide information important for transition to the best recovery procedure. If no activity has caused an alarm the transition will be to 2BEP-1. If activity exists then a transition to 2BEP-3 will be required. So, it is equally important to sample ALL SGs for activity. A. is correct as described above and in 2BEP-2. Date Written: 3/26/2006 Author: M. Jorgensen App. Ref: B. is incorrect since all are equally important and, with 2B open to the environment, this one could be even more likely to develop high activity.

C. is incorrect since this would not allow sampling the 3 SGs that are being used for plant temperature control and may be steaming to atmosphere. D. is incorrect since this sample will be directed the appropriate procedure and is not the concern in 2BEP-2.

REACTOR OPERATOR

Page 1 Page 3

QUESTION: 001 (1.00) The plant was restarted following a forced outage. Loop flow measurement has determined the C RCP impeller has degraded such that its RCS loop flow has DECREASED by 5% from its original value. The other three RCS loop flows remain UNCHANGED.

Which one of the following would be a result of the decreased flow rate in the C loop? a. b. c. d. Demand on the Pressurizer variable heaters at 2235 psig will be lower. The reactor core will operate closer to DNB when at full power. Delta-T in the C RCS loop at full power will be lower. Steam pressure in the C S/G at full power will be higher.

QUESTION: 002 (1.00) The following plant conditions exist: Core alterations are in progress on Unit 1. Conditions are such that Emergency Boration is required. All offsite power has been lost. Both ESF Buses are energized.

Which one of the following Emergency Boration methods should be used? a. b. c. d. Emergency Boration valve 1CV8104 Normal Boration path through the blender Refueling Water Storage Tank (RWST) Manual Emergency Boration valve 1CV8439

REACTOR OPERATOR

Page 2

QUESTION: 003 (1.00) Given the following conditions: The RCS is water solid in MODE 5 The A RH pump is operating in shutdown cooling and RCS temperature is being maintained at 180F via throttling of the 'A' train RH Hx valves. Instrument Air is lost to the 'A' RH Hx room.

Assuming NO operator action, which of the following describes the plant response over the next ten minutes? 'A' RHR suction pressure will ... a. b. c. d. NOT change because there is NO change in RHR flow. DECREASE because RH606 fails OPEN and RH618 fails CLOSED. DECREASE because RH606 fails CLOSED and RH618 fails OPEN INCREASE because RH606 fails OPEN and RH618 fails CLOSED

REACTOR OPERATOR

Page 3

QUESTION: 004 (1.00) Given the following plant conditions: Unit 2 has experienced a large break LOCA During the ECCS injection phase the 2B RH pump tripped due to a overheated bearing that seized The crew has now transitioned to 2BEP ES-1.3, TRANSFER TO COLD LEG RECIRCULATION

In this situation the concern is _____(1)_____, therefore 2BEP ES-1.3 will direct valve alignments such that the 2A RH pump will be injecting into ____(2)_____ . a. (1) inadequate core flow (2) all 4 cold legs, and supplying suction for only the 2 SI pumps. all 4 cold legs, and supplying suction for only the 2 CV pumps. only 2 cold legs, and supplying suction for both CV and both SI pumps. only 2 cold legs, and supplying suction for only the 2 CV pumps.

b.

(1) inadequate core flow

(2)

c.

(1) 2A RH pump runout

(2)

d.

(1) 2A RH pump runout

(2)

QUESTION: 005 (1.00) The plant is operating at 40% power with all systems in a normal lineup. Annunciator 1-12-B7, "PRT PRESS HIGH" alarms. PRT pressure is currently 6 psig. In order to clear this high pressure alarm, which of the following would be the appropriate action? a. b. c. d. Align and verify the start of a RCDT pump. Align the PRT to drain to the RCDT. Vent the PRT to the Waste Gas header. Vent the PRT to the RCDT.

REACTOR OPERATOR

Page 4

QUESTION: 006 (1.00) The unit has just experienced a spurious Containment Phase B actuation. In regard to RCP components ... a. b. c. d. seal cooling is still being provided; motor bearing cooling is NOT. BOTH motor bearing cooling and seal cooling are still being provided. NEITHER motor bearing cooling NOR seal cooling is being provided. motor bearing cooling is still being provided; seal cooling is NOT.

QUESTION: 007 (1.00) The following conditions exist on Unit 1:

A load rejection has occurred from 100% power Reactor power is now 80% Pressurizer level is 56% Pressurizer pressure is 2275 psig RCS Tave is 582F

What Pressurizer Pressure Control System indications are expected on the Main Control Board? a. Backup and proportional heaters are fully on. b. c. d. Proportional heaters have modulated on. Pressurizer spray valves AND pressurizer PORVs are open. Pressurizer spray valves have modulated open.

REACTOR OPERATOR

Page 5

QUESTION: 008 (1.00) Given: Unit 1 P-7 Bypass Permissive light is LIT

Which of the following conditions will cause a Unit 1 reactor trip? a. b. c. d. PZR Level of 94% PZR Pressure of 2390 psig All 4 RCP breakers open PZR Pressure of 1880 psig

QUESTION: 009 (1.00) Given the following plant status: Unit 1 is at full power Last shift Containment Pressure Channel II failed and the Tech Spec required actions were performed A few seconds ago Containment Pressure Channel III failed high

Which of the following automatic actions would be expected to be observed on 1PM06J? a. b. c. d. Closure of all MSIVs only. Closure of all MSIVs and start of ECCS pumps. Start of ECCS pumps only. Closure of all MSIVs, start of CS pumps, and start of ECCS pumps.

REACTOR OPERATOR

Page 6

QUESTION: 010 (1.00) Assume the following conditions exist on Unit 2: A reactor trip has occurred from full power PZR pressure is 1820 psig and steadily decreasing Steps 1 through 3 of 2BEP-0, Reactor Trip or Safety Injection, have been properly verified

Per step 4 of 2BEP-0, an operator checks the Safety Injection annunciators and the Safety Injection Bypass Permissive annunciator, NONE OF WHICH ARE ILLUMINATED, and NO SI equipment has automatically actuated. For this reactor trip event, SI has... a. NOT occurred and is NOT required. The operator should transfer to 2 BEP ES-0.1, Reactor Trip Response. occurred and is required. occurred but is NOT required. The operator should immediately terminate SI by depressing both trains reset pushbuttons on 2PM06J. NOT occurred but is required. The operator should manually initiate SI via the switch on 2PM05J or 2PM06J.

b. c.

d.

QUESTION: 011 (1.00) A loss of Bus 131X will limit the availability of the Reactor Containment Fan Cooler (RCFC) Fans to ... a. b. c. d. 1A and 1C low speed only. 1A and 1B low speed only. 1B and 1D high or low speed only. 1B and 1C high or low speed only.

REACTOR OPERATOR

Page 7

QUESTION: 012 (1.00) The 2A, 2B, and 2D RCFCs are operating in high speed. The 2C RCFC is in standby.

The following indications are observed on the Unit 2 RCFC Dry Bulb temperatures: 2A RCFC Inlet Temperature - 119F 2B RCFC Inlet Temperature - 118F 2C RCFC Inlet Temperature - 127F 2D RCFC Inlet Temperature - 121F

Per the Containment Air Temperature Technical Specification: a. The action requirement must be applied because the average of ALL the RCFC temperatures exceeds the LCO upper limit. NO action is necessary because ALL the RCFC temperatures are within their appropriate LCO limit(s). The action requirement must be applied because ONE of the OPERATING RCFC's temperatures is above the LCO upper limit. NO action is necessary because the average temperature of ALL OPERATING RCFC's is below the LCO upper limit.

b.

c.

d.

QUESTION: 013 (1.00) Consider each of the following situations separately; assume Mode 1 operations. Over the next 20 minutes, which of the following situations would be expected to raise the humidity level in containment? a. b. c. d. A RCFC is swapped from low to high speed operation on PM06J. Containment Phase B Isolation is manually actuated via 2/2 switches on PM06J. CV8160, Letdown Orifices Outlet Header Isolation valve, inadvertently closes. A #1 seal failure is experienced on a RCP.

REACTOR OPERATOR

Page 8

QUESTION: 014 (1.00) The time is 0300. The plant is in MODE 3 at 555F, EOL, following a trip from 100% power moments ago. The following events occur: The Unit NSO reports that Tave has begun decreasing at a rate of approximately 1F/min. The crew receives a report that one of the SG Safety Valves on the "C" SG is passing steam to the atmosphere. The Unit Supervisor confirms an RCS cooldown rate of 5F in the last 5 minutes.

Assuming the RCS cooldown rate is constant, and NO actions are taken in response to the stuck open safety valve, which of the following conditions will exist by time 0400? 1. 2. 3. 4. a. b. c. d. Main Steamline Isolation. Tech Spec limit for RCS Cooldown Rate will be exceeded. The actual amount by which the reactor is shutdown will decrease. A Pressurized Thermal Shock (PTS) challenge to reactor vessel integrity will occur. 1 and 3 2 Only 3 Only 2 and 4

REACTOR OPERATOR

Page 9

QUESTION: 015 (1.00) Given the following plant conditions: Unit 2 is performing a reactor startup at End of Core Life (EOL) per 2BGP 100-2A1, REACTOR STARTUP Reactor power is stable at 1E-3% while taking critical data Steam Header pressure transmitter 2PT-507 fails HIGH

Reactor power will ________ ; and the transient can be mitigated if the board operator places . a. increase; one or more S/G PORV contollers in manual and increases demand a Steam Dump Bypass Interlock switch to OFF/RESET. the Steam Dump Mode Selector switch to Tave position. the Steam Dump Main Steam Header Pressure Controller in manual and reduces demand.

b. c. d.

decrease; decrease; increase;

QUESTION: 016 (1.00) Which of the following describes component response to a Main Feed pump LOW NPSH signal: a. The CD152 valve (CD pump recirc) OPENS and CD157A and B (Gland Steam Condenser Bypass Valves) CLOSE. The standby CD/CB pump starts, then the standby CD/CB aux oil pumps start. The standby CD/CB aux oil pump starts, then the standby CD/CB pump starts. The HD46A and B (HD Pump Discharge Valves) OPEN and the CD210A and B (Condensate Polisher Bypass Valves) CLOSE.

b. c. d.

REACTOR OPERATOR

Page 10

QUESTION: 017 (1.00) Given the following: Unit 1 experienced a reactor trip and Safety Injection from full power when DC Bus 112 deenergized. While performing step 5 to Verify FW Isolation of 1BEP-0, Reactor Trip Or Safety Injection the NSO noted the 1B FW Pump did NOT automatically trip, and its TURB LOSS OF DC TO TRIP lite is illuminated.

Under these circumstances, what action must be performed to trip the 1B FW Pump? a. b. c. Manually trip 1B FW Pump via the MCB pushbutton. Manually initiate FW isolation via the MCB pushbutton. Select Speed Setter Control Mode then depress the DECREASE SPEED pushbutton for 1B FW Pump. Dispatch a NLO to locally trip 1B FW Pump via its overspeed trip plunger.

d.

QUESTION: 018 (1.00) Given the following plant conditions: Unit 2 just entered Mode 1 during plant startup. The Start Up FW Pump is maintaining S/G levels via the Feed Reg Bypass Valves in automatic. A grid disturbance has resulted in a Loss of Offsite Power (LOOP) for Unit 2.

One minute later, by what method is RCS heat being removed, and what is maintaining the heat sink? a. b. c. d. Natural Circulation; Aux Feedwater System Forced Circulation; Main Feedwater System Forced Circulation; Aux Feedwater System Natural Circulation; Main Feedwater System

REACTOR OPERATOR

Page 11

QUESTION: 019 (1.00) Following a reactor trip with NO SI, a transition is made to BEP ES-0.1, Reactor Trip Response Which of the following describes the basis for the RNO actions of step 2, MAINTAIN RCS TEMPERATURE that directs: (1) Verifying total feed flow is greater than 500 GPM; (2) then throttling it once a minimum S/G water level is obtained? a. (1) Ensure enough feedwater flow for decay heat removal; (2) then limit overcooling of the RCS. (1) Ensure enough flow for Aux Feed Pump protection; (2) then limit runout of the Aux Feed pumps. (1) Ensure enough feedwater flow for decay heat removal; (2) then limit runout of the Aux Feed pumps. (1) Ensure enough flow for Aux Feed Pump protection; (2) then limit overcooling of the RCS.

b.

c.

d.

REACTOR OPERATOR

Page 12

QUESTION: 020 (1.00) The following conditions exist on Unit 1: Bus 141 is powered from its normal source D/G 1A surveillance is being performed with the D/G paralleled to the bus

What would occur if a failure of the undervoltage relay results in a sensed undervoltage condition on Bus 141? SAT feeder breaker ACB 1412 ... a. will open but D/G feeder breaker ACB 1413 will remain closed. The Safe Shutdown loads will NOT sequence and CANNOT be manually started from the control room. and D/G feeder breaker ACB 1413 remain closed. The Safe Shutdown loads will NOT sequence and CANNOT be manually started from the control room. and D/G feeder breaker ACB 1413 will open. After a short delay, ACB 1413 will close and the Safe Shutdown loads will sequence. will open but D/G feeder breaker ACB 1413 will remain closed. The Safe Shutdown loads will sequence normally.

b.

c.

d.

QUESTION: 021 (1.00) The purpose of the 125 VDC battery is to supply 125 VDC to: a. b. c. d. ESF and non-ESF DC buses during loss of all AC power. ESF and non-ESF DC buses during normal operation. Only ESF DC buses during normal operation. Only ESF DC buses during loss of off-site power.

REACTOR OPERATOR

Page 13

QUESTION: 022 (1.00) Given the following sequence of events: Following a reactor trip, Unit 1 in MODE 3 at 557F, with the crew performing steps of 1BEP ES-0.1, 'REACTOR TRIP RESPONSE'. Annunciator 1-21-E10, '125V DC PNL 111/113 VOLT LOW' alarms. The MCB indication for DC Bus 111 indicates 0 volts Pressurizer Spray Valve 1RY455B is stuck open and RCS pressure is LOWERING.

To stop the RCS depressurization, the 1____ RCP breaker must be opened. This can be done locally at the breaker _________________ . a. b. c. d. C; only C; or remotely D; only D; or remotely

REACTOR OPERATOR

Page 14

QUESTION: 023 (1.00) Given the following: A Unit 2 Loss of Offsite Power event (LOOP) was caused by a fault on SAT 242-2. Both 2A and 2B Diesel Generators (D/Gs) started on their respective bus undervoltage condition. 2A D/G output breaker ACB 2413 closed in to energize bus 241. 2B D/G output breaker ACB 2423 did NOT close and bus 242 is still deenergized The MCB BUS ALIVE indicator lite for the 2A D/G is lit. The MCB BUS ALIVE indicator lite for the 2B D/G is NOT lit.

Which of the following is the reason the 2B D/G did NOT automatically energize bus 242; and what can be done to reenergize bus 242? The status of the 2B BUS ALIVE lite indicates the D/G did not achieve proper _________ to auto close ACB 2423; and in order to reenergize bus 242 it will be necessary to ________________. a. VOLTAGE; go to RAISE on the Voltage Regulator control switch until proper VOLTAGE is obtained go to RAISE on the Governor Adjust switch until proper FREQUENCY is obtained use the unit crosstie breaker ACB 2424 restore offsite power

b.

FREQUENCY;

c. d.

VOLTAGE; FREQUENCY;

REACTOR OPERATOR

Page 15

QUESTION: 024 (1.00) Given the following conditions: Containment Mini-Purge Supply and Exhaust Systems are in operation. Containment Fuel Handling Incident Rad Monitor, RE-AR012J spikes high.

In response to this Radiation Monitor spike ... a. ALL Containment Mini-Purge Supply and Exhaust valves close and the running fans remain running. ONE train of Containment Mini-Purge Supply and Exhaust valves close and the running fans remain running. ALL Containment Mini-Purge Supply and Exhaust valves close and the running fans trip. ONE train of Containment Mini-Purge Supply and Exhaust valves close and the running fans trip.

b.

c.

d.

QUESTION: 025 (1.00) Both Units are at 100% power, with the 1A and 2B SX pumps running. The 1A SX pump trips on over current. In accordance with 1BOA Pri-7, SX Malfunction, the initial action by the Unit 1 NSO should be to... a. b. verify the 1B SX pump automatically starts. coordinate with Unit 2 NSO to start the 2A SX pump and open 1SX005 and 2SX005, the Unit 0 CC HX Inlet valves. verify open 1SX033 and 1SX034, the SX Pump Crosstie valves. manually start the 1B SX pump.

c. d.

REACTOR OPERATOR

Page 16

QUESTION: 026 (1.00) Given: Both units are operating normally at 100% power Unit 2 station air compressor (SAC) is supplying all instrument and service air needs The remaining two SACs are in standby Maintenance is being performed in the Unit 2 turbine building A workman drops a toolbox from a raised platform The dropped toolbox severs the 4-inch turbine building Service Air header

Which of the following describes the plant response? a. b. c. Only unit 2 will trip due to an immediate loss of instrument air header pressure. BOTH units will trip due to an immediate loss of instrument air header pressure. BOTH units will trip, but only after instrument air header pressure bleeds down due to instrument air loads. Only unit 2 will trip, but only after instrument air header pressure bleeds down due to instrument air loads.

d.

REACTOR OPERATOR

Page 17

QUESTION: 027 (1.00) On 5/23/1996, Byron Station Unit 1 experienced a Loss of Offsite Power (LOOP) when a fault occurred in a Bus Duct. Prior to the event: Unit 1 was in Mode 5 Unit 2 was at 100% power The Unit 2 Station Air Compressor (SAC) was Out of Service The 0C Non-Essential Service Water Pump (WS pp) was Out of Service

Two minutes after the LOOP occurred, operators manually tripped Unit 2 on decreasing steam generator levels as secondary system air operated valves began moving to their failure positions. Prior to the Unit-2 manual trip, had the Unit-2 SAC been in standby instead of Out of Service, it would ... a. b. c. need to be manually started by placing the local control switch in COMP RUN. need to be shutdown until backup cooling water from FP was manually aligned. automatically start on low Instrument Air Receiver pressure, but soon trip on HIGH OIL or AIR TEMPERATURE. automatically start on low Service Air Receiver pressure, but trip immediately on SURGE.

d.

REACTOR OPERATOR

Page 18

QUESTION: 028 (1.00) The BAR for Annunciator 2-1-B2, CNMT HATCH DOOR SEAL TROUBLE lists the 3 alarm inputs as coming from: SUPPLY AIR PRESSURE HIGH SUPPLY AIR PRESSURE LOW SUPPLY AIR FLOW HIGH.

If the CNMT HATCH DOOR SEAL TROUBLE annunciator came in due to high air FLOW and low air PRESSURE, then cleared 8 minutes later, which of the following should be considered valid? a. b. c. d. A Containment Hatch Air Lock is INOPERABLE. A Containment Hatch Air Lock Door interlock has failed. The Containment Overall Leakage Rate has increased. Personnel have just passed through the Air Lock Doors.

REACTOR OPERATOR

Page 19

QUESTION: 029 (1.00) Step 9 of BEP-1, LOSS OF REACTOR OR SECONDARY COOLANT, directs the operator to check all steam generator pressures STABLE OR INCREASING. If any steam generator pressure is DECREASING, the operator is directed to return to step 1 of BEP-1. Which of the following describes why the operator should NOT proceed past step 9 while a steam generator is still depressurizing? a. b. BEP-1 provides NO guidance for faulted steam generator isolation past step 9. The RCS cooldown rate must be under control in order for subsequent BEP-1 steps to be effectively implemented. SI Termination criteria could initially NOT be met. The crew then would be directed to BEP ES-1.2, POST LOCA COOLDOWN AND DEPRESSURIZATION which contains more restrictive termination criteria. SI Termination criteria does NOT exist in BEP ES-1.2, POST LOCA COOLDOWN AND DEPRESSURIZATION. A loop back to step 1 ensures that SI Termination criteria is met in BEP-1.

c.

d.

REACTOR OPERATOR

Page 20

QUESTION: 030 (1.00) Charging, letdown, and PZR level control system are in automatic. A&C letdown orifices are in service. The following conditions exist: Letdown Hx Outlet Flow Charging Header Flow Total seal flow to RCPs FI-132 118 gpm FI-121 127 gpm FI-142-145 30 gpm

The controlling PZR level channel fails high to an indicated 100% level. Which of the following describes the short term effect on total RCP seal injection flow, assuming NO operator action? Total seal injection flow ... a. b. c. d. decreases to 0 gpm. decreases to about 12 gpm. remains about 30 gpm. increases to about 50 gpm.

QUESTION: 031 (1.00) The emergency procedures require RCPs to be tripped during certain primary loss of coolant conditions. Prior to tripping RCPs, it must be verified that ECCS flow is occurring to the core. As long as a controlled cooldown is NOT in progress, low ____(1)____ is used to indicate the LOCA conditions and _____(2)_____ flow indication can be used to indicate adequate ECCS flow to the core. a. b. c. d. (1) PZR pressure (1) RCS pressure (1) PZR pressure (1) RCS pressure (2) High Head SI (2) Charging header (2) Charging header (2) SI pump discharge

REACTOR OPERATOR

Page 21

QUESTION: 032 (1.00) A step in BEP-1, LOSS OF REACTOR OR SECONDARY COOLANT has the crew place the Hydrogen Monitors in service per BOP PS-9, POST LOCA CONTAINMENT HYDROGEN MONITORING SYSTEM OPERATION. Which of the following actions will be necessary to obtain valid indication of the containment hydrogen concentration: 1. 2. 3. 4. 5. 6. a. b. c. d. Verify/Reset SI Verify/Reset Containment Phase A Isolation and open isolation valves Verify/Reset Containment Phase B Isolation and open isolation valves Switch the Hydrogen Monitors from OFF to ON on panel 1PM12J Place the HI/LO Range Switch to HI Wait several minutes before taking readings 1, 2, 4 2, 5, 6 2, 4, 6 3, 4, 5

QUESTION: 033 (1.00) A failure HIGH of 0RT-AR039J, the Fuel Handling Building Crane Radiation Monitor, will inhibit movement of the Fuel Handling Building Cranes ... a. b. c. d. upward motion only. downward and lateral motion. upward and lateral motion. downward motion only.

REACTOR OPERATOR

Page 22

QUESTION: 034 (1.00) The crew is preparing for a Reactor Startup. PT-507, the Main Steam Header Pressure Channel, has a range of 0 to 1500 psig:

At what potentiometer value should PK-507, the Steam Dump Main Steam Header Pressure Controller be set in order to automatically maintain NO-LOAD Tave? a. b. c. d. 6.66 turns 7.28 turns 7.38 turns 7.43 turns

QUESTION: 035 (1.00) Per 2BOA TG-8, TURBINE TRIP BELOW P8, following a Unit 2 Turbine Trip below 30% power ... a. BOTH Aux Feedwater pumps are started/verified running to ensure adequate SG levels are maintained. the control rods are required to be maintained in automatic to control primary system temperature. a manual reactor trip is required if the condenser is NOT available to maintain the heat sink. the reactor is stabilized at 25-30% power to minimize xenon transients.

b.

c.

d.

REACTOR OPERATOR

Page 23

QUESTION: 036 (1.00) An Assist NSO initiated a liquid release in accordance with BCP 400-TWX01, LIQUID RADWASTE RELEASE FORM FOR RELEASE TANK 0WX01T. A few minutes later the same NSO noted CW Blowdown flowrate is indicating 8000 gpm. The NSO contacted the Radwaste Panel to check the release status and was informed the liquid release from 0WX01T was still in progress. NO alarms are currently lit on the Center Desk annunciator panels.

Under these circumstances ... a. the release failed to automatically isolate; the NSO should initiate the manual interlock function for 0PR001J, the Liquid Radwaste Effluent Radiation Monitor from 0PM01J. a low flow alarm failed to come in on the MCB; the NSO should open 0CW018A/B, the CW Blowdown Spray valves to increase blowdown flow. the release failed to automatically isolate; the NSO should direct the Radwaste Operator to close 0WX353, the Liquid Release Isolation valve. a low flow alarm failed to come in on the MCB; the NSO should direct the Radwaste Operator to reduce release tank flow rate to 80% of the original value.

b.

c.

d.

REACTOR OPERATOR

Page 24

QUESTION: 037 (1.00) The following plant conditions exist: Instrument Air Header pressure is at 50 psig and decreasing.

0BOA SEC-4, Loss of Instrument Air, Attachment A, SAC Startup During Abnormal Conditions, directs connecting a nitrogen bottle for backup control air to the Station Air Compressor to be started. This is done because on a loss of Instrument Air, the Station Air Compressors ... a. b. c. d. Unloader Valve fails open and the Compressor Discharge Valve fails open. Unloader Valve fails closed and the Compressor Discharge Valve fails closed. Unloader Valve fails closed and the Compressor Discharge Valve fails open. Unloader Valve fails open and the Compressor Discharge Valve fails closed.

QUESTION: 038 (1.00) An NLO is performing Outside Rounds and reports to the Control Room that the door hinges for the 0B Fire Pump room are broken. In this situation, this is an impairment of... a. b. c. d. a fire door. the ventilation when the Fire Pump is running. a flood seal. a security barrier.

REACTOR OPERATOR

Page 25

QUESTION: 039 (1.00) The crew has diagnosed a Pressurizer (PZR) Vapor Space Accident. The following is the procedural flowpath followed: BEP-0, REACTOR TRIP OR SAFETY INJECTION BEP-1, LOSS OF REACTOR OR SECONDARY COOLANT BEP ES-1.2, POST LOCA COOLDOWN AND DEPRESSURIZATION

90 minutes after the reactor trip, performing the COOLDOWN and DEPRESSURIZATION of the plant per BEP ES-1.2 will result in ... a. b. c. d. a stable flowrate out the vapor space leak, and decreasing PZR level. a reduction in flowrate out the vapor space leak, and increasing PZR level. a reduction in flowrate out the vapor space leak, and PZR level offscale high. a stable flowrate out the vapor space leak, and stable PZR level.

QUESTION: 040 (1.00) Consider plant parameter response to each of the following accidents separately: Small Break LOCA Faulted Steam Generator inside Containment Steam Generator Tube Rupture

Which of the following plant parameters is used to differentiate the small break LOCA from either the Faulted Steam Generator inside Containment or the Steam Generator Tube Rupture events? Only the small break LOCA would exhibit a change in: a. b. c. d. Containment airborne activity. Pressurizer pressure. Containment pressure. Steam Generator level.

REACTOR OPERATOR

Page 26

QUESTION: 041 (1.00) Given the following: A loss of RCP seal cooling resulted in hot seal package conditions The reactor was tripped, then all RCPs were tripped The plant has been cooled down to <250F on natural circulation.

Select from the following options to complete the statement describing how the RCP seal packages will be further cooled in accordance with BOA RCP-2, Attachment A, 'RCP HOT SEAL PACKAGE COOLDOWN'. Establishing ____(1)_____, maintaining <1F/min cooldown rate until ____(2) ___ temperature is less than 235F and ____(3) _____ temperature is less than 225F. (1) a. b. c. d. seal injection; thermal barrier cooling; thermal barrier cooling; seal injection; (2) lower bearing; lower bearing; seal outlet; seal outlet; (3) seal outlet seal outlet lower bearing lower bearing

REACTOR OPERATOR

Page 27

QUESTION: 042 (1.00) The plant is operating at 100% power. An NLO reports to the control room that the charging line containment isolation valve (CV8105) has a significant bonnet leak, approx. 20 to 30 gpm. The SM directs the control room operators to isolate letdown, secure normal charging, and then place excess letdown in service. NO changes are to be made in the Seal Return flowpath. Under these conditions, excess letdown is required to: a. b. c. Restore VCT hydrogen-control capability. Permit maintenance of a constant RCS inventory/PZR level. Provide a means of purifying reactor coolant by aligning flow through the mixed-bed demineralizers. Ensure an adequate cooling water flow rate through the RCP seals while charging flow is isolated.

d.

QUESTION: 043 (1.00) While performing actions of BOA S/D-2, SHUTDOWN LOCA, the operator is directed to check RCS hot leg temperatures less than 260F prior to aligning any RH pump suction to the RWST. For example, aligning a RH pump to the RWST with suction piping temperature of 340F ... a. b. c. d. exceeds the RH pump's design temperature. may cause damage to the HUT if the RH suction relief valve were to lift. may cause steam binding at the RH pump suction when aligned to the RWST. exceeds the RH heat exchanger's heat removal capacity.

REACTOR OPERATOR

Page 28

QUESTION: 044 (1.00) Given the following: The LEVEL HIGH alarm for annunciator 1-2-A5, CC SURGE TANK LEVEL HIGH LOW came in a few minutes ago when Unit 2 started a second CC pump for a test. A Safety Injection has subsequently occurred on Unit 1. While checking the Component Cooling pumps 1A and 1B running, the operator notices annunciator 1-2-A5 CC SURGE TANK LEVEL HIGH LOW is slow flashing. The Unit 1 Component Cooling Surge Tank level is now DROPPING.

Assuming the level decrease continues, the operator should verify the following CC system response: a. Demineralized water auto makeup starts at 50%. Primary water auto makeup starts at 42%. Demineralized water auto makeup starts at 55%. Primary water auto makeup starts at 50%. Primary water auto makeup starts at 50%. Demineralized water auto makeup starts at 42%. Primary water auto makeup starts at 55%. Demineralized water auto makeup starts at 50%.

b.

c.

d.

REACTOR OPERATOR

Page 29

QUESTION: 045 (1.00) The unit is at 100% power, steady state conditions. A malfunction on the Master Pressurizer Pressure Controller has caused Reactor Coolant System pressure to deviate 80 psig from its normal setpoint. This malfunction will result in a change in ... a. b. c. d. PRT parameters. the Overpower Delta T trip setpoints. VCT level. Pressurizer Vapor Space temperature.

QUESTION: 046 (1.00) Given the following: The plant experienced a Steam Generator Tube Rupture with Loss of Reactor Coolant. A degraded ECCS has resulted in the LOSS of core heat transfer from BOTH forced circulation and natural circulation.

If vessel level continues to decrease below the hot leg penetrations and boiling occurs in the core, which of the following describes the prevalent core heat transfer mechanism? a. Condensation of vapor in the head, which is cooled by fans in containment, and draining back to the core. Slug flow via the cold legs through the loop seal and flashing across the RUPTURED and FAULTED SG U-tube break. Condensation of vapor from the bubble at the hot leg side of the INTACT SGs U-tubes which then drains back to the core via the hot legs. Partial natural circulation flow characterized by liquid pulses flowing from the cold leg over the INTACT SGs U-tubes and into the hot legs.

b.

c. d.

REACTOR OPERATOR

Page 30

QUESTION: 047 (1.00) The WOG Background Document for CA-2.1, UNCONTROLLED DEPRESSURIZATION OF ALL STEAM GENERATORS lists a "High Level Action Summary" of several actions that are to be taken to combat this event. The action to "Control Feed Flow" is intended to ___(1)___ . The action to "Terminate SI Flow" is intended to ___(2)___ . Overall the main goal during this event is maintaining ___(3)___ . a. (1) prevent overfilling the steam generators (2) conserve RWST inventory for Containment Spray usage (3) Containment Integrity (1) minimize further RCS cooldown (2) prevent repressurization of the RCS (3) RCS integrity (1) maintain steam generator levels (2) allow primary and secondary pressures to equalize (3) an adequate heat sink for decay heat removal (1) maintain a thermal stratification layer in the steam generators (2) minimize boron plate out on heat transfer surfaces (3) integrity of the steam generator U-tubes

b.

c.

d.

REACTOR OPERATOR

Page 31

QUESTION: 048 (1.00) Should a loss of Main Feedwater occur, proper Steam Generator levels can be maintained by starting the Aux Feedwater Pump(s) and controlling flow via the AF005s, the AF Pump Discharge Flow Control Valves. Depending on plant status, the flow control valves may need to be positioned from various locations such as the Main Control Board (MCB) potentiometer, the Remote Shutdown Panel (RSP) potentiometer, or locally. The methodology to control flow is different for each because: When controlled from the MCB potentiometer, AF005 positioning is based on _____(1)_____; when controlled from the RSP potentiometer, AF005 positioning is based on _____(2) ; and when controlled locally, a handwheel allows the operator to force the valve ___(3)____ upon failing the Instrument Air supply. a. (1) 0 to 100% loop flow (2) 0 to 100% valve position (3) closed (1) 0 to 100% loop flow (2) 0 to 100% valve position (3) open (1) 0 to 100% valve position (2) 0 to 100% loop flow (3) open (1) 0 to 100% valve position (2) 0 to 100% loop flow (3) closed

b.

c.

d.

REACTOR OPERATOR

Page 32

QUESTION: 049 (1.00) 1 BCA-0.0, LOSS OF ALL AC POWER is in progress. The crew is performing the step to crosstie Bus 141 to Bus 241. The following breakers have been placed in PULL OUT: ACB 1411, the Non-ESF bus tie ACB 1412, the SAT feed ACB 1414, the Reserve feed

Which ONE of the following annunciators COULD be LIT and still allow Bus 141 to be crosstied: a. b. c. d. 1-21-A7, BUS 141 FD BRKR 1412 TRIP 1-21-C7, BUS 141 OVERLOAD OR VOLT LOW 1-21-B8, BRKR 1414 CROSS-TIE OVERCURRENT 1-21-B9, DG 1A OVERLOAD

REACTOR OPERATOR

Page 33

QUESTION: 050 (1.00) Given the following: Unit 1 has experienced a SAT fault that resulted in a Loss of Offsite Power (LOOP). Unit 1 is in Mode 3 at 557F and 2235 psig. 1BOA ELEC-4, 'LOSS OF OFFSITE POWER' is being performed. Attachment C, 'RESTORATION OF RCS PRESSURE CONTROL' is being performed to restore PZR Heaters.

Assuming NO jumpers were used to defeat interlocks, which of the following describes an alignment that would energize a PZR Backup Heater Group: a. Bus 143 crosstied to bus 141 that is energized by ___(1)___ . Closed associated 4KV heater transformer high and low side breakers. Heater group contactor control switch position of ___(2)___ . (1) 1A D/G (2) 'ON' (1) Bus 241 (2) 'ON' (1) 1A D/G (2) 'AUTO' (1)Bus 241 (2) 'AUTO'

b.

c.

d.

REACTOR OPERATOR

Page 34

QUESTION: 051 (1.00) Given: Unit 1 is in Mode 3 at 557F. Annunciator 1-21-E10 "125V DC PNL 111/113 VOLT LOW" alarms. Annunciator 1-4-A5 "BUS 111 INVERTER TROUBLE" alarms. Annunciator 1-4-C5 "BUS 113 INVERTER TROUBLE" alarms. Power Range Channel N-43 Drawer is dark (Deenergized).

Which of the following is indicated by the above? a. b. c. d. DC bus 113 is deenergized; only instrument bus 113 is deenergized. DC bus 113 is deenergized; instrument bus 111 and 113 are deenergized. DC bus 111 is deenergized; instrument bus 111 and 113 are deenergized. DC bus 111 is deenergized; only instrument bus 111 is deenergized.

QUESTION: 052 (1.00) The AF-2 breaker (125 VDC Feed from Battery) to DC Bus 211 has tripped and will not reclose. From the below selections, choose which correctly describes the alternate flowpath used to reenergize DC bus 211. a. Bus 141 to 131X, thru battery charger 111 to battery and DC bus 113, thru bus 113 crosstie breaker, thru bus 211 crosstie breaker. Bus 242 to 232X, thru battery charger 212 to battery and DC bus 212, thru bus 212 crosstie breaker, thru bus 211 crosstie breaker. Bus 241 to 231X to 231X2, thru bus 211 inverter to DC bus 211. Bus 141 to 131X, thru battery charger 111 to battery and DC bus 111, thru bus 111 crosstie breaker, thru bus 211 crosstie breaker.

b.

c. d.

REACTOR OPERATOR

Page 35

QUESTION: 053 (1.00) Given the following: 1BOA PRI-7, ESSENTIAL SERVICE WATER MALFUNCTION is in progress. Attachment A is being used to perform a SX system crosstie between the units.

The procedural direction is to verify/open the respective RCFC SX Supply and Return valves, 1SX016_ and 1SX027_, prior to starting the standby SX pump. The reason for having these valves open is ... a. b. c. d. to ensure a flowpath is available for containment chiller cooling. that they provide the crosstie flowpath. that they are part of the starting interlocks for the pump. to ensure a flowpath is available for Diesel Generator cooling.

QUESTION: 054 (1.00) Given the following Unit 2 plant conditions: Unit 2 was in Mode 3 at 2235 psig and 557F with primary system and secondary system parameters being controlled automatically. The Instrument Air (IA) Header became depressurized and the crew entered 0/1/2BOA SEC-4, 'LOSS OF INSTRUMENT AIR'. During the loss of IA, the Unit 2 operators performed actions directed by 2BOA SEC-4. The IA problem has been corrected and IA Header pressure is now 50 psig and INCREASING.

Which of the following MCB controllers should be placed in MANUAL with 0% demand to prevent an undesired transient? a. b. c. d. 2AF005A thru H, Aux Feedwater Flow Controllers 2RY455B and C, PZR Spray Valve Controllers 2CC130A/B, Letdown Heat Exchanger Temperature Control Valve Controller 2CV121, Centrifugal Charging Pump Flow Controller

REACTOR OPERATOR

Page 36

QUESTION: 055 (1.00) The following conditions exist on Unit 1: A loss of coolant accident has occurred. RWST Level is 35% and DECREASING. 1BCA-1.1, LOSS OF EMERGENCY COOLANT RECIRCULATION is in progress. 1B RH pump has TRIPPED on overcurrent. Attempts are being made to establish Cold Leg Recirculation capability. SI has been RESET. The NSO is questioning the ECCS valve alignment.

Which of the following is PREVENTING 1SI8811A, Train A SI Recirc Sump Isolation valve from being MANUALLY OPENED? a. b. c. d. 1SI8812A, Train A RWST to RH Suction valve is CLOSED. 1SI8812A, Train A RWST to RH Suction valve is OPEN. 1CS001A, Train A RWST to CS Suction valve is CLOSED. 1CS009A, Train A Containment Recirc Sump to CS Suction valve is OPEN.

QUESTION: 056 (1.00) Given the following: 1BFR-H.1, RESPONSE TO LOSS OF SECONDARY HEAT SINK is in progress. The Operators are performing step 4, trying to establish Aux Feedwater to at least one S/G.

Per the Operator Action Summary (OAS) page, which one of the following conditions would result in the need to jump ahead in the procedure to initiate Bleed and Feed? a. b. c. d. Low PZR pressure Low S/G Narrow Range level NO Centrifugal Charging pumps available Increasing loop Delta-T

REACTOR OPERATOR

Page 37

QUESTION: 057 (1.00) Per 2BOA REFUEL-2, REFUELING CAVITY OR SPENT FUEL POOL LOSS, in order to initiate evacuation of non-essential personnel from affected areas: _____(1)_____ if the affected area is the containment (CNMT). _____(2)_____ if the affected area is the Fuel Handling Building (FHB). a. (1) (2) b. (1) (2) Actuate the CNMT evacuation alarm from the MCB and make an announcement over the plant page Make an announcement over the plant page Make an announcement over the plant page Actuate the FHB evacuation alarm from the MCB and make an announcement over the plant page Insure the CNMT evacuation alarm has automatically sounded and contact the CNMT Coordinator to initiate the evacuation. Insure the FHB evacuation alarm has automatically sounded and make an announcement over the plant page Insure the CNMT evacuation alarm has automatically sounded and make an announcement over the plant page Insure the FHB evacuation alarm has automatically sounded and contact the Fuel Handling Supervisor to initiate the evacuation.

c.

(1) (2)

d.

(1) (2)

REACTOR OPERATOR

Page 38

QUESTION: 058 (1.00) Given the following plant conditions: Unit 1 is at full power. Based on primary and secondary plant indication, 1BOA SEC-8, STEAM GENERATOR TUBE LEAK, has been entered. 1CV121, the CV Pumps Discharge Header Flow Control Valve, and 1CV182, the Seal Injection Flow Control Valve, have been throttled full open. Pressurizer level is still DROPPING.

Prior to manually initiating a reactor trip and SI, which of the following actions is acceptable in an attempt to maintain Pressurizer level? a. b. c. d. Start a Second Charging Pump. Establish 75 GPM Letdown. Begin a Unit Load Reduction. Open 1SI8801A and B, the Cold Leg Injection Valves.

QUESTION: 059 (1.00) Which of the following Process Radiation Monitors has an INTERLOCK FUNCTION designed to prevent an accidental offsite liquid radwaste release? a. b. c. d. 0RE-PR010, the Station Blowdown Rad Monitor 1RE-PR002, the RCFC 1A and 1C SX Outlet Rad Monitor 1RE-PR011, the Containment Atmosphere Rad Monitor 0RE-PR041, the Condensate Polisher Sump Discharge Rad Monitor

REACTOR OPERATOR

Page 39

QUESTION: 060 (1.00) Given the following: Both units were manually tripped from mode 1. The Control Room has been evacuated, 0/1/2 BOA PRI-5, CONTROL ROOM INACCESSIBILITY procedures have been entered and local control has been established from the respective Units Remote Shutdown Panel (RSP).

It has been decided to place both units in Mode 4. To prevent an automatic SI, it will be necessary to block the ... a. Steamline Low Pressure SI by depressing the block pushbuttons at the RSP prior to Main Steamline pressure reaching 640 psig. Pressurizer Low Pressure SI by depressing the block pushbutton at the RSP prior to Pressurizer pressure reaching 1930 psig. Steamline Low Pressure SI by placing jumpers in the SSPS cabinets prior to Main Steamline pressure reaching 640 psig. Pressurizer Low Pressure SI by placing jumpers in the SSPS cabinets prior to Pressurizer pressure reaching 1930 psig.

b.

c.

d.

QUESTION: 061 (1.00) Meeting LCO 3.5.2, ECCS-OPERATING, helps to ensure the ECCS Acceptance Criteria (10 CFR 50.46) of 2200F peak cladding temperature will NOT be exceeded during a LOCA. The limit of 2200F is selected as this is ... a. b. c. d. 500F below the clad melt point of 2700F. 1000F below the fuel melt temperature of 3200F. the temperature above which thermal conductivity of the clad decreases significantly. the temperature above which the zircaloy-water reaction is greatly accelerated.

REACTOR OPERATOR

Page 40

QUESTION: 062 (1.00) Given the following plant conditions for Unit 1: A LOCA has occurred. The crew manually tripped the Reactor, manually actuated Safety Injection, entered and performed 1BEP-0, REACTOR TRIP AND SAFETY INJECTION. A transition was made to 1BEP-1, LOSS OF REACTOR OR SECONDARY COOLANT. The crew is now in 1BEP ES-1.1, SI TERMINATION, preparing to perform the step to REALIGN CENT CHG PUMPS(s) when the following was observed: RCS pressure: 1250 psig SI Pump 1A flow: 0 gpm SI Pump 1B flow: 150 gpm

What is the status of the ECCS? a. SI Pump 1A flow is LOWER than expected and SI Pump 1B flow is HIGHER than expected. SI Pump 1A flow is LOWER than expected and SI Pump 1B flow is AS expected. SI Pump 1A is indicating the expected flow rate with its miniflow valve OPEN. SI Pump 1B is indicating the expected flow rate with its miniflow valve CLOSED. BOTH SI Pumps are reading LESS THAN the expected flow.

b. c.

d.

REACTOR OPERATOR

Page 41

QUESTION: 063 (1.00) 1BFR-H.2, RESPONSE TO STEAM GENERATOR OVERPRESSURE, contains a CAUTION: " If AFFECTED SG narrow range level increases to greater than 93% (81% ADVERSE CNMT), steam should NOT be released from the AFFECTED SG(s)". WHICH of the following is the reason for this caution? a. To prevent moisture separator damage due to two phase flow from an overfilled steam generator. To prevent damage to valves and piping from water hammer if steam is released from the steam generator. Steam flow indication would be inaccurate due to moisture collection in steam flow transmitters. Steam flow would have a high moisture content and could cause inaccurate main steam line radiation readings.

b.

c.

d.

QUESTION: 064 (1.00) During various Design Basis Accidents (DBAs), water is collected in the containment sump so it is available for long term core and/or containment cooling via the emergency core cooling or containment spray recirculation system. During a major accident, indications of water addition to the containment water volume from which of the following sources is UNEXPECTED per BFR-Z.2, RESPOND TO CONTAINMENT FLOODING? a. b. c. d. Condensate Storage Tank Safety Injection Accumulators Primary Water Storage Tank Refueling Water Storage Tank

REACTOR OPERATOR

Page 42

QUESTION: 065 (1.00) Assume the following plant conditions exist on Unit 1: The unit tripped from 100% power when a switchyard failure caused a loss of offsite power. 1BEP ES-0.2, NATURAL CIRCULATION COOLDOWN is in progress to perform a natural circulation cooldown and depressurization of the RCS.

For which of the following situations should 1BEP ES-0.3, NATURAL CIRCULATION COOLDOWN WITH STEAM VOID IN VESSEL (WITH RVLIS) be used instead of 1BEP ES-0.2? a. NO RCPs will be able to be restarted prior to cooling down the plant to less than 200F. Normal Pressurizer Spray is unavailable for use in depressurizing the RCS. The required high rate of plant cooldown and depressurization results in less than 100% level in the vessel head. The SI Accumulators are unable to be isolated.

b. c.

d.

QUESTION: 066 (1.00) Consider each of the following situations individually. Assume Mode 1 operation. Which of the following has a Completion Time of "Immediately" for the Tech Spec Required Action of "Initiate action to restore one train to OPERABLE status"? a. b. c. d. 2 ECCS Trains are inoperable. 2 AF Trains are inoperable. 2 CS Trains are inoperable. 2 VC Filtration Trains are inoperable.

REACTOR OPERATOR

Page 43

QUESTION: 067 (1.00) An NSO has been tasked to perform an activity that involves directing NLOs in the field to manipulate equipment. Which of the following is the expected method of ensuring the most recent revision of the procedure is being used to perform the activity? Use the ... a. b. c. d. NLOs field copy. Electronic Central Files (ECF) copy. Electronic Document Management System (EDMS) copy. spare copy that was printed out the last time the task was performed.

QUESTION: 068 (1.00) BGP 100-3, POWER ASCENSION has a Turbine Generator Limitation and Action to ensure that, when within 5% of a power limit, MW OUT is to be selected at the DEHC panel. What is the basis for the direction of operating with MW OUT? Operating in MW OUT mode REDUCES ... a. turbine Governor Valve oscillations that would occur from changing main steam pressure. turbine load changes from grid frequency perturbations. reactor power changes since generator output is held constant. the chances of exceeding power limits due to degrading secondary efficiency.

b. c. d.

REACTOR OPERATOR

Page 44

QUESTION: 069 (1.00) BOP RC-1, STARTUP OF A REACTOR COOLANT PUMP, has a Limitation and Action for starting an RCP with RCS cold leg temperatures less than 350F in that the temperature in the Steam Generators must be less than 50F above RCS temperature. This Limitation and Action is designed to ... a. b. c. d. prevent an overpressure event of the RCS. prevent excessive cooldown rate of the Steam Generators. maintain RCP seal parameters within normal operating range. ensure sufficient net positive suction head at suction of the RCP.

QUESTION: 070 (1.00) An Estimated Critical Condition (ECC) has been calculated for a Beginning of Life (BOL), Negative Moderator Temperature Coefficient (- MTC) reactor startup that is to be performed 5 hours after a trip from a 60 day full power run. Which of the following events or conditions will result in the ACTUAL critical control rod height being LOWER than the PREDICTED control rod height in the ECC? (Consider each item separately) a. b. The startup is delayed 2 hours due to a passing thunderstorm. The Nuclear Engineer used the EOL Integrated Rod Worth Curve instead of the BOL Curve for the ECC calculation. A new boron sample shows a current boron concentration 20 ppm higher than that used in the ECC calculation. Main Steam Header pressure is decreased by 100 psi just prior to criticality.

c.

d.

REACTOR OPERATOR

Page 45

QUESTION: 071 (1.00) 2BGP 100-3, POWER ASCENSION for Unit 2 has a step to open 2FW039A, B, C, and D (Feedwater Preheater Bypass Valves ) at 80% power. Why does 1BGP 100-3 POWER ASCENSION for Unit 1 NOT direct a similar action? 1FW039A, B, C, and D are ... a. opened earlier in the power ascension when feedwater control is switched to the Feed Reg Valves. only open at low power when it is necessary to close the respective 1FW009 valve. NOT allowed to be opened while in Mode 1. normally open when tempering flow is required.

b.

c. d.

QUESTION: 072 (1.00) During repair of a Reactor Coolant Filter connection, radiation levels suddenly increased from 15 mRem/hr to 400 mRem/hr. The worker ... a. recognizes the increased radiation levels, but continues the repair since levels are less than 1 Rem/hr and is NOT an ALARA concern. identifies the increasing general area dose rates and immediately leaves the area to inform the Radiation Protection Department based on ALARA concerns. continues the work since the repair is covered under a general RWP, and the estimated time of exposure for the job is 4 hours. continues the work because the Radiation Protection Department is scheduled to perform a survey of the job site in 4 hours AND the ALARA review was performed prior to commencing the repair work.

b.

c.

d.

REACTOR OPERATOR

Page 46

QUESTION: 073 (1.00) The NRC Legal Radiation Exposure limit for an Adult Occupational Worker at Byron Station is ___(1)___ TEDE. The Exelon Exposure limit for an Emergency Worker to PROTECT VALUABLE PROPERTY should the Byron site enter the Emergency Plan due to an accident event, and with proper authorization, is ___(2)___ TEDE. a. (1) 5 Rem/year (2) 10 Rem (1) 5 Rem/year (2) 25 Rem (1) 2 Rem/year (2) 10 Rem (1) 2 Rem/year (2) 25 Rem

b.

c.

d.

QUESTION: 074 (1.00) While operating at 50% power, a scan of the Main Control Board reveals the following: Reactor Power = 50% RCS Tavg = 575F PZR Press. = 1860 psig PZR Level = 40% Rod Control = MANUAL

Which ONE of the following is the correct procedure to enter based on the above information? a. b. c. d. BOA PRI-1, 'EXCESSIVE PRIMARY PLANT LEAKAGE'. BOA PRI-12, 'UNCONTROLLED DILUTION'. BEP-1, 'LOSS OF REACTOR OR SECONDARY COOLANT'. BEP-0, 'REACTOR TRIP OR SAFETY INJECTION'.

REACTOR OPERATOR

Page 47

QUESTION: 075 (1.00) Which ONE of the following statements completes the following sentence? Concerning usage rules of LOW-LEVEL STEPS in Emergency Procedures,_________are used to signify those steps that must all be completed, but can be in any order. a. b. c. d. closed bullets lower case letters open bullets diamonds

(********** END OF EXAMINATION **********)

REACTOR OPERATOR ANSWER: 001 (1.00) B REFERENCE: ILT Fundamentals Lesson Plan Power Distribution Chapter 3, Thermal Limits; page 15-16. I1-PD-XL-03 modified High 003000K501 ..(KAs) ANSWER: 005 (1.00) A REFERENCE: P & ID M-60, sheet 6 & M-70 sheet 1 BOP RY-3, page 3 note: BOP RY-4 page 2 BAR 1-12-B7 Bank High 007000A102 ..(KAs)

Page 48 ANSWER: 009 (1.00) B REFERENCE: ILT Systems LP Ch 61, pages 15-16, 21 ILT Systems Ch 40, pages 31-32 New High 013000A302 ..(KAs)

ANSWER: 002 (1.00) C REFERENCE: BOP AB-E1 page 3 1BOA PRI-2, EMERGENCY BORATION bank High 004000K201 ..(KAs)

ANSWER: 006 (1.00) A REFERENCE: ILT Systems LP Ch 19, page 32 ILT Systems LP Ch 13, page 20 & 21 New High 008000K303 ..(KAs)

ANSWER: 010 (1.00) D REFERENCE: 2BEP-0, page 5 Bank High 013000 2.4.49 ..(KAs)

ANSWER: 003 (1.00) B REFERENCE: ILT Systems LP Ch 18 page 12; ILT Systems Ch 53 page 52 Bank High 005000K603 ..(KAs)

ANSWER: 007 (1.00) D REFERENCE: ILT Systems LP Ch 14, pages 8 - 10 Bank High 010000A401 ..(KAs)

ANSWER: 011 (1.00) C REFERENCE: BOP VP-E1A page 3, BOP VP-E1B page 3 New Low 022000K201 ..(KAs)

ANSWER: 004 (1.00) C REFERENCE: 2BEP ES-1.3 steps 5 and 8; Horsenote ECCS-2 WOG ERG background for ES-1.3, page 4 New High 006000A201 ..(KAs)

ANSWER: 008 (1.00) B REFERENCE: ILT Systems LP Ch 60b, pages 13 thru 16 Bank High 012000K406 ..(KAs)

ANSWER: 012 (1.00) D REFERENCE: LCO 3.6.5, SR 3.6.5.1 2BOSR 0.1-1,2,3 page D12 Bank High 022000 2.1.33 ..(KAs)

ANSWER: 013 (1.00) B REFERENCE: ILT Systems Ch 59, page 11 New High 026000A104 ..(KAs)

REACTOR OPERATOR ANSWER: 014 (1.00) A REFERENCE: steam tables ILT Systems LP Ch 23 page 9-10 bank High 039000A105 ..(KAs) ANSWER: 018 (1.00) A REFERENCE: ILT Systems LP Ch 13, page 18-19 ILT Systems LP Ch 4, pages 34-37 New High 061000K412 ..(KAs)

Page 49 ANSWER: 022 (1.00) C REFERENCE: BEP ES-0.1 step 6b RNO (page 8) for direction to trip 1D RCP for this situation. ILT Systems LP Ch 13, page 18 for RCP control power BOA ELEC-5, Symptoms or Entry Conditions (page 1), and Attachment A (page 6). Bank High 2.4.31 063000 ..(KAs)

ANSWER: 015 (1.00) D REFERENCE: ILT Systems Ch 24, page 18-19 ILT Fundamentals Ch RT3 BAP 300-1, OP-AA-100, Conduct of Operations Manual, Byron Addendum C.1.h on page 4. New High 039000A205 ..(KAs)

ANSWER: 019 (1.00) A REFERENCE: ILT Procedure LP page 57 for BEP ES-0.1 step 2 WOG Background Document for ES-0.1 step 1 Modified Low 061000A302 ..(KAs)

ANSWER: 016 (1.00) C REFERENCE: ILT Systems LP Ch 25, page 48-49, 55 bank Low 056000K103 ..(KAs)

ANSWER: 020 (1.00) A REFERENCE: ILT Systems LP Ch 9, page 45 Horse Note DG-2 bank High 062000K102 ..(KAs)

ANSWER: 023 (1.00) C REFERENCE: ILT Systems LP Ch 9, pages 28-29, 33-34 BOA ELEC-3, pages 2, 27-28 6E-2-4004B, 6E-2-4021A New High 064000A202 ..(KAs)

ANSWER: 017 (1.00) D REFERENCE: ILT Systems LP Ch 37b, page 34 New High 059000A401 ..(KAs)

ANSWER: 021 (1.00) A REFERENCE: ILT Systems Ch 8a, pages 5-6, 13 Bank Low 063000K103 ..(KAs)

ANSWER: 024 (1.00) D REFERENCE: ILT Systems LP Ch 42, page 52-53 ILT Systems LP Ch 49, page 30 Bank Low 073000K301 ..(KAs)

ANSWER: 025 (1.00) D REFERENCE: 1BOA Pri-7, page 2, step 1 RNO Bank Low 076000A401 ..(KAs)

REACTOR OPERATOR ANSWER: 026 (1.00) C REFERENCE: ILT Systems LP Ch 53, pages 55, 57, TPs 4 & 5 Based on actual Braidwood event Bank High 078000K102 ..(KAs) ANSWER: 030 (1.00) B REFERENCE: ILT Systems Ch 14 pages 50-51, Ch 15a pages 29-30 Bank High 011000K302 ..(KAs)

Page 50 ANSWER: 035 (1.00) C REFERENCE: 2BOA TG-8 LP for steps 2, 3, 4 Bank High 045000K518 ..(KAs)

ANSWER: 027 (1.00) B REFERENCE: 0BOA SEC-5, step 3 RNO (page 3 of 13) ILT Systems LP Ch 53, pages 67, 69 New High 078000K403 ..(KAs)

ANSWER: 031 (1.00) D REFERENCE: BEP-0 step 26/page 23 or OAS New Low 016000K107 ..(KAs)

ANSWER: 036 (1.00) C REFERENCE: BAR 0PL01J-7-B6 (Radwaste Panel) BCP 400-TWX01, page 23 Modified High 068000A204 ..(KAs)

ANSWER: 028 (1.00) D REFERENCE: BAR 2-1-B2 SR 3.6.2.1 Note 1 New High 103000K105 ..(KAs)

ANSWER: 032 (1.00) C REFERENCE: BEP-1 steps 11c/d on page 17 BOP PS-9 pages 2-3 New Low 028000A403 ..(KAs)

ANSWER: 037 (1.00) D REFERENCE: ILT Systems LP Ch 53, page 48 Bank Low 079000K401 ..(KAs)

ANSWER: 029 (1.00) C REFERENCE: Westinghouse Background Document for EP-1, step 9 (page 86) Bank High 002000 2.4.6 ..(KAs)

ANSWER: 033 (1.00) A REFERENCE: ILT Systems LP Ch 49, page 27 Bank Low 034000K602 ..(KAs)

ANSWER: 038 (1.00) A REFERENCE: BAP 1100-3A3 page 20 New Low 086000A103 ..(KAs)

ANSWER: 034 (1.00) B REFERENCE: ILT Systems Ch 24, pages 6-7 New High 041000A305 ..(KAs)

ANSWER: 039 (1.00) C REFERENCE: WOG ERG ES-1.2, pages 47, 54, 73, 74, 76 New High 000008K102 ..(KAs)

REACTOR OPERATOR ANSWER: 040 (1.00) A REFERENCE: BEP-0, steps 27, 28, 29 New High 000009A210 ..(KAs) ANSWER: 045 (1.00) D REFERENCE: ILT Systems LP CH 14, page 2 New High 000027K101 ..(KAs)

Page 51 ANSWER: 050 (1.00) A REFERENCE: ILT Systems Ch 14 pages 8-9 ILT Systems Ch 4 page 89 BOA ELEC-4, pages 8 & 24 New High 000056A1.3 ..(KAs)

ANSWER: 041 (1.00) C REFERENCE: BOA RCP-2, Attachment A, pages 2 thru 5 of 5 Bank Low 000017A207 ..(KAs)

ANSWER: 046 (1.00) C REFERENCE: Critical Safety Function Analysis Text, Module 2, Lesson 2, page 2-18 Modified High 000038K104 ..(KAs)

ANSWER: 042 (1.00) B REFERENCE: ILT Systems LP Ch 15a, page 34 Bank High 000022K303 ..(KAs)

ANSWER: 047 (1.00) B REFERENCE: WOG Background Document for ECA-2.1, pages 3, 17-20 New High 00WE12K3.1 ..(KAs)

ANSWER: 051 (1.00) A REFERENCE: BAR 1-21-E10 BAR 1-4-A5 and C5 1BOA ELEC-2, page 1 Symptoms or Entry Conditions for loss of an instrument bus. Horse Note I&C-5 Bank High 000057A204 ..(KAs)

ANSWER: 043 (1.00) C REFERENCE: BOA S/D-2, Attachment D, Note prior to Step 1 Bank Low 000025K101 ..(KAs)

ANSWER: 048 (1.00) A REFERENCE: ILT Systems LP Ch 26, pages 32-33 Bank Low 000054K303 ..(KAs)

ANSWER: 052 (1.00) D REFERENCE: Horsenote AC-7 Bank Low 000058A101 ..(KAs)

ANSWER: 044 (1.00) A REFERENCE: BAR 1-2-A5 Bank Low 000026 2.4.50 ..(KAs)

ANSWER: 049 (1.00) B REFERENCE: BCA-0.0, step 9 New High 000055 2.4.50 ..(KAs)

ANSWER: 053 (1.00) C REFERENCE: ILT Systems LP Ch 20 page 33 New Low 000062K303 ..(KAs)

REACTOR OPERATOR ANSWER: 054 (1.00) B REFERENCE: 1/2 BOA SEC-4 pages 2 thru 6 Modified High 000065 2.1.23 ..(KAs) 0 0 0 0 3 7 2 . 1. 3 2 . . ( K A s )

Page 52 ANSWER: 059 (1.00) D REFERENCE: TRM Table 3.11.a, pages 3.11.a-9 thru 11 or ILT Systems Chapter 49 LP, page 36 New Low 000059K201 ..(KAs)

ANSWER: 055 (1.00) B REFERENCE: ILT Systems LP Ch 18, page 11 Bank Low 00WE11K2.1 ..(KAs)

ANSWER: 056 (1.00) C REFERENCE: 1BFR-H.1 OAS page Ne w Low 00WE05 2.4.49 ..(KAs)

ANSWER: 060 (1.00) C REFERENCE: BOA PRI-5 pages 28-30, 61-62 ILT Systems LP Ch 61 pages 15-16 New Low 000068K314 ..(KAs)

ANSWER: 057 (1.00) A REFERENCE: BOA REFUEL-2 step 1a New Low 000036A103 ..(KAs)

ANSWER: 061 (1.00) D REFERENCE: ILT Systems LP Ch 58, pages 3-4 ILT Fundamentals LP Ch PD1 TS Bases B 3.5.2 page 4 Bank Low 0 0 0 0 7 4 2 . 2. 2 5 . . ( K A s )

ANSWER: 058 (1.00) B REFERENCE: BOA SEC-8 step 1b RNO (page 2) Modified Low

ANSWER: 062 (1.00) D REFERENCE: ILT Systems LP Ch 58, page 9-10 Bank High 00WE02K1.1 ..(KAs)

ANSWER: 063 (1.00) B REFERENCE: WOG Background Document for Caution prior to step 4 of FR-H.2 (page 16 of Document) Bank Low 00WE13K3.3 ..(KAs)

ANSWER: 064 (1.00) C REFERENCE: WOG Background Document for FR-Z.2, page 2 & 8 New Low 00WE15A1.3 ..(KAs)

ANSWER: 065 (1.00) C REFERENCE: 1BEP ES-0.2 Note at top of page 8, step 14 New Low 00WE09A2.1 ..(KAs)

ANSWER: 066 (1.00) B REFERENCE: LCO 3.7.5 Condition C on page 3.7.5-1 New Low 194001 2.1.11 ..(KAs)

REACTOR OPERATOR ANSWER: 067 (1.00) C REFERENCE: HU-AA104-101, Procedure Use and Adherence", page 2. New Low 194001 2.1.21 ..(KAs) ANSWER: 070 (1.00) D REFERENCE: BCB Fig 8c for distractor A BCB Fig 2 for BOL & EOL for distractor B BCB Table 1-5 for distractor C BCB Fig 5A for answer D Bank High 194001 2.2.34 ..(KAs)

Page 53 ANSWER: 073 (1.00) A REFERENCE: RP-AA-203, page 2 and page 7 New Low 194001 2.3.4 ..(KAs)

ANSWER: 068 (1.00) D REFERENCE: ILT Systems LP Ch 37a, page 22 New High 1 9 40 01 2. 1. 3 2 . . ( K A s)

ANSWER: 069 (1.00) A REFERENCE: ILT Systems LP Ch 13, page 31for discussion of LCO 3.4.6 Note. Bank Low 194001 2.2.25 . .( KAs)

ANSWER: 071 (1.00) B REFERENCE: 1BGP 100-3, step F.40 (pages 51-53) New Low 194001 2.2.3 ..(KAs)

ANSWER: 074 (1.00) D REFERENCE: BEP-0, page 1, SYMPTOMS OR ENTRY CONDITIONS BAR 1-11-C3 Bank High 194001 2.4.2 ..(KAs)

ANSWER: 072 (1.00) B REFERENCE: RP-AA-1002 Reference: Bank Low 194001 2.3.2 ..(KAs)

ANSWER: 075 (1.00) A REFERENCE: BAP 1310-10, HU-AA-104-101, Procedure Use and Adherence, Byron Addendum" page 13 New Low 194001 2.4.19 ..(KAs)

(********** END OF EXAMINATION **********)

REACTOR OPERATOR

Page 54

ANSWERKEY MULTIPLE CHOICE

001

016 C 017 D 018 A 019 A 020 A 021 A 022 C 023 C 024 D 025 D 026 C 027 B 028 029 C 030 B

031 D 032 C 033 A 034 B 035 C 036 C 037 D 038 A 039 C 040 A 041 C 042 B 043 C 044 A 045 D

046 C 047 B 048 A 049 B 050 A 051 A 052 D 053 C 054 B 055 B 056 C 057 A 058 B 059 D 060 C

061 D 062 D 063 B 064 C 065 C 066 B 067 C 068 D 069 A 070 D 071 B 072 B 073 A 074 D 075 A

002 C 003 B 004 C 005 A 006 A B 2 007 D S 3 008 B 009 B 010 011 012 013 D C D B

014 A 015 D

INITIAL SUBMITTAL
MCGUIRE EXAM 2000-301 50-369/2000-301 AND 50-370/2000-301 MAY 8 - 12, MAY 19, MAY 22 - 25, 2000

INITIAL SUBMITTAL RO WRITTEN EXAMINATION

14

A i0.1 E 0, !C ie 7 L' 0_54 0 0

McGuire Sample Plan

ES-401

PWR RO Examination Outline

ES-401-4

Facility: McGuire K 1 2 2 1 K 2 2 3 0 K 3 4 4 0

Date of Exam: 5/19/00 K/A Catego K K 4 5 Points K A 6 _1 3 2 0 A 2 2 4 1 A 3

Exam Level: RO A 4 Point G 3 2 1 Total 16 17 3 36 23 20 8 Target 16 17 3

Tier 1 Emergency & Abnormal Plant Evolutions

Group 1 2 3 Tier Totals 1

5 1 2 1

5 1 1 1

8 2 3 1 3 3 0 2 1 1 2 1 0

5 3 2 0

7 3 3 2 3 1 0 2 2 1

6 1 1 1 -

36 23 20 8

2 Plant Systems

2 3 Tier Totals

51 4 3 6 6 Cat 1 3 4 3 Cat 2 4 5 8 Cat 3 3 4 5 Cat 4 3 13 3 51 13 Totals:

3 Note:

Generic Knowledge and Abilities

* Attempt to distribute topics among all K/A categories; select at least one topic from every K/A category within each tier. * Actual point totals must match those specified in the table. * Select topics from many systems; avoid selecting more than two or three K/A topics from a system unless they relate to plant-specific priorities. * Systems (evolutions within each group are identified on the associated outline. * The shaded areas are not applicable to the category/tier. ** Denotes plant specific, high priority K/As

Summary

For Official Use Only

3/21/00

1 ES-401 11 a Emarioncy and Abnormal Mont EvoAnions - 714r 1/Group 1 KM Toplc(s) Imp.

IKK K A A 0 EJAPE 0 I Name I Safety Function I .1_ ,3 Ability to interpret control room indications to verify the status and operation of system 3.5/3.6 1 G O 3 ..... 1 1 2 Points 0

000005 Inoperable/Stuck Control Rod P_4.46 and how operator actions and directives affect plant system mations

000015117 RCP Malfunction 1 IV Knowledge of the operational implications of the following concepts as they apply to Kwee nldo oe f gt h orn pa el a o i t mts i pnte cf ii h lao o

2.09

Ability to determine and interpret...when to secure RCP5 on high stator temperatures 34/3.51

000024 Emergency Bonbon it , 1.02 000025 Loss of Component Cooling Water I VIII, between boron addition and reactor power . 1 424 Knowledge of loss of Coding water procedures
O

If

as '

W/E09 Natural Circ. 11V

1_1

the...components, capacity end function of emergency systems

3.0/3.4 _

3.6/3.9

... 000027 Pressurizer Pressure Control System Malfunction/III . , 2.10 Ability to determine and interpret. ..P2R heeler energixed/dt-energized condition3.3/3.6 1

33/33

000040 Steam Line Rupture - Excessive Heat Transfer / IV , Knowledge of the Interrelationships betweencomponents and functions of control and safety systems including Instrumentation, signals, interlocks, failure modes. and 2.1 Knowledge of the reasons for the following responses ...loss of steam dump 000051 Loss of Condenser Vacuum I W Knowledge of the reasons for the following responses ...actions contained in EOP for 00 tn 05t 0S 5a o i 3.01 capability upon loss of condenser vacuum 21/3.1* 1
6
a

1,05

VLIE05 RCS Overcooling . PTS I IV

automatic and manual features

A b y t l io

3.4/17

0000132 Lass of Nuclear Service Water I IV


F' A

Knowledge artful reasons for the following responses ...ircianti contained in EOP for plant fire omits Knowledge of the reasons for the following responses ...maintenance of SG levels , , using AFW control valves Knowledge of the Interrelationships between,.. personnel access hatch and 4.0/4.3 1 13/4.1 1_
3

000057 Plant Fire On-stie I IX 000065 Control Room Evac. / V111 , 3.07

3.04

000059 (W/E14) Loss of CTMT Integrity I V2.CI3

r 000076 H1.11 Reactor CoolaM Activity/ IX

4.11 Knowledge of abnormal condition procedures

34/35

c o co __

900074 (VIE0433E07) Mad. Core Cooling I IV

, 1.02

, Ability to operate and/or monitor ...RCS cooldown rate

3.9/4.2

emergency access hatch

2.13'/2.0

wild *twos lumen,/ PWR RO Examination Outline Fonn ES-401-3

McGuire Sample Plan

1 0 4 E S PWR RO Examination Outfit,' 2 Emergency and Abnormal Plant Evolutions - Tier 1/Group 2
K EIAPE i i Name I Safety Function 1 K 2 K 3 A 1 A 2 11 KEA Topictsit

Form ES-4014
1= 11 111iIMI

Bank
Imp, Points Question

000001 Continuous Rod Withdrawal I I

1.05

Ability to operate andlor monitor reactor trip switches Knowledge of the reasons for the following responses ...Tech spec limits Tave

.3/4.2

308

000003 Dropped Control Rod II

3.07

3.6(3 0

600

000007 Reactor Trip - Stabilization - Recovery I I Knowledge of the operational implications of the following concepts ...thermodynamics and flow characteristics ol open or leaking valves

000008 Pressurizer Vapor Space Accident / III

1.01

3.2117

311

000009 Small Break LOCA I III Knowledge of the operational implications of the following concepts ...netural cliculation and cooling including reflux baling

000011 Large Break LOCA MI

1.01

4.1(4.4

801

W/E04 LOCA Outside Containment 1111

W/E03 LOCA Cooldown - Depress. ) IV Knowledge for the reasons for the following responses... RO or SRO function within the control room learn as appropriate to the assigned posh' In such tawdry that procedures are adhered to and the limitations in the facilities eballite and amendments are not violated Ability to determine and Interpret the following., Adherence to appropriate Procedures and operation within limitations In the facilities license and amendments Ability to determine and interpret . how long P2R level can be maintained within limns

WIE111 Loss of Emergency Coolant Rocky I IV

3.4

3.6/3.8

51

WIEO2 SI Termination I III

2.2

3.5(4.0

802

22 Loss of Reactor Coolant Makeup / II

2,04,

2.9/3.8

803

000025 Loss of RHR System / IV Knowledge of the interreietionships between... breakers, relays and disconnects

000029 Anticipated Transient w/o Scram I I

2.08

2.9'/3.1"

241

000032 Loss of Source Range NI I VII

'I 1.02 Ability to operate and/orand/ormonitor ...level trip bypass 30 1 . * _ . " 31 _ 1 604

000033 Loss of intermediate Range NI I VII

000037 Steam Generator Tube Leak / III

Ability to evaluate plant performance and make optcational judgments based on operating characteristics, reactor behavior and instrument 1.7 interpretation Knowledge of the reasons for the following responses ... Automatic actions associated with high radioactivity In SIG sample lines Knowledge of Vie masons Na the following responses .._ actions contained in EOPs Ability to determine and interpret the following...sdherence to appropriate procedures and operation Within limitations in the realities license and amendments

3.714.4

605

Steam Generator Tube Rupture! III

3,03

31314.0"

806

000054 Loss of Main Feedwater I IV WIE05 Inadequate Heat Transfer -Loss of Secondary Heat Sink! IV

3.04

4.4/4.8

191

2.2

3.7/4.3

471

000058 Loss of DC Power

!VI

-a

2,22 Knowledge of limiting conditions for operation and safety limits .-

3.4/4/1

608

000059 Accidental Liquid Rachvaste Rat. I IX

2.61

Knowledge of the interrelationships between...radioactive liquid monitors

2.7r2.8

609

000060 Accidental Gaseous Rodwaste Rel. I IX

2.02

Ability to determine and interpret...the possible location of a radioactive gas leak with the assistance of PEO, health physics and chemistry personnel 3.1/4.0

811

000061 ARM System Alarms I VII

. Knowledge of the interrelationships between...facility's hest removal systems, inducing Primary coolant, emergency coolant, decay heal removal systems, and relations between the proper operation of these systems to operation of the facility 4 2, 4 2 Group Point Total:

.....

/EIS High Containment Radiation/1X KM Category Totals: 2

2.2 , 3. _

2.8/3.0 17,

1 17.

812 17

EAPEs TI G2

For Official Use Only

3/21100

001L

S100 esn 113Pa -10A

L.I.sad13

:11110j. 000d dram.

I.

5 .

I. :RI eloi

0 .14.0. vrA

Al 11011410014 LueWuleLLI00. SL WM

Al I lunsseid-Joeo icilLiettlin UnIKS i 31M

IPA 1 JIV lueumesui Jo eloi 5900013

6I.

WE/.1-E

il eupueLep of eta% weept ete Jo elm 'Butioopqns Jo uoquyep ' stdewoo 0L..monoi au' yo suoevoqdwieuotpuedo @Le io eBpepi.001 uoenueLunesul Ougioddne pus suomedo etmen,1 Jo pollens ul 'Demo citz all 1000 pepuedo welsh 1pposi eemots lent Lem uogeo!untuumo 'ewe BuRpueu poi wog swop se Lions LueppOe BulleueLl Pm) BUynp w00.11011U07 all 1.11 BellnP 01:Ito elSpep.ouN edwnd Buylmui, SOA3 Joi irJoleopu! Buluuru pue suetetuwe -pudieete pug eulttueleP 01 AlIllesi (0011/01VO4

COt

IA/ aimed ees-YO Jo seoi ggee00

Let

ET/9'E

111A 11009100V runpusH lend 9E0000

CO

L'EaZ

a0 r
-i -

II! UOIPUMItri !Mtn lazianssom ez0000

uoeserto

speod

'dm

ZC)5 V)41 VC I11

00140u0A 40.111S i owliN IR 3dV/3


L t

sing

Cfr101911, 11,11 - ruognioA3 Judd ietwouqv pug .L2uolLredug C-1.01,-S3 uuod oulTWO 1,0104/iWir3 OH tiMd

korsa,

ueld*Owne111100111

McGuire Sample Plan

ES-401 4 3 System R/ Name K K 2 K 3 4

PWR RO Examination Outline Plant System! - Tim 2 Grow A A A A KK K 0 1 2 .......3 4 ....11 X

Fano ES-401-3 1
IVA Toplc(s)

Bank Imp. Points Question

001 Control Rod Drive

2.09

Ability to predict the Impacts of the following malfunction or operation...and based on those predictions, use procedures to correct, control or mitigate the consequences of... station blackout Ability to predict and/or monitor changes In parameters (to prevent exceeding design limits) associated with operating the..,RCP standpipe levels Ability lo predict and/or monitor changes in parameters do prevent exceeding design limes) associated with operating the...RCS pressure and temperature

3.9/4.0

522

003 Reactor Coolant Pump

1.10

2,5/2.7

623

004 Chemical Volume Control

i 59

3.0/3.0

625

019 Engineered Safety Features Actuation

2.01

Knowledge 01 bus power Supplies to... ESFASisafeguards equipment COMM

3.9./3.43

827

016 Nuclear Instrumentation

6.03

Knowledge of the effect that a loss or malfunction will have on ...component interconnections Knowledge of the following operational implications...saturation and subcooling of wider

2.6/3.0

152

017 In-core Temperature Monitor

5.02

3.7/4.0

404

022 Containment Cooling

3.01

Ability to monitor automatic operation of the, _initiation of safeguards mode of operation 41/4.3 Knowledge of the cited that a loss or malfunction will have on ...upper and lower doors Knowledge of physical connections and/or cause end effect reletionshIps...MFW Ability to predict the impacts of the following malfunction or operation... and based on those predictions. use procedures to coned, control or mitigate the consequences of... Failure of feedwnter control system Knowledge of design feature(s) and/or interlock(s) which provide for.., turbine trio including overspend

526

026 ice Condenser

6.01

3.4"13.6"

630

066 Condensate

1.03

2.6'12.6

531

069 Main Feedwalar

2.11

3.0139'

539

061 AuxiliarpErnergeney Feedwater

4.07

3.1'0.3'

634

068 Liquid Red Waste

3.02

Ability to monitor automatic operation of the...automatic deletion

3.6/3.8

407

071 Waste Gee Disposal

4.46 Ability to verify that alarms are consident with plant conditions Ability to predict and/or monitor changes In parameters (to prevent exceeding design limits) associated with operating the...radiation levels Ability to manually operate and/or monitor in the control rope... major components Knowledge of design features) and/or interlock(s) which provide for...safety injection block

3_513.6

489

072 Area Radiation Monitoring

1,01

34/3_6

672

072 Area Radiation Monitoring

4.02

2.5"0.5

535

013 Engineered Safety Features Actuation

4.12

3,7/3.9

412

071 Waste Ciao Disposal

4.05

Knowledge of deslon features) and/or Interlock(s) which provide for...point of release 2.7/3.0 ANON to predict the impacts of the following malfunction or operation._ and based On those predictions, use procedures to correct, control or mitigate the consequences of...loss of condensate pumps

607

066 Condensate

2.04

2.6/2.9'

415

013 Engineered Safety Features Actuation

3.02

Knowledge of the affect that a loss or malfunction ...will have on ...RCS

4.3/4.5

645

069 Main Friedman

4.11

Ability to manually operate and/or monitor In the control room... recovery from automatic feedwater isolation 3_10.3

150

004 Chemical Volume Control

354

Knowledge of the effect that a loss or malfunction ...will have on ...RCPS Knowledge of the following operational Implications...Impedance 01011ductility temperature in plant operations

3,7/3.0

647

001 Chemical Volume Control

5.10

3.20.7

677

003 Reactor Coolant Pump KM Category Totals: 1 I_ 2 3 2 2 3 3

3.01 3 2 1

Ability to monitor automatic operation of the...seal injection Sow Group Point Total

3,3/3.2 ,
.

1 23

652 23

23

SYSTEMS T2 G1

For Official Use Only

3/21/00

Generics

For Official Use Only

3/21/00

NRC Official Use Only

Nuclear Regulatory Commission Reactor Operator Licensing Examination


McGuire Nuclear Station

This document is removed from Official Use Only category on Date of examination

NRC Official Use Only

Question #1

McGuire Nuclear Station

RO Exam

Bank Question: 51 1 Pt(s)

Answer: C

A large break LOCA is in progress and the operators are responding in E-I (Reactor Trip or Safety Injection). Given the following conditions: ND pump IA is tagged out of service for maintenance. Containment pressure is 14 psig. FWST level is below the swap over setpoint.

When shifting to cold leg recirc using ES-1.3 (Transfer to Cold Leg Recirc), valve 1NI-184B (RB Sump to Train 1B ND & NS) fails to open. The operators implement ECA-1.1 (Loss of Emergency Coolant Recirculation). FR-Z.1 (Response to High Containment Pressure) requires both NS pumps to be in operation. ECA-1.1 limits the operators to only one NS pump in step 11. Which of these two procedures takes priority under these conditions and what is the basis for this requirement? A. FR-Z.1 takes priority because a total loss of ND causes the NS system to become relatively more important to reduce containment pressure. FR-Z.1 takes priority because it was implemented in response to a red path and FRPs always have priority over ECA procedures. ECA-1.1 takes priority because it conserves FWST water level as long as possible for injection while providing sufficient NS flow to mitigate containment pressure.

B.

C.

D.

ECA-1.1 takes priority because ECA procedures always have priority over FRPs.

Distracter Analysis: A. Incorrect: ECA-1.1 takes priority over FR-Z.1 Plausible: Although a loss of ND and containment sump recirc causes a loss of the containment heat sink, the supply for NS comes from the FWST which will be drawn down until containment sump recirculation can be established. B. Incorrect: ECA-1.1 takes priority over FR-Z.1 Plausible: FRPs normally take priority over most EOPs C. Correct answer D. Incorrect: ECAs do not always have priority over FRPs.

Ques_051

For Official Use Only Page 1

Question #1

McGuire Nuclear Station

RO Exam

Plausible: Some ECAs take priority e.g. ECA-0.0 has priority over FRPs in that F-0 is not applicable until transition out of ECA-0.0.

Ques_051

For Official Use Only Page 2

Question #2

McGuire Nuclear Station

RO Exam

Bank Question: 60
1 Pt(s)

Answer: A

Unit 2 was operating at 100% power when an electrical fire started inside the auxiliary building cable spreading room. What type of fire suppression system is installed inside the cable spreading area and what are the hazards to personnel if they enter this room? A. A manual deluge (Mulsifyre) System is installed. An electrical shock hazard exists due to the use of water to combat an electrical fire. An automatic sprinkler system is installed. An electrical shock hazard exists due to the use of water to combat an electrical fire. An automatic Halon system is installed. An asphyxiation hazard exists due to the presence of Halon gas. A manual Cardox system is installed. An asphyxiation hazard exists due to the presence of carbon dioxide gas.

B.

C.

D.

Distracter Analysis: A. B. C. Correct Answer: Incorrect: A manual deluge Mulsifyre system is installed Plausible: an electrical shock hazard exists Incorrect: A manual deluge Mulsifyre system is installed Plausible: Halon gas is generally used in areas in which electrical fires are the predominant risk because it does not create a shock hazard Incorrect: A manual deluge Mulsifyre system is installed Plausible: Cardox gas is a personnel hazard although all the CARDOX systems have been replaced with HALON, the pull switches still say CARDOX in some areas (like the diesel generators)

D.

Ques_060

For Official Use Only Page 3

Question #3

McGuire Nuclear Station

RO Exam

Bank Question: 82 1 Pt(s)

Answer: D

Unit 2 is recovering from a loss of 120 VAC instrument bus 2EKVA due to the loss of the 2EVIA static inverter. 2EKVA has been reenergized from the alternate supply. After repairs to inverter 2EVIA are completed, the operator is directed to restore the 2EKVA bus to the normal line up. Which one of the following actions is necessary to restore the electrical lineup to a normal operating configuration after tags are cleared? A. Manually transfer bus power from static inverter 2EVIB back to static inverter 2EVIA. Enable the automatic transfer of power from static inverter 2EVIB back to 2EVIA. Enable the automatic transfer of power from regulated power center 2KRP back to static inverter 2EVIA.

B.

C.

D.

Manually transfer power from regulated load center 2KRP back to static inverter 2EVIA.

Distracter Analysis: A. Incorrect: Power is not transferred to 2EVIB this is the normal power supply to 2EKVB Plausible: If the candidate thinks that 2EKIB can be used to supply 2EKVA Incorrect: There is no automatic transfer associated with these static inverters Plausible: There are automatic bus transfers for some of the 120 VAC power supply breakers Incorrect: There is no automatic transfer between 2KRP and 2EVIA Plausible: 2KRP is the correct alternate supply for 2EKVB if 2EVIA is not operating and there is an auto transfer switch between the normal and alternate power supplies for 2KRP. Correct answer

B.

C.

D.

Ques_082

For Official Use Only Page 4

Question #4

McGuire Nuclear Station

RO Exam

Bank Question: 111 1 Pt(s)

Answer: B

Unit I experienced a LOCA into containment at 0200. Given the following sequence of events: 0200 LOCA starts 0205 The OSM declared an Alert 0210 The OSM designated you as the Control Room Communicator and directed you to prepare the initial notification messages for his review. 0220 You complete filling out the first notification form What are the maximum allowable notification times required by RP/0/A/5700/002? States and Counties A . B . C . D . 0215 0220 0225 0225 NRC 0300 0305 0300 0305

Distracter Analysis: A. Incorrect: only 10 minutes from alert declaration time and less than 1 hours for NRC Plausible: 15 minutes and 1 hour from start of the event Correct Answer: Incorrect: 20 minutes and 55 minutes from time of alert declaration Plausible: if the candidate thinks that the 15 minute clock starts when he/she is told to complete the initial notification form Incorrect: 20 minutes from time of alert declaration Plausible: 1 hour NRC notification from the alert declaration is correct

B. C.

D.

Ques_111

For Official Use Only Page 5

Question #5

McGuire Nuclear Station

RO Exam

Bank Question: 120

Answer: C

I Pt(s)

Unit 1 was operating at 100% power. Given the following motor driven auxiliary feedwater pump operating parameters: M) Discharge Pressure (ft water) 3325 Suction Pressure (ft water) 75 420 Pump flow rate (gpm) 210 3325 75 480 (M) 3010 75 520 9/1Q 2950 75 560

What is the onset (earliest time) of pump cavitation conditions? REFERENCES PROVIDED Curve 8.4 of enclosure 4.3 to OP/1/A/6100/22 A. B. C. D. 0200 0210 0220 0230

Distracter Analysis: A. Incorrect: - cavitation has not yet occurred: 3325 - 75 = 3250 in Plausible: - below pump characteristic curve but above NPSH requirement Incorrect: - cavitation has not yet occurred: 3325-75 = 3250 Plausible: - point is above the pump characteristic curve but still below NPSH curve Correct answer: below NPSH curve 3010 75 2935 in Incorrect: cavitation has already occurred Plausible: - if the candidate does not remember to subtract the suction pressure from the discharge pressure: 3010 from answer C is above the NPSH curve

B.

C. D.

Ques_120

For Official Use Only Page 6

Question #6

McGuire Nuclear Station

RO Exam

Bank Question: 125 1 Pt(s)

Answer: C

A worker is preparing to enter a high radiation area to work on a valve in the reactor building. During the pre-job briefing, RP states that the expected whole body radiation level are as follows: Dose rate in the center of the room 20 ft away = 200 mrem/hr Dose rate 18 inches from valve = 700 mrem/hr Contact reading = 1100 mrem/hr

How should the area around the valve be classified? A. B. C. The room is a radiation area; the valve is a hot spot The room is a high radiation area; valve is NOT a hot spot The room is a high radiation area; the valve is a hot spot

D.

The room is an extra high radiation area; the valve is NOT a hot spot

Distracter Analysis: A hot spot is an area where the dose rate on contact is > 5x general area radiation but > 100 mrem/hr. In this case 5 x 200 = 1000 mrem/hr < 1100 mrem/hr on contact. A. Incorrect: 200 mrem/hr general area dose rate > 100 mrem/hr = high radiation area Plausible: if the candidate does not know that the lower limit for a high radiation area is 100 mrem/hr and the valve is a hot spot Incorrect: The valve is a hot spot Plausible: the room is a high radiation area if the candidate thinks that the definition of a hot spot is > 5x general area dose rate when measured 18 inches from the contact reading Correct Answer: Incorrect: The room is not an extra high radiation area Plausible: if the candidate thinks that the definition of a hot spot is > 5x general area dose rate when measured 18 inches from the contact reading

B.

C. D.

Ques_125

For Official Use Only Page 7

Question #7

McGuire Nuclear Station

RO Exam

Bank Question: 150 1 Pt(s)

Answer: D

Unit 1 is responding to a reactor trip from 100% power. Main feedwater isolation occurred at 0150 due to a safety injection signal. Given the following plant conditions at the following times: Time Tavg R, Trip Breakers A SG NR level B SG NR level C SG NR level D SG NR Level 0200 551 open 84% 79% 80% 82% 0201 552 open 82% 81% 79% 80% 0205 554 open 82% 76% 75% 75% 0210 555 shut 81% 72% 71% 71%

At 0200, safety injection was reset. The operators depress the FWI reset pushbutton at 0200, 0201 and again at 0205. When is the earliest time that the operators will regain control of main feedwater components?

A. B.
C. D.

0200 0201
0205 0210

Distracter Analysis: This question is a modification of a 1997 NRC exam question and subsequent bank test question that asked when MFI could be reset if actuation was caused by reactor trip and low T-ave. The correct answer was "A". If MFI was caused by a safety injection, reset cannot occur until after reactor trip breakers are closed A. Incorrect: - MFI caused by SI initiation requires RTB to be shut and SI reset before MFI reset can occur. Plausible: - MFI isolation can be reset with actuation signals present if actuation occurred due to P-4/low T-ave or P-14 (hi S/G level) without a reactor trip Incorrect: - MFI caused by SI initiation requires RTB to be shut and SI reset before MFI reset can occur Plausible: - P14 has cleared on A S/G < 83%

B.

Ques_150

For Official Use Only Page 8

Question #7 C.

McGuire Nuclear Station

RO Exam

D.

Incorrect: - MFI caused by SI initiation requires RTB to be shut and SI reset before MFI reset can occur Plausible: -low Tave (< 553F) has cleared Correct answer - need to shut RTBs to get SI FWI signal to reset need not depress FWI reset pushbutton for this to occur.

Ques_150

For Official Use Only Page 9

Question #8

McGuire Nuclear Station

RO Exam

Bank Question 152 1 Pt(s)

Answer: C

The operators are conducting a reactor startup. Given the following indications on the source range (SR) and intermediate range (IR) excore nuclear instruments: Time SR "A" (cps) SR "B" (cps) IR "A" (amps) IR "B" (amps) 0200 1.5x104 1.4x104 7.6x10-ll 7.9x1011 0205 2.5x104 2.3x104 1.1x1e 9.0x1e 0210 2.8x104 2.7x104 1.5x1e 1.Ix1e 0215 1.0x105 9.8)(104 7.0x10" 7.5x10-10

What is the earliest time that the operators should block the source range nuclear instruments? A . B . 0200 0205

C . D .

0210 0215 Distracter Analysis: The objective behind this question is to determine if the candidate can differentiate between when they CAN block SR high flux (because P-6) is in and when they are ALLOWED to block SR high flux - after observing "proper" overlap between IR and SR - i.e. one decade. They will not observe one decade of overlap until both IR NI channels are > 1E-10 amps because they come on scale at 1E-11 amps. A. Incorrect: < 1E-10 amps in both IR channels Plausible: Source range nuclear instruments should be blocked by the time the level is 10" CPS by the training material Incorrect: only IR A channel has reached 1 decade of observed overlap with the SR Plausible: this is when P6 is in and they can physically block SR high flux Correct answer: proper overlap has been observed on IR B

B.

C.

Ques_152

For Official Use Only Page 10

Question #8
D.

McGuire Nuclear Station

RO Exam

Incorrect: - will reach the SR high flux trip setpoint at 1E10 cps Plausible: - if the candidate does not know P6 or if he is confusing the high flux trip setpoint with the P6 setpoint

Ques_152

For Official Use Only Page 11

Question #9

McGuire Nuclear Station

RO Exam

Bank Question: 191 1 Pt(s)

Answer: B

Unit 1 was operating at 100% power when a total loss of feedwater occurred. The operators reached step 35 of FR-H.1(Response to Loss of Secondary Heat Sink) which states: IF AT ANY TIME while in this procedure any S/G W/R level goes below 12% (17% ACC), THEN GO 1Q Enclosure 10 (Hot/Dry Steam Generator Limits) Given the following conditions: Loop A 0 150 Loop B 15 555 Loop C 9 530 Loop D 10 545

S/G (WR) [%1 NC T uot [F]

Containment pressure is 3.4 psig The TD CA pump is available to feed the S/Gs

Which one of the following statements correctly describes the bases for the restrictions for restoring feedwater flow following feed and bleed in FR-H.1? A. Restore flow to the A S/G because loop A T-hot is the lowest of the loops and this will reduce the chance of thermal shocking the S/G tube sheet. Flow should not be restored to the B and C S/Gs because they will be reserved for use later to provide a steam supply for the TD CA pump. Restore flow to the B S/G because B S/G level is the highest and this will reduce the chance of thermal shocking the S/G tube sheet. Flow should be preferentially restored to the B or C S/G to maintain the TD CA pump steam supply. Restore flow to the C S/G because loop C T-hot is less than loop B T-hot and this will reduce the chance of thermal shocking the S/G tube sheet. Flow should be preferentially restored to the B or C S/G to maintain the TD CA pump steam supply. Restore flow to the D S/G because the D S/G is higher than A S/G level, which will reduce the risk of thermal shock. Flow should not be restored to the B and C S/Gs because they will be reserved for use later to provide a steam supply for the TD CA pump.

B.

C.

D.

Ques_191

For Official Use Only Page 12

Question #9

McGuire Nuclear Station

RO Exam

Distracter Analysis: There was a change in this procedure since the last NRC exam. The previous guidance was not to feed a SIG when Thot > 550 F and to select the B and C SIGs for restoration of flow. Now the guidance is to select the S/G that has the highest apparent level and to preferentially select the B or C S/G. A. Incorrect: T-hot should not be used to determine which S/G should receive flow. It is not a reliable means of determining S/G shell temp in a dry stagnant loop. Plausible: The apparent temp of the A loop is the lowest and it may appear that the chance of thermal shock is lessened. Correct answer: feed the S/G that has the highest level and preferentially feed B & C S/Gs to maintain steam supply to the TD CA pump. Incorrect: C S/G has a lower SIG level than B S/G Plausible: C SIG has a lower T-hot than B S/G Incorrect: No basis for reserving the B & C S/Gs for restoring flow Plausible: There is a high probability that restoring feed to a dry S/G could rupture the tube sheet due to thermal stress. It makes sense to select a S/G that is NOT used to supply steam to the TD CA pump for the initial restoration of the heat sink.

B.

C. D.

Ques_191

For Official Use Only Page 13

Question #10

McGuire Nuclear Station

RO Exam

Bank Question: 241 1 Pt(s)

Answer: D

Unit 1 is operating at 100% power when the supply breaker from 1LXG to Control Rod Drive MG set #2 opens. Which one of the following sequence of events will occur to the reactor trip breakers A or B (RTA/B) and the reactor trip bypass breakers A or B (BYA/B)? A. B. C. D. RTA and BYB will open RTB and BYA will open BYA and BYB will open No breakers will open

Distracter Analysis: A. Incorrect: the rod drive MG sets are run in parallel losing one MG set will not cause any reactor trip breakers to open. Plausible: If the candidate thinks that the 1B rod drive MG set provides control power RTA and BYB. Incorrect: the rod drive MG sets are run in parallel losing one MG set will not cause any reactor trip breakers to open. Plausible: If the candidate thinks that the 1B rod drive MG set provides control power RTB and BYA. Incorrect: the rod drive MG sets are run in parallel losing one MG set will not cause any reactor trip breakers to open. Plausible: If the candidate thinks that the 1B rod drive MG set provides control power BYA and BYB. Correct answer: the rod drive MG sets are run in parallel losing one MG set will not cause any reactor trip breakers to open.

B.

C.

D.

Ques_241

For Official Use Only Page 14

Question #11

McGuire Nuclear Station

RO Exam

Bank Question: 242 1 Pt(s)

Answer: A

Unit 1 is operating at 100% power. Given the following conditions on the lA NCP: Time 0200 312 Motor winding temp (F): Pump shaft vibration (mils): 15 #1 seal AP (psid): 201 #1 seal outlet temp (F): 201 0210 315 16 196 226 0220 320 18 223 236 0230 324 21 235 240

What is the earliest time that the operators are required to trip NCP-1A? A. B. 0200 0210

C. D.

0220 0230

Distracter Analysis: Objective to determine if the candidate can analyze the above conditions and select the correct time to trip the NCP. Parameters are selected to plausibly distract on the basis of the different set points. A. B. Correct: Must trip when motor winding temperature exceeds 311 F Incorrect: reached trip set point at 0200 Plausible: NCP #1 seal differential pressure is less than limit of 200 psid Incorrect: reached trip set point at 0200 Plausible: NCP #1 seal outlet temp exceeds the limit of 235 F Incorrect: reached trip set point at 0200 Plausible: pump shaft vibration limit reached at 20 mils

C. D.

Ques 242

For Official Use Only Page 15

Question #12

McGuire Nuclear Station

RO Exam

1 Pt(s) Bank Question: 243 Answer: C

Unit 1 is conducting a plant startup in Mode 1. The operators have reached 8% power when a momentary electrical transient occurs resulting in the following conditions: Bus Frequency (Hz) Voltage (VAC) 1TA 55 6410 1TB 60 6900

1TC 55 6410

1TD 60 6900

Which one of the following sequences would occur? A. A reactor trip does NOT occur and NCPs 1A and 1C trip on under-frequency while NCPs 1B and 1D continue running. A reactor trip occurs and NCPs 1A and 1C trip on undervoltage while NCPs 1B and 1D continue running. A reactor trip does NOT occur and all four NCPs trip on underfrequency. A reactor trip occurs and all four NCPs trip on under-frequency

B.

C.

D.

Distracter Analysis: A. Incorrect: all 4 NCPs trip due to the NC pump monitor system action Plausible: only 2 pumps have a low frequency condition Incorrect: all 4 NCPs trip due to the NC pump monitor system action - the reactor dies not trip as power is below P-7 (10%) Plausible: only 2 pumps have a low voltage condition Correct answer Incorrect: the reactor does not trip below P-7 (10%) Plausible: all four NCPs trip due to under-frequency on 2/4 NCPs

B.

C. D.

Ques_243

For Official Use Only Page 16

Question #13

McGuire Nuclear Station

RO Exam

Bank Question: 264 1 Pt(s)

Answer: D

Unit 2 was operating to 60% power when an ATWS event occurred that lifted all 3 pressurizer safety relief valves. T-ave peaked at 680 F and the NC pressure transient reached 2675 psig. Which one of the following describes the safety limit value and allowable action time applicable to this transient? A. B. 2635 psig / 5 minutes 2735 psig / 5 minutes

C.
D.

2635 psig / 1 hour


2735 psig 11 hour

Distracter Analysis: A. B. Incorrect: safety limit is 2735 psig, allowable action time is 1 hour Plausible: psychometric balance Incorrect: allowable action time is 1 hour Plausible: the safety limit is correct and the allowable action time is correct for modes 3-6 Incorrect: safety limit is 2735 psig Plausible: allowable action time is correct Correct Answer:

C. D.

Ques_264

For Official Use Only Page 17

Question #14

McGuire Nuclear Station

RO Exam

Bank Question: 298 1 Pt(s)

Answer: D

Unit 1 was operating at 100% power. Given the following conditions: Pressurizer pressure controller is selected to "1-2" Pressurizer pressure controls are in AUTO Pressurizer pressure channel I detector fails LOW

Which one of the following describes the plant response with no operator action? A. B. High pressurizer pressure reactor trip will occur. PORV 1NC-34A will maintain NC system pressure 80 to 100 psig above normal. PORV 1NC-34A will maintain NCS pressure from 100 psig above normal to 50 psig below normal.

C.

D.

PORVs 1NC-32B and 1NC-36B maintain NC system pressure 80 to 100 psig above normal.

Distracter Analysis: This question was modified from a question on the Catawba NRC Exam from 1997. The stem was changed from position 3-2 to position 1-2 and distracter C was changed and the correct answer was changed. A. Incorrect: no trip will occur Plausible: would be the correct answer for pressure control in the 32 position this was the correct answer on the 1997 Catawba NRC exam Incorrect: NC-34A will not open Plausible: the plant pressure control band is correct but NC-34A only opens if pressurizer pressure channel I fails high, not low right pressure, wrong PORV Incorrect: NC-34A will not open Plausible: NC-34A opens if pressurizer pressure channel I fails high and the pressure control band is correct for NC-34A Correct answer

B.

C.

D.

Ques 298

For Official Use Only Page 18

Question #15

McGuire Nuclear Station

RO Exam

Bank Question: 307


1 Pt(s)

Answer: C

Unit 1 was operating at 100% power when a crud burst occurred. Given the following events and conditions: EMF-48 (Reactor Coolant Hi Rad) trip 2 alarm 1EMF-18 (Reactor Coolant Filter 1A) trip 2 alarm

Which one of the following actions is required to reduce coolant activity due to a crud burst in the NC system? A. B. C. D. Purge the VCT with nitrogen Place/ensure both mixed bed demineralizers are in service Increase letdown flow Add hydrogen to the reactor coolant

Distracter Analysis: A. Incorrect: Will not correct a high NC activity from a crud burst Plausible: One of the subsequent actions in AP/18 is to purge the VCT to the waste gas system with Hydrogen. In addition, Nitrogen is used to purge the VCT for shutdown. It is likely that a candidate could mix up these purges. Incorrect: Do not want to load crud particles into BOTH mixed bed demineralizers Plausible: Mixed bed demins will filter crud particles and remove fission product ionic impurities - this action required for fuel element failure/high fission product activity in AP/18 - but not for crud burst Correct: Will increase removal rate of crud particles by increased filtration. Incorrect: Will not remove crud burst particulate activity Plausible: Used to scavenge Oxygen from the NC coolant and thus reduce the corrosion rates and crud production in the RCS. However, this does not affect crud burst particulates that are already in the NC system coolant.

B.

C. D.

Ques_307

For Official Use Only Page 19

Question #16

McGuire Nuclear Station

RO Exam

Bank Question: 308 1 Pt(s)

Answer: D

Unit 1 is operating at 100% power. Given the following conditions: Rod control is in manual Control Bank D is at 200 steps If the rods in control bank D start stepping out at 8 steps per minute, what one of the following actions is required at this time? A. Select Control Bank D on the rod selector switch and manually insert Control Bank D Select "AUTO" on the Bank Select Switch and see if rod motion stops Commence emergency boration

B.

C.

D.

Trip the reactor

Distracter Analysis: A. Incorrect: The correct response is to trip the reactor for a rod withdrawal Plausible: this action could stop the rod withdrawal, as the rods in signal should over-ride the rods out signal Incorrect: Trip the reactor is the correct response. Plausible: If the malfunction was in the manual section of the rod control circuitry, this could stop the rods. If the rod control was in auto then going to manual would be the correct answer. This reverses that thought process. Incorrect: Trip the reactor is the correct response Plausible: This action would be required to insert negative reactivity if the trip did not work Correct answer: Immediate action in step 3 of AP-14

B.

C.

D.

Ques_308

For Official Use Only Page 20

Question #17

McGuire Nuclear Station

RO Exam

Bank Question: 311 1 Pt(s)

Answer: A

Unit 1 is operating at 50% power. Given the following conditions: Pressurizer pressure is 2235 psig Pressurizer Relief Tank (PRT) pressure is 20 psig PRT temperature is 125 F PRT level is 81/G The PRT is being cooled by spraying from the RMWST A pressurizer code safety valve is suspected of leaking by it's seat

What temperature would be indicated on the associated safety valve discharge RTD if the code safety were leaking by? REFERENCES PROVIDED: Steam Tables A. B. 258-262 F 228-232 F

C. D.

161-165 F 123 -127F

Distracter Analysis: A. B. Correct answer Incorrect: Temp is too low - the correct temp is 260 F Plausible: If the candidate makes the mistake of not correcting for atmospheric pressure by failing to adding 14.6 psi to the PRT pressure and uses 20 psia. Incorrect: Temp is too low - the correct temp is 260 F Plausible: If the candidate reverses the correction for atmospheric pressure by subtracting 14.6 psi from PRT pressure of 20 psig to get 5 psia. Incorrect: Temp is too low - the correct temp is 260 F Plausible: If the candidate thinks that the discharge temperature will be at the same temperature as the PRT fluid.

C.

D.

Ques 311

For Official Use Only Page 21

Question #18

McGuire Nuclear Station

RO Exam

Bank Question: 330 1 Pt(s)

Answer: B

During a cold startup, the NCPs are limited to 3 consecutive starts in any 2hour period. There is an additional requirement of a minimum idle period of 30 minutes between restarts. What is the reason for these limitations? A. This restriction assures that the oil temperature will decrease to design specifications between restart attempts. This restriction prevents overheating the motor windings due to high starting currents. This restriction allows the NCP seals to fully reseat between NCP oil lift pump cycles. This restriction prevents operators from restarting without a deliberate approach to ensure that all precautions and interlocks have been satisfied.

B.

C.

D.

Distracter Analysis: A. Incorrect: The reason is stator-winding temperatures Plausible: if the candidate remembers this as a high temperature concern - another adverse consequence of multiple starts on motors Correct Answer: Incorrect: The reason is stator-winding temperatures Plausible: NCP seal seating is the basis for a different cold start precaution seal leakoff limit after seal replacement Incorrect: The reason is stator-winding temperatures Plausible: this sounds plausible a motherhood statement that does not apply

B. C.

D.

Ques_330

For Official Use Only Page 22

Question #19

McGuire Nuclear Station

RO Exam

Bank Question: 338 1 Pt(s)

Answer: C

Which one of the following statements complies with the requirements of OMP 4-3 regarding the rules of usage for abnormal procedures {APs) when the EOPs have been implemented? A. B. APs may not be implemented when EOPs have been entered. Only one AP at a time may be implemented when EOPs have been implemented. Concurrent implementation of APs when EOPs are in use is not allowed. APs may be implemented concurrently with EOPs. However, the APs were written assuming that SI has not actuated and operators must be careful when using APs if SI has occurred. APs may be implemented concurrently with EOPs with the exception of events where SI has actuated. APs were written assuming the SI had not occurred and cannot be used if SI has actuated.

C.

D.

Distracter Analysis: A. Incorrect: APs may be entered after EOPs have been started Plausible: Many plants have this provision - symptomatic EOPs should address all significant safety challenges without requiring APs Incorrect: No limitation on the number of APs Plausible: Makes sense to limit the number of concurrent procedures in use Correct answer Incorrect: No explicit prohibition against use of APs when SI has actuated BUT there is a caution and the APs were written for the situation where SI has NOT occurred. Plausible: APs were written for the situation where SI has NOT occurred.

B.

C. D.

Ques_338

For Official Use Only Page 23

Question #20

McGuire Nuclear Station

RO Exam

Bank Question: 353


1 Pt(s)

Answer: D

A male worker needs to repack a valve in an area that has the following radiological characteristics: The worker's present exposure is 1800 mrem for the year. General area dose rate = 65 mrem/hr Airborne contamination concentration = 20 DAC

The job will take 4 hours with a mechanic wearing a full-face respirator. It will only take 2 hours if the mechanic does NOT wear the respirator. Which of the following choices for completing this job would maintain the workers exposure within the Station ALARA requirements? A. The worker should wear the respirator otherwise he will exceed 25% of the DAC limit. The worker should NOT wear the respirator because the dose received will exceed neither NRC nor site dose limits. The worker should wear the respirator because the total TEDE dose received will be less than if he does not wear one. The worker should NOT wear the respirator because the total TEDE dose received will be greater than if he wears one.

B.

C.

D.

Distracter Analysis: Radiation exposure comparison: Without respirator DDE = 65 mrem/hr x 2 hr = 130 mrem From airborne contamination: CEDE = 20 DAC 2 hr x 2.5 mrem/DAC-hr = 100 mrem TEDE = 130 + 100 = 230 mrem from job Total exposure for year = 1800 + 230 = 2030 mrem With respirator DDE = 65 mrem/hr x 4 hr = 260 mrem CEDE = 0 TEDE = 260 mrem Total exposure for year = 260 + 1800 = 2060 mrem

Ques_353

For Official Use Only Page 24

Question #20

McGuire Nuclear Station


(with respirator) (without respirator) TEDE = 2060 mrem > 2030 mrem = do NOT use a respirator A.

RO Exam

B.

C.

D.

Incorrect: Will not exceed 25% the DAC limit - this is not how DAC is applied to exposure limits Plausible: 25% DAC is the limit at which an area requires posting as a high airborne contamination area. Incorrect: The dose will exceed station admin limits of 2000 mrem Plausible: if the candidate does not know the station admin limit or miscalculates the dose received Incorrect: The exposure will be greater if you wear the respirator Plausible: If the candidate incorrectly computes the exposure - this was the correct answer on the 1997 Catawba NRC exam Correct answer

Ques_353

For Official Use Only Page 25

Question #21

McGuire Nuclear Station

RO Exam

Bank Question: 401 Pt(s)

Answer: D

Unit 2 was operating at 100% when a single control rod in control bank D drops into the core due to a failed CDRM. The SRO directs that the dropped rod be recovered. Which one of the following prevents the remaining rods in the control bank from being withdrawn while the dropped rod is being recovered? A. The rod control non-urgent failure alarm will actuate when the dropped rod is withdrawn blocking all rod motion. C-11 actuated when the rod dropped and will prevent outward rod motion by control bank D The Lift Coil Disconnect Switch is opened on the dropped rod to electrically isolate it from control bank D

B.

C.

D.

The Lift Coil Disconnect Switches are opened on control bank D rods that did not drop

Distracter Analysis: A. Incorrect: the non-urgent failure alarm does not actuate unless you lose a power supply to a logic or power cabinet Plausible: - the urgent failure alarm actuates when the rod is withdrawn and this would block rod motion for all rods on the opposite power cabinet to the dropped rod. Incorrect: - C-11 will not allow any auto rod motion but allows manual motion C-11 has not actuated under these conditions (bank D at top of core) Plausible: - C-11 will prevent outward rod motion in bank D Incorrect: - will not be able to pick up the dropped rod Plausible - if the candidate was not familiar with the actions of the lift coil disconnect switch - it will disconnect the rod from bank D but the rod cannot then be withdrawn with the switch open Correct answer

B.

C.

D.

Ques_401

For Official Use Only Page 26

Question #22

McGuire Nuclear Station

RO Exam

Bank Question: 404


1 Pt(s)

Answer: A

Unit 1 was responding to a small break LOCA. Containment pressure reached 3.5 psig. The Subcooling Margin Monitor currently indicated +35 F. Which of the following statements best describes the status of subcooling in the core? A. B. C. The core is subcooled by 35 F The core is superheated by 35 F The core is superheated by more than 35 F due to the effects of adverse containment conditions The core is subcooled by more than 35 F due to the effects of adverse containment conditions

D.

Distracter Analysis: This is a modified question from a previous NRC exam. The original question asked what the core conditions were if ICCM was reading 35 F. The original answer was "C". Note: the upper limit for measuring superheat is 35 F. The lower limit is +200 F. Although the ICCM was designed for ACC inputs, this option was never used because the pressure transmitters were located outside of containment A. B. C. Correct: Incorrect: - subcooling is 35 F Plausible: - if the candidate reverses the meaning of the indication (i.e. means subcooled, + means superheated) Incorrect: - subcooling is 35 F Plausible: - if the candidate reverses the meaning of the indication this was the answer on the NRC exam in 1997. Incorrect: - subcooling is 35 F Plausible: - if the candidate reverses the meaning of the indication.

D.

Ques_404

For Official Use Only Page 27

Question #23

McGuire Nuclear Station

RO Exam

Bank Question: 407

Answer: C

1 Pt(s)

Unit 1 has a liquid radioactive waste release in progress from the Ventilation Unit Condensate Drain Tank (VUCDT) through the RC system. All lineups and authorizations have been properly made in accordance with OP/O/B/6200/35 using the normal path. 2 RC pumps are the minimum required under LWR document. Given the following initial conditions: 2 RC pumps are running Controlling EMF properly adjusted for trip 1 and trip 2 settings No other releases are in progress

What automatic actions would terminate the release? A. B. WM-46 will close automatically if 1 RC pump trips WM-46 will close automatically when EMF-44 (VUCDT) reaches the trip 2 setpoint WL-320 and WP-35 will close automatically if 1 RC pump trips

C.

D.

WL-320 and WP-35 will close automatically when EMF-49 (Liquid Waste) reaches the trip 2 setpoint

Distracter Analysis: Used a similar question on the last NRC exam but modified the stem and the answer. The answer from the previous exam was "D". 1EMF-44 is the controlling EMF, not 1 EMF-49. A. Incorrect: WM-46 is isolated and not used anymore as a release path. Plausible: - RC pump interlock will actuate - set at 2 pumps (minimum required on LWR document). WM-46 was formerly the normal release path Incorrect: - WM-46 receives a closing signal from EMF-44 but this is not the normal path for a release. WM-46 is isolated and not used anymore. Plausible: - this was formerly the normal release path EMF44 sends a closing signal to WM-46 but the valve is no longer in service. Correct: - RC pump interlock will actuate - set at 2 pumps (minimum required on LWR document).

B.

C.

Ques_407

For Official Use Only Page 28

Question #23 D.

McGuire Nuclear Station

RO Exam

Incorrect: - EMF-49 does not trip WL-320 is not used to monitor the release from the VUCDT Plausible: - EMF-49 would monitor and isolate a liquid release from the Waste Monitor Tank (WMT) This was the correct answer from the last NRC exam except the monitor referenced was I EMF-44 instead of 1 EMF-49.

Ques_407

For Official Use Only Page 29

Question #24

McGuire Nuclear Station

RO Exam

Bank Question: 412 1 Pt(s)

Answer: A

Unit 1 is responding to a LOCA. Given the following initial conditions: A reactor trip and safety injection actuation occurred at 0150 MSIVs are shut. Phase B containment isolation has occurred

The operators reach step 2 in ES-1.1 (SI Termination) requiring a reset of the safety injection signal. Given the following parameter trends at 0200: NC pressure = dropped to 1850 psig then stabilized at 1951 psig Steamline pressure = 771psig - decreasing slowly Containment pressure = 2.2 psig - decreasing slowly

Given the following sequence of operator actions: 0202 Blocks the low steam line pressure MSI signal 0203 Blocks the low PZR pressure SI signal 0204 Resets the phase B isolation signal What is the earliest time that depressing the SI reset pushbuttons (trains A and B) would reset safety injection? A . B . 0200 0202

C . D .

0203 0204 Distracter Analysis: This question will test if a candidate understands that safety injection can be reset even with valid SI actuation signals still present (not blocked). The only restrictions are the 60-second timer and P-4 (RTBs open). A. Correct answer - safety injection can be reset after a 60 second timer has elapsed and the train related reactor trip breaker has opened (P-4). None of the SI signals being present will prevent reset of SI. Once reset, only manual SI is available

Ques_412

For Official Use Only Page 30

Question #24
B.

McGuire Nuclear Station

RO Exam

C. D.

Incorrect: - SI already reset at 0200 Plausible: - the steam line low pressure MSI can be blocked < P-11 doesn't effect SI Incorrect: - SI already reset at 0202 Plausible: - this will block the low pressurizer pressure SI signal Incorrect: - SI already reset at 0202 Plausible: - this action will block hi-hi containment pressure SI signal

Ques_412

For Official Use Only Page 31

Question #25

McGuire Nuclear Station

RO Exam

1 Pt(s) Bank Question: 415 Answer: A

Unit 1 is operating at 100% power when an electrical transient causes several condensate system pumps to trip. The operators take action to regain the system. Given the following conditionsand events: Start CF pump lA Suction Pressure (psig) CF pump 1B Suction Pressure (psig) # Hotwell Pumps running # Condensate Booster Pumps running 451 448 2 2 10 seq 238 235 3 0 20 sec 30 sec 235 225 220 225 3 2 1 2

What is the earliest time (if any) that BOTH main feedwater pumps will have tripped? A. B. 10 seconds 20 seconds

C. 3 0 seconds D. Feedwater pumps will continue to run under these conditions

Distracter Analysis: This is a modification of a question from the 1999 exam where the stern conditions were different and the answer was B. A. B C. Correct: Loss of all three CBPs causes a trip of both CF pumps Incorrect: - both CF pumps trip at 10 sec when 3/3 condensate booster pumps trip - trip is irrespective of suction pressure Incorrect: - trip occurred earlier at 10 sec Plausible: - if candidate does not know both CF pump trip on a loss of CBPs, then this is the first time that both CF pumps would trip on a loss of suction pressure (<230 psig) Incorrect: - tripping criteria met at 10 sec Plausible: - if the candidate does not know CF pump trip set points

D.

Ques_415

For Official Use Only Page 32

Question #26

McGuire Nuclear Station

RO Exam

Bank Question: 447 1 Pt(s)

Answer: D

Unit 1 is shutdown, Mode 6, in a refueling outage. Given the following conditions: Containment airlock doors are both open A full shift of qualified maintenance personnel are available inside containment The Refueling SRO is in the control room The Fuel Handling Supervisor is inside containment

Refueling has been completed and the Fuel Handling Supervisor (who is not a qualified SRO) requests permission to latch all control rods to prepare for the reactor startup. What additional requirements must be met (if any) to proceed with latching rods? A. Latching rods may proceed at the discretion of the Fuel Handling Supervisor. Latching rods may not proceed until after containment integrity has been restored. Latching control rods may not proceed until after the Refueling SRO arrives inside containment to supervise.

B.

C.

D.

Latching control rods may not proceed until after the Refueling SRO arrives inside containment and containment integrity has been restored.

Distracter Analysis: A. Incorrect: - the Refueling SRO is required to supervise this evolution and containment integrity must be restored Plausible: - if the candidate does not recognize that latching rods is a core alteration or doesn't recognize that this requires containment integrity to be established B. Incorrect: - the Refueling SRO is required to supervise this evolution Plausible: - if the candidate does not recognize that latching rods is a core alteration C. Incorrect: - containment integrity must first be established Plausible: - core alterations requires SRO coverage and containment integrity D. Correct answer

Ques_447

For Official Use Only Page 33

Question #27

McGuire Nuclear Station

RO Exam

Bank Question: 451


1 Pt(s)

Answer: B

Unit 1 is shutdown in a refueling outage. Given the following events and conditions: A VI header rupture occurs The VI system completely depressurizes. VI-820 was open at the time of the rupture. The VS system was in a normal lineup

What effect does a total loss of the VI system have on the VS system? A. VI-820 will auto-close as VI header pressure decreases below 90 psig and the VS air compressor will start automatically at 82 psig to maintain VS header pressure VI-820 will auto-close as VI header pressure decreases below 82 psig and the VS air compressor must be manually started to maintain VS header pressure Check valves in the VI - VS cross-connect line will close to isolate VS system pressure before it drops below 90 psig VS pressure in the Fire Protection Pressurizer Tank will be lost until a VS air compressor can be started.

B.

C.

D.

Distracter Analysis: A. Incorrect: - the VS air compressor does not automatically start to maintain pressure - VI-820 auto-closes at 82 psig not 90 psig Plausible: - The VI system is safety significant, VI-820 does close at 82 psig and there is a separate VS air compressor which has an automatic startup feature but it just is normally in "off' and requires operator action to start. B. Correct answer C. Incorrect: - there are no check valves in this line Plausible: - this is another possible method to prevent depressurizing the VS header at some plants. D. Incorrect: - the RF system tank is pressurized with VS air - but is maintained isolated from the VI header Plausible: - if the candidate does not know that the RF system air tank is isolated from the VS header.

Ques451

For Official Use Only Page 34

Question #28

McGuire Nuclear Station

RO Exam

Bank Question: 465 1 Pt(s)

Answer: A

Unit 1 is operating at 15% power going to 100% power. The operators just completed synchronizing the main generator on the power grid. Which one of the following sequences describes the correct operator actions for increasing the main generator load? A. Select MW IN Raise the GV limit from 17% to 120% Depress LOAD RATE pushbutton and enter desired load rate Depress the REFERENCE pushbutton and enter the load Depress the GO pushbutton Select MW IN Raise the GV limit from 17% to 120% Depress STANDARD pushbutton and enter desired load and load rate using the keypad Depress the GO pushbutton Select MW OUT Raise the GV limit from 17% to 100% Depress STANDARD pushbutton and enter desired load and load rate using the keypad Depress the GO pushbutton Select MW OUT Raise the GV limit from 17% to 100% Depress the REFERENCE pushbutton and enter the load Depress LOAD RATE pushbutton and enter desired load rate Depress the GO pushbutton Distracter Analysis: A. Correct answer: B. Incorrect: - do not use the STANDARD button Plausible: - this is designed to work correctly but is not used at McGuire C. Incorrect: - MW must be IN - do not use the STANDARD button Plausible: - if the candidate does not understand the MW IN feedback loop D. Incorrect: - MW must be IN for feedback loop Plausible: if the candidate does not understand the MW IN feedback loop

B.

C.

D.

Ques_465

For Official Use Only Page 35

Question #29

McGuire Nuclear Station

RO Exam

Bank Question: 469

Answer: B

1 Pt(s)

Unit 1 is in the process of making a radioactive gaseous waste release from the waste gas decay tank in accordance with OP/0/A/6200/18. Given the following conditions: MRIRR = 35 CFM 1EMF-50(L) trip 1 setpoint 1.0E5 CPM lEMF-50(L) trip 2 setpoint = 2.0E5 CPM 1 EMF-36(L) is out of service The operators reset 1EMF-50(L) whenever procedural direction allows
0200 22 1.1E5

Time Release rate (CFM) 1EMF-50(L) (CPM)

Q215 37 2.1E5

0230 32 2.2E5

0245 41 2.2E5

What was the earliest time that the operators were required to terminate (and not restart) the gaseous release.
A. 0200 - cannot release with lEMF-36(L) out of service

B.

0215 - must terminate due to exceeding MRIRR

C. D.

0230 - must terminate due to 2" trip of lEMF-50(L) 0245 - must terminate after 3rd trip of lEMF-50(L)

Distracter Analysis: A. Incorrect: - allowed to conduct this release as long as 1EMF-50(L) is in service Plausible: - 1 EMF-36(L) is the plant vent monitor and is normally in the release path. This has an automatic trip associated with it. Correct answer - release rate > MRIRR and EMF-50 trips and closes release path Incorrect: - release already stopped at 0215 Plausible: - and this is only the 2nd trip of EMF-50 - the operators could reset EMF-50 and restart this release at 0215 if they had not exceeded MRIRR Incorrect: - release already stopped at 0215 due to exceeding allowed release rate Plausible: - This action would be allowed if 1 EMF-36(L) was in service

B. C.

D.

Ques_469

For Official Use Only Page 36

Question #30

McGuire Nuclear Station

RO Exam

Bank Question: 471 1 Pt(s)

Answer: D

Unit 1 is responding to a LOCA. Given the following events and conditions: Completed E-0 (Reactor Trip or Safety Injection) Entered E-1 (Loss of Reactor or Secondary Coolant) The STA reported the following valid critical safety functions: Subcriticality - orange path Integrity - red path Heat Sink - red path All other CSFs are green or yellow

What procedure should be operator select? A. B. Remain in E-1 (Loss of Reactor or Secondary Coolant) Transition immediately to FR-S.1 (Response to Nuclear Generation /ATWS) Transition immediately to FR-P.1 (Response to Imminent Pressurized Thermal Shock Condition)

C.

D.

Transition immediately to FR-H.1 (Response to Loss of Secondary Heat Sink)

Distracter Analysis: A. Incorrect: - must transition to CSFs Plausible: - if candidate does not know restrictions and applicability of F-0 Incorrect: - Orange path does not have priority over red paths Plausible: - if candidate does not know rules of usage Incorrect: - Integrity does not have priority over Heat Sink Plausible: - if candidate does not know CSF rules of usage Correct answer: - Heat sink does not have priority over core cooling

B. C. D.

Ques_471

For Official Use Only Page 37

Question #31

McGuire Nuclear Station

RO Exam

Bank Question: 477 1 Pt(s)

Answer: B

Unit 1 is cooling down for a refueling outage. Given the following conditions and events: NCPs 1A and 1C are in operation Tave = 360 F Pressurizer pressure = 390 psig Pressurizer level = 75% SIG levels = 35% in all four S/Gs

If PORV NC-32 failed a surveillance test (the valve failed to open and could not be cycled manually) which of the following Tech Spec actions (if any) are required within one hour? A. B. C. Close and maintain power to the block valve for PORV NC 32 Close and remove power from the block valve for PORV NC 32 Immediately suspend all actions that could lead to a water solid pressurizer

D.

No actions are required within one hour - restore PORV NC 32 to an operable status within 7 days

Distracter Analysis: A. Incorrect: - answer incomplete - must also close the block valve and remove power Plausible: - appropriate for a failed PORV in mode 3 that can be manually cycled Tech Spec 3.4.11 condition A Correct answer Incorrect: - not an applicable tech spec requirement in mode 3 Plausible: - appropriate for mode 5, 6 Tech Spec 3.4.12 condition E Incorrect: - not an applicable Tech Spec requirement in mode 3 Plausible: - appropriate for a failed PORV in mode 4 - Tech Spec 3.4.12 condition D

B. C. D.

Ques_477

For Official Use Only Page 38

Question #32

McGuire Nuclear Station

RO Exam

Bank Question: 501


1 Pt(s)

Answer: D

Unit 2 was operating at 100% power when a terrorist attack in the control room caused the operators to rapidly evacuate to the Auxiliary Shutdown Panel. The operators were not able to perform AP/17 (Loss of Control Room) actions prior to evacuation at 0200. The terrorists tripped the turbine but did not operate any other controls. There are no other local operator actions taken. Given the following steam generator narrow range levels:

2A S/G NR 2B S/G NR 2C S/G NR 2D S/G NR

NM 65% 64% 63% 65%

DMZ 37% 38% 39% 38%

0204 22% 23% 25% 26%

N_Qb. 15% 18% 16% 20%

0208 25% 26% 24% 27%

Which one of the following statements describes the complete list of running feedwater pumps when the operators first arrive at the ASP at 0210 to take local control of the plant? A. B. C. Both motor driven CA pumps Both motor drive CA pumps and the turbine drive CA pump Both motor driven CA pumps and both CF pumps (in roll-back hold) Both motor driven CA pumps, the turbine driven CA pump and both CF pumps (in roll-back hold)

D.

Distracter Analysis: The lo-lo setpoint for SGWL is 17%. This causes: Reactor trip - on 1 of 4 S/Gs in 2 of 4 channels MD CA pumps auto-start - on 1 of 4 S/Gs in 2 of 4 channels TD CA pump auto-start on 2 of 4 S/Gs in 2 of 4 channels A. Incorrect: CF pumps will not trip this is done by a local operator action in AP-17, TD CA pump auto-starts. Plausible: MD CA pumps will start when S/G levels < 17% on 1/4 S/Gs Incorrect: The CF pumps will continue to run until tripped by local operator action in AP -17 Plausible: The MD and TD CA pumps auto start

B.

Ques_501

For Official Use Only Page 39

Question #32
C.

McGuire Nuclear Station

RO Exam

D.

Incorrect: The TD CA pump will auto start. Plausible: The MD CA pumps auto start and the CF pumps remain running Correct answer:

Ques_501

For Official Use Only Page 40

Question #33

McGuire Nuclear Station

RO Exam

Bank Question: 531 1 Pt(s)

Answer: C

Unit 2 is operating at 75% power when a load rejection occurs. Which one of the following statements correctly describes the response of 2CM-420 (Load Rej Byp) to this transient? A. 2CM-420 closes to prevent condensate water from being diverted to the suction of the hotwell booster pumps from the condensate booster pumps to assure minimum flow to the CF pumps. 2CM-420 closes to prevent diversion of water from the "C" heater drain tank back to the UST thereby ensuring sufficient CF pump suction pressure. 2CM-420 opens to divert condensate flow directly to the condensate booster pump suction to ensure that CF pumps have sufficient suction pressure.

B.

C.

D.

2CM-420 opens to divert condensate flow, bypassing around the condensate booster pumps, directly to the CF pumps to assure minimum flow requirements.

Distracter Analysis: A. Incorrect: CM-420 opens - does not close. Does not prevent water from being recirculated around the hotwell pumps. Plausible: this function is performed by CM-407 which opens to assure minimum flow around the hotwell pumps to prevent water hammer on the CM system during startup. Incorrect: CM-420 opens - does not close. Does not prevent a loss of water to the condensate booster pump suction. Plausible: CM-227 opens to recirc condensate from the C feedwater heater to the USTs to assure minimum recirc flow on the CBPs Correct answer Incorrect: CM-420 does not provide a flow path around the condensate booster pumps directly to the CF pumps to meet minimum flow requirements Plausible: CM-420 opens to provide bypass flow but directly to the CBPs not the CF pumps

B.

C. D.

Ques_531

For Official Use Only Page 41

Question #34

McGuire Nuclear Station

RO Exam

Bank Question: 538 1 Pt(s)

Answer: C

Unit 1 is operating at 28% power during a plant startup to 100%. Given the following conditions on the 1C steam generator: Main feedwater regulating valve (FRV) is in AUTO control at 25% open Bypass FRV is in MANUAL control at 100% open Steam flow channel I fails high

Which one of the following statements correctly describes the plant response for the 1C steam generator FRVs?? A. Main FRV modulates open to increase feedwater flow and steam generator water level increases to the high level alarm setpoint. Main FRV modulates shut to reduce feedwater flow and steam generator level decreases to the low level alarm setpoint. Main FRV modulates open to increase feedwater flow but sufficient level error signal develops to restore CF flow to normal without reaching the high level alarm setpoint.

B.

C.

D.

Main FRV modulates shut to reduce feedwater flow but sufficient level error signal develops to restore CF flow to normal without reaching the low level alarm setpoint.

Distracter Analysis: A. Incorrect: SIG water level does not increase to the high level alarm as level error quickly overcomes flow mismatch Plausible: CF control valves open to 120% Incorrect: FRVs do not modulate shut and SG water level does not fall to the low level alarm Plausible: If the candidate reverses the effect of the instrument failure - this is what happens for a steam flow transmitter failing low Correct answer Incorrect: FRVs do not modulate shut Plausible: level error does overcome flow mismatch and level will be restored

B.

C. D.

Ques_538

For Official Use Only Page 42

Question #35

McGuire Nuclear Station

RO Exam

Bank Question: 591 1 Pt(s)

Answer: D

Unit 2 was operating at 5% power during a plant startup when the following sequence of actions occurred. Opened 2NV-265B Started Boric Acid Transfer pump #2A

If no other operator actions occurred, which of the following statements correctly describes the response of reactor power and control rods? A. Power remains at 5% Control rods drive in Power remains at 5% Control rods do not move Power decreases Control rods drive in Power decreases Control rods do not move Distracter Analysis: The candidate must recognize that: The sequence of operations amounts to emergency boration of the reactor. Control rods are in manual at this point during the startup Power decreases due to boron addition A. B. C. D. Incorrect: power will decrease Plausible: control rods would drive in IF they were in auto control Incorrect: power will decrease Plausible: control rods will not move Incorrect: control rods are in manual and will not move Plausible: power will decrease Correct answer

B.

C.

D.

Ques_591

For Official Use Only Page 43

Question #36

McGuire Nuclear Station

RO Exam

Bank Question: 592 1 Pt(s)

Answer: B

Unit 1 was cooling down in Mode 4 when the I Al KC pump trips. Given the following conditions: Both trains of KC were initially in operation 1A2 KC pump was secured due to high KC flow Both trains of ND were aligned for RIR shutdown cooling NCS temperature was 205 F

If train A KC pumps cannot be restarted, which one of the following list of actions is the complete list of actions that must be taken to prevent damage to equipment? A. B. Stop ND pump lA Stop ND pump lA Isolate ND flow through the lA ND heat exchanger Cross-connect KC flow to the 1A ND heat exchanger Cross-connect KC flow through the lA ND Pump mechanical seal heat exchanger

C.

D.

Stop ND pump lA Isolate KC flow through the letdown heat exchanger

Distracter Analysis: Upon a loss of KC to an operating ND train, AP/21 requires two actions (per Foldout page): Stop the associated ND pump Isolate flow to the associated ND HX A. Incorrect: Must also stop flow to the ND HX per AP/21 Plausible: action to stop the lA ND pump is correct. There is a separate operating precaution to maintain flow through the ND HX > 2000 gpm to prevent water hammer but it does not apply to this case. Correct answer Incorrect: cannot cross-connect B train KC flow to the A train ND 11X under these conditions AP/21 specifies that flow must be stopped to the ND HX. Plausible: There is a precaution to ensure that KC flow is maintained to ND mechanical seal HX for all operating ND pumps Incorrect: no need to secure flow the letdown HX

B. C.

D.

Ques_592

For Official Use Only Page 44

Question #36

McGuire Nuclear Station

RO Exam

Plausible: this would be required if KC was lost when the plant was at power and NCS temp was higher to prevent flashing in the letdown line.

Ques 592

For Official Use Only Page 45

Question #37

McGuire Nuclear Station

RO Exam

Bank Question: 593 1 Pt(s)

Answer: B

Unit 2 was operating at 99% power when a steamline rupture occurred. Given the following events and conditions: 0200 The operators enter AP/01 (Steam Leak) 0200 The operators reduce turbine load to match Tave and Tref 0201 The operators start a second NV pump and isolate letdown 0202 NLOs start investigating for the location of the steam leak 0203 "P/R OVER POWER ROD STOP" alarm the RO reports that power has turned and is decreasing. 0204 STA reports pressurizer level is decreasing and cannot be maintained 0205 The turbine building operator reports that the line to the atmospheric dump valves has a steam leak and cannot be isolated If no safety injection has occurred, pressurizer pressure is maintained and no reactor trip signals are received prior to 0205, which one of the following operator responses is correct? A. B. Manually trip the reactor at 0203 Manually trip the reactor at 0204

C. D.

Manually trip the reactor at 0205 Commence a rapid down power using AP/04 at 0205

Distracter Analysis: A. Incorrect: no requirement to trip the reactor because reactor power has turned and is decreasing. Not approaching the overpower automatic reactor trip at 109% in 2 of 4 channels. Plausible: shows a power mismatch reactor power reaches 103% on I of 4 PR channels to cause C-2. OMP 4-3 requires the operator to trip when an automatic safeguards action setpoint is approached to avoid challenging the automatic safeguards function. Correct answer required to trip under AP/01 (and many other procedures) if you cannot maintain pressurizer level with 2 NV pumps Incorrect: required to trip when PZR level cannot be maintained Plausible: if the candidate thinks that a reactor trip is required because the steam leak was not isolated. Incorrect: required to trip when PZR level cannot be maintained

B.

C.

D.

Ques_593

For Official Use Only Page 46

Question #37

McGuire Nuclear Station

RO Exam

Plausible: this would be the correct answer if not required to trip at 0204.

Ques 593

For Official Use Only Page 47

Question #38

McGuire Nuclear Station

RO Exam

Bank Question: 594 1 Pt(s)

Answer: C

Which one of the following statements correctly describes the operation of the condenser dump valves during a loss of condenser vacuum? A. Condenser steam dump valves do not open because the C-7A arming signal is blocked. Condenser steam dump valves isolate on a P4 signal when the reactor trips. Condenser steam dump valves isolate upon a loss of C-9 signal when condenser pressure drops below 20 inches of vacuum.

B.

C.

D.

Condenser steam dump valves continue to dump steam to the condenser until condenser reaches atmospheric pressure.

Distracter Analysis: A. Incorrect: C-7A will arm on a 10% step change in load Plausible: If the C-7A interlock did not pick up and arm the condenser dump valve, they would not open Incorrect: The P4 signal does not close the condenser dump valves Plausible: A P4 signal would block the atmospheric dump valves Correct answer Incorrect: The condenser dump valves would close on loss of C-9 Plausible: The condenser dump valves normally open for a reactor trip.

B. C. D.

Ques_594

For Official Use Only Page 48

Question #39

McGuire Nuclear Station

RO Exam

Bank Question: 596 1 Pt(s)

Answer: C

Units 1 and 2 were operating at 100% power when a fire broke out in the back of the control room. Given the following conditions: The fire has not effected or degraded any control systems Heavy black smoke is throughout the control room The SRO implements AP/17 (Loss of Control Room)

Which one of the following statements correctly describes the operator response to this event? A. Immediately trip both unit turbines and reactors and evacuate the control room to the auxiliary shutdown panels. Evacuate the control room; trip both unit turbines and reactors on the way to the auxiliary shutdown panel. Evacuate the control room: proceed to the auxiliary shutdown panels and trip both unit turbines and reactors when directed by the SROs at the auxiliary shutdown panels. D. Evacuate the control room, proceed to the safe shutdown facility and trip both unit turbines and reactors when directed by the SRO at the standby shutdown facility.

B.

C.

Distracter Analysis: A. Incorrect: There is a specific caution in AP-17 that warns against tripping the reactor until the SRO is stationed at the ASP and the SRO directs the reactor be tripped. Plausible: This could be a conservative thing to do before evacuating. Many plants require the reactor to be tripped prior to evacuation. Incorrect: There is a specific caution in AP-17 that warns against tripping the reactor until the SRO is stationed at the ASP and the SRO directs the reactor be tripped. Plausible: If the candidate does not recognize this caution. This would be a convenient and expeditious action to take. This was the old AP-17 response and is now the AP/24 response.

B.

Ques_596

For Official Use Only Page 49

Question #39
C. D.

McGuire Nuclear Station

RO Exam

Correct answer Incorrect: Evacuate to the ASP not the SSF Plausible: The operators would evacuate to the SSF if the fire degraded control systems

Ques_596

For Official Use Only Page 50

Question #40

McGuire Nuclear Station

RO Exam

Bank Question: 597 Pt(s)

Answer: D

Unit 2 was shutdown in Mode 4, cooling down to a refueling outage. The following annunciator lights are provided for identification purposes in answering the question below: Annunciators on panel 2AD-10: E-I = Upper Cont. Airlock Aux. Door Open F-1 = Upper Cont. Airlock Rx. Door Open E-2 = Lower Cont. Airlock Aux. Door Open F-2 = Lower Cont. Airlock Rx. Door Open Annunciators on panel 2AD-13: A-8 = VE Door Open Approval was given for normal passage into the containment to perform work no approval has been given for any compensatory measures. Which one of the following alarm conditions requires corrective action under MSD 585, (Reactor Building Personnel Access and Material Control for Modes 1, 2, 3 and 4)? A. B. C. D. 2AD-10 E-1 and 2AD-10 E-2 actuated 2AD-10 F-1 and 2AD-10 F-2 actuated 2AD-10 E-1 and 2AD-10 F-2 actuated 2AD-10 E-2 and 2AD-13 A-8 actuated

Distracter Analysis: A. Incorrect: Allowable to have one door open in each airlock Plausible: 2 airlock doors are open at the same time B. Incorrect: Allowable to have one door open in each airlock Plausible: 2 airlock doors are open at the same time C. Incorrect: Allowable to have one door open in each airlock Plausible: 2 airlock doors are open at the same time D. Correct answer although only one containment airlock door is open, the alarm on the VE annulus door if left open for > 2minutes, requires compensatory security action because an ECCS phase B actuation will auto-start the VE system to establish a negative pressure in the annulus which can't occur if the annulus door is open.

Ques_597

For Official Use Only Page 51

Question #41

McGuire Nuclear Station

RO Exam

Bank Question: 598 1 Pt(s)

Answer: C

Unit 1 was responding to an internal flow blockage condition in the core that required a reactor trip and entry into FR-C.2 (Response to Degraded Core Cooling). Step 15.e of FR-C.2 states: Dump steam to condenser from intact S/Gs while maintaining cooldown rate in NC T-colds less than 100 F in an hour. Given the following times and temperatures during the event: Time 0200 NC T-cold F 557 Time 0300 NC T-cold F 520 Time 0400 NC T-cold F 420 0210 560 0310 495 0410 405 0220 565 0320 468 0420 390 0230 558 0330 467 0430 371 0240 540 0340 444 0440 350 0250 530 0350 428 0450 320 0300 520 0400 420 0500 310

If the cooldown started at 0230, what time did the operators first exceed the cooldown limit of FR-C.2? A . B . C . D . 0240 0320 0350 0450 Distracter Analysis: A. Incorrect: cooldown rate was 45 F for 1 hour - did not exceed 100 F in one hour Plausible: 108 F/hr instantaneous cooldown rate for the 10-minute interval exceeded 100 F/hr. In addition, the applicant has to consider the NCS temperature prior to the trip. Incorrect: cooldown rate was 97 F for 1 hour - did not exceed 100 F in one hour but came very close.

B.

Ques598

For Official Use Only Page 52

Question #41

McGuire Nuclear Station

RO Exam

C. D.

Plausible: 162 F/hr instantaneous cooldown rate for the 10-minute interval exceeded 100 F/hr. Correct Answer: cooldown rate was 102 OF in one hour the instantaneous cooldown rate was only 96 F/hr. Incorrect: although the cooldown rate was 108 F for 1 hour, the operators exceeded the limit at 0350 not the first time Plausible: Exceed both the instantaneous rate and the 1-hour rate. If the applicant misses the correct calculation for answer C, this is the next time when the cooldown rate is exceeded.

Ques_598

For Official Use Only Page 53

Question #42

McGuire Nuclear Station

RO Exam

Bank Question: 600


1 Pt(s)

Answer: C

Unit 1 was conducting a reactor startup with NC pressure at 2200 psig and intermediate range level at 1.5x10' amps when a control rod in control bank "A" drops into the core. Given the following events and conditions: The reactor remains critical at 1.0x10 amps during the recovery of the control rod. AP/14 (Rod Control Malfunction) is implemented to recover the dropped rod. Tave drops to 550F while recovering the rod.

Which one of the following statements correctly describes the required actions (if any)? A. No additional action is required as long as the reactor does not increase power above 5%. Within 30 minutes, adjust power range N/Is to increase reactor power so that reactor power and thermal power best estimate are equal. Within 30 minutes increase NC system temperature above 551 F or commence a normal shutdown.

B.

C.

D.

Immediately trip the reactor and enter E-0 (Reactor Trip or Safety Injection).

Distracter Analysis: A. Incorrect: Tech Spec 3.4.2 is applicable in mode 2 when critical. Plausible: The change from mode 2 to mode occurs when power exceeds 5%. If the candidate thinks that ITS 3.4.2 only applies in mode 1, this would be a plausible mistake. Incorrect: Thermal power would indicate lower, not higher than reactor power due to increased thermalization of the neutrons. While NI adjustment is a problem, this action does not comply with tech spec 3.4.2. Plausible: This was a recent event {July 1, 1998) at McGuire but the temperature remained under 551 F for only 4 minutes. The concern expressed in the lessons learned report was for the NI power to thermal power mismatch. Correct answer

B.

C.

Ques_600

For Official Use Only Page 54

Question #42
D.

McGuire Nuclear Station

RO Exam

Incorrect: An immediate reactor trip is NOT required. AP/14 requires a controlled shutdown to mode 3 - but with temp only 1 F below minimum required for criticality, the best choice is C. Shutting down to mode 3 is not a distracter. Plausible: Seems like an appropriate response to finding yourself below the minimum temperature for criticality a conservative response.

Ques_600

For Official Use Only Page 55

Question #43

McGuire Nuclear Station

RO Exam

Bank Question: 601 1 Pt(s)

Answer: A

Which one of the following selections correctly describes reflux boiling flow path during a large break LOCA. Steam enters the (1) ____ of S/G U-tubes where the steam condenses and re-enters the core area via the S/G _(2)

U)
A. B. C. D. hot leg_ hot leg cold leg cold leg

a)
hot leg cold leg hot leg cold leg

Distracter Analysis: A. B. C. D. Correct answer Incorrect: steam returns via the hot leg Plausible: the first part of the answer is correct Incorrect: the steam enters the hot leg Plausible: the second part of the answer is correct Incorrect: cold legs are not affected during reflux boiling Plausible: psychometric balance

Ques_601

For Official Use Only Page 56

Question #44

McGuire Nuclear Station

RO Exam

Bank Question: 602 1 Pt(s)

Answer: A

Unit 2 is responding to a small break LOCA in ES-1.1 (SI Termination). Given the following plant conditions: NCPs tripped Pressurizer level is steady Only one train of ECCS is injecting Loop A temperatures are representative of all 4 loops Steam generator pressures are the same as steam header pressure

Which one of the following sets of plant parameters is indicative of natural circulation occurring in the steam generators per enclosure 2 of ES-1.1? Time A . Steam Header Pressure (psig) NC System Pressure (psig) Loop A T-hot (F) Loop A T-cold (F) Steam Header Pressure (psig) NC System Pressure (psig) Loop A T-hot (F) Loop A T-cold (F) Steam Header Pressure (psig) NC System Pressure (psig) Loop A T-hot (F) Loop A T-cold (F) Steam Header Pressure (psig) NC System Pressure (psig) Loop A T-hot (F) Loop A T-cold (F) 0200 1042 1968 579 548 1042 1968 579 548 1042 1968 579 548 1042 1968 579 548 0205 1009 1964 574 544 1009 1972 582 544 1047 1964 574 549 1047 1972 582 544 0210 976 1960 569 540 976 1975 585 540 1050 1960 569 548 1050 1975 585 540 0215 945 1958 564 536 945 1981 595 536 1052 1958 564 550 1052 1981 595 536

B .

C .

D .

Distracter Analysis: The following conditions support natural circulation: SIG pressure stable of decreasing T-hot stable or decreasing T-cold stable or decreasing NC subcooling > 0 - NC pressure may trend up or down.

Ques_602

For Official Use Only Page 57

Question #44
A.

McGuire Nuclear Station

RO Exam

B. C.

D.

Correct: This shows indication of natural circulation flow occurring decreasing SIG pressure, T-cold at SIG saturation conditions and decreasing, T-hot decreasing. Incorrect: T-hot is increasing while steam pressure is decreasing Plausible: Steam pressure and T-cold are both decreasing Incorrect: Steam pressure is increasing and T-cold is tracking along with this trend. Temperature difference is decreasing indicating that heat removal rate is decreasing. This is a classic case of gas binding Plausible: T-hot is decreasing. Incorrect: Steam pressure increasing and T-hot is increasing. Plausible: T-cold is decreasing

Ques 602

For Official Use Only Page 58

Question #45

McGuire Nuclear Station

RO Exam

Bank Question: 603 1 Pt(s)

Answer: B

Unit 1 was operating at 100% power when the 1 A NV pump failed. Given the following events and conditions: 1B NV pump was tagged out of service for maintenance The Positive Displacement NV pump was tagged out of service The plant is at normal operating temperature, pressure and level Normal letdown is in service on the 75 gpm orifice Identified leakage is at the Tech Spec Limit Unidentified leakage is 1 cc/hr

If no operator actions are taken, how much time will elapse before the pressurizer level reaches the low level alarm and the heaters trip? REFERENCES PROVIDED: Curve Book Encl 7.38 (PZR Volume vs. Level) ITS 3.4.13 page 1 A. B. Less than 45 minutes 45 -55 minutes

C. D.

55 - 65 minutes Longer than 65 minutes

Distracter Analysis: The NC coolant will let down to the VCT at a rate of 75 gpm until 17% is reached in the pressurizer. At 17%, the PZR low-level alarm will isolate letdown. Pressurizer level 55% = 7800 gal Pressurizer at 17% = 2900 gal Letdown flow = 75 gpm until isolation at 17% PZR level Identified leakage = 10 gpm (includes NCP seal leakage to NCDT) Unidentified leakage is negligible and may be ignored NCP seal leak off= 4x3 = 12 gpm into VCT Total flow rate out of the NC system = 97 gpm until letdown isolation Time to reach 17% PZR level = (7800-2900 gal) / (97 gpm) = 50.5 minutes

Ques_603

For Official Use Only Page 59

Question #45
A.

McGuire Nuclear Station

RO Exam

B. C.

D.

Incorrect: too short Plausible: if the candidate adds total NCP #1 seal injection flow instead of seal leak off (8x4=32 gpm) to the letdown leak rate 75 gpm and Tech Spec leak rate 10 gpm Time = 41 minutes, misreads the pressurizer level tank curve or makes another mistake. Correct answer: Time to reach 17% PZR level = (7800-2900) / (75 + 10 +12 gpm) = 50.5 minutes Incorrect: time is too long Plausible: if candidate forgets to add in max allowable Tech Spec leakage or NCP seal leak off Time to 17% PZR level = (7800-2900) / (75+10gpm) = 57.6 min Time to 17% PZR level = (7800-2900)1(75+12 gpm) = 56.3 min Incorrect: too long Plausible: if the candidate does not consider the letdown rate of 75 gpm and only considers the Tech Spec leak rate 10 gpm and/or NCP seal leakoff, misreads the pressurizer level tank curve or makes another mistake.

Ques_603

For Official Use Only Page 60

Question #46

McGuire Nuclear Station

RO Exam

Bank Question: 604 1 Pt(s)

Answer: D

Unit 1 was operating at 25% power following a reactor startup when intermediate range channel N35 failed. Given the following conditions and events: N35 repairs have been made and N35 is being returned to service N36 reads1.5x104 amps The N35 "level trip" switch was returned to the "normal" position

If all power range nuclear instruments and N36 have been properly adjusted, which of the following operator conditions (if any) would cause the reactor to trip? A. N35 "Operation Selector" switch was left in "10-' " position after retesting N35 was significantly under-compensated

B.

C. D.

N35 control power fuses were never reinstalled A reactor trip would not occur

Distracter Analysis: At 10%, the operators manually block the hi IR Rx trip by procedure after P-10 is enabled on 2 of 4 PR detectors > 10% A. Incorrect: - the operation selector switch is taken out of the circuit when the level trip switch is taken to normal and all IR high flux Rx trips are blocked by P-10. Plausible: - if the candidate thinks that a test signal can be inserted with level trip switch in the normal position Incorrect: - The IR high flux trip is blocked by P-10 Plausible: inserting a test signal can cause a trip signal to be generated from the IR drawer but will not go to SSPS Incorrect: - the reactor is above P-10 and although undercompensation of N35 could cause the high flux setpoint to be reached, the IR trips are disabled by P-10 Plausible: - If the candidate does not recognize that a N36 level of 1.5x104 amps is above P-10 Correct answer

B.

C.

D.

Ques_604

For Official Use Only Page 61

Question #47

McGuire Nuclear Station

RO Exam

Bank Question: 605 I Pt(s)

Answer: B

Unit 2 was operating at 100% power when a reactor trip occurred. The reactor trip caused the initiation of a tube leak in the 2B SIG. The leak rate was 100 gpm. Given the following conditions: 2EMF-33 (Condenser Air Ejector Exhaust) alarms in trip 2

If all the automatic features operate as designed (without operator intervention), which one of the following indications will provide the best indication (most sensitive and timely) to confirm that a S/G tube leak has occurred? A. B. Comparing S/G feed flow to steam flow mismatch Observing 2EMF-10, 11, 12 and 13 (steamline hi rad)

C. D.

Observing 2EMF-34 (S/G sample line lo range) Observing 2EMF-71, 72, 73, 74 (N16 leakage)

Distracter Analysis: A. Incorrect: Not a sensitive method of comparison requires large gpm leak rates before this is noticeable. Plausible: This method will show gross SGTRs Correct answer: normally, EMF-71-74 are the most sensitive monitors. But these monitors detect N'' y radiation that has a high energy (7 MeV) y that only is generated when the reactor is operating at power (requires a neutron flux). Incorrect: SIG sample line will isolate at EMF-33 trip 2 the sample line can only be lined up to 1 SIG at a time. If the leak is not in that S/G, there will be no indication of anything after isolation. Prior to isolation, it may show an increasing trend due to a general build up of activity in the feedwater. Plausible: This would be a good answer if the automatic isolation did not occur Incorrect: most sensitive method as it detects N'' y radiation Plausible: This was the correct answer for the 1997 NRC exam when the premise of the question had the reactor was operating at 100% power. In this question, the reactor has tripped and neutron flux has decreased causing the N'' y to decay off (TY2 is 7 seconds)

B.

C.

D.

Ques_605

For Official Use Only Page 62

Question #47

McGuire Nuclear Station

RO Exam

so that by the time that the steam line monitors see the contents of the SIG, the 1\1' y has decayed away.

Ques_605

For Official Use Only Page 63

Question #48

McGuire Nuclear Station

RD Exam

Bank Question: 606 1 Pt(s)

Answer: D

Unit 1 was operating at 100% when a steam generator tube rupture occurred in the 1B S/G. Given the following list of valves in the SIG sample and blowdown systems: 1NM-267 SIG Sample HDR RAD Monitor Inlet Isolation Blowdown Blowoff Automatic Isolation Valves o 1BB-119 = from the lA S/G o 1BB-120 = from the 1B S/G o 1BB-121 = from the IC S/G o 1BB-122 = from the 1D S/G S/G Sample HDR to Conventional Sample System valves o 1NM-269 = from the 1A S/G o 1NM-270 = from the 1B SIG o 1NM-271 = from the 1C S/G o 1NM-272 = from the 1D S/G

Which one of the following statements correctly describes the complete set of valves that would automatically close?

A.
B. C.

1NM - 267
1NM-267, 1BB-120, 1NM-270 1NM-267 1NM-269, 1NM-270, 1NM-271, 1NM-272

D.

1NM-267 1BB-119, 1BB-120, 1BB-121, 1BB-122, 1NM-269, 1NM-270, 1NM-271, 1NM-272

Distracter Analysis: A. Incorrect: insufficient Plausible: closes sample header B. Incorrect: insufficient partial list Plausible: these valves would isolate sample flow from the 1B S/G with the ruptured tube C. Incorrect: insufficient partial list does not include blowdown system valves Plausible: all sample system (NM) valves isolate D. Correct answer complete list

Ques_606

For Official Use Only Page 64

Question #49

McGuire Nuclear Station

RO Exam

Bank Question: 607 1 Pt(s)

Answer: B

Unit 1 was conducting an approved release from WGDT "B". Which one of the following conditions would automatically terminate this release? A. B. C. D. lEMF-35L (unit vent particulate monitor) trip 2 IEMF-36L (unit vent gas) trip 2 IEMF-36H (unit vent gas) trip 2 lEMF-37 (unit vent iodine) trip 2

Distracter Analysis: A. Incorrect: lEMF-36L terminates the release Plausible: lEMF35L monitors the unit vent for radioactive particulate and will stop the aux building unfiltered exhaust fans Correct Answer: Incorrect: 1EMF-36L terminates the release Plausible: 1 EMF-36H has no automatic isolation functions strictly for monitoring vent exhaust during an accident Incorrect: EMF-36L terminates the release Plausible: 1EMF-37L monitors the unit vent for radioactive iodine and will stop the aux building unfiltered exhaust fans

B. C.

D.

Ques_607

For Official Use Only Page 65

Question #50

McGuire Nuclear Station

RO Exam

Bank Question: 608 1 Pt(s)

Answer: D

OP/O/A/6350/001C, (250 VDC Auxiliary Power System) contains the following precaution: The DC bus ties will normally remain open. They are only to be closed during equalization charges of batteries, or on a loss of a battery or battery charger. " Which one of the following is the basis for this precaution? A. Prevents damage to the battery chargers resulting from both battery chargers simultaneously supplying the same bus at different voltage outputs. Prevents overloading one battery if the battery terminal voltages are significantly different which would lead to excessive hydrogen evolution and a possible explosive hazard. Ensures both battery chargers are operated in parallel to be able to reach the terminal voltage (-271 VDC) required for an equalization charge. Ensure DC channels remain independent of each other and that a fault on one bus does not adversely affect the other bus

B.

C.

D.

Distracter Analysis: A. Incorrect: The reason is DC channel independence Plausible: If 2 battery chargers were run in parallel, they could fight each other if they had vastly different voltage output characteristics. B. Incorrect: The reason is DC channel independence Plausible: If the candidate thinks that batteries with different charge states could fight each other if connected in parallel. C. Incorrect: The reason is DC channel independence - during an equalization charge, the battery being charged is charged from the standby charger due to the high termination voltage required to finish the charge. Plausible: DC ties are closed during an equalization charge to allow one battery to power both buses to allow the charged battery to achieve termination voltage which is higher than normal voltage and may damage equipment if applied on the bus. D. Correct answer

Ques_608

For Official Use Only Page 66

Question #51

McGuire Nuclear Station

RO Exam

Bank Question: 609 1 Pt(s)

Answer: C

The Unit 1 SRO was monitoring a release from the waste monitor tank. Which one of the following alarms would terminate this release automatically? A. B. C. D. IEMF-31 (Turbine Bld Sump Disch) trip 2 1EMF-44(L) (Cont Vent Drn Tank Out) trip 2 1EMF-49(L) (Liquid Waste Disch) trip 2 1EMF-50(L) (Waste Gas Disch) trip 2

Distracter Analysis: A. Incorrect: does not monitor the WMT release path Plausible: would terminate a liquid release from the turbine building sump Incorrect: does not monitor the WMT release path Plausible: would terminate a liquid release from the VUCDT using the same automatic valves as the 1EMF-49 (WP-35 and WP-46) Correct answer: closes WP-35 and WP-46 to stop the release. Incorrect: does not monitor the WMT release path Plausible: would terminate a release from the WGTD

B.

C. D.

Ques_609

For Official Use Only Page 67

Question #52

McGuire Nuclear Station

RO Exam

Bank Question: 611 1 Pt(s)

Answer: D

Unit 2 has just completed a plant shutdown after a record run when a leak was suspected from the relief valve on the waste gas decay tank that had been placed in service at the start of the shutdown. The tank was empty prior to being placed in service for the shutdown. The SRO directs RP to confirm the existence and determine the location of the suspected leak. Which one of the following statements would be an effective method of locating the leak in the waste gas system? A. Radiological Protection could monitor for alpha particle emission from the radioactive decay of entrained tritium gas. Radiological Protection could monitor for flammable levels of Hydrogen gas that accumulate in the WGDTs from purging the VCT. Radiological Protection could monitor for ammonia (NH4) gas from the breakdown of ammonium hydroxide (NH4OH), which is added to the NC system for pH control.

B.

C.

D.

Radiological Protection could monitor for beta/gamma emission from the radioactive decay of particulate from long-lived fission product gaseous isotopes.

Distracter Analysis: A. Incorrect: Tritium gas does not emit alpha particles will not detect Tritium by monitoring for alpha emission. Plausible: Tritium builds up in the waste gas system from reactor operations and has a relatively long half-life. Incorrect: waste gas system recombiners remove Hydrogen during shutdown prior to storage in a WGDT. The Hydrogen gas concentration is reduced below flammable levels prior to storage in the WGDT to assure that it is safe to release to the environment. Plausible: Hydrogen gas is removed from the VCT, PRT and NCDT by the waste gas system during shutdown. Incorrect: Ammonium Hydroxide is not added to the NC system for chemistry control of pH_ It is added to the condensate system Plausible: Ammonia gas is produced in the NC system by the breakdown of Hydrazine (N1H4) when temperature is raised above 250 F during startup. Ammonia gas builds up in the pressurizer and

B.

C.

Ques_611

For Official Use Only Page 68

Question #52

McGuire Nuclear Station

RO Exam

D.

enters the waste gas system during degas operations. It is not removed in the waste gas decay system. Correct answer: The waste gas system would contain fission product gasses.

Ques_611

For Official Use Only Page 69

Question #53

McGuire Nuclear Station

RO Exam

Bank Question: 612 Pt(s)

Answer: A

A large break LOCA occurred on Unit 1. The operators entered ECA-1.1 (Loss of Emergency Coolant Recirculation) for a complete loss of emergency coolant recirculation due to a blockage in the containment sumps, causing large increase in containment temperatures and pressures. Which one of the following parameter changes would indicate that significant core uncovery was occurring? A. B. C. Source range instruments show a rapid increase Power range instruments show a rapid increase 1EMF-51/52 (Containment TRN A/B High Range) shows a rapid increase IEMF-9 (Rx Bld Incore Inst Rm) radiation ARM shows a rapid increase

D.

Distracter Analysis: The loss of containment cooling will cause core uncovery. All of the answers will show indications of degrading conditions inside containment. A. B. Correct answer Incorrect: power range instruments are calibrated to detect high levels of neutron flux and to compensate for gamma flux. Plausible: An increase in power range output would indicate recriticality, not uncovery. Incorrect: EMF-51/2 (Containment High Range Radiation Monitor) would rise as fission products are transported into the containment atmosphere but would not show a rapid increase when core uncovery occurs. The loss of shielding effect (water shielding EMF-51/2) would be very small compared to the other effects primarily the amount and location of fission products in the containment atmosphere. Plausible: EMF-51 /2 will increase throughout the accident Incorrect: The reactor building incore instrument room is essentially the same area as the Seal Table from the SAMGs. The radiation increase is indicative of a core melt and failure of an incore instrument tube Plausible: Used by SAMGs

C.

D.

Ques 612

For Official Use Only Page 70

Question #54

McGuire Nuclear Station

RO Exam

Bank Question: 613 1 Pt(s)

Answer: B

Unit 2 was operating at 100% when the following indications occurred: Pressurizer level began decreasing 1 A NV Pump ammeter showed running amps decreased Normal letdown was in service

If all automatic control system appeared to operate normally, which one of the following conditions would cause the 1 A NV pump running amps to decrease to the minimum value? A. B. C. D. 2NV-238 (Charging Line Flow Control) failed open 2NV-238 (Charging Line Flow Control) failed closed 2M-241 (Seal Inj Flow Control) failed open 2NV-241 (Seal Inj Flow Control) failed closed

Distracter Analysis: A. Incorrect: pump amps and pressurizer level would initially increase Plausible: If the candidate does not understand the charging flow path or does not understand the relationship between pump amps and flow. Correct answer: This would block the charging flow path and pumps amps would reduce to minimum as all the charging flow was diverted through NV-150 and NV-151 miniflow valves Incorrect: this would increase charging flow, which would increase charging pump amps. It would also increase pressurizer level, which would cut back on charging pump speed to offset the flow increase and stabilize the pressurizer level. Plausible: If the candidate thought that this could divert charging flow or did not understand the relationship between pump amps and flow. Incorrect: NV-241 closing would increase backpressure on the charging system, which would divert more charging flow through the NCP seals. However, the drop in pressurizer level would act to increase the running speed of the charging pump to compensate for the flow reduction. The overall effect would be to increase pump amps as flow would remain the same but at a higher backpressure.

B.

C.

D.

Ques_613

For Official Use Only Page 71

Question #54

McGuire Nuclear Station

RO Exam

This would also cause letdown isolation due to the loss of NV flow through the regenerative heat exchanger. Plausible: If the candidate does not consider the effect of the charging pump speed control circuit.

Ques_613

For Official Use Only Page 72

Question #55

McGuire Nuclear Station

RO Exam

Bank Question: 615 1 Pt(s)

Answer: D

E-1 (Loss of Reactor or Secondary Coolant) step 19 directs the operator to realign the ND system to the hot leg flow path. Which one of the following statements correctly describes the basis for this realignment? A. Realigns the ECCS flow to take suction on the containment sump to avoid draining the FWST. Realigns the ECCS flow to cool the reactor vessel upper internals package. Realigns the ECCS flow to pass through the ND heat exchangers to remove decay heat. Realigns the ECCS flow to reverse flow through the core to address the consequences of boron stratification/plate out.

B.

C.

D.

Distracter Analysis: A. Incorrect: ECCS flow realigned to containment sump much earlier than 6 hours and realigned from injection to cold leg recirc not hot leg recirc Plausible: describes the basis for cold leg recirc mode. Incorrect: also correct for cold leg recirc not the basis for shift to hot leg recirc Plausible: describes an inconsequential result of the realignment. Incorrect: also correct for cold leg recirc not the basis for shift to hot leg recirc Plausible: describes another basis of the cold leg recirc path. Correct:

B.

C.

D.

Ques_615

For Official Use Only Page 73

Question #56

McGuire Nuclear Station

RO Exam

Bank Question: 616 1 Pt(s)

Answer: A

Unit 1 is operating at full power. Given the following events and conditions on the NCPs: An OAC alarm indicates loss of KC flow to the to the NCPs. The KC supply outside containment isolation valve (1KC-338) is closed. Seal injection flow rate to each NCP is 8 gpm. What are the likely consequences if the operators do not respond to this alarm? A. B. C. D. The NCPs should operate without KC indefinitely. The NCP motor bearings will overheat causing motor damage. The NCP stator windings will overheat causing motor damage. The NCPs will experience seal failure within 3-5 minutes.

Distracter Analysis: This failure condition is not explicitly covered in the training materials however the candidates should be familiar with the component failure. When KC flow is stopped to the NCPs, this will stop cooling flow to the motor lube oil coolers for the bearings. This will cause lube oil temperatures to exceed allowable values leading to bearing failure. A. Incorrect: Although the NCP thermal barriers can be operated indefinitely as long as seal injection flow is maintained, the loss of KC flow to the motor coolers will cause motor bearing temperatures to overheat. Plausible: The NCP seals will operate indefinitely without KC Correct Answer : Motor bearings will overheat Incorrect: NCP Stator windings are cooled by air coolers Plausible: if the candidate forgets that the NCP stators are air cooled Incorrect: seal failure will not occur as long as seal injection is maintained. Plausible: If seal injection is lost to a pump along with KC flow to the thermal barrier, this would be true.

B. C.

D.

Ques_616

For Official Use Only Page 74

Question #57

McGuire Nuclear Station

RO Exam

Bank Question: 619 1 Pt(s)

Answer: B

Unit 1 is recovering from a loss of offsite power in ES-0.2 (Natural Circulation Cooldown). The operators reach step 17 which states: IF AT ANY TIME cooldown rate must be raised to greater than 50F in an hour, THEN GO TO EP/1/A/5000/ES-0.3 (Natural Cooldown with Steam Void in Vessel) Given the following plant conditions: T-hot = 560 F NC Pressure = 1250 psig RVLIS = 100% upper range, 64% lower range The subcooling margin monitor on the Plasma Display is reading "INVALID" All other plant equipment is operating as designed

The SRO has asked the RO to determine the subcooling margin in the core to support his determination of the contingency action statement above. Which statement correctly describes the condition of the core? REFERENCES PROVIDED: Steam Tables Curve Book Curves 1.10B, 1.10C A. B. The core is superheated by more than 15 F The core is superheated between 5 F and 15F

C. D.

The core is in a saturated condition (5 F) The core is subcooled by more than 5F

Distracter Analysis: When evaluating conditions for EOP transitions, the operators are required to use the curves in the Data Book instead of steam tables. These curves include an instrument error offset of 20 F. If the operators refer to steam tables to evaluate subcooling, the core conditions are subcooled (ignoring instrument error). If they refer to the curve, it will show that the core is in

the saturation region which means it is below the saturation curve and below the subcooled region which means it is superheated.

Ques_619

For Official Use Only Page 75

Question #57

McGuire Nuclear Station

RO Exam

A.

B. C.

D.

Incorrect: The core is 12 F superheated Plausible: If the candidate uses Curve 1.10C, he/she will arrive at a superheat value of 24F Correct: Incorrect: The core is 12 OF superheated Plausible: If the candidate misreads curve 1.10B and determines that the conditions are in the "SATURATED" region because this is what the graph indicates. Incorrect: The core is 12 F superheated Plausible: If the candidate uses steam tables, he/she will determine that the core is subcooled by 12 F. This is not allowable because it does not include instrument error.

Ques 619

For Official Use Only Page 76

Question #58

McGuire Nuclear Station

RO Exam

Bank Question: 620 1 Pt(s)

Answer: C

Unit 1 was operating at 100% power when a loss of VI system air pressure occurred. Which one of the following statements correctly describes the condition of the upper containment airlock seals? A. The seals will slowly depressurize. They can be manually reinflated using SA system air pressure. The seals will slowly depressurize. They can be manually reinflated using the VB system The seals will remain pressurized by an air supply from local air tanks. The seals will remain pressurized by a backup line from the SA system.

B.

C.

D.

Distracter Analysis: A. B. C. D. Incorrect: The seals will not depressurize Plausible: VI provides air to the seal supply Incorrect: The seals will not depressurize Plausible: VI provides air to the seal supply Correct: Incorrect: There is no backup connection to the SA system Plausible: The seals will remain pressurized on a loss of VI.

Ques_620

For Official Use Only Page 77

Question #59

McGuire Nuclear Station

RO Exam

Bank Question: 621


1 Pt(s)

Answer: B

Unit 2 was responding to a large break LOCA in E-1 (Loss of Reactor or Secondary Coolant). Given the following events and conditions: The 4160/600 VAC supply transformer to load center 2ELXD failed Motor control center 2EMXD was deenergized

Which one of the following statements correctly describes the actions needed to start containment air return fan 2B? A. B. Transfer 2ELXD to transformer 2ELXB Transfer 2ELXD to transformer 2ELXF

C. D.

Transfer 2EMXD to transformer 2ELXF Manually start air return fan 2B

Distracter Analysis: Containment air fan 2B is used to mix the containment atmosphere during a LOCA to enhance removal of Iodine and other fission products by containment spray. Fan 2B is powered from motor control center 2EMXD through load center 2ELXD. The fan is automatically started by a safety injection signal. A. B. C. Incorrect: Prohibited by a Kirk Key interlock Plausible: This would physically repower the fan Correct: Incorrect: Not physically possible Plausible: If the transfer could be physically done, it would repower fan 2B Incorrect: Will not start no power to the fan Plausible: If the candidate does not determine which motor control center powers containment air return fan 2B

D.

Ques_621

For Official Use Only Page 78

Question #60

McGuire Nuclear Station

RO Exam

Bank Question: 622 1 Pt(s)

Answer: D

Unit 2 was operating at 5% power during a plant startup when a total loss of AC power (station blackout) occurred. Given the following events and conditions: The plant was operating within normal limits and bands All protection systems operated as designed All emergency diesel generators failed to start No safety injection occurred No operator action was taken

Which one of the following statements correctly describes the response of the reactor trip system? A. No automatic reactor trip would occur and the reactor would remain critical. The shunt coils in the reactor trip and bypass breakers would energize and a reactor trip would occur. The under-voltage and shunt coils in the reactor trip breakers would energize and a reactor trip would occur. The CRDMs would deenergize and the rods would drop into the core.

B.

C.

D.

Distracter Analysis: The loss of the safety bus would cause the rod drive MG sets to lose power and power would be lost to the CRDMs. Below 10% power, the at-power reactor trips are bypassed so no automatic trip would occur. SSPS would remain energized from the 120 VAC Instrument bus but would not get a trip signal. A. Incorrect: CRDM power would be lost from the rod drive MG sets Plausible: If the candidate thinks that the rod drive MG sets are powered from the 120 VAC instrument bus or a DC bus. Incorrect: No trip signal would be generated below 10% power, P-7. Plausible: SSPS remains energized and could generate a trip signal Incorrect: No trip signal would be generated below 10% power, P-7. Plausible: SSPS remains energized and could generate a trip signal. Correct:

B. C. D.

Ques_622

For Official Use Only Page 79

Question #61

McGuire Nuclear Station

RO Exam

Bank Question: 623 1 Pt(s)

Answer: A

Unit 2 was operating at 100% power when an alarm was received on the 2B NCP standpipe level. Which one of the following statements correctly describes the cause of the standpipe level alarm? A. A high standpipe level indicates excessive leakoff through the #2 seal. A high standpipe level indicates reduced leakoff through the #3 seal. A low standpipe level indicates excessive leakoff through the #2 seal.

B.

C.

D.

A low standpipe level indicates reduced leakoff through the #3 seal.

Distracter Analysis: A. B. Correct: Incorrect: the standpipe level maintains a backpressure on the #2 seal reduced leakoff by the #3 seal has no effect on standpipe level as this leakoff goes directly to the RCOT Plausible: If the candidate thinks that the leakoff from the #3 seal effects standpipe level. Incorrect: excessive leakoff from the #2 seal would lead to a higher standpipe level as the standpipe maintains a backpressure on the #2 seal. Plausible: if the candidate thinks that excessive leakoff would cause standpipe level to drop or thinks that the standpipe is on the #3 seal.. Incorrect: reduced leakoff from the #3 seal would not lead to a lower standpipe level as the standpipe maintains a backpressure on the #2 seal. Plausible: If the candidate thinks that flow from the #3 seal goes to the standpipe

C.

D.

Ques_623

For Official Use Only Page 80

Question #62

McGuire Nuclear Station

RO Exam

Bank Question: 624 1 Pt(s)

Answer: D

Unit 2 was operating at 100% power with train B components in service. If a high strainer differential pressure alarm occurs on the 2B RN pump, what statement describes the RN system alignment upon completion of all automatic actions? A. 2A RN pump is running from the SNSWP 2A RN strainer is in service 2A RN pump is running from the low level intake 2B RN strainer is in backwashing 2B RN pump is running from the SNSWP 2B RN strainer is in backwashing D. 2B RN strainer is backwashing Distracter Analysis: A. Incorrect: The low DIP alarm causes the 2B strainer to backwash Plausible: IF the candidate thinks that the low DIP alarm on the B strainer causes the running RN trains to shift to the A train Incorrect: the RN pumps do not shift for a high DIP alarm Plausible: the RN pumps would be manually shifted to the SNSWP for a low suction pressure alarm, not a high DIP alarm Incorrect: the suction does not shift to the SNSWP Plausible: the 2B RN strainer backwashes Correct:

B.

C.

B.

C. D.

Ques_624

For Official Use Only Page 81

Question #63

McGuire Nuclear Station

RO Exam

Bank Question: 625 1 Pt(s)

Answer: B

Unit I was operating at 5% power following a reactor startup after a refueling outage. Given the following conditions and events: A mixed bed demineralizer that had been isolated at the end of the last fuel cycle was placed in service T-ave = 558 F All systems are aligned normally for the existing plant conditions

What will be the effect (if any) on T-ave? A. B. C. D. T-ave will increase due to the exchange of Lithium ions T-ave will increase due to the exchange of boric acid T-ave will decrease due to the exchange of boric acid T-ave will not change

Distracter Analysis: Placing a mixed bed demineralizer in service will change reactivity by exchanging boric acid (borate ions) for OH ions. The mixed bed demineralizer was exposed to low boric acid concentrations at the end of core life prior to the shutdown so it reached equilibrium saturation conditions of very low concentrations of boric acid. When placed in service at beginning of core life, it will exchange boric acid for OH ions and reduce the boric acid concentration in the NC system. A. Incorrect: T-ave will increase due to exchange of boric acid Plausible: pH may increase but not temp due to the exchange of Li ions for hydroxyl ions depending on the saturation state of the resin. Correct Answer: Incorrect: T-ave will increase due to exchange of boric acid Plausible: If the candidate reverses the effect of this action Incorrect: T-ave will increase due to exchange of boric acid Plausible: This is the correct answer for cation bed resin which exchanges Lithium for OH - but not boric acid

B. C. D.

Ques_625

For Official Use Only Page 82

Question #64

McGuire Nuclear Station

RO Exam

Bank Question: 626


1 Pt(s)

Answer: A

During a reactor start-up, the following conditions are noted: The NC system is at normal operating pressure and temperature. Four NCPs are running. NCS temperature is being controlled using the steam dumps. The reactor power is 5%. Which of the following describes the change in actual plant parameters if the main steam header pressure transmitter fails high? A. Steam dump demand increases, actual steam header pressure decreases, and NCS temperature decreases. Steam dump demand decreases, actual steam header pressure increases, and NCS temperature increases. Steam dump demand increases, actual steam header pressure increases, and NCS temperature decreases. Steam dump demand decreases, actual steam header pressure decreases, and NCS temperature increases.

B.

C.

D.

Distracter Analysis: A. B. Correct: Incorrect: Steam dump demand increases not decreases. Plausible: Parameter changes are consistent with steam dump demand decrease. Incorrect: Steam pressure decrease not increases - change is not consistent with increased demand. Plausible: reflects indicated rather than actual pressure. Incorrect: Steam dump demand increases not decreases. Plausible: psychometrically balanced distracter.

C.

D.

Ques_626

For Official Use Only Page 83

Question #65

McGuire Nuclear Station

RO Exam

Bank Question: 627 1 Pt(s)

Answer: C

Unit 1 was operating at 100% power when panel board 1 EKVB was unintentionally deenergized. Which one of the following lists of ESS loads was deenergized. A. Process Protection Channel I Safeguards Test Cabinet Train A SSPS Channel I (Trains A&B) SSPS Train A Output Cabinet Auxiliary Safeguards Cabinet Train A Process Protection Channel IV Safeguards Test Cabinet Train B SSPS Channel IV (Trains A&B) SSPS Train B Output Cabinet Auxiliary Safeguards Cabinet Train Process Protection Channel II SSPS Channel II (Trains A & B) Process Protection Channel III SSPS Channel III (Trains A & B)

B.

C.

D.

Distracter Analysis: A. B. C. D. Incorrect: powered from EKVA Plausible: if the candidate does not know the power supply Incorrect: powered from EKVD Plausible: if the candidate does not know the power supply Correct: Incorrect: powered from EKVC Plausible: if the candidate does not know the power supply

Ques_627

For Official Use Only Page 84

Question #66

McGuire Nuclear Station

RO Exam

Bank Question: 628 1 Pt(s)

Answer: C

Unit 2 was responding to a small-break LOCA in E-1 (Loss of Reactor or Secondary Coolant). Given the following conditions: Containment pressure = 0.7 psig (at peak pressure for the event) 2ETA was deenergized due to a bus fault The VI header inside containment was depressurized and isolated due pipe rupture The VI system outside containment remained pressurized Which one of the following statements correctly describes the positions of valves 2RV-79A and 2RV-80B? REFERENCES PROVIDED Station Drawing MCFD-1604-03.00 Flow Diagram of RV A. B. C. D. 2RV-79A is open, 2RV-80B is open 2RV-79A is shut, 2RV-79A is open 2RV-79A is shut 2RV-80B is open 2RV-80B is shut 2RV-SOB is shut

Distracter Analysis: These valves are RV containment isolation valves. They are air operated and will close on a high-high containment pressure signal (phase B isolation). They fail closed upon loss of operating air pressure. There are no backup nitrogen accumulators to provide operating pressure (as with the PORVs) even though they are safety-related valves. In this question, the valves will not auto close because pressure remains below the phase B actuation point (3.0 psig) but they will close due to a loss of operating air pressure. A. Incorrect: the valves are shut due to the loss of VI air pressure Plausible: if the candidate thinks that they fail open or that they are electrically operated Incorrect: the valves are shut due to the loss of VI air pressure Plausible: if the candidate thinks that they are electrically operated and fail shut

B.

Ques_628

For Official Use Only Page 85

2 May 2000

2:02PM Brian Haagensen PSHA FAX: 860 739 0333

PAGE 2

Question #74

McGuire Nuclear Station

RO Exam

1 Pt(s)

Unit 1 was operating at 60% power. Given the following events and conditions: Pressurizer pressure decreased to 1940 prig. The SSPS train A low PZR pressure trip logic relay failed to actuate.

What effect would this failure have on the function of the reactor protection system? A. The reactor would not trip because the Train A logic relay would not remove power from the UV coil for RTA. The reactor would not trip because the Train B logic relay would not remove power from the UV coil for RTA. The reactor would trip because the Train B logic, relay would remove power from the UV coil for RTB. The reactor would trip because the Train B logic relay would remove power from the UV coil for RTA.

B.

C.

D.

Ques_638.doc

For Official Use Only

Page 74

2 Ma'y 2000

2:02PM Brian Haayensen PSHA FAX: 860 739 0333

PAGE 3

Question #74

McGuire Nuclear Station

RO Exam

Bank Question: 638


1 Pt(s)

Answer: C

Unit 1 was operating at 60% power. Given the following events and conditions: Pressurizer pressure decreased to 1940 psig. The SSPS train A low PZR pressure trip logic relay failed to actuate.

What effect would this failure have on the function of the reactor protection system? A. B. The reactor would not trip because the Train A logic relay would not remove power from the UV coil for RTA. The reactor would not trip because the Train B logic relay would not remove power from the UV coil for RTA. The reactor would trip because the Train B logic relay would remove power from the UV coil for RTB. The reactor would trip because the Train B logic relay would remove power from the UV coil for RTA.

C.

D.

Distracter Analysis: A. Incorrect: The reactor will nip. Plausible: based on misunderstanding of RPS redundancy. SSPS Train A had the failure so it makes sense that this could potentially fail to cause the trip Incorrect: The reactor will trip. Plausible: based on misunderstanding of RPS redundancy. Correct: Incorrect: l'he Train B logic does not affect RTA. Plausible: based on misunderstanding of RPS redundancy. if the candidate thinks that SSPS Train B opens RTA.

B. C. D.

Ques_638.doc

For Official Use Only

Page 91

Question #66
C.

McGuire Nuclear Station

RO Exam

D.

Correct Answer: 2RV-79A remains open without a phase B signal and 2RV-80B fails closed due to the loss if VI pressure inside containment. Incorrect: 2RV-79A remains open because no phase B isolation was generated Plausible: if the candidate thinks that a phase B signal was generated or if he does not recognize that VI pressure is maintained outside of containment - or if he thinks 2RV-79A is powered from 1ETA and fails closed.

Ques_628

For Official Use Only Page 86

Question #67

McGuire Nuclear Station

RO Exam

Bank Question: 629 I Pt(s)

Answer: A

With Unit 1 was operating at 75% power with rods in automatic control when turbine load drops 10%. Which of the following correctly indicates the change in plant parameters when the transient is complete? A. B. T-ave decreases approximately 2 to 3F due to decreased Tref. Tave stays the same due to automatic rod motion.

C. D.

Tave increases approximately 2 to 3 F due to rod motion. Tave stays the same due to decreased Tref.

Distracter Analysis: T-ave is ramped from 557 F at 0% power to 585 F at 100% power. A 10% drop in load causes a 10% reduction in T-ave within the operating band - so T-ave would drop by 10% of 28 F or 3F. A. Correct: B. Incorrect: Tave decreases 10%. Plausible: If you think rod control compensates for load change by maintaining T-ave C. Incorrect: Tave decreases 10%. Plausible. If you think Tref change reflects load increase. D. Incorrect: Tave decreases 10%. Plausible: If you think Tref decreases and compensates for load decrease.

Ques_629

For Official Use Only Page 87

Question #68

McGuire Nuclear Station

RO Exam

Bank Question: 630


Pt(s)

Answer: A

Which of the following statements correctly describes the major effect of a failure of a large number of lower ice condenser doors to open for an unisolable main steam line break with an associated SGTR (tube rupture on the faulted S/G) accident inside containment. A. Containment peak pressure would be higher and would be achieved sooner in the event. Containment peak pressure would be higher but would be achieved later in the event. Containment sump water inventory would not be adequate after shift to recirculation mode.

B.

C.

D.

Containment sump water inventory would not be adequate to maintain long-term subcriticality during the cooldown.

Distracter Analysis: This question is a variation on a bank question which is normally asked from the perspective of a LBLOCA. However, the analysis is also true for a faulted ruptured S/G inside containment. The difference in these events is that the MSL break is a HELBIC and the SGTR is a small break LOCA so this would cause the water in the containment sump to include the secondary water, which is unborated. In addition, the amount of water from the S/G would be less than the water from a LBLOCA. A. B. Correct: Incorrect: containment pressure would peak sooner Plausible: if the candidate thought that the ice condenser melt was delayed due to the door not opening causing the release of the cold water to occur later Incorrect: sufficient water is added from the FWST during the injection phase Plausible: the ice water would not be immediately available as melting would be delayed. In addition, the amount of water that enters the sump from the SIG would be less than the water from a LBLOCA. A SGTR is essentially a SBLOCA and water would not be released from the core at a rapid rate to add to the sump before swap over was required. Incorrect: the FWST would provide sufficient borated water to maintain shutdown throughout the process

C.

D.

Ques_630

For Official Use Only Page 88

Question #68

McGuire Nuclear Station

RO Exam

Plausible: release of the borated water from the ice melt would be delayed and thus would not be available until after melting. In addition, the water from the SIG is unborated.

Ques_630

For Official Use Only Page 89

Question #69

McGuire Nuclear Station

RO Exam

Bank Question: 631 1 Pt(s)

Answer: D

The Unit 2 NV system cold leg flow path balance test procedure throttles high pressure injection flow between a minimum value to limit ___(1) ___ and a maximum value to limit ____ (2) ___ (1) Pump overheating Break flow Pump overheating Break flow a) pipe erosion pipe erosion pump runout pump runout

A. B. C. D.

Distracter Analysis: A. Incorrect: pump overheating is not a problem at the higher flow rates and pipe erosion is not a limiting problem for this system Plausible: low flow rates can cause pump overheating and high flow rates can cause pipe erosion Incorrect: safe-end erosion is not a limiting factor Plausible: break spillage is the basis for the minimum throttle limit Incorrect: pump overheating is not a problem at the higher flow rates and pipe erosion is not a limiting problem for this system Plausible: pump runout is the basis for the higher throttle setting Correct:

B. C.

D.

Ques_631

For Official Use Only Page 90

Question #70

McGuire Nuclear Station

RO Exam

Bank Question: 632 1 Pt(s)

Answer: D

Unit I was responding to a station blackout in ECA-0.0 (Loss of all AC Power). What pressurizer heaters are available to control reactor pressure? A. B. C. D. Group A backup heaters can be controlled from the SSF. Group B backup heaters can be controlled from the ASP. Group C backup heaters can be controlled from the ASP. Group D backup heaters can be controlled from the SSF.

Distracter Analysis: Only about 10% of Group D heaters will be available under these conditions A. Incorrect: Group A has no power and is controlled from the ASP. Plausible: If confuses group A with D and keys on SSF as the power source. Incorrect: Group B has no power. Plausible: If associates group B with its correct local control station. Incorrect: Group C has no power and is controlled from the MCB. Plausible: If doesn't know power supplies and chooses based on logical extension of normal pressure control. Correct:

B. C.

D.

Ques_632

For Official Use Only Page 91

Question #71

McGuire Nuclear Station

RO Exam

Bank Question: 633 1 Pt(s)

Answer: B

Unit 2 was operating at 100% power when the pressurizer spray valve failed open. Given the following conditions: PZR Channel Select is in 1-2 position PZR Pressure Control in AUTO NCS Pressure is 2245 psig

Which one of the following describes the response of the PZR pressure control system to these conditions? A. PZR pressure does not decrease because the spray valves will not open below 2260 psig PZR pressure decreases to 1945 psig where the RPS reactor trips. PZR pressure decreases to 2210 psig, where the backup heaters take control of pressure.

B.

C.

D.

PZR pressure decreases to 2185 psig where the spray line block valves close.

Distracter Analysis: A. B. C. D. Incorrect: the spray valves open and pressure decreases. Plausible: based on a misunderstanding of how the controller works. Correct: Incorrect: Backup heaters come on but do not control pressure. Plausible: based on misconception of an IPE system "degas" mode. Incorrect: There are no block valves. Plausible: based on possible confusion between spray and PORV systems.

Ques_633

For Official Use Only Page 92

Question #72

McGuire Nuclear Station

RO Exam

Bank Question: 634 1 Pt(s)

Answer: A

Which one of the following statements correctly describes the response of the turbine driven CA pump if turbine speed exceeds 4500 rpm? A. A mechanical flyweight assembly unlatches a trip hook on the turbine stop valve A mechanical flyweight assembly unlatches a trip hook on the turbine governor valve The Woodward governor will generate a signal that trips the turbine stop valve tripping latch assembly

B.

C.

D.

The Woodward governor will generate a trip signal that trips the turbine governor valve

Distracter Analysis: A. B. C. D. Correct Answer: Incorrect: turbine stop valve trips Plausible: the mechanical flyweight assembly causes the trip Incorrect: the Woodward governor does not cause the trip Plausible: the turbine stop valve is the valve that actually trips Incorrect: the Woodward governor does not cause the trip Plausible: this is the mechanism by which the turbine speed is limited

Ques_634

For Official Use Only Page 93

Question #73

McGuire Nuclear Station

RO Exam

Bank Question: 635 1 Pt(s)

Answer: B

Unit 1 was conducting refueling in Mode 6. RP requested the control room operator to independently verify the adjustment of the trip 2 setpoint for 1EMF-16 (Containment Refueling Bridge) area radiation monitor. If the trip 2 setpoint was required to be set at 1/2 decade above the background and background radiation levels were 3.0 mR/hr, what is the correct value for the trip 2 setpoint?

A.
B. C. D.

6 mR/hr
9 mR/hr 12 mR/hr 15 mR/hr

Distracter Analysis: V2 decade is 3.16x background above background that is commonly considered 3x background for setting trip 2 setpoints. A. Incorrect: too low - the correct value is 3x3 = 9 mR/hr Plausible: if the candidate thinks that 1/2 decade = twice background levels Correct Answer: 3x3 mR/hr = 9 mR/hr Incorrect: too high - 9 mR/hr is the correct answer Plausible: if the candidate adds the background level to the calculation i.e. 3x3+3 = 12 mR/hr Incorrect: too high 9 mR/hr is correct Plausible: if the candidate thinks that 1/2 decade is 5 x background (as one decade is 1/2 of 10 X background) 5x3 = 15

B. C.

D.

Ques_635

For Official Use Only Page 94

Question #74

McGuire Nuclear Station

RO Exam

Bank Question: 637 1 Pt(s)

Answer: D

Unit I was conducting a reactor startup. Given the following conditions: All shutdown rod banks have been fully withdrawn at 222 steps Control bank "A" rods are being withdrawn at 80 steps. The RP1 Urgent Failure Annunciator alarms.

Which of the following conditions would cause this alarm? A. A control bank "A" rod is misaligned from its bank position by 8 steps. Data "A" failure has occurred on one or more rods in shutdown bank "A". The rod control bank overlap unit has detected an improper rod step sequence.

B.

C.

D.

A rod in shutdown bank "C" has dropped into the bottom of the reactor.

Distracter Analysis: A. Incorrect: requires a position deviation > 12 steps - and will not withdraw control bank C at this point - overlap unit restricts Plausible: if candidate does not know the criteria for deviation alarm Incorrect: Will not cause an urgent failure alarm Plausible: Will cause a non-urgent failure alarm Incorrect: will not cause an urgent failure Plausible: could confuse with a Non-Urgent Failure alarm. Correct Answer:

B. C. D.

Ques_637

For Official Use Only Page 95

. 2 Ma,:y 2800

2:02P11 Brian Haagensen PSHA FAX: 860 739 0333

PAGE 2

Question #74

McGuire Nuclear Station

RO Exam

1 Pt(s)

Unit 1 was operating at 60% power. Given the following events and conditions: Pressurizer pressure decreased to 1940 psi g. The SSPS train A low PZR pressure trip logic relay failed to actuate.

What effect would this failure have on the function of the reactor protection system? A. The reactor would not trip because the Train A logic relay would not remove power from the UV coil for RTA. The reactor would not trip because the Train B logic relay would not remove power from the UV coil for RTA. The reactor would trip because the Train B logic relay would remove power from the UV coil for RTB. The reactor would trip because the Train B logic relay would remove power from the UV coil for RTA.

B.

C.

D.

Ques_638.doc

For Offidal Use Only

Page 74

2 Ma'y 2008

2:02PM Brian Haagensen PSHA FAX: 860 739 0333

PAGE 3

Question #74

McGuire Nuclear Station

RO Exam

Bank Question: 638


1 Pt(s)

Answer: C

Unit 1 was operating at 60% power. Given the following events and conditions: Pressurizer pressure decreased to 1940 psig. The SSPS train A low PZR pressure trip logic relay failed to actuate.

What effect would this failure have on the function of the reactor protection system? A. The reactor would not trip because the Train A logic relay would not remove power from the UV coil for RTA. The reactor would not trip because the Train B logic relay would not remove power from the UV coil for RTA. The reactor would trip because the Train B logic relay would

B.

C.

remove power from the UV coil for RTB. D. The reactor would trip because the Train B logic relay would remove power from the UV coil for RTA.

Distracter Analysis: A. Incorrect: The reactor will nip. Plausible: based on misunderstanding of RPS redundancy. SSPS Train A had the failure so it makes sense that this could potentially fail to cause the trip incorrect: The reactor will trip. Plausible: based on misunderstanding of RPS redundancy. Correct: incorrect: The Train B logic does not affect RTA. Plausible: based on misunderstanding of RPS redundancy. If the candidate thinks that SSPS Train B opens RTA.

B. C. D.

Ques_638.doc

For Official Use Only

Page 91

Question #75

McGuire Nuclear Station

RO Exam

Bank Question: 638 1 Pt(s)

Answer: C

Unit 1 was operating at 60% power. If the SSPS train A low PZR pressure trip logic relay fails to respond to a valid sensor trip signal, what effect would this failure have on the function of the reactor protection system? A. The reactor would not trip because the Train A logic relay would not remove power from the UV coil for RTA. The reactor would not trip because the Train B logic relay would not remove power from the UV coil for RTA. The reactor would trip because the Train B logic relay would remove power from the UV coil for RTB. The reactor would trip because the Train B logic relay would remove power from the UV coil for RTA.

B.

C.

D.

Distracter Analysis: A. Incorrect: The reactor will trip. Plausible: based on misunderstanding of RPS redundancy. SSPS Train A had the failure so it makes sense that this could potentially fail to cause the trip Incorrect: The reactor will trip. Plausible: based on misunderstanding of RPS redundancy. Correct: Incorrect: The Train B logic does not affect RTA. Plausible: based on misunderstanding of RPS redundancy. If the candidate thinks that SSPS Train B opens RTA.

B. C. D.

Ques_638

For Official Use Only Page 96

Question #77

McGuire Nuclear Station

RO Exam

Bank Question: 640 1 Pt(s)

Answer: A

Unit 2 is responding in E-1 (Loss of Reactor or Secondary Coolant) to a LOCA inside containment. Given the following conditions: Phase B containment isolation actuated Containment pressure remained above 3 psig The FWST level decreased to 20 inches. Which of the following best describes the steps necessary to prevent damaging the NS pumps? A. B. Reset NS, stop the NS pumps. Reset CPCS, stop the NS pumps.

C. D.

Reset containment phase B isolation, stop the NS pumps. Override CPCS, stop the NS pumps.

Distracter Analysis: A. B. Correct: Incorrect: There is no CPCS reset. Plausible: based on confusion between NS and CPCS actuation logic. Incorrect: phase B will not reset - > 3 psig Plausible: based on confusion between phase B and NS actuation logic can reset NS. Incorrect: Overriding CPCS will not reset phase B or NS. Plausible: based on confusion with CPCS failure actions.

C.

D.

Ques_640

For Official Use Only Page 98

Question #78

McGuire Nuclear Station

RO Exam

Bank Question: 641 1 Pt(s)

Answer: B

Unit 1 was shutdown in Mode 6. A containment purge was in progress using the VP system. Which one of the following conditions would automatically isolate the containment purge line? A. B. C. 1EMF-36(L) (Unit Vent Gas) trip 2 setpoint exceeded. Manual containment spray actuation signal Containment Protection Control (CPCS) setpoint (0.35 psig) exceeded Containment Evacuation Alarm actuation

D.

Distracter Analysis: A. Incorrect: does not shutdown VP Plausible: if the candidate makes the mistake of thinking that the purge release path goes through the unit vent - isolates the waste gas release path by closing WM-60. Correct: manual NS causes a containment ventilation isolation signal Incorrect: CPCS does not secure VP Plausible: this would indicate that containment was being pressurized Incorrect: does not secure VP Plausible: indicative of a criticality problem inside containment

B. C.

D.

Ques_641

For Official Use Only Page 99

Question #79

McGuire Nuclear Station

RO Exam

Bank Question: 642

Answer: D

1 Pt(s)

Unit 1 was operating at 100% power when a design basis earthquake caused a loss of all AC power (station blackout). Given the following events and conditions: The suction line to the 1A KF pump sheared during the earthquake Spent fuel pool (SFP) makeup was aligned from the FWST to compensate for any loss of SFP level as required. FWST level was at 300 inches. The operators entered ECA-0.0 (Loss of All AC Power) Which of the following events would cause spent fuel pool level to continue to decrease? A. The ruptured suction line on the lA KF pump caused the water to be siphoned out of the spent fuel pool. The loss of containment and spent fuel pool ventilation fans caused a change in the differential pressure between the spent fuel pool and the reactor cavity. The FWST gravity makeup line to the spent fuel pool was not properly isolated and water has been siphoned back into the FWST. The standby makeup pump was in operation.

B.

C.

D.

Distracter Analysis: A. Incorrect: The suction line inlet is very close to the surface of the spent fuel pool and a rupture in the line would not cause an appreciable drop in level Plausible: If the suction line inlet was not designed to prevent this event, it would drain the pool Incorrect: The reactor cavity isolation devices are closed. Plausible: based on misunderstanding plant-operating conditions. The spent fuel pool level will change if there is a change in differential pressure between containment and the refueling building has occurred in the past. Incorrect: FWST makeup does not reverse siphon until below level is below 100 inches.

B.

C.

Ques_642

For Official Use Only Page 100

Question #79

McGuire Nuclear Station

RO Exam

D.

Plausible: based on misunderstanding the SFP/FWST design criteria. Correct: The standby makeup pump takes suction from the spent fuel pool

Ques_642

For Official Use Only Page 101

Question #80 #81

McGuire Nuclear Station

RO Exam

Bank Question: 643 1 Pt(s)

Answer: D

Unit 1 was operating at 100% power when a steam generator tube rupture occurred in the 1B S/G. If the operators respond properly in E-3 (Steam Generator Tube Rupture) and isolate the 1B S/G, which of the following conditions are indicative of successful isolation prior to commencing the initial cooldown of the NC system? A. S/G level decreases as SIG water flows back through the break into the NC system. S/G pressure decreases as steam generator pressure equalizes with NC system pressure. S/G level decreases as SIG water flows back through the break into the NC system. SIG pressure increases as steam generator pressure equalizes with NC system pressure. S/G level increases as NC system coolant water flows through the break into the SIG. SIG pressure decreases as steam generator pressure equalizes with NC system pressure. S/G level increases as NC system coolant water flows through the break into the S/G. S/G pressure increases as steam generator pressure equalizes with NC system pressure.

B.

C.

D.

Distracter Analysis: A. B. C. D. Incorrect: S/G level increases and S/G pressure increases Plausible: provided for psychometric balance Incorrect: S/G level increases Plausible: S/G pressure increases correctly Incorrect: S/G pressure increases Plausible: S/G level increases correctly Correct Answer:

Ques_643

For Official Use Only Page 102

Bank Question: 644 1 Pt(s)

Answer: A

Unit 2 has tripped due to instrument technician error during a surveillance test. The moisture separator reheaters (MSRs) did not reset. Assuming no operator action, what effect would this failure have on the plant response to this transient? A. The NCS would be overcooled because the main steam supply to the MSRs would not isolate. Safety injection will actuate on low SG pressure because MSR steam supply valve 2SM-15 fails to close. Safety injection will actuate on low SG pressure because the main steam supply to the MSRs would not isolate. The NCS would be overcooled because MSR steam supply valve 2SM-15 fails to close.

B.

C.

D.

Distracter Analysis:

A. B.

C.

D.

Correct: Incorrect: Unit 2 does not have low SG pressure safety injection. Plausible: The cooldown will continue until MSIV closure on low SG pressure. A plausible answer for Unit 1. Incorrect: Unit 2 does not have low SG pressure safety injection. Plausible: The cooldown will continue until MSIV closure on low SG pressure. A plausible answer for Unit 1 which does have low SG pressure safety injection. Incorrect: 2SM-15 closure is a manual action if the MSRs do not reset. Plausible: based on a misunderstanding of the MSR system configuration.

Ques_644

For Official Use Only Page 103

Question #82

McGuire Nuclear Station

RO Exam

Bank Question: 645 1 Pt(s)

Answer: B

Unit 1 was operating at 100% power when a 'A inch break on a pipe in the NC system occurs inside containment. Given the following conditions: Operators properly implement: o E-0 (Reactor Trip) o E-1 (Loss of Reactor or Secondary Coolant) o ES-1.1 (Safety Injection Termination) o ES-1.2 (Post LOCA Cooldown and Depressurization) One train of safety injection fails to actuate All other automatic safety and control features function as designed Which one of the following statements correctly describes NC system pressure at the time that the cooldown is started in ES-1.2? A. NC system rapidly repressurizes and pressurizer level may go solid unless safety injection is promptly terminated NC system pressure stabilizes to a value above steam generator pressure but below 2235 psig. NC system pressure reaches equilibrium with steam generator pressure. NC system pressure stabilizes below steam generator pressure.

B.

C.

D.

Distracter Analysis: A. Incorrect: The NC system does not rapidly repressurizes with minimum SI flow Plausible: this occurs with 2 trains of safety injection in operation Correct Answer: Incorrect: One train of SI flow is sufficient to raise pressure above S/G pressure for a V2 inch break Plausible: This will occur for SGTRs Incorrect: One train of SI flow is sufficient to raise pressure above S/G pressure for a 'A inch break Plausible: This occurs for medium sized LOCAs larger than 1 inch

B. C.

D.

Ques_645

For Official Use Only Page 104

Question #83

McGuire Nuclear Station

RO Exam

Bank Question: 646 1 Pt(s)

Answer: D

Unit 2 is operating at 85% power, when a condenser low vacuum alarm occurs. Main condenser vacuum is steady at 24 inches. Which of the following statements correctly describes three possible causes of this condition? A. Low auxiliary steam pressure, vacuum pump failure, or low RC temperature Vacuum breaker leakage, low RC temperature, or low auxiliary steam pressure Vacuum pump failure, boot seal leakage, or vacuum breaker leakage

B.

C.

D.

Vacuum breaker leakage, low auxiliary steam pressure, boot seal leakage

Distracter Analysis: A. Incorrect: Vacuum pumps are not in service at 85% and low RC improves condenser vacuum. Plausible: based on misunderstanding of vacuum control mechanisms. Incorrect: low RC improves condenser vacuum. Plausible: based on misunderstanding of vacuum control mechanisms. Incorrect: Vacuum pumps are not in service at 85%. Plausible: based on misunderstanding of vacuum control mechanisms. Correct:

B.

C.

D.

Ques_646

For Official Use Only Page 105

Question #84

McGuire Nuclear Station

RO Exam

Bank Question: 647 1 Pt(s)

Answer: A

Unit 1 was operating at 100% power the following trends were noted: 0200 87 75 8.0 >400 230 3.0 10 50% 55% 0205 88 75 6.5 >400 233 2.5 15 50% 53% 0210 87 75 6.2 >400 234 2.2 18 50% 54%

Charging Flow (gpm) Letdown Flow (gpm) 1A-1D NCP seal injection (gpm) 1A-1D NCP #1 seal d/p (psid) 1A-1D #1 seal outlet temp (F) 1A-1D #1 seal leakoff flow (gpm) 1A-1D seal water inj filter dip (psid) VCT Level Pressurizer level

Which one of the following conditions would cause the parameter trends? A. B. C. D. #1 seal injection filter becoming clogged VCT pressure increased NV-241 was manually adjusted in the closed direction NV-238 was manually adjusted in the open direction

Distracter Analysis: Seal leakoff is decreasing as indicated by reduced seal d/p, leakoff flow and leakoff temp increasing. However, seal injection filter d/p is also increasing which indicates that the clogged seal injection filter causes the reduction. Pressurizer level decreases initially as less seal injection enters the NC system which causes total charging flow to increase. This is corrected by pressurizer level control. A. B. Correct Answer: seal filter d/p increasing while seal leakoff decreasing Incorrect: if VCT pressure increased, the seal injection filter dip would decrease Plausible: seal leakoff shows a decrease I n c o r r ec t: # 1 s ea l l ea k o ff a nd fi l t e r di p w o ul d i nc re a se s e a l l e ak o ff t em p w o ul d de c re as e Pl ausi b l e: o pe ni ng N V- 2 4 1 w o ul d c a use l e a k o ff t o de c re as e Incorrect: seal injection filter dip would not increase Plausible: all other parameters would trend as indicated except pressurizer level decrease.

C.

D.

Ques_647

For Official Use Only Page 106

Question #85

McGuire Nuclear Station

RO Exam

Bank Question: 648


1 Pt(s)

Answer: B

Which one of the following correctly describes the normal loading of the 125VDC vital battery chargers? A. (1) Battery on "charge", (1) 125VDC DC distribution center, (1) 125VDC DC panel board, (1) 120VAC AC static inverter. (1) Battery on "float", (1) 125VDC DC distribution center, (2) 125VDC DC panel boards, (2) 120VAC AC static inverters. (1) Battery on "float", (1) 125VDC DC distribution center, (1) 125VDC DC panel board, (1) 120VAC AC static inverter. (1) Battery on "charge", (2) 125VDC DC distribution centers, (2) 125VDC DC panel boards, (2) 120VAC AC static inverters.

B.

C.

D.

Distracter Analysis: A. Incorrect: There are 2 panel boards and inverters, one per unit. Plausible: based on misunderstanding of float, and a one unit only perspective. Correct: Incorrect: There are 2 panel boards and inverters, one per unit. Plausible: based on one unit only perspective. Incorrect: There is only one distribution center normally aligned. Plausible: based on a misunderstanding of float, and battery capacity versus normal alignment.

B. C. D.

Ques_648

For Official Use Only Page 107

Question #86

McGuire Nuclear Station

RO Exam

Bank Question: 651 I Pt(s)

Answer: D

Which of the following actions occur when an emergency diesel generator reaches 95% of rated speed during an emergency start? A. Generator field flashed and voltage and frequency automatically controlled. Low lube oil pressure trip reinstated and starting air secured. Generator field flashed, and starting air secured. Low lube oil pressure trip reinstated and voltage and frequency automatically controlled.

B. C. D.

Distracter Analysis: A. B. C. D. Incorrect: Field flash occurs at 40%. Plausible: based on a logical progression for auto start logic. Incorrect: Starting air is secured at 40%. Plausible: based on misunderstanding of starting air reset setpoint. Incorrect: Field flash and starting air secured occur at 40%. Plausible: based on misunderstanding of correct setpoint. Correct:

Ques_651

For Official Use Only Page 108

Question #87

McGuire Nuclear Station

RO Exam

Bank Question: 652 1 Pt(s)

Answer: C

Which one of the following statements explains the reason for closing all four NC pump seal return isolation valves when NCS Pressure is less than 100 psig? A. Prevent backflow from the NV System through the seal injection filter to the NC Pump Seals. Ensure proper seal injection flow (at least 3 gpm) through No. 1 Seal during low system pressure operating applications. Limit the introduction of contaminants/particulates to the NCP Seals, due to backflow through the seal return filter.

B.

C.

D.

Ensure adequate seal cooling by lowering seal leakoff backpressure during low system pressure operation.

Distracter Analysis: A. Incorrect: the seal injection filter is on the charging side of the #1 seal package. Not possible to back flow from the NV system into the seals through this filter Plausible: This would be correct if it was the seal return filter Incorrect: Seal return isolation valves have no effect on seal injection flow they simply divert the flow after it passes through the seal. Plausible: this is the basis for OPENING the seal bypass valves when pressure is > 100 psig and < 1000 psig Correct Answer: Incorrect: Prevents particulate contamination of the NCP seals Plausible: Would reduce backpressure on the NCP seals - : this is the basis for OPENING the seal bypass valves when pressure is > 100 psig and < 1000 psig

B.

C. D.

Ques_652

For Official Use Only Page 109

Question #88

McGuire Nuclear Station

RO Exam

Bank Question: 654 1 Pt(s)

Answer: A

Which one of the following interlocks is designed to prevent a water hammer in the RC piping if the RC pumps trip? A. The vacuum breaker valves automatically open if all RC pumps trip. There is a 45 second time delay before pump discharge valves close to allow coast down. The pump discharge valves remain open until flow decreases below a preset value.

B.

C.

D.

The pump discharge valves close over 120 seconds while the pump is coasting down.

Distracter Analysis: A. B. C. D. Correct: Incorrect: There is no time delay. Plausible: based on a logical alternative. Incorrect: The valves close regardless of RC flow rate. Plausible: based on a logical alternative. Incorrect: Not designed to preclude water hammer. Plausible: based on a logical alternative.

Ques_654

For Official Use Only Page 110

Question #89

McGuire Nuclear Station

RO Exam

Bank Question: 655

Answer: B

1 Pt(s)

On May 19th, the NLO was directed by the unit supervisor to perform a sequence of steps using a working copy of a procedure in progress that had previously been correctly validated against the controlled copy on May 1'. Which one of the following statements correctly describes the required actions of the NLO? A. Perform just the designated steps as directed using the existing working copy. Re-validate the working copy of the procedure and perform just the designated steps from the existing working copy. Obtain a new working copy of the procedure and perform just the designated steps from the new working copy.

B.

C.

D.

Obtain a new working copy of the procedure and inform the shift supervisor that all procedure steps must be performed or validated from the first step in the procedure.

Distracter Analysis: A. Incorrect: the working copy cannot be used unless it has been validated every 14 days Plausible: if the candidate thinks that the validation requirement exceeds 14 days Correct Answer: Incorrect: There is no requirement to obtain a new working copy if the validation has exceeded 14 days in addition this would now have working 2 copies of the procedure with completed steps initialed on each would be hard to keep track of the configuration control Plausible: if the candidate thinks that once the working copy has exceeded its validation requirement, it must be replaced Incorrect: There is no requirement to obtain a new working copy if the validation has exceeded 14 days - or to revalidate all steps in the procedure Plausible: This answer is overly conservative but some candidates might select the most conservative answer if they do not know the requirement.

B. C.

D.

Ques_655

For Official Use Only Page 111

Question #90

McGuire Nuclear Station

RD Exam

Bank Question: 656 1 Pt(s)

Answer: D

Units 1 & 2 are operating at 100% power when all off-site AC power is lost. Only one emergency diesel generator starts (2ADG) and loads the safety bus. Which of the following best describes the function of the Standby Shutdown Facility in responding to this event? A. Provides alternate power to supply makeup to the NCP seals at Unit 2. Provides an alternate location for plant shutdown due to control room inhabitability at Unit 1. Provides alternate power to reach and maintain HOT SHUTDOWN conditions at Unit 2. Provides alternate power to align valves for primary and secondary makeup at Unit 1. Distracter Analysis: A. B. C. Incorrect: Unit 2 did not lose NCP seal flow. Plausible: based on correct answer for Unit 1. Incorrect: Neither unit suffered inhabitability, an ASP function. Plausible: based on confusion with ASP function. Incorrect: SSF not needed for HOT SHUTDOWN as the 2A DG has powered 2ETA Plausible: based on correct answer for Unit I. Correct:

B.

C.

D.

D.

Ques_656

For Official Use Only Page 112

Question #91

McGuire Nuclear Station

RO Exam

Bank Question: 658

Answer: D

1 Pt(s)

Unit 2 is in Mode 6 and refueling operations are currently in progress. Given the following plant conditions: The Fuel Handling Manipulator Crane Operator has lowered the mast onto the fuel assembly located at H-8 in preparation for offloading. All conditions/indications on the fuel handling manipulator crane are satisfied for latching the fuel assembly located at H-8, in accordance with procedure. Which one of the following describes the responsibility of the OATC (Operator at the Controls) in the control room, associated with moving the fuel assembly? A. Verifies proper instrument air pressure available for latching the fuel assembly and relays this information to the fuel handling SRO. Verifies the procedural step for the Fuel Handling SRO and initials the step after satisfactory completion of the step. Grants permission to the Fuel Handling SRO prior to latching the fuel assembly. D. Grants permission to the Fuel Handling SRO prior to unloading the fuel assembly out of position 11-8.

B.

C.

Distracter Analysis: A. Incorrect: not an OATC responsibility required by fuel handling procedure Plausible: VI is an essential plant support system Incorrect: not an OATC responsibility required by fuel handling procedure Plausible: procedural compliance is often a requirement for the OATC Incorrect: not an OATC responsibility required by fuel handling procedure Plausible: This is the responsibility of the Fuel Handling SRO this was previously the responsibility of the OATC no longer. Correct Answer:

B.

C.

D.

Ques 658.

For Official Use Only Page 113

Question #92

McGuire Nuclear Station

RO Exam

Bank Question: 661 1 Pt(s)

Answer: A

Units 1 and 2 are at 100% power. Given the following conditions: Unit 2 has experienced 2 fuel pin failures. The mechanical seal has failed on NI pump 2B. The NI-2B pump room general area is 200 mrem/hr. In order to reach the N1-2B pump room the workers must transit through 6 rem/hr high radiation area for 1 minute and return. Worker A has an accumulated annual dose of 400 mrem, respectively.

How long can worker A participate in the seal repair on NI Pump 2B without exceeding the alert flag exposure limit for external exposure? A. B. C. No longer than 5 hours No longer than 5.5 hours No longer than 6 hours

D.

No longer than 7 hours

Distracter Analysis: The candidate should determine that the alert flag exposure limit is 80% of 2000 mrem admin limit = 1600 mrem Transient exposure is 200 mrem (6000mrem/hr x 2160hr). (During transit to and from job). 400 mrem + 200 mrem = 600 mrem 1600 mrem 600 mrem = 1000 mrem allowable before reaching alert flag exposure admin limit 1000 mrem 1200 mrem/hr = 5 hours A. B. C. D. Correct: Incorrect: The answer is 5 hours Plausible: based on calculating a one-way transit dose. Incorrect: The answer is 5 hours. Plausible: based on no transit dose. Incorrect: The answer is 5 hours. Plausible: based on using admin limit (2000) versus alert flag.

Ques_661

For Official Use Only

Page 114

Question #93

McGuire Nuclear Station

RO Exam

Bank Question: 666 1 Pt(s)

Answer: D

Unit 1 was operating at 100% power when a total loss of offsite power occurred. Given the following events and conditions: The diesel generators started and loaded as designed The operators completed E-0 (Reactor Trip Response) The operators reached step II of ES-0.1 (Natural Circulation Cooldown) which requires the cooldown of the NC system

Which one of the following components are necessary to prevent the formation of a void in the reactor vessel while cooling down the plant? A. B. C. D. NI pumps Head vent VL/VU fans CRDM fans

Distracter Analysis: A. Incorrect: NI pumps are not required - SI is blocked Plausible: If the candidate thinks that SI is required to prevent void in reactor vessel Incorrect: not required for cooldown Plausible: head vent would relieve a void in the reactor vessel but would not prevent a void Incorrect: not required for cooldown Plausible: these are another set of fans in containment that provide cooling to various components Correct answer

B.

C.

D.

Ques_666

For Official Use Only Page 115

Question #94

McGuire Nuclear Station

RO Exam

1 Pt(s) Bank Question: 668 Answer: D

Unit 2 is conducting a core reload and one hundred thirty fuel assemblies have been loaded into the core. The following data has been recorded upon completion of each assembly reload sequence group: Reload Sequence Group 9 10 11 12 13 14 15 16 17 18 19 Source Range Count Rate 300 360 400 425 4 6 0 520 600

If the reactivity added by each load sequence group was the same, during which reload group (if any) would you predict that the reactor would reach criticality? A. B. C. 9 D. 16 18 1

The reactor will not reach criticality.

Distracter Analysis: Using the thumb rule that if the count rate doubles, the reactor is Y2 way to criticality, the following calculation shows: 300-600 counts count rate doubles between reload sequences 10 and 15 have only 4 more load groups to go - will not go critical. A. Incorrect: The reactor will not go critical Plausible: based on misapplication of the thumb rule (count rate doubles at criticality). Incorrect: The reactor will not go critical Plausible: if the candidate does not understand or misapplies the thumb rule

B.

Ques_668

For Official Use Only Page 116

Question #94
C.

McGuire Nuclear Station

RO Exam

Incorrect: The reactor will not go critical Plausible: if the candidate does not understand or misapplies the thumb rule D. Correct:

Ques 668

For Official Use Only Page 117

Question #95

McGuire Nuclear Station

RO Exam

Bank Question: 669 1 Pt(s)

Answer: C

Unit 2 was operating at 90% after a start-up from a refueling outage. A PORV is found to be leaking and the PORV block valve was shut. Given the following PRT conditions: Level 78% Pressure 8 psig Temperature 115F What action is required to restore normal operating conditions to the PRT? A. B. Vent/purge the PRT to containment. Vent/purge the PRT to the waste gas system.

C. D.

Initiate cooling of the PRT. Lower the PRT level .

Distracter Analysis: The priority of action to reduce pressure is: 1. Cool the PRT 2. Reduce level 3. Purge to waste gas A. Incorrect: cannot be performed at power as the vent valve is inside containment and is inaccessible at power - not a good option Plausible: venting to containment would accomplish the required action Incorrect: venting will not address the hi temp problem - first priority action is to cool the PRT Plausible: Venting would reduce pressure Correct: Incorrect: Will reduce pressure BUT will not reduce temperature Plausible: reducing level will reduce pressure ad is the 211d priority of action to be taken.

B.

C. D.

Ques_669

For Official Use Only Page 118

Question #96

McGuire Nuclear Station

RO Exam

Bank Question: 670 1 Pt(s)

Answer: B

Which one of the following statements correctly describes the requirement and allowable time to start the standby makeup pump during a loss of all AC power (station blackout). A. The operators must start the standby makeup pump within 10 minutes to provide makeup water to the pressurizer to prevent reaching the pressurizer low level alarm. The operators must start the standby makeup pump within 10 minutes to provide makeup water to NCP seals to prevent seal degradation. The operators must start the standby makeup pump within 15 minutes to provide makeup water to the pressurizer to prevent reaching the pressurizer low level alarm.

B.

C.

D.

The operators must start the standby makeup pump within 15 minutes to provide makeup water to NCP seals to prevent seal degradation.

Distracter Analysis: A. B. C. Incorrect: reason for action is to protect NCP seals Plausible: partially correct - time commitment is 10 minutes Correct answer Incorrect: time commitment is 10 minutes Plausible: adding water to the reactor does maintain pressurizer level Incorrect: time commitment is 10 minutes Plausible: partially correct - does protect NCP seals

D.

Ques_670

For Official Use Only Page 119

Question #97

McGuire Nuclear Station

RO Exam

Bank Question: 67/


1 Pt(s)

Answer: C

Unit 1 was releasing the contents of a waste gas decay tank in accordance with an approved release permit. If 1EMF-50(L) (Waste Gas Disch) failed high during the release, which one of the following actions must be taken? A. Stop the release and complete repairs on 1EMF-50(L) before restarting. Stop the release and recalculate the trip set points using 1EMF-50(H) as the release path monitor Restart the release using IEMF-36(L) (Unit Vent Gas) as the release path monitor. Continue the release using IEMF-36(L) as the release path monitor

B.

C.

D.

Distracter Analysis: A. Incorrect: not required to use 1EMF-50(L) as the only qualified release path monitor Plausible: if the candidate does not recognize that 1EMF-36(L) can be used to monitor the release path Incorrect: 1EMF-50(11) does not automatically trip WM-46 and cannot be used as a waste gas release path monitor Plausible: if the candidate thinks that substituting the high range of 1EMF-50(L) provides the same automatic protection Correct answer Incorrect: the release would be terminated when lEMF-50(L) tripped Plausible: if the candidate did not recognize that 12EMF-50(L) provided an automatic trip of WM-46 and terminated the release.

B.

C. D.

Ques_671

For Official Use Only Page 120

Question #98

McGuire Nuclear Station

RO Exam

Bank Question: 672 1 Pt(s)

Answer: C

Which one of the following conditions would cause 1EMF-51A (Containment TRN A (Hi Range)) to increase. A. B. C. D. An increase in alpha radiation from a tritium leak A cloud of radioactive gas that emits beta radiation An increase in gamma flux from a failed fuel event An increase in neutron radiation from a criticality event

Distracter Analysis: A. Incorrect: does not respond to alpha radiation - nor does tritium emit and alpha particle Plausible: a type of radiological hazard - provided for psychometric balance Incorrect: does not respond to beta radiation Plausible: some detectors respond to beta such as scintillation detectors Correct answer Incorrect: does not respond to neutron radiation Plausible: would seem appropriate to measure neutron radiation for criticality events

B.

C. D.

Ques_672

For Official Use Only Page 121

Question #99
B ank Question: 677 1 Pt(s)

McGuire Nuclear Station


Answer: B

RO Exam

Unit 1 was shutdown in Mode 5 preparing for a refueling outage. Given the following conditions and events: LTOPs key switches set to "LOW PRESSURE" PORVs are in AUTO mode NC temperature = 190 F cooling down to 170 F NC system in shutdown cooling on ND train A NV pump 1A is operating NV pump 1B was started at 0200 for an NV pump swap

Which one of the following statements correctly describes the required operator actions (if any) at 0215? A. This condition is allowed to continue for 15 more minutes to complete the NV pump swap. Immediately verify ND suction relief valve is operable. Immediately verify NCS cooldown rate is > 20 F/hr.

B. C.

D.

Immediately open one PORV and associated block valve.

Distracter Analysis: A. Incorrect: operation of 2 NV pumps is only allowed for 15 minutes while swapping pumps Plausible: if the candidate does not know the tech spec limit for swapping NV pumps is 15 minutes Correct Answer: Incorrect: The cooldown rate must be < 20 F/hr not > 20 F/hr Plausible: if the candidate does not understand the concern for PTS in this condition Incorrect: One PORV will NOT allow sufficient vent path to provide cold over-pressure protection for 2 NV pumps - and power must be removed from the valves Plausible: opening BOTH PORVs will provide sufficient protection.

B. C.

D.

Ques_677

For Official Use Only Page 122 QUESTIONS REPORT for HL-14 NRC RO DRAFT EXAM 1. 001K6.03 001/2/2/RODS - TRIP BREAKERS/C/A - 3.7/NEW/R/NRC RO/TNT / RLM Given the following: The reactor trips from 100% power. One reactor trip breaker fails to open. The BPLP C-7 status light is illuminated. Which ONE of the following would be CORRECT regarding RCS Tave control assuming no operator action and all other systems function as designed? A. If Rx. Trip Breaker "A" failed to open, ARVs control Tave at 562 degrees F. B. If Rx. Trip Breaker "B" failed to open, ARVs control Tave at 562 degrees F. C. If Rx. Trip Breaker "A" failed to open, steam dumps control Tave at 557 degrees F.

D. If Rx. Trip Breaker "B" failed to open, steam dumps control Tave at 557 degrees F.
Feedback

K/A 001 Control Rod Drive System: K6.03 Knowledge of the effect of a loss or malfunction on the following CRDS components. Reactor trip breakers, including controls. K/A MATCH ANALYSIS Question gives a plausible scenario where a Reactor Trip Breaker fails to open after a reactor trip. Candidate must determine the effects on a plant control system (steam dump controls) to determine the final RCS Tave and method of control. ANSWER / DISTRACTOR ANALYSIS A. Incorrect. Steam dumps would arm on C-7. Plausible candidate may recall P-4 train "A" arms dumps but not recall C-7 would also arm the dumps. Dumps should control Tave at 559 degrees F in this situation and not ride on the ARVs at 562 degrees F. B. Incorrect. Steam dumps would arm on C-7 and P-4 Train "A". Plausible candidate may confuse which train arms dumps and not recall C-7 also arms the dumps. Dumps should control Tave at 557 degrees F in this situation and not ride

on the ARVs at 562 degrees F.


Monday, August 13, 2007 10:00:32 AM 1

QUESTIONS REPORT for HL-14 NRC RO DRAFT EXAM C. Correct. Steam dumps would arm on C-7 and P-4 Train "B" would shift their operation to the "plant trip mode". Dumps should control Tave at 557 degrees F in this situation. D. Incorrect. Steam dumps would arm on C-7 and P-4 Train "A". Plausible candidate may confuse which train shifts dumps to the plant trip mode. If Train "B" trip breaker failed to open, steam dumps operate in the "load reject mode". Dumps would control Tave at 559 degrees F in this situation. REFERENCES Power Point Presentation, V-LO-PP-21201, Revision # 2, Steam Dumps (slides 59 - 88 in particular cover this question) Vogtle Text, V-LO-TX-21201, Revision # 1, Steam Dumps (pages # 14, 20 - 22, 34 - 36 in particular cover this question) VEGP learning objectives: LO-PP-21201-07, Discuss how the Steam Dump System will respond to a Reactor Trip from Reactor Power level. LO-PP-21201-11, Identify all the conditions that will arm the Steam Dump System and when they're normally activated. LO-PP-21201-17, Discuss how the Steam Dump System will respond to a large temperature error signal in the Tavg Mode of operation.
Notes

Response form Answers MCS Time: 1 Points: 1.00 Version: 0 1 2 3 4 5 6 7 8 9 Answer: CADBDADDCB Scramble Range: A- D

Monday, August 13, 2007 10:00:32 AM

QUESTIONS REPORT for HL-14 NRC RO DRAFT EXAM 2. 003G2.4.11 001/1/2/DROPPED ROD - AOP/MEM - 3.4/MODIFIED/R/NRC RO/TNT / RLM Given the following conditions / events: Plant was at 100%. A rod drops in CBD, crew enters AOP-18003-C, "Rod Control System Malfunction". Crew is preparing to realign the dropped rod to the other rods in CBD. All lift coils have been disconnected for the unaffected rods in CBD. Bank selector switch is selected to CBD. ROD CONTROL URGENT FAILURE annunciator illuminates at the start of rod pull and the RO acknowledges the alarm. Which ONE of the following is CORRECT concerning the "ROD CONTROL URGENT FAILURE" annunciator and actions the crew should take in accordance with 18003-C? A. EXPECTED alarm, continue the rod pull. A multiplexing failure occurs due to the lift coils of all the other rods in the bank being disconnected. B. EXPECTED alarm, continue the rod pull. A regulation failure occurs due to the lift coils of all the other rods in the bank being disconnected. C. UNEXPECTED alarm, stop the rod pull. A multiplexing failure occurs due to the lift coils of all the other rods in the bank being disconnected. D. UNEXPECTED alarm, stop the rod pull. A regulation failure occurs due to the lift coils of all the other rods in the bank being disconnected.

Monday, August 13, 2007 10:00:32 AM

QUESTIONS REPORT for HL-14 NRC RO DRAFT EXAM

Feedback

K/A 003 Dropped Control Rod: G2.4.11 Knowledge of abnormal condition procedures. K/A MATCH ANALYSIS Question gives a plausible scenario where a dropped rod has occurred. The candidate must pick the correct reason for a ROD CONTROL URGENT FAILURE alarm and whether rod withdrawal may continue. ANSWER / DISTRACTOR ANALYSIS A. Incorrect. Rod pull is allowed, however the annunciator is caused by a regulation failure, not multiplexing. B. Correct. Rod withdrawal is allowed, regulation is cause of Urgent Failure. C. Incorrect. Alarm is expected, action is from AOP if alarm inhibits rod motion. Plausible candidate may choose this as Urgent Failure normally inhibits rod motion. Rod withdrawal is also allowed. D. Incorrect. Alarm is expected. However, action is from RNO for rod motion inhibited. Plausible the candidate could think the alarm must be reset to allow motion and think rod withdrawal is not allowed. REFERENCES AOP-18003-C, "Rod Control Malfunction" section A for Dropped Rods in Mode 1. (Step A22 in particular AER and RNO and preceding note.) LO-LP-60303-03-003 from Vogtle LO Active Exam Bank V-LO-PP-27101, "Rod Control System" Power Point pages 85 and 86 in particular) V-LO-TX-27101, "Rod Control System" Vogtle Text page # 55 VEGP learning objectives: LO-PP-27201-20, Describe what happens upon receipt of a Rod Control Sytems Urgent Failure; include how rod motion is inhibited. LO-PP-60303-03, State why an urgent failure alarm will
Monday, August 13, 2007 10:00:32 AM

sound during a dropped rod retrieval.


Notes

QUESTIONS REPORT for HL-14 NRC RO DRAFT EXAM


Response form

Answers MCS Time: 1 Points: 1.00 Version: 0 1 2 3 4 5 6 7 8 9 Answer: BCBCBBACDD Scramble Range: A- D

Monday, August 13, 2007 10:00:32 AM

QUESTIONS REPORT for HL-14 NRC RO DRAFT EXAM 3. 003K5.02 001/2/1/RCP - CSTDWN RCS/C/A - 2.8/MODIFIED/RINRC RO/TNT / RLM The unit is at 100% power: - RCP # 3 trips due to an operator error on the QMCB . - The crew verifies reactor trip. Which ONE of the following would be CORRECT regarding the affected loop # 3 SG level and RCS delta T INITIAL responses following the RCP trip? A. SG level would shrink lower than the other SGs immediately after the RCP trips. RCS loop delta T would be lower than the other RCS loop delta T's. B. SG level would swell higher than the other SGs immediately after the RCP trips. RCS loop delta T would be lower than the other RCS loop delta T's. C. SG level would swell higher than the other SGs immediately after the RCP trips. RCS loop delta T would be higher than the other RCS loop delta T's. D. SG level would shrink lower than the other SGs immediately after the RCP trips. RCS loop delta T would be higher than the other RCS loop delta T's.

Monday, August 13, 2007 10:00:32 AM

QUESTIONS REPORT for HL-14 NRC RO DRAFT EXAM


Feedback

K/A 003 Reactor Coolant Pumps: K5.02 Knowledge of the operational implications of the following concepts as they apply to the RCS. Effects of RCP coastdown on RCS parameters. K/A MATCH ANALYSIS Question gives a plausible scenario with an RCP trip. The candidate must choose the correct SG level response and RCS delta T response immediately following the RCP trip (during RCP coastdown). ANSWER / DISTRACTOR ANALYSIS A. Correct. SG level will shrink immediately following the RCP trip due to less heat input. Loop delta T will lower due to no heat removal through the affected loop. B. Incorrect. Plausible candidate could think SG level would swell due to no heat removal from the loop. Delta T response is correct. C. Incorrect. Plausible candidate could think SG level would swell due to no heat removal from the loop and think delta T could rise for the same reason. D. Incorrect. SG level response is correct. Plausible candidate could think no heat removal from the loop could cause a higher delta T. REFERENCES Vogtle May 2006 NRC RO Retake Exam question # 2 (used as a base to modify) Various Vogtle LO Active Bank questions (LO-LP-60305-01, 02, and 03)

VEGP learning objectives:


LO-LP-60305-01 Describe how and why, during a partial loss of flow condition, the affected loop steam generator level will be affected. LO-LP-60305-09 Given the entire AOP, describe: a. Purpose of selected steps. b. How and why the step is being performed. c. Expected response of the plant / parameter(s) for the step.
Monday, August 13, 2007 10:00:32 AM

Notes

QUESTIONS REPORT for HL-14 NRC RO DRAFT EXAM


Response form

Answers MCS Time: 1 Points: 1.00 Version: 0 1 2 3 4 5 6 7 8 9 Answer: ADD B D A AD B C Scramble Range: A- D

Monday, August 13, 2007 10:00:32 AM

QUESTIONS REPORT for HL-14 NRC RO DRAFT EXAM 4. 004K6.09 001/2/1/CVCS - VCT DIVERT/C/A - 2.8BANK/R/NRC RO/TNT / RLM Given the following: The plant is at 40% power with 120 gpm letdown in service. VCT level transmitter, LT-112 fails HIGH. At the time of failure VCT level transmitter LT-185 reads 50%. What ONE of the following is CORRECT if NO operator action is taken ? VCT level lowers ...... A. until charging pumps lose suction and start to cavitate. B. faster than auto makeup input and charging suction shifts to the RWST. C. with NO auto makeup capability and charging suction shifts to the RWST. D. until auto makeup starts and maintains VCT level between 30% and 50%.

Monday, August 13, 2007 10:00:32 AM

QUESTIONS REPORT for HL-14 NRC RO DRAFT EXAM


Feedback

K/A 004 Chemical Volume Control System (CVCS): K6.09 Knowledge of the effect of a loss or malfunction on the following CVCS components. Purpose of VCT divert valve. K/A MATCH ANALYSIS Question asks plausible scenario on the effect of a failure of VCT LT-112A. ANSWER / DISTRACTOR ANALYSIS A. Correct. LT-112 failing high will cause letdown divert to RHUT via LV-112A. VCT level will lower, RWST auto swapover is defeated and charging pumps lose suction due to VCT auto makeup capability is defeated (2 / 2 coincidence) B. Incorrect. Plausible the candidate may invert the instrument that initiates auto makeup to the VCT with a swap to the RWST on low level with full letdown divert. C. Incorrect. Plausible the candidate may invert the instrument that initiates auto makeup to the VCT and no makeup occurs. RWST auto swapover is defeated and requires a 2 / 2 coincidence. D. Incorrect. Plausible the candiate may invert the instrument that initiates auto makeup to the VCT and level will cycle between 30 to 50% with auto makeup. REFERENCES ARP-17007-1/2, window E05 for VCT HI / LO LEVEL annunciator. LO-PP-09200-01-001 from Vogtle LO Active Exam Bank. LO-PP-09100 CVCS Letdown (pages 113 and 129 in particular) VEGP learning objectives: LO-PP-09100-03, State the purpose and describe the control signals, setpoints, and any interlocks for the following:

b. VCT divert valve, LV-112A


Notes Monday, August 13, 2007 10:00:32 AM

10

QUESTIONS REPORT for HL-14 NRC RO DRAFT EXAM


R e s p o nse form Answers MCS Time: 1 Points: 1.00 Version: 0 1 2 3 4 5 6 7 8 9 Answer: AADACBDCDA Scramble Range: A- D

Monday, August 13, 2007 10:00:32 AM

11

QUESTIONS REPORT for HL-14 NRC RO DRAFT EXAM 5. 005A4.03 001/2/1/RHR- TEMP, HTRS, FLO/MEM - 2.8/NEW/R/NRC RO/TNT / RLM Given the following Unit 1 conditions: Two RCPs are in service Both Trains of RHR are in service HV-606 and HV-607 (RHR Hx outlets) are 50% open. RHR flow is 3000 gpm per train. PRZR level is being maintained at 25%. RCS is stable at 180 degrees F and 340 psig. All mode 4 entry requirements have been completed. The Shift Supervisor has directed an RCS heatup to 330 degrees F to begin. Which ONE of the following describes the CORRECT actions to take to commence the MAXIMUM heatup rate allowed in accordance with UOP 12001-C "Unit Heatup to Hot Shutdown (Mode 5 to Mode 4)"? A. Adjust FV-0618 / FV-619 to establish a heatup rate not to exceed the procedural limit of 50 F per hour. B. Adjust HV-606 / HV-607 to establish a heatup rate not to exceed the procedural limit of 100 F per hour. C. Adjust HV-606 / HV-607 to establish a heatup rate not to exceed the procedural limit of 50 F per hour. D. Adjust FV-618 / FV-619 to establish a heatup rate not to exceed the procedural limit of 100 F per hour.

Monday, August 13, 2007 10:00:32 AM

12

QUESTIONS REPORT for HL-14 NRC RO DRAFT EXAM


F e edback

K/A 005 Residual Heat Removal System (RHR): A4.03 Ability to manually operate and / or monitor in the control room. RHR temperature, PRZR heaters and flow, and nitrogen. K/A MATCH ANALYSIS Question gives a plausible scenario where a heatup to Mode 4 from 180 degrees F has been initiated. The candidate must determine the proper heatup rate to establish using RHR and the correct RHR valves to use. ANSWER / DISTRACTOR ANALYSIS A. Incorrect. Per Tech Specs and procedure there is a 100 F hour heatup rate limit. Plausible the candidate may confuse heatup rate limit or confuse the valves used to establish the heatup rate. HV-606 / 607 are the valves to use to throttle cooling. B. Correct. Heatup limited to 100 F per hour and HV-606 / 607 are the correct valves. C. Incorrect. Per Tech Specs and procedure there is a 100 F hour heatup rate limit. This is the correct valves to use but the wrong heatup rate. D. Incorrect. Per Tech Specs and procedure the heatup limit is 100 F per hour and this part is correct. However, this is the wrong RHR valves to use for starting heatup. Valves should be HV-606 / 607.. REFERENCES UOP-12001-C, Steps 4.5.1, 4.5.2, and 4.5.3 for Mode 4 entry and heatup to 330 F. VEGP learning objectives: L O L P 61209-12, Describe basically how an RCS heatup is performed.
Notes Response form Answers

MCS

Time: 1

Points:

1.00

Version: 0 1 2 3 4 5 6 7 8 9 Answer: BB B CADC CAB

Scramble Range: A- D

Monday, August 13, 2007 10:00:33 AM

13

QUESTIONS REPORT for HL-14 NRC RO DRAFT EXAM 6. 005K2.03 001/2/1/RHR-BOUNDARY MOVS/MEM - 2.7/NEW/R/NRC RO/TNT / RLM Given the following conditions: Unit 1 at 100% power. The following alarm status exists on the Unit 1 QEAB. - ALB34 window E06 for STARTER 1CD1I5N TROUBLE is illuminated. - ALB34 window F06 for INVERTER 1CD115 TROUBLE is illuminated. ALB34 window E07 for STARTER 1DD1I6N TROUBLE is extinguished. ALB34 window F07 for INVERTER 1DD116 TROUBLE is extinguished. Which ONE of the following is CORRECT regarding the status of the RHR pump loop suction valve power supplies ? A. Train "A" RHR pump loop suction power status is correct, the valve is energized. B. Train "A" RHR pump loop suction power status is correct, the valve is de-energized. C. Train "B" RHR pump loop suction power status is correct, the valve is energized. D. Train "B" RHR pump loop suction power status is correct, the valve is de-energized.

Monday, August 13, 2007 10:00:33 AM

14

QUESTIONS REPORT for HL-14 NRC RO DRAFT EXAM


Feedback

K/A 005 Residual Heat Removal System (RHR): K2.03 Knowledge of the bus power supplies to the following. RCS pressure boundary motor-operated valves. K/A MATCH ANALYSIS Question gives a plausible scenario with a wrong unit clearance results in misoperation of an RHR loop suction power supply. Candidate must choose from indications which loop suction power supply status is correct. ANSWER / DISTRACTOR ANALYSIS A. Incorrect. Annunciator status is correct for Train "A" loop suction. The valve is normally de-energized at power. Plausible the candidate could confuse the annunciator status and / or the valve power status. B. Correct. Annunciators normally illuminated at power (green windows normally lit like RCP oil lift pumps). Valve is normally de-energized. C. Incorrect. Annuciators normally illuminated at power with the valve deenergized. Plausible the candidate could confuse the annunciator status and / or the valve power supply status. D. Incorrect. Annuciators normally illuminated at power with the valve deenergized. Plausible the candidate could confuse the annunciator status. REFERENCES ARP 17034 for QEAB, windows E06, E07, F06, and F07. SOP-13011-1, Residual Heat Removal System, pages 12, 13, 17, and 18. {This is steps 4.1.4.b(3), 4.1.7.m. VEGP learning objectives:

LO-PP-12101-12 Briefly describe the RHR system alignment during normal power operations and during RCS cooldown.
Notes

Response form

Monday, August 13, 2007 10:00:33 AM

15

QUESTIONS REPORT for HL-14 NRC RO DRAFT EXAM


A n swers MCS Time: 1 Points: 1.00 Version: 0 1 2 3 4 5 6 7 8 9 Answer: BCDAABAADD Scramble Range: A- D

Monday, August 13, 2007 10:00:33 AM

16

QUESTIONS REPORT for HL-14 NRC RO DRAFT EXAM 7. 006A4.02 001/2/1/ECCS- VALVES/C/A - 4.0/BANK/R/NRC RO/TNT / RLM Given the following sequence of events: An SI occurs on Unit 1 The crew enters 19011-C, "SI Termination" SI is reset and the ECCS pumps shut down The Rx. Trip Breakers were cycled to defeat the P-4 seal in. Containment Sump to RHR Pump Suction valves HV-8811A and HV-8811B are shut RWST level over time lowers to less than 39% Which ONE of the following is CORRECT regarding the status of HV-8811A / B valve positions and the RWST auto swapover function? A. Automatically open, this function is always active and cannot be defeated. B. Remain closed, this function is defeated by resetting the SI actuation signal. C. Remain closed, this function was defeated by the cycling of the trip breakers. D. Automatically open, this function is defeated by the RWST SI reset handswitches.

Monday, August 13, 2007 10:00:33 AM

17

QUESTIONS REPORT for HL-14 NRC RO DRAFT EXAM


Feedback

K/A 006 Emergency Core Cooling System (ECCS): A4.02 Ability to manually operate and / and or monitor in the control room. Valves K/A MATCH ANALYSIS Question gives a plausible scenario where an SI actuation has occurred and SI has been reset and Rx. Trip Breakers cycled to clear P-4 seal in. Candidate has to choose the correct response of RWST auto swapover function if RWST level lowers to < 39%. ANSWER / DISTRACTOR ANALYSIS A. Incorrect. RWST SI Reset can defeat this function. Plausible candidate may not recall this since the switches are not manipulated that often. Valves coming open part would be correct. B. Incorrect. Valves would automatically open. Plausible candidate could confuse the RWST SI Reset function with resetting of SI. C. Incorrect. Plausible candidate could confuse the cycle of the Rx. Trip Breakers to clear P-4 seal in with other functions such as FWI or SI on Hi Hi SG level. D. Correct. In this condition the function would still occur and the valves would open until the RWST SI reset handswitches are reset. REFERENCES LO-PP-13101-04-008 Vogtle LO Active Bank question. LO-PP-13101 "ECCS" Power Point (pages 57 and 58 in particular) VEGP learning objectives: LO-PP-13101-03, Describe all valve interlocks associated with the ECCS system.

HV-8811A / B
Notes

Monday, August 13, 2007 10:00:33 AM

18

Response form

QUESTIONS REPORT for HL-14 NRC RO DRAFT EXAM


A n swers MCS Time: 1 Points: 1.00 Version: 0 1 2 3 4 5 6 7 8 9 Answer: DADCDCCBC A Scramble Range: A- D

Monday, August 13, 2007 10:00:33 AM

19

QUESTIONS REPORT for HL-14 NRC RO DRAFT EXAM 8. 006K6.19 001/2/1/ECCS-MODE CHANGE/MEM - 3.7/BANKJR/NRC RO/TNT / RLM Given the following conditions: The plant is at 360 degrees F with a cooldown in progress for a refueling outage. SI pump A has been declared INOPERABLE due to pump seizure. Based on the above conditions, which ONE of the following would be CORRECT and why? A. Mode 4 entry is allowed. The minimum requirements of one high head subsystem and one RHR subsystem is available. B. Mode 4 entry is allowed. The minimum requirements of one high head subsystem, one SI subsystem and one RHR subsystem is available. C. Mode 4 entry is not allowed. The minimum requirements of two high head subsystems and two SI subsystems is not available. D. Mode 4 entry is not allowed. The minimum requirements of two high head subsystems, two SI subsystems and two RHR subsystems is not available.

Monday, August 13, 2007 10:00:33 AM

20

QUESTIONS REPORT for HL-14 NRC RO DRAFT EXAM


F e edback

K/A 006 Emergency Core Cooling System (ECCS): K6.19 Knowledge of the effect of a loss or malfunction of the following will have on the ECCS: HPI / LPI systems (mode change) K/A MATCH ANALYSIS Question gives a plausible scenario where a Unit 2 cooldown for a refueling outage is in progress. The candidate must determine the minimum required ECCS trains for Mode 4 entry. ANSWER / DISTRACTOR ANALYSIS A. Correct. One high head and one low head available allows the mode change. B. Incorrect. Plausible the candidate may think the SIP subsystem is also required for this mode as minimum required equipment. C. Incorrect. Plausible the candidate may think 2 high head and 2 intermediate head systems required for this mode and the change would not be allowed. D. Incorrect. Plausible the candidate may think all ECCS systems required for the mode and would prevent a mode change. REFERENCES Technical Specifications and bases for 3.5.3 ECCS - Shutdown. LO-PP-13101-14-009 from Vogtle LO Active Exam bank. VEGP learning objectives: L O PP-13101-14, State the Technical Specification, LCO, bases, applicability, and one hour or less actions for the Emergency Core Cooling Systems.
Notes Response form

Answers MCS Time: 1 Points: 1.00 Version: 0 1 2 3 4 5 6 7 8 9 Answer: ACACACAABC Scramble Range: A- D

Monday, August 13, 2007 10:00:33 AM

21

QUESTIONS REPORT for HL-14 NRC RO DRAFT EXAM 9. 0071(4.01 001/2/1/QUENCH TK COOLING/MEM - 2.6/BANK/R/NRC RO/TNT / RLM Which ONE of the following is a CORRECT method to cooldown the PRT in accordance with 13004-1/2, "Pressurizer Relief Tank Operation"? A. Drain the PRT to the CNMT sump while making up to the PRT from the RMWST. B. Drain the PRT to the CNMT sump while making up to the PRT from Demin Water. C. Recirculate the PRT through the RCDT using NSCW to cool the heat exchanger. D. Recirculate the PRT through the RCDT using ACCW to cool the heat exchanger.

Monday, August 13, 2007 10:00:33 AM

22

QUESTIONS REPORT for HL-14 NRC RO DRAFT EXAM


Feedback

K/A 007 Pressurizer Relief Tank (Quench Tank): K4.01 Knowledge of PRTS design feature(s) and / or interlock(s) which provide for the following: Quench tank cooling. K/A MATCH ANALYSIS Question gives a plausible scenario where the PRT has high temperature. The candidate must pick the correct description of the 8 hour PRT cooldown method. ANSWER / DISTRACTOR ANALYSIS A. Incorrect. This method not described in procedures, plausible candidate could confuse with bleed and feed actions of other procedures such as CCW. B. Incorrect. This method not described in procedures and the makeup source is wrong too. Plausible candidate could confuse the cooldown methods and many of our makeup sources are from the DWST. C. Incorrect. Method describes is correct but cooling water to RCDT Hx is wrong. Plausible candidate could confuse cooling medium as NSCW services many loads including loads in the containment where RCDT is located. D. Correct. Method and cooling medium for RCDT are both correct. REFERENCES V-LO-PP-16301-09-003 Vogtle LO Active Exam Bank Harris February 2006 NRC RO Exam question # 8 VC Summer October 2006 NRC SRO Retake Exam question # 13 Harris March 2004 NRC RO Exam question # 23 LO-PP-16301 "Pressurizer and Pressurizer Relief Tank" (pages 20 and 21 in particular) V-LO-PP-04101, "ACCW" slide # 12 in particular VEGP learning objectives: LO-PP-16301-09, Describe the methods for cooling the PRT.

Monday, August 13, 2007 10:00:33 AM

23

QUESTIONS REPORT for HL-14 NRC RO DRAFT EXAM


N o t es R e sponse form Answers MCS Time: 1 Points: 1.00 Version: 0 1 2 3 4 5 6 7 8 9 Answer: DAD AA C B BAC Scramble Range: A- D

Monday, August 13, 2007 10:00:33 AM

24

QUESTIONS REPORT for HL-14 NRC RO DRAFT EXAM 10. 008A4.10 001/2/1/CCW - TWO COOLERS/MEM - 3.1/NEW/R/NRC RO/TNT / RLM Regarding the CCW and ACCW system heat exchangers: Which ONE of the following is CORRECT regarding when you need both of either systems heat exchangers in service? A. ACCW - to provide necessary cooling to both Diesel Generators during an LOSP event on both RATs. B. CCW - to provide necessary cooling to both RHR pump seal water heat exchangers during the ECCS injection phase of a DBA LOCA. C. ACCW - to provide necessary cooling to all RCPs during a small break LOCA event. D. CCW - to provide necessary cooling to the Spent Fuel Pool Cooling System to maintain SFP temperature < 120 degrees F during a full core offload.
F e edback

K/A 008 Components Cooling Water System (CCW): A4.10 Ability to manually operate or monitor in the control room: Conditions that require the operation of two CCW coolers. K/A MATCH ANALYSIS Question gives plausible scenarios for the candidate to choose involving ACCW and CCW that require the candidate to determine which event requires both trains of the Hx to be in service. ANSWER / DISTRACTOR ANALYSIS A. Incorrect. Plausible the candidate may recall the ACCW / NSCW Hx. alignment where NSCW is sometimes tagged out to the ACCW Hx with one train bypassed. One train of ACCW Hx. is adequate and does not affect NSCW flow to the DGs. B. Incorrect. Although CCW normally aligend for seal cooling to RHR pumps, during the ECCS injection phase with RWST low water temperature CCW cooling would not be required. During Cold and Hot Leg recirculation phases it would. Plausible the candidate may think CCW required during injection phase. C. Incorrect. Plausible the candidate may think both ACCW HX necessary to cool RCPs during a small break LOCA. D. Correct. Design of SFP is to be maintained less than 120 degrees F with a full core
Monday, August 13, 2007 10:00:33 AM

25

QUESTIONS REPORT for HL-14 NRC RO DRAFT EXAM offload and both Hx in service. For this you would need both trains of CCW in service. REFERENCES V-LO-PP-25102, Spent Fuel Pool Cooling (slide # 12 in particular) V-LO-PP-04101, ACCW slide # 12 in particular shows both Hx to limit to 120 degrees F V-LO-PP-10101, CCW slide # 9 in particular shows CCW cools SFP Hx. SOP-130111/2, Residual Heat Removal System, Prerequisites & Initial Conditions 3.3 VEGP

learning objectives:
LO-PP-25101-03, Explain how the Spent Fuel Pool and Purification System performs the following functions: a. cooling LO-PP-25101-06, Explain the function of the following components: b. SFPCPS heat exchangers
N o t e s

Response form Answers MCS Time: 1 Points: 1.00 Version: 0 1 2 3 4 5 6 7 8 9 Answer: DC AB CAB DB A Scramble Range: A- D

Monday, August 13, 2007 10:00:33 AM

26

QUESTIONS REPORT for HL-14 NRC RO DRAFT EXAM 11. 008K3.03 001/2/1/CCW - RCP/MEM - 4.1/BANK/R/NRC RO/TNT / RLM Given the following: Unit 2 is at 30% power. ACCW pump # 1 is tagged out. - At 0115, ACCW pump # 2 trips - The RCP temperatures and vibrations are being monitored. - The current time is 0130. Which ONE of the following is CORRECT regarding operation of the RCPs? A. Trip the reactor, trip any RCP with shaft vibration in excess of 5 mils. B. Trip the reactor, trip any RCP that has # 1 seal leakoff greater than 5 gpm. C. Trip the reactor, trip all Reactor Coolant Pump due to total loss of ACCW. D. Trip the reactor, trip any RCP if stator temperature exceeds 230 degrees F.

Monday, August 13, 2007 10:00:34 AM

27

QUESTIONS REPORT for HL-14 NRC RO DRAFT EXAM


Feedback

K/A 008 Components Cooling Water System (CCW): K3.03 Knowledge of the effect that a loss or malfunction of the CCWS will have on the following: RCP K/A MATCH ANALYSIS Question gives a plausible scenario where ACCW flow is lost. The candidate must choose the correct action to take regarding RCP operations. ANSWER / DISTRACTOR ANALYSIS A. Incorrect. Plausible candidate may confuse this with frame vibration limit of 5 mils. Limit for shaft vibration is 20 mils. B. Incorrect. Plausible candidate may think seal degradation a reason to trip the RCP which would be in the abnormal operating range. However, this is not an immediate trip criteria until greater than 5.5 gpm. C. Correct. All RCPs should be stopped if ACCW flow lost for more than 10 minutes. This would require a reactor trip, then stopping of the RCPs per AOP for Total Loss of ACCW and SOP-13003-1/2 for RCP Operations. D. Incorrect. Plausible candidate may confuse this with seal water inlet temperature limit of 230 degrees F. REFERENCES LO-LP-60318-05-003 from Vogtle LO Active Exam Bank VC Summer April 2004 NRC RO Exam question # 8 SOP-13003-1/2, "Reactor Coolant Pump Operation", Precaution & Limitation 2.2.10, pages 29 and 30 for Operation with a Seal Abnormality Flow Chart Figures 1 & 2 AOP-18022-C, "Loss of Auxiliary Component Cooling Water", step # 6. VEGP learning objectives: LO-LP-16401-07, List the RCP components that are cooled by the ACCW system. LO-LP-60318-05, Describe the operator actions required during a loss of ACCW with the plant in operation and the RCP temperature limits are exceeded.

Monday, August 13, 2007 10:00:34 AM

28

QUESTIONS REPORT for HL-14 NRC RO DRAFT EXAM


N o t e s

Response form Answers MCS Time: 1

Points:

1.00

Version: 0 1 2 3 4 5 6 7 8 9 Answer: CABDBDABDB

Scramble Range: A- D

Monday, August 13, 2007 10:00:34 AM

29

QUESTIONS REPORT for HL-14 NRC RO DRAFT EXAM 12.


009EK1.01 001/1/1/SM LOCA - NAT CIRC/MEM - 4.2/BANK/R/NRC RO/TNT / RLM

Which ONE of the following describes reflux cooling flow following a small break LOCA? A. Liquid heated by the core is subsequently cooled inside the steam generator tubes and returned to the core via gravity counterflow along the bottom of each partially filled hot leg pipe. B. Steam produced inside the core is condensed in the steam generator tubes and returned to the core via gravity counterflow along the bottom of each partially filled hot leg pipe. C. Steam produced inside the core is condensed in the steam generator tubes and returned to the core via natural circulation flow along the bottom of each partially filled cold leg pipe. D. Liquid heated by the core is subsequently cooled inside the steam generator tubes and returned to the core via natural circulation flow along the bottom of each partially filled cold leg pipe.

Monday, August 13, 2007 10:00:34 AM

30

QUESTIONS REPORT for HL-14 NRC RO DRAFT EXAM


F e edback

K/A 009 Small Break LOCA EK1.01 Knowledge of the operational implications of the following concepts as they apply to the small break LOCA. Natural circulation and cooling, including reflux. K/A MATCH ANALYSIS Question gives a plausible scenario where a small break LOCA exists and asks the cooling mechanism for reflux cooling. The candidate must determine the proper description of reflux cooling from the choices listed. ANSWER / DISTRACTOR ANALYSIS A. Incorrect. Reflux flow occurs after core boiling starts. Steam has formed over the top of the core. Student may select due to liquid flow and reflux flow. Hot leg pipe would be the correct transport pipe. B. Correct. Reflux flow is the transport of steam to the SGs whiere it is condensed and travels back to the core along the hot leg pipe. C. Incorrect. Steam produced in the core is transported via the hot leg pipe where it is cooled and travels back along the same hot leg pipe to the core. Natural circulation has stopped at this point. Student may see this as natural circulation, however flow will not return in the cold leg pipe. D. Incorrect. Reflux flow occurs after core boiling starts. Steam has formed over top of the core. Student may select due to liquid flow and reflux flow. Cold leg pipe is also incorrect. REFERENCES Watts Bar July 2004 NRC RO exam question # 2. LO-PP-36101 "MCD - Mitigating Core Damage" slide # 26 & 27 in particular. VEGP learning objectives: LO-PP-36101-11, "Define the term "Pool Boiling". LO-PP-36101-15, "State the effectiveness of steam cooling versus water cooling in core heat removal".
Notes

Monday, August 13, 2007 10:00:34 AM

31

QUESTIONS REPORT for HL-14 NRC RO DRAFT EXAM

Response form Answers MCS Time: 1 Points: 1.00 Version: 0 1 2 3 4 5 6 7 8 9 Answer: BCCDCADDBB Scramble Range: A- D

Monday, August 13, 2007 10:00:34 AM

32

QUESTIONS REPORT for HL-14 NRC RO DRAFT EXAM 13.


0.10K3.02 001/2/1/PRZ PRESS - RPS/C/A - 4.0/BANK/R/NRC RO/TNT / RLM

Given the following conditions and events on Unit 1: The unit is at 100% power. Pressurizer pressure control is selected to the 457 / 456 position. PRZR pressure is 2235 psig. 1PT-457 fails high Which ONE of the following is the CORRECT plant / system response? A. PORV 1PV-455 opens, both spray valves remain closed and all PRZR heaters will energize. Pressure will stabilize near 2185 psig. B. PORV 1PV-456 opens, both spray valves remain closed and all PRZR heaters will energize. Pressure will stabilize near 2185 psig. C. PORV 1PV-455 opens, both spray valves open and all PRZR heaters turn off. Pressure will continue to decrease causing a reactor trip and safety injection. D. PORV 1PV-456 opens, both spray valves open and all PRZR heaters turn off. Pressure will continue to decrease causing a reactor trip and safety injection.
F e edback

K/A 010 Pressurizer Pressure Control System (PZR PCS): K3.02 Knowledge of the effect that a loss or malfunction of the PZR PCS will have on the following: RPS K/A MATCH ANALYSIS Question gives a plausible scenario with a PRZR pressure instrument failing high. The candidate must determine from the control switch selection and the channel failure the proper plant response. ANSWER / DISTRACTOR ANALYSIS A. Incorrect. This would be the response if PT-456, failed high with the current control channel selection but with the wrong PORV opening. Plausible the candidate could confuse the channel selection. B. Incorrect. This would be the correct plant response if PT-456 failed high with the
Monday, August 13, 2007 10:00:34 AM

33

QUESTIONS REPORT for HL-14 NRC RO DRAFT EXAM current control channel selection. Plausible candidate could confuse the channel selection. C. Correct. This is the correct plant response which would result in an RPS actuation. D. Incorrect. This would be the correct plant response but with the wrong PORV opening. Plausible the candidate could confuse the channel selection. REFERENCES LO-LP-60301-10-007 Vogtle LO Active Exam Bank LO-PP-16303, Pressurizer Pressure Control slides 29, 36, 37, 151, and 152 in particular. VEGP learning objectives: LO-LP-60301-10, Given that the channel selector switch is in the NORMAL position (455/456), describe how and why the plant will respond to the following pressure instrument failures. Consider each separately and include the effects of the Pressurizer Pressure Control System response, alarms, RPS, and ESFAS actuations. a. b. c. d. 455 fails high 455 fails low 456 fails high 456 fails low

N otes Monday, August 13, 2007 10:00:34 AM

34

Response form

QUESTIONS REPORT for HL-14 NRC RO DRAFT EXAM 14. 011EK2.02 001/1/1/LG LOCA - PUMPS/C/A - 2.6/BANK/R/NRC RO/TNT / RLM Given the following conditions: A large break LOCA occurred. Operators have just completed 19013-C, "Transfer to Cold Leg Recirculation". A loss of offsite power occurs, the diesel generators start and load the class 1 E 4160 buses. Which ONE of the following describes the actions required for this condition in accordance with 19010-C, Loss of Reactor or Secondary Coolant? A. Ensure all ECCS pumps are started by the blackout sequencer. B. Manually start the RHR pumps, then manually start the SIPs as needed. C. Place in Pull to Lock the SIPs and CCPs until the RHR pumps are started by the blackout sequencer. D. Ensure both RHR pumps are started by the blackout sequencer, then manually s t a r t CCPs and SIPs as needed.
Feedback

K/A 011 Large Break LOCA EK2.02 Knowledge of the interellations between the following and a large break LOCA. Pumps. K/A MATCH ANALYSIS Question gives a plausible scenario during a DBA LOCA where a LOSP occurs after aligning to Cold Leg Recirculation. Candidate must choose the correct response to address the ECCS pumps after the DGs start and reload the buses. ANSWER / DISTRACTOR ANALYSIS A. Incorrect. All ECCS pumps are not started by the sequencer after SI is reset. SI is reset early in 19013-C, Tranfer to Cold Leg Recirculation. Plausible candidate may not think SI reset since not stated in the stem. Blackout sequencer does not start RHR pumps and SI pumps. B. Correct. RHR pumps and SI pumps would have to be manually started, they do not

get a start signal from the blackout sequencer.


Monday, August 13, 2007 10:00:34 AM

35

QUESTIONS REPORT for HL-14 NRC RO DRAFT EXAM C. Incorrect. RHR pumps would not be started by blackout sequencer after SI reset. Plausible candidate may not recall SI has been reset after completion of 19013C. Placing CCPs / SIPs in PTL since they would not have suction is plausible. D. Incorrect. RHR pumps not started by blackout sequencer. Plausible candidate may not recall SI reset and feel pumps would start. CCPs would not require manual start on blackout sequencer but SIPs would. REFERENCES Vogtle 2002 NRC RO exam question # 19. 19013-C, "Transfer to Cold Leg Recirculation" step # 4 19010-C, "Loss of Primary or Secondary Coolant" step # 13 VEGP learning objectives: LO-LP-37111-08, Using EOP- 19010-C as a guide, briefly describe how each step is accomplished. LO-LP-37111-09, Given a NOTE or CAUTION statement from the EOP, state the bases for that NOTE or CAUTION statement. LO-LP-37113-02, Using EOP-19013-C as a guide, briefly describe how each step is accomplished. LO-LP-37113-05, Given a NOTE or CAUTION statement from the EOP, state the bases for that NOTE or CAUTION statement.
N o t e s R esponse form Answers MCS Time: 1 Points: 1.00 Version: 0 1 2 3 4 5 6 7 8 9 Answer: BABCCDCCDC Scramble Range: A- D

Monday, August 13, 2007 10:00:34 AM

36

QUESTIONS REPORT for HL-14 NRC RO DRAFT EXAM 15. 011K2.01 001/2/2/PRZR LVL- CHRG PUMPS/C/A - 3.6/NEW/R/NRC RO/TNT / RLM Given the following conditions: Plant at 100% power, all systems in normal alignment. LOSP occurs when RAT 1A and RAT 1B trip open during a severe thunderstorm. Several minutes later a reactor trip occurs. DG1B trips on low lube oil pressure just after the reactor trip. Which ONE of the following is CORRECT regarding the charging pumps status? A. NCP - running, CCP "A", running, CCP "B" stopped. B. NCP - stopped, CCP "A" stopped, CCP "B" running. C. NCP - stopped, CCP "A" running, CCP "B" stopped. D. NCP - running, "CCP "A", stopped, CCP "B" running.

Monday, August 13, 2007 10:00:34 AM

37

QUESTIONS REPORT for HL-14 NRC RO DRAFT EXAM


F e edback

K/A 011 Pressurizer Level Control System: K2.01 Knowledge of bus power supplies to the following: Charging Pumps K/A MATCH ANALYSIS Question gives a plausible scenario where an LOSP, reactor trip, and trip of a running DG occur. The candidate has to determine the status of all the charging pumps. ANSWER / DISTRACTOR ANALYSIS A. Incorrect. Plausible the candidate may think the NCP running and CCP "A". The NCP would be de-energized on bus transfer to a dead RAT post reactor trip. B. Incorrect. Plausible the candidate may recognize the NCP is de-energized and not recognize CCP "B" is de-energized. C. Correct. The NCP is de-energized on fast bus transfer to a dead bus, CCP "A" should have started on LOSP sequence. CCP "B" should be stopped due to DG1B tripping de-energizing it's power supply. D. Incorrect. Plausible the candidate may think the NCP running,however, the NCP is de-energized on fast bus transfer to a dead bus. Also plausible the candidate may invert the power supplies to the CCPs. REFERENCES LO-PP-09200, "CVCS Charging" slides # 22 and # 23 Simplified Electrical Drawing from LO-PP-01101, "Electrical Distribution" slide # 20 VEGP learning objectives: L O P P -09200-02, State the power supply for the charging pumps.
Notes Response form Answers

MCS

Time: 1

Points:

1.00

Version: 0 1 2 3 4 5 6 7 8 9 Answer: CAC CBABB AA

Scramble Range: A- D

Monday, August 13, 2007 10:00:34 AM

38

QUESTIONS REPORT for HL-14 NRC RO DRAFT EXAM 16. 012G2.4.31 001/2/1/RP S-ALARMS/ANNUNC/C/A - 3.3/NEW/R/NRC RO/TNT / RLM Given the following: The following annunciator windows illuminate. ALB06 window A06 for CNMT HI-1 PRESS ALERT ADVERSE CNMT - ALB06 window B06 for CNMT HI-2 PRESS ALERT ALB06 window C06 for CNMT HI-3 PRESS ALERT The HI-1, HI-2, and HI-3 MLBs for PI-0934 are all illuminated. Containment Pressure Recorder PR-0934 reads 40 psig. QMCB meter for PI-0934 reads 40 psig. The following annunciator windows are extinguished. ALB06 window DO6 for CNMT SPRAY ACTUATION - ALB09 window D06 for HI CNMT PRESS SI RX TRIP ADVERSE CNMT (first out) The HI-1, HI-2, and HI-3 MLBs for PI-0935, PI-0936, and PI-0937 are extinguished. QMCB meters for PI-0935, PI-0936, and PI-0937 all read 2.5 psig. Which ONE of the following operator actions would be CORRECT? A. Actuate Containment Spray since to containment pressure is > 21.5 psig. B. Perform a "crew briefing" to inform the crew of the failed pressure channel. C. Perform a "crew briefing" to inform the crew to use "Adverse CNMT" values. D. Manually start / align Containment Spray using the pump and valve handswitches.

Monday, August 13, 2007 10:00:34 AM

39

QUESTIONS REPORT for HL-14 NRC RO DRAFT EXAM


F e edback

K/A 012 Reactor Protection System (RPS): G2.4.31 Knowledge of annunciators, alarms, and indictations, and use of the response instructions. K/A MATCH ANALYSIS Question gives a plausible scenario during a LOCA where a Containment pressure channel fails high resulting in annunciators, MLBs, indicators showing PI-0934 reading 40 psig. Pressure channel 0934 is an input into the SI-Rx Trip circuitry. ANSWER / DISTRACTOR ANALYSIS A. Incorrect. Plausible since several annunciators, MLBs, meters, and the recorder indicate > 21.5 psig and is RNO for Initial Operator Actions of E-0. B. Correct. Per indications and annunciator response procedures, a single failed pressure channel has occurred. C. Incorrect. Plausible the candidate may see the Adverse Containment annunciator and inform the crew to use adverse values. D. Incorrect. Plausible since several annunciators, MLBs, meters, and the recorder indicate > 21.5 psig and is RNO for Initial Operator Actions of E-0. REFERENCES Annunciator response for ALB06 windows A06, B06, C06, D06. Annunciator response for ALB09 window D06. LO-PP-15101, "Containment Spray" slide # 10 VEGP learning objectives: LO-PP-15101-02, Describe what will actuate the Containment S p r ay System, including coincidences and setpoint.
N otes Response form Answers

MCS

Time: 1

Points:

1.00

Version: 0 1 2 3 4 5 6 7 8 9 Answer: B AB ACDDC CB

Scramble Range: A- D

Monday, August 13, 2007 10:00:34 AM

40

QUESTIONS REPORT for HL-14 NRC RO DRAFT EXAM 17. 012K5.02 001/2/1/RPS - POWER DENSITY/MEM - 3.1/NEW/R/NRC RO/TNT / RLM Which ONE of the following correctly describes the Over Power Delta T reactor trip? A. The setpoint will adjust on change in Tave, Pressurizer Pressure, and Delta I. Protects against DNB. B. The setpoint will adjust on change in Tave, rate of change of Tave, and Delta I. Protects against power density (KW / ft). C. The setpoint will adjust on change in Tave, Pressurizer Pressure, the Delta I input is zeroed out. Protects against DNB. D. The setpoint will adjust on change in Tave, rate of change of Tave, the Delta I input is zeroed out. Protects against power density (KW / ft).

Monday, August 13, 2007 10:00:35 AM

41

QUESTIONS REPORT for HL-14 NRC RO DRAFT EXAM


Feedback

K/A 012 Reactor Protection System (RPS): K5.02 Knowledge of the operational implications of the following concepts as they apply to the RPS. Power Density K/A MATCH ANALYSIS Question asks the candidate which parameters cause the Over Power Delta T reactor trip setpoint to adjust and what protection is afforded the reactor ? ANSWER / DISTRACTOR ANALYSIS A. Incorrect. This is the inputs and description for the OT delta T reactor trip. Plausible the candidate could confuse the bases and inputs for the trip. B. Incorrect. Delta I input is zeroed out and will not cause the setpoint to adjust. The bases for the trip is correct. Plausible candidate may recognize Delta I is an input but not realize it is zeroed out in the setpoint calculation. C. Incorrect. This is the inputs and description for the OT delta T reactor trip with Delta I incorrectly zero'd out. Plausible the candidate may recognize the OT delta T inputs and confuse Delta I being zeroed out or confuse the bases. D. Correct. Inputs and bases for OP delta P. REFERENCES Vogtle Tech Specs and Bases for 3.3.1 Reactor Trip Instrumentation V-LO-PP-28103, "Reactor Trip and ESFAS Signals" Power Point slide # 101 VEGP learning objectives:

LO-LP-39207-04, Describe the Bases for any given Tech Spec in section 3.3
Notes Response form Answers

MCS

Time: 1

Points:

1.00

Version: 0 1 2 3 4 5 6 7 8 9 Answer: DCDDAB CB BB

Scramble Range: A- D

Monday, August 13, 2007 10:00:35 AM

42

QUESTIONS REPORT for HL-14 NRC RO DRAFT EXAM 18. 013K1.13 001/2/1/ESFAS - HVAC/C/A - 2.8/BANK/R/NRC RO/TNT / RLM Given the following: Both Units are at 3565 MWt with all systems in their normal alignment. Unit 1 annunciator windows for INTMD RADIATION ALARM and HIGH RADIATION are illuminated. The SRDC shows Control Room Air Intake Rad Monitor 1RE-12116 yellow and red indication lights are illuminated. Unit 2 has NO alarms associated with radiation monitors. Which ONE of the following describes the CORRECT Control Room HVAC response? A. Only Unit 1 Train B CREF unit starts and both ESF chillers start. B. Unit 1 Train A and B CREF units start and both ESF chillers start. C. Both Unit 1 CREF units start and only the Train B ESF Chiller Starts. D. Both Units Train B CREF units start, both Trains of ESF chillers start on both units.

Monday, August 13, 2007 10:00:35 AM

43

QUESTIONS REPORT

for HL-14 NRC RO DRAFT EXAM


Feedback

K/A 013 Engineered Safety Features Actuation System (ESFAS): K1.13 Knowledge of the physical connections and / or cause effect relationships between the ESFAS and the following systems. HVAC K/A MATCH ANALYSIS Question gives a plausible scenario where a HIGH radiation alarm is received on the Control Room Air Intake Radiation Monitor. The candidate has to determine the correct Control Room HVAC response. ANSWER / DISTRACTOR ANALYSIS A. Correct. Unit 1 Train B CREF starts and both ESF chillers start. B. Incorrect. Only Train B CREF starts, plausible the candidate would think that both CREFs units start on high rad such as FHB HVAC does by starting all units. C. Incorrect. Plausible the candidate may think both CREF units start and know only the Train B ESF Chiller starts. D. Incorrect. Plausible since control rooms are crosstied that the candidate would know the B CREF and both chillers start and think they should start on both units. REFERENCES LO-PP-23301-05-003 Vogtle LO Active Exam Bank LO-PP-23301, Control Room HVAC slide # 17, 21, and 22 LO-PP-28103, Reactor Trip and ESFAS signals slide # 147 VEGP learning objectives:

LO-PP-23301-05, List the actuating signals for CRI including indications and automatic actions.
Notes

Monday, August 13, 2007 10:00:35 AM

44

Response form

QUESTIONS REPORT for HL-14 NRC RO DRAFT EXAM


A n swers MCS Time: 1 Points: 1.00 Version: 0 1 2 3 4 5 6 7 8 9 Answer: ABADCACCDB Scramble Range: A- D

Monday, August 13, 2007 10:00:35 AM

45

QUESTIONS REPORT for HL-14 NRC RO DRAFT EXAM 19. 014A1.03 001/2/2/DRPI-LOW LIMITS/C/A - 3.6/BANK/R/NRC RO/TNT / RLM The RO withdraws control bank C rods to establish conditions for a dilution to criticality. Prior to the rod withdrawal the following conditions existed: DRPI: 42 steps Group 1 Step Counter: 42 steps Group 2 Step Counter: 41 steps After the rod withdrawal the following conditions exist: DRPI: 42 steps Group 1 Step Counter: 56 steps Group 2 Step Counter: 55 steps Based on these indications, which ONE of the following CORRECTLY describes the status of the "ROD DEV" and "ROD BANK LO-LO LIMIT" annunciators? REFERENCE PROVIDED A. The "ROD DEV" annunciator alarms and the "ROD BANK LO-LO LIMIT" annunciator clears. B. The "ROD DEV" annunciator alarms and the "ROD BANK LO-LO LIMIT" annunciator remain illuminated. C. The "ROD DEV" annunciator does not alarm and the "ROD BANK LO-LO LIMIT" annunciator remains illuminated. D. The "ROD DEV" annunciator does not alarm and the "ROD BANK LO-LO LIMIT" annunciator clears.

Monday, August 13, 2007 10:00:35 AM

46

QUESTIONS REPORT for HL-14 NRC RO DRAFT EXAM


Feedback

K/A 014 Rod Position Indication System (RPI): A1.03 Ability to predict and / or monitor changes in parameters (to prevent exceeding design limits) associated with operating the RPIS controls, including: PDIL, PPDIL K/A MATCH ANALYSIS Question gives a plausible scenario during rod withdrawal where the RO notes a discrepancy between demand (step counter) and actual (DRPI) indications. Candidate also has to determine if the rods are above or below the Rod Bank Lo-Lo Limit and the alarm condition. ANSWER / DISTRACTOR ANALYSIS A. Correct. Rod Dev should be lit at > 11 steps, Rod Bank Lo-Lo should clear at 46 steps on CBC. B. Incorrect. Rod Dev alarm status is correct. Plausible the candidate could confuse the Rod Bank Lo Limit annunciator setpoint with the Rod Bank Lo-Lo Limit setpoint. C. Incorrect. Plausible the candidate may not correlate the DRPI status with the Rod Dev annunciator or confuse the control bank for which Lo-Lo limit clears. D. Incorrect. Plausible the candidate may not correlate the DRPI status with the Rod Dev annunciator. Plausible the candidate could confuse the Rod Bank Lo Limit annunciator setpoint with the Rod Bank Lo-Lo Limit setpoint. REFERENCES Vogtle May 2005 NRC RO Exam question # 18 same KA #. NOTE: This is one of 3 questions of 4 allowed re-used from previous 2 RO exams. ARP 17010 window D06 rod ROD DEV COLR Figure 3 for Rod Insertion Limits VEGP learning objectives: LO-PP-27101-21, State the alarms associated with the rod insertion limits; include set oints and source of the set oints.
Notes Monday, August 13, 2007 10:00:35 AM

47

QUESTIONS REPORT for HL-14 NRC RO DRAF T EXAM


Response form Answers MCS Time: 1 Points: 1.00 Version: 0 1 2 3 4 5 6 7 8 9 Answer: AB B AB AAB AC Scramble Range: A- D

Monday, August 13, 2007 10:00:35 AM

48

QUESTIONS REPORT for HL-14 NRC RO DRAFT EXAM 20. 015/017AA1.11 001/1/1/RCP MALF - INDICATOR/MEM - 2.5/NEW/R/NRC RO/TNT / RLM Given the following: RCS LOCA in progress. RCP Immediate Trip Criteria was met The RCPs were all stopped. Later, procedure directs restart of an RCP. Which ONE of the following is CORRECT regarding the method to stop / start the RCPs using the QMCB handswitches? A. Open the non-1E handswitches first when stopping the pumps. Close the non-1E handswitches first when starting the pumps. B. Open the non-1E handswitches first when stopping the pumps. Close the lE handswitches first when starting the pumps. C. Open the 1 E handswitches first when stopping the pumps. Close the non-1E handswitches first when starting the pumps. D. Open the 1 E handswitches first when stopping the pumps. Close the non-1E handswitches first when starting the pumps.

Monday, August 13, 2007 10:00:35 AM

49

QUESTIONS REPORT for HL-14 NRC RO DRAFT EXAM


Feedback

K/A 015 / 017 RCP Malfunctions AA1.11 Ability to operate and / or monitor the following as they apply to the Reactor Coolant Pump Malfunctions (Loss of RC Flow): RCP on / off and run indicators K/A MATCH ANALYSIS Question gives a plausible scenario during an RCS LOCA where the RCPs would be stopped. The procedures in many scenarios have the RCPs started later if possible. The candidate has to pick the correct starting sequence to open / close the RCP handswitches. ANSWER / DISTRACTOR ANALYSIS A. Incorrect. Plausible the candidate may know to open non-1E first for stopping the RCP and think the same is true for starting. B. Correct. The RCP non-1E handswitches should be manipulated first when stopping an RCP as protection for the electrical penetrations into containment. The 1 E breakers are closed first when starting an RCP. This would prevent a possible electrical arc on a fault from damaging the penetration. C. Incorrect. Plausible the candidiate may invert the proper order for manipulating the RCP breakers and think it would be the opposite for the other evolution. D. Incorrect. Plausible candidate may invert the proper order for breaker manipulation while knowing he is supposed to operate the same breaker first for either evolution. REFERENCES SOP-13003-1/2, "Reactor Coolant Pump Operation" steps 4.1.2.15 and 4.2.1.4. VEGP learning objectives: LO-PP-16401-09 Describe the following for the RCP supply breakers. a. Breaker arrangement

c. Protection features
Notes Monday, August 13, 2007 10:00:35 AM

50

QUESTIONS REPORT for HL-14 NRC RO DRAF T EXAM


Response form Answers MCS Time: 1 Points: 1.00 Version: 0 1 2 3 4 5 6 7 8 9 Answer: BDCDBBACAD Scramble Range: A- D

Monday, August 13, 2007 10:00:35 AM

51

QUESTIONS REPORT for HL-14 NRC RO DRAFT EXAM 21. 016A4.02 001/2/2/NNIS - RECORDERS/C/A - 2.7/NEW/R/NRC RO/TNT / RLM Given the following conditions: Unit 1 is at 100% power. All control systems are in normal alignment. All SG level, steam flow and feed flow instruments controlling channel selector switches EXCEPT ONE are positioned pointing to the Channel I direction. Controlling channel for SG # 4 steam flow is pointing to the Channel II direction. Channel II SG pressure instrument for SG # 4 fails low. As the control systems respond to the above conditions, which ONE of the following would be CORRECT regarding the feedwater flow and steam flow recorder indications for SG # 4? A. Recorder feed and steam flow indications would both lower. B. Recorder feed flow indication would lower, steam flow indication would rise. C. Recorder feed flow indication would rise, steam flow indication would lower. D. Recorder feed and steam flow indications would both rise.

Monday, August 13, 2007 10:00:35 AM

52

QUESTIONS REPORT for HL-14 NRC RO DRAFT EXAM


Feedback

K/A 016 Non-Nuclear Instrumentation System (NNIS): A4.02 Ability to manually operate or monitor in the control room: Recorders K/A MATCH ANALYSIS Question gives a plausible scenario where all SG levels, steam and feed flow controlling channels are selected to the normal position (left - channel I). SG # 4 controlling steam flow channel is selected to channel II. The candidate has to determine the effect of a SG # 4 channel II pressure channel failure on SG # 4 feedwater flow and recorder steam flow indication. ANSWER / DISTRACTOR ANALYSIS A. Correct. Channel II steam pressure failure would affect the feedwater flow and the recorder steam flow indication, both would lower. B. Incorrect. Plausible the candidate may confuse the direction of the steam flow indication on the failure. C. Incorrect. Feedwater flow would be lower and plausible the candidate could invert the direction, steam flow direction is correct. D. Incorrect. Plausible candidate could confuse the direction and think both feed and steam flow indications could rise. REFERENCES A0P018001-C, section F for Failure of SG Pressure Instrumentation V-LO-PP-18101, "Condensate and Feedwater" slides 156 - 160. VEGP learning objectives: LO-PP-18101-22, Discuss the SGWLC SYstem to include: a. The 3 inputs into the system LO-PP-18101-23, Discuss how the SGWLC system will respond to the following controlling channel failures: d. Steam flow channel fails low

Monday, August 13, 2007 10:00:35 AM

53

QUESTIONS REPORT

for HL-14 NRC RO DRAFT EXAM


Notes Response form Answers MCS Time: 1 Points: 1.00 Version: 0 1 2 3 4 5 6 7 8 9 Answer: ACCBDCCDCC Scramble Range: A- D

Monday, August 13, 2007 10:00:35 AM

54

QUESTIONS REPORT for HL-14 NRC RO DRAFT EXAM 22. 017K3.01 001/2/2/CETS-NAT CIRC INDICA/C/A - 3.5/BANK/R/NRC RO/TNT / RLM Which ONE of the following is the CORRECT indication representation if 5 core exit TC's are failed due to an OPEN circuit? A. The Core Exit Temperature indications will be HIGHER than actual. RCS Subcooling will indicate MORE subcooling than actual. B. The Core Exit Temperature indications will be HIGHER than actual. RCS Subcooling will indicate LESS subcooling than actual. C. Core Exit Temperature indications will indicate LOWER than actual. RCS Subcooling will indicate MORE subcooling than actual. D. Core Exit Temperature indications will indicate the SAME as actual. RCS Subcooling will indicate the SAME as actual.

Monday, August 13, 2007 10:00:36 AM

55

QUESTIONS REPORT for HL-14 NRC RO DRAFT EXAM


F e edback

K/A 017 In-core Temperature Monitor (ITM): AK3.01 Knowledge of the effect that a loss or malfunction of the ITM system will have on the following: Natural Circulation Indications K/A MATCH ANALYSIS Question gives a plausible scenario following a reactor trip where the operator is attempting to verify natural circulation. Five (5) CETs are failed due to an open circuit which would cause them to fail low. Candidate has to determine how failure would affect indications used for natural circulation verification. It would not since CETs only take the highest five (5) indications. ANSWER / DISTRACTOR ANALYSIS A. Incorrect. Plausibe since the CETs are failed, but the failed CET would indicate low and not high, therefore, top 5 temps used to determine subcooling are unaffected. B. Incorrect. Plausible since the CETs are failed, but the failed CET would indicate low and not high, therefore, top 5 temps used to determine subcooling are unaffected. C. Incorrect. Plausible since the CETs are failed low, but the subcooling calculation uses the 5 highest CET temperatures, not the lowest. D. Correct. The failed CETs are not used to process the subcooling calculation, since they are not used, the top 5 CETs would indicate the proper subcooling margin. REFERENCES Harris February 2006 NRC RO Exam, question # 19. Harris March 2004 NRC RO Exam, question # 70. V-LO-PP-17301, Incore TCs and Moveable Incore Detectors slide # 6 VEGP learning objectives: LO-PP-17301-01, Discuss the Operation of the Core Exit Thermocouples to include: a . H o w
Notes

t h e

o p e r a t o r

c a n

t e l l

w h e n

t h e y

f a i l

This question was rated C/A on the Harris exam with a difficulty rating of 3.0

Monday, August 13, 2007 10:00:36 AM

56

QUESTIONS REPORT for HL-14 NRC RO DRAF T EXAM


Response form Answers MCS Time: 1 Points: 1.00 Version: 0 1 2 3 4 5 6 7 8 9 Answer: DB C DD CD C AD Scramble Range: A- D

Monday, August 13, 2007 10:00:36 AM

57

QUESTIONS REPORT for HL-14 NRC RO DRAFT EXAM 23. 022A2.01 001/2/1/CTMT COOL - FAN CURR/MEM - 2.5/BANKJR/NRC RO/TNT / RLM Given the following conditions: Main steamline break has occurred inside Containment. The crew is performing Initial Operator Actions of E-0, Reactor Trip or SI. The RO reports the Containment Coolers are running in FAST speed. Which ONE of the following describes the CORRECT required action (if any) and the reason for the decision? A. Allow the coolers to continue running in FAST speed to assist in the prevention of explosive hydrogen pockets. B. Shift the coolers to SLOW speed to reduce the electrical load on ABO4 and BB06 1 E electrical buses in the event of an LOSP. C. Shift the coolers to SLOW speed to prevent fan motor overcurrent due to the more dense containment atmospheric conditions. D. Allow the coolers to continue running in FAST speed to offset the loss of NSCW flow to the Reactor Cavity Coolers and Auxiliary Coolers.

Monday, August 13, 2007 10:00:36 AM

58

QUESTIONS REPORT for HL-14 NRC RO DRAFT EXAM


F e edback

K/A 022 Containment Cooling System: A2.01 Ability to (a) predict the impacts of the following malfunctions or operations on the CCS; and (b) based on those predictions, use procedures to correct, control, or mitigate the consequences of those malfunctions or operations. Fan motor over-current K/A MATCH ANALYSIS Question gives a plausible scenario where the containment coolers are running in FAST speed during a steam line break in containment. The candidate must determine the proper action to take and the reason why. ANSWER / DISTRACTOR ANALYSIS A. Incorrect. Plausible since one of the design features of the containment cooling system is to prevent explosive hydrogen pockets from forming. BUT, coolers should be running in SLOW speed to prevent from tripping on overcurrent due to the more dense containment atmospheric conditions. B. Incorrect. Plausible the candidate may think a slower speed would reduce load to the lE electrical switch gear. The reason for slow speed stated in A above. C. Correct. SLOW speed is to protect the containment coolers from the more dense atmosphere during DBA events inside containment. D. Incorrect. Plausible since NSCW to Cavity and Aux. coolers does isolate on an SI and coolers may need to operate in FAST to offset the cooling water loss. REFERENCES V-LO-PP-29101, Containment HVAC Systems slide # 22 V-LO-PP-29101-14-01 Vogtle LO Active Exam Bank VEGP learning objectives: LO-PP-29101-04, State how the Containment atmosphere changes following a LOCA and why. LO-PP-29101-13, State why two speeds are provided for the Containment Coolers and when each speed is used.
Notes

Monday, August 13, 2007 10:00:36 AM

59

QUESTIONS REPORT for HL-14 NRC RO DRAFT EXAM


R e s p o nse form Answers MCS Time: 1 Points: 1.00 Version: 0 1 2 3 4 5 6 7 8 9 Answer: CAACBACDCD Scramble Range: A- D

Monday, August 13, 2007 10:00:36 AM

60

QUESTIONS REPORT for HL-14 NRC RO DRAFT EXAM 24. 022AK1.03 001/1/1/LOSS RX MU - PRZR LV/C/A - 3.0/MODIFIED/R/NRC RO/TNT / RLM Given the following conditions: - Reactor power is at 100%. ALB07 window A05 for REGEN HX LETDN HI TEMP alarm lit. ALB07 window B06 for CHARGING LINE HI / LO FLOW alarm lit. ALB07 window D03 LETDN HX OUTLET HI TEMP alarm lit. ALB08 window F06 for RCP SEAL WATER INJ LO FLOW alarm lit. All other plant conditions are normal. Which ONE of the following CORRECTLY explains the given conditions? A. PRZR level would be rising, HV-0182 seal flow control valve has failed shut. B. PRZR level would be lowering, HV-0182 seal flow control valve has failed open. C. PRZR level would be rising, FV-0121 charging flow control valve has failed open. D. PRZR level would be lowering, FV-0121 charging flow control valve has failed shut.

Monday, August 13, 2007 10:00:36 AM

61

QUESTIONS REPORT for HL-14 NRC RO DRAFT EXAM


F e edback

K/A 022 Loss of Rx. Coolant Makeup AK1.03 Knowledge of the operational implications of the following concepts as they apply to the Loss of Reactor Coolant Pump Makeup. Relationship between charging flow and PRZR level. K/A MATCH ANALYSIS Question gives several charging, letdown, and RCP seal alarms that are consistent with a loss of charging flow. Candidate must pick out the correct pressurizer level response and malfunction which caused the indications. ANSWER / DISTRACTOR ANALYSIS A. Incorrect. PRZR level response is correct for HV-0182 failed shut, however, the annunciators listed are not consistent with this condition. Seal water flow lo annunciator would be illuminated, the others would not. Plausible candidate may invert valve failure direction on charging flow / PRZR level effects B Incorrect. PRZR level response is correct for HV-0182 failed open, however, the annunciators listed are not consistent with this condition. Seal water flow lo annunciator would not be illuminated, the others would. Plausible candidate may invert valve failure direction on charging flow / PRZR level effects. C. Incorrect. PRZR level response is correct for FV-0121 failed open, however, the annunciators listed are not consistent with this condition. None of the annunciators listed would be illuminated except possibly Charging Line Hi / Lo flow. D. Correct. PRZR level response and annunciators would be correct for the failure. REFERENCES LO-PP-09100-005-003 Vogtle LO Active Exam Bank question. V-LO-PP-09200, CVCS Charging System VEGP learning objectives: LO-PP-09200-01, State the purpose and describe the control signals, setpoints, and any interlocks for the following: c. charging flow control valve, FV-121 d. seal injection flow control valve, FV-182
Monday, August 13, 2007 10:00:36 AM

62

QUESTIONS REPORT for HL-14 NRC RO DRAFT EXAM


N o t e s

Response form Answers MCS Time: 1 Points: 1.00 Version: 0 1 2 3 4 5 6 7 8 9 Answer: DAADB AB BDC Scramble Range: A- D

Monday, August 13, 2007 10:00:36 AM

63

QUESTIONS REPORT for HL-14 NRC RO DRAFT EXAM 25. 022G2.4.4 001/2/1/CTMT COOL - FAN CURR/C/A - 4.0/MODIFIED/R/NRC RO/TNT / RLM The unit is at 100% when the following occurs: ALB01, window F05 for CNMT HI TEMP illuminates. The BOP operator starts additional Containment Coolers per the ARP guidance. Later in the shift: ALB01, window F06 for CNMT HI MSTR illuminates. ALB62, window E05 for CNMT CLR COND LEAK illuminates. Containment relative humidity and temperatures are slowly rising. No other alarms are present. Containment pressure has risen from 0.1 psig to 0.6 psig and is rising. PRZR level and pressure and RCS Tavg have remained stable. Which ONE of the following is CORRECT regarding these indications? A. NSCW leak, perform a CNMT Cooler Condensate Collection Calculation. B. RCS leak, perform a leak rate and enter AOP-18004 for RCS Leakage C. Feedwater leak, enter AOP-18008 for Secondary Leakage D. Steam leak, enter AOP-18008 for Secondary Leakage

Monday, August 13, 2007 10:00:36 AM

64

QUESTIONS REPORT for HL-14 NRC RO DRAFT EXAM


F e edback

K/A 022 Containment Cooling System: G2.4.4 Ability to recognize abnormal indications for system operating parameters which are entry-level conditions for emergency and abnormal operating procedures. K/A MATCH ANALYSIS Question gives a plausible scenario with containment temperature, moisture, and humidity rising with associated alarms. Candidate has to choose the event and corrective action in response. ANSWER / DISTRACTOR ANALYSIS A. Incorrect. NSCW leak would not cause pressure, temperature, and humidity to rise. Plausible candidate could think NSCW leak causing humidity and pressure to rise. B. Incorrect. RCS leak not present due to no radionuclides. Plausible candidate may think symptoms of RCS leak and not recognize radiation not present. C. Correct. Feedwater line leak would give these indications. Different from steam leak due to RCS cooldown would be present for a steam leak. D. Incorrect. Plausible candidate could think a small steam leak present but Tave stable would rule this out. REFERENCES ARP-17001 windows E06 and F06 for Containment Hi Temp and Containment Hi Moist ARP-1706262, window E05 for CNMT CLR COND LEAK illuminates. LO-OR-60308-03-001 from Vogtle LO Active Exam Bank Vogtle May 2006 NRC RO Exam question # 60 Vogtle May 2005 NRC RO Exam question # 37 VEGP learning objectives: LO-LP-60308-04, Discuss the parameters that distinguish primary coolant leakage from secondary coolant leakage.
Notes

Monday, August 13, 2007 10:00:36 AM

65

QUESTIONS REPORT for HL-14 NRC RO DRAFT EXAM


R e s p o nse form Answers MCS Time: 1

Points:

1.00

Version: 0 1 2 3 4 5 6 7 8 9 Answer: CCBAAB C AA C

Scramble Range: A- D

Monday, August 13, 2007 10:00:36 AM

66

QUESTIONS REPORT for HL-14 NRC RO DRAFT EXAM 26.


026A1.04 001/2/1/C. SPRAY - HUMIDITY/MEM - 3.1/NEW/R/NRC RO/TNT / RLM

To prevent challenging containment design pressure and rendering equipment inoperable due to high moisture (humidity), the Containment Spray System ________ A. suction valves have to be manually opened following a manual spray actuation. B. discharge valves have to be manually opened following a manual spray actuation. C. relays are "de-energize to actuate" preventing inadvertent actuation during testing. D. actuation requires operation of 2 of 2 handswitches at either control board location.

Monday, August 13, 2007 10:00:36 AM

67

QUESTIONS REPORT for HL-14 NRC RO DRAFT EXAM

Feedback

K/A 026 Containment Spray System (CSS): A1.04 Ability to predict and / or mitigate changes in parameters (to prevent exceeding design limits) associated with operating the CSS controls including: Containment humidity K/A MATCH ANALYSIS Question asks plausible methods to prevent an inadvertent spray actuation from challenging containment design pressure and rendering equipment inoperable due to high moisture / humidity. Candidate has to choose a correct design feature that prevents inadvertent spray actuation. ANSWER / DISTRACTOR ANALYSIS A. Incorrect. Plausible candidate may think suction valves required to be opened following a manual spray actuation by confusing with RHR suctions which auto open on RWST lo lo level. Suction valves normally open. B. Incorrect. Discharge valves auto open following a spray actuation. Plausible the candidate may confuse by to manually align for recirculation mode instead of the semi-auto swapover like RHR suctions from RWST. C. Incorrect. Relays are "energize to actuate" preventing loss of power from initiating a spray actuation. Plausible candidate could confuse with other system bistables which are "de-energize to actuate". D. Correct. 2 of 2 handswitches at either QMCB location is required for spray actuation. REFERENCES V-LO-PP-15101, Containment Spray System slides 10 and 17 Tech Specs 3.3.2 ESFAS for C. Spray Instrumentation and Bases VEGP learning objectives: LO-PP-15101-02, Describe what will actuate the Containment Spray System, including coincidence and set point. LO-PP-15101-04, List all components that receive a Containment Spray actuation signal and their change in status.
Notes

Monday, August 13, 2007 10:00:36 AM

68

QUESTIONS REPORT for HL-14 NRC RO DRAF T EXAM


Response form Answers MCS Time: 1 Points: 1.00 Version: 0 1 2 3 4 5 6 7 8 9 Answer: DCCDBADCCA Scramble Range: A- D

Monday, August 13, 2007 10:00:36 AM

69

QUESTIONS REPORT for HL-14 NRC RO DRAFT EXAM 27. 026AA1.05 001/1/1/CCW SURGE TANK/C/A - 3.1/BANIC/R/NRC RO/TNT / RLM The following conditions exist for Unit 1 Train "A" CCW system: - CCW pumps # 1 and # 3 are running. - CCW pump # 5 handswitch is in AUTO. A single CCW level transmitter on the Train "A" surge tank fails LOW. Which ONE of the following describes a CORRECT system response to the failure? A. One of the running CCW pumps trip, the standby COW pump starts. The reactor makeup water valve to the CCW surge tank auto opens. B. One of the running CCW pumps trip, the standby CCW pump starts. The demin water makeup valve to the CCW surge tank remains shut. C. Both the running CCW pumps trip, the standby CCW pump starts and trips. The demin water makeup valve to the CCW surge tank auto opens. D. Both the running CCW pumps trip, the standby CCW pump starts and trips. The reactor makeup water valve to the CCW surge tank remains shut.

Monday, August 13, 2007 10:00:37 AM

70

QUESTIONS REPORT for HL-14 NRC RO DRAFT EXAM


F e edback

K/A 026 Loss of Component Cooling Water (CCW): AA1.05 Ability to operate and / or monitor the following as they apply to the Loss of Component Cooling Water: The CCWS surge tank, including level control and level alarms, and radiation alarms. K/A MATCH ANALYSIS Question gives a plausible scenario where a single CCW Surge Tank level transmitter fails low. The candidate must pick the correct effect on the pumps and the auto makeup valve to the tank. ANSWER / DISTRACTOR ANALYSIS A. Incorrect. Pump response is correct, however, Reactor Makeup Water does not have an auto open function. Plausible candidate may know pump response and think auto makeup should occur from RMWST. B. Correct. Pump response / Demin Water auto makeup valve responses are correct. C. Incorrect. Only one pump would trip and Demin Water auto makeup response is incorrect too. Plausible candidate may think surge tank level low would trip all pumps and call for auto makeup from Demin Water Storage Tank. D. Incorrect. Only one pump would trip but part about Reactor Makeup water valve is correct. Plausible candidate may realize makeup valve stays shut but confuse the pump trip circuitry interlocks. REFERENCES LO-PP-10101-04-011 Vogtle LO Active Bank question. LO-PP-10101, Component Cooling Water (CCW) slides 16, 20 and 21 VEGP learning objectives: LO-PP-10101-04, From memory, describe the expected system response and operator corrective actions for each of the following: d .
Notes

Monday, August 13, 2007 10:00:37 AM

71

QUESTIONS REPORT for HL-14 NRC RO DRAFT EXAM


R e s p o nse form Answers MCS Time: 1

Points:

1.00

Version: 0 1 2 3 4 5 6 7 8 9 Answer: BDAC CAB CB A

Scramble Range: A- D

Monday, August 13, 2007 10:00:37 AM

72

QUESTIONS REPORT for HL-14 NRC RO DRAFT EXAM 28. 027AK3.02 001/1/1/PRZR LVL CONT - ALT/MEM - 2.9/NEW/R/NRC RO/TNT / RLM Given the following plant conditions: The plant is at 100% power. PRZR Pressure Channel control selected to the 455 / 456 position. PRZR Pressure transmitter PT-455 fails high. PRZR Master Controller output is reading 100%. Control selector switch has been placed to the 457 / 456 position. Which ONE of the following actions is CORRECT per 18001-C, "Primary Instrumentation Malfunction"? A. Leave Master Controller output at 100%, select recorder to channel 457. B. Take Master Controller output to 25% in manual, select recorder to channel 457. C. Leave Master Controller output at 100%, select recorder to channel 456. D. Take Master Controller output to 25% in manual, select recorder to channel 456.

Monday, August 13, 2007 10:00:37 AM

73

QUESTIONS REPORT for HL-14 NRC RO DRAFT EXAM


Feedback

K/ A 027 Pressurizer Pressure Control System (PZR PCS) Malfunction: AK3.02 Knowledge of the reasons for the following responses as they apply to the Pressurizer Pressure Control Malfunctions: Verification of alternate transmitter and / or plant computer prior to shifting flow chart transmitters. K/A MATCH ANALYSIS Question gives a plausible scenario with failure of controlling PRZR Pressure Channel PT-455. Candidiate must pick the correct choice to place the channel back in AUTO. ANSWER / DISTRACTOR ANALYSIS A. Incorrect. Master controller should be placed to 25% demand prior to placing the control system back in auto. Recorder should be selected to same channel as the controlling channel which would be 457. B. Correct. These are the correct actions per the AOP. C. Incorrect. Master controller should be placed to 25% demand prior to placing the control system back in auto. Recorder should be selected to the primary controlling channel, plausible the candidate may confuse the channels. D. Incorrect. This is the correct AOP action for the Master Controller. Recorder should be selected to the primary controlling channel, plausible the candidate may confuse the channels. REFERENCES AOP-18001, Primary Instrumentation Malfunction section C for Failure of Pressurizer Pressure Instrumentation steps C5 and RNO and C11. VEGP learning objectives: LO-PP-16303-02, Describe how the response of pressurizer pressure control to the following failures:

b. controlling primary channel fails high


Notes

Monday, August 13, 2007 10:00:37 AM

74

Response form

QUESTIONS REPORT for HL-14 NRC RO DRAFT EXAM


A n swers MCS Time: 1

Points:

1.00

Version: 0 1 2 3 4 5 6 7 8 9 Answer: BBBBCABACC

Scramble Range: A- D

Monday, August 13, 2007 10:00:37 AM

75

QUESTIONS REPORT for HL-14 NRC RO DRAFT EXAM 29. 028A1(2.02 001/1/2/PRZR LVL - SNSRS/C/A - 2.6/MODIFIED/R/NRC RO/TNT / RLM The plant is operating at 50% power with all control systems in automatic, except for Rod Control. A leak on the REFERENCE LEG of a pressurizer level transmitter occurs. The Letdown System remains in service for the duration of the event.

Which ONE of the following is a CORRECT description of the failure where letdown would remain in service indefinitely? (assuming no operator actions are taken) A. B. C. D. I 4 LI-459 has failed LOW with the control selector switch in the 461 / 460 position. LI-460 has failed LOW with the control selector switch in the 459 / 461 position. LI-459 has failed HIGH with the control selector switch in the 459 / 460 position. L

60 has failed HIGH with the control selector switch in the 461 / 460 position.
Feedback

K/A 028 Pressurizer Level Malfunction: AK2.02 Knowledge of the interrelations between the Pressurizer Level Control Malfunctions and the following: Sensors and detectors K/A MATCH ANALYSIS Question gives a plausible scenario with a reference leg break on a PRZR level control instrument. Candidate has to choose the correct description of the failure (high / low) and which one Letdown would remain in service for an indefinite time. ANSWER / DISTRACTOR ANALYSIS A. Incorrect. Reference leg failure would result in high failure versus low. Plausible that the candidate could invert the failure direction. Channel is also not selected which make even more plausible. B. Incorrect. Reference leg failure woud result in a high failure versus low. Plausible that candidate could invert the failure direction. Channel is also not selected which makes it even more plausible. C. Incorrect. High failure part is correct. However, with controlling channel failed high, charging flow would reduce until letdown isolates on PRZR low level < 17%.

Monday, August 13, 2007 10:00:37 AM

76

QUESTIONS REPORT for HL-14 NRC RO DRAFT EXAM D. Correct. High failure would result, with LT-460 failed high and not the controlling channel, a PRZR HI LEVEI alarm would be the only result. REFERENCES LO-PP-16302-07-03, Vogtle LO Active Exam Bank LO-PP-16302-02-11, Vogtle LO Active Exam Bank V-LO-PP-16302, Pressurizer Level Control Power Point slide # 47 Watts Bar September 2006 NRC RO exam question # 35 Prairie Island August 2005 NRC RO exam question # 21 (this was a NOT question) VEGP learning objectives: LO-PP-16302-02, Describe how the response of pressurizer level control to the following failures: b. controlling (primary and secondary) channel fails high LO-PP-16302-05, Describe the consequences of a leak on or isolation of the common reference lea.
Notes R e s p o nse form Answers MCS Time: 1 Points: 1.00 Version: 0 1 2 3 4 5 6 7 8 9 Answer: DAADB AC CB C Scramble Range: A- D

Monday, August 13, 2007 10:00:37 AM

77

QUESTIONS REPORT for HL-14 NRC RO DRAFT EXAM 30. 029A2.04 001/2/2/PURGE-HP NOTIFY/MEM - 2.5/NEW/R/NRC RO/TNT / RLM Given the following timeline: 0200 on 08-16-2007 HP / Chemistry samples the containment atmosphere for a mini-purge relief. 0300 on 08-16-2007 Shift Supervisor approves the release permit. Due to manpower shortage on day shift, the mini-purge relief is not performed. 0330 on 08-17-2007 Control Room operators are ready to initiate the mini-purge relief. Which ONE of the following is CORRECT regarding the mini-purge relief evolution? A. The relief can be started immediately. B. The relief can be started at any time. C. The relief cannot be started since more than 24 hours have passed from the containment air sample. D. The relief cannot be started since more than 24 hours have passed since the approval of the permit.

Monday, August 13, 2007 10:00:37 AM

78

QUESTIONS REPORT for HL-14 NRC RO DRAFT EXAM


Feedback

K/A

029 Containment Purge System (CPS): A2.04 Ability to (a) predict the impacts of the following malfunctions or operations on the Containment Purge System; and (b) based on those predictions, use procedures to correct, control, or mitigate the consequences of those malfunctions or operations: Health physics sampling of containment atmosphere. K/A MATCH ANALYSIS Question gives a plausible scenario timeline where a mini-purge relief (release) evolution is approved by HP / Chemistry but a manpower shortage delays the initiation of the release. The candidate must determine if the release is allowed to be started. ANSWER / DISTRACTOR ANALYSIS A. Incorrect. The relief cannot be started since > 24 hours have passed since the chemistry sample was taken. B. Incorrect. The relief cannot be started at any time since > 24 hours have passed since the chemistry sample was taken. C. Correct. The relief cannot be started since > 24 hours have passed since the chemistry sample was taken. D. Incorrect. The relief cannot be started since > 24 hours have passed since the chemistry sample was taken, while true > 24 hours has passed since permit approval, this is not a restriction on start of the relief. REFERENCES SOP-13125-1/2, "Containment Purge System" VEGP learning objectives: LO-PP-29101-08, Describe routine actions taken to adjust Containment pressure and temperature.
Notes Response form
Monday, August 13, 2007 10:00:37 AM

79

QUESTIONS REPORT for HL-14 NRC RO DRAFT EXAM


A n swers MCS Time: 1

Points:

1.00

Version: 0 1 2 3 4 5 6 7 8 9 Answer: CDCBDADADA

Scramble Range: A- D

Monday, August 13, 2007 10:00:37 AM

80

QUESTIONS REPORT for HL-14 NRC RO DRAFT EXAM 31. 029EA1.05 001/1/1/ATWT - BIT OUTLET SW/C/A - 3.7/MODIFIED/R/NRC RO/TNT / RLM The following plant conditions exist: Unit 2 has experienced an Anticipated Transient Without Trip (ATWT) and has implemented FRP-19211, Response to Nuclear Power Generation ATWT. A Charging Pump is running. Boric Acid Transfer Pump # 1 is tagged out. Boric Acid Transfer Pump # 2 trips on start. SI has NOT actuated at this time. The SS has directed the RO to establish Emergency Boration in accordance with SOP-13009, "CVCS Reactor Makeup Control System". Which ONE of the following actions would establish a CORRECT emergency boration flow path in accordance with the SOP? (Assume 12 gpm seal return flow) A. 1) Open HV-8104 EMERGENCY BORATE Valve. 2) Adjust charging flow controller FIC-0121 to obtain > 42 gpm flow through the Normal Charging flow path.

B. 1) Open LV-0112D and LV-0112E RWST TO CHARGING PUMP SUCT valves. 2) Adjust charging flow controller FIC-0121 to obtain > 42 gpm flow through the Normal Charging flow path.

C. 1) Open FV-110A BA to Blender and FV-110B BLENDER OUTLET TO CHARGING PUMPS SUCT. 2) Adjust charging flow controller FIC-0121 to obtain > 100 gpm flow through the Normal Charging Path.

D. 1) Open LV-0112D and LV-0112E RWST TO CHARGING PUMP SUCT valves and HV-8801A and HV-8801B BIT DISCHARGE ISOLATION valves. 2) Verify BIT flow (FI-0917A), plus total seal injection flow, minus total seal r e t u rn flow is > 100 gpm.
Feedback

K/A
Monday, August 13, 2007 10:00:37 AM

81

QUESTIONS REPORT for HL-14 NRC RO DRAFT EXAM 029 Anticipated Transient Without Scram (ATWS): EA1.05 Ability to operate and monitor the following as they apply to an ATWS. BIT outlet valve switches. K/A MATCH ANALYSIS Question gives a plausible scenario with an ATWT in progress. Neither Boric Acid Transfer Pump is available. Candidate must choose a correct emergency boration flow path that would achieve Emergency Boration Flow. ANSWER / DISTRACTOR ANALYSIS A. Incorrect. Without BA Transfer Pumps available there would be no flow through HV8104. Plausible the candidate may not realize BA Transfer Pump impact on flow path. Flow rates given would satisfy the flow path if BA Transfer Pumps available. B. Incorrect. LV-112D and LV-112E would satisfy the flow path requirements but the minimum flow requirement via this path would be 100 gpm. Plausible candidate could recognize a correct flow path but confuse the flow rate requirements. C. Incorrect. FV-0110A and FV-0110B would not have flow through this path without the Boric Acid Transfer Pumps available. Plausible candidate may not realize BA Transfer Pump impact on the flow path and confuse the flow rate requirements. D. Correct. Opening LV-112D and LV-112E would establish boration flow from RWST and flow requirements would be satisfied with 100 gpm to BIT. 100 gpm used to sound more like choice B to make question symetrical with choices and NOT be a NOT question. REFERENCES 19211-C, Nuclear Power Generation ATWT page 4 13009-1/2, CVCS Makeup Control System section 4.9 for Emergency Boration pages 38 through 41. LO-PP-09300-06-001, 003, and 004 from Vogtle LO Active Exam Bank VEGP learning objectives: LO-PP-09300-06, Describe all emergency flow path a. borated water source and discharge flow path b. minimum flow requirements
Monday, August 13, 2007 10:00:37 AM

82

QUESTIONS REPORT for HL-14 NRC RO DRAF T EXA M


Notes Response form Answers MCS Time: 1

Points:

1.00

Version: 0 1 2 3 4 5 6 7 8 9 Answer: D C D AD DBACB

Scramble Range: A- D

Monday, August 13, 2007 10:00:37 AM

83

QUESTIONS REPORT for HL-14 NRC RO DRAFT EXAM 32. 032AK1.01 001/1/2/SR MS - VOLT CHANGE/C/A - 2.5/BANK/R/NRC RO/TNT / RLM Given the following conditions: Reactor Startup in progress. Startup is on hold due to a problem with SR N-31. SR N-32 indicates 1000 cps. SR N-31 is in Level Trip Bypass Which ONE of the following will occur if the control power fuse for SR N-31 blows? A. A reactor trip will occur on SR Hi Flux. B. Rod withdrawal is blocked in automatic or manual. C. SR indication for N-31 is lost on the QMCB and NIS cabinets D. Rod withdrawal is blocked in automatic only.

Monday, August 13, 2007 10:00:37 AM

84

QUESTIONS REPORT for HL-14 NRC RO DRAFT EXAM


Feedback

K/A 032 Loss of Source Range Nuclear Instrumentation AK1.01 Knowledge of the operational implications of the following concepts as they apply to Loss of Source Range Nuclear Instrumentation: Effects of voltage changes on performance. K/A MATCH ANALYSIS Question gives a plausible scenario where a SR instrument is lost due to loss of control power below P-6 while in "Bypass". The candidate must choose the correct response to the instrument failure. ANSWER / DISTRACTOR ANALYSIS A. Correct. Loss of control power de-energizes bistables and intiates a reactor trip signal as the 1 / 2 coincidence is made up for SR Hi Flux Trip. B. Incorrect. Source range instruments do not have rod stops associated with them. Plausible since SR / IR instruments combined that candidate may think C-1 rod stop would be affected. C. Incorrect. Not all indications are lost since the instrument power is still available. Plausible candidate may think control power causes a loss of indication. D. Incorrect. Source range instruments do not have rod stops associated with them. Plausible since SR / IR instruments combined that candidate may think C-1 rod stop would be affected. REFERENCES Vogtle 2002 NRC RO Exam question # 38. VEGP learning objectives: LO-PP-17201-01, Discuss the operation of the Source & Intermediate range detectors to include: g. Power supplies (also including the effects on loss of instrument or control power) LO-PP-17201-02, Discuss the operation of the Source & Intermediate range switches. a. Level Trip Bypass
Monday, August 13, 2007 10:00:38 AM

85

Notes

QUESTIONS REPORT for HL-14 NRC RO DRAF T EXAM


Response form Answers MCS Time: 1 Points: 1.00 Version: 0 1 2 3 4 5 6 7 8 9 Answer: AACBBDBDCD Scramble Range: A- D

Monday, August 13, 2007 10:00:38 AM

86

QUESTIONS REPORT for HL-14 NRC RO DRAFT EXAM 33. 033A3.01 001/2/2/SFPCS-TCVS/MEM - 4.2/MODIFIED/R/NRC RO/TNT / RLM Given the following conditions: Unit 1 refueling outage in progress. A total loss of CCW has occurred. Per AOP-18030-1, Loss of Spent Fuel Pool Cooling", the SS has directed that "Feed and Bleed" be initiated. Which ONE of the following is CORRECT regarding temperature control during the feed and bleed evolution? A. Train A is the preferred train, simultaneous feed and bleed can be performed, SFP temperature is controlled using an auto Temperature Control Valve (TCV). B. Either Train A or Train B allows for simultaneous feed and bleed to be performed, SFP temperature is controlled using an auto Temperature Control Valve (TCV). C. Train B is the preferred train, simultaneous feed and bleed can be performed, SFP temperature is controlled using manually throttled valves, no auto TCV exists. D. Either Train A or Train B allows for simultaneous feed and bleed to be performed, SFP temperature is controlled using manually throttled valves, no auto TCV exists.
F e edback

K/A 033 Spent Fuel Pooling Cooling System (SFPCS): A3.01 Ability to monitor automatic operation of the Spent Fuel Pool Cooling System including: Temperature Control Valves K/A MATCH ANALYSIS Question gives a plausible scenario requiring SFP feed and bleed. The candidate has to determine which train is preferred and whether manual control or an auto TCV exists to control SFP temperature. ANSWER / DISTRACTOR ANALYSIS A. Incorrect. Plausible candidate may not know Train B is preferred, due to makeup valve arrangement train B is only train simultaneous feed and bleed can be performed. Candidate may think a TCV exists a some point in the flow path. B. Incorrect. Plausible candidate may not know Train B is preferred, due to makeup
Monday, August 13, 2007 10:00:38 AM

87

QUESTIONS REPORT for HL-14 NRC RO DRAFT EXAM valve arrangement train B is only train simultaneous feed and bleed can be performed. Candidate may think a TCV exists a some point in the flow path. C. Correct. Train B is preferred train and manual valve manipulation controls the spent fuel pool temeprature. D. Incorrect. Plausible candidate may not know Train B is preferred, due to makeup valve arrangement train B is only train simultaneous feed and bleed can be performed. Candidate may know manual manipulation only is performed but not know there is a preferred train. REFERENCES AOP-18030-C, "Loss of Spent Fuel Pool Cooling" LO-LP-60322-05-001 and 003 Vogtle LO Active Exam Bank questions. VEGP learning objectives: LO-PP-25102-09, Describe the impacts of the following conditions: b. Loss of CCW to the SFPCS heat exchangers. LO-LP-60322-02, Describe why you are cautioned in AOP-18030-C not to align the Spent Fuel Pool System to provide bleed to the RWST if spent fuel pool level is less than 217 ft elevation.

Notes Monday, August 13, 2007 10:00:38 AM Response form

88

QUESTIONS REPORT for HL-14 NRC RO DRAFT EXAM 34. 035K1.02 001/2/2/SG-MRSS/C/A - 3.2/NEW/R/NRC RO/TNT / RLM A reactor trip has occured and the plant has entered Mode 3. Steam dump valve 1PV-507C is stuck open and cannot be shut. All SG Pressures and PT-507 Steam Header Pressure are lowering. The BOP operator actuates SLI using the Train "A" SLI handswitch only. Which ONE of the following is CORRECT regarding the plant response? A. BOTH Trains MSIVs and Bypasses close. Steam Generator Pressures will stabilize or rise. B. ONLY Train "A" MSIVs and Bypasses close. Steam Generator Pressures will continue to lower. C. BOTH Trains MSIVs and Bypasses close. Steam Header Pressure read on PT-507 will stabilize or rise. D. ONLY Train "A" MSIVs and Bypasses close. S t e a m Header Pressure read on PT-507 will continue to lower.
Feedback

K/A

035 Steam Generator System (SGS): K1.02 Knowledge of the physical connections and / or cause effect relationships between the S/Gs and the following systems: MRSS K/A MATCH ANALYSIS Question asks a plausible scenario with a steam dump valve stuck open and the BOP taking action to stop an uncontrolled cooldown by performing an SLI. Candidate must pick the correct response of the MSIVs and the SG Pressures or Steam Header Pressure response. ANSWER / DISTRACTOR ANALYSIS A. Correct. Either SLI handswitch taken to actuate will cause BOTH trains of MSIVs and Bypasses to shut. SG pressures should begin to rise once the steam flow path has been isolated.

B. Incorrect. Either SLI handswitch taken to actuate causes BOTH trains of MSIVs and
Monday, August 13, 2007 10:00:38 AM

89

QUESTIONS REPORT for HL-14 NRC RO DRAFT EXAM Bypasses to shut. Plausible candidate may think only Train "A" isolates. Also, Plausible candidate could still think a steam flow path exists via bypasses and SG pressures still be lowering. C. Incorrect. MSIV and Bypass response is correct. However, PT-507 pressure should continue to lower with steamlines isolated and steam dump valve stuck open. Plausible candidate could confuse location of PT-507 relative to MSIVs. D. Incorrect. Either SLI handswitch taken to actuate causes BOTH trains of MSIVs and Bypasses to shut. Plausible candidate may think only Train "A" isolates. PT507 response is correct. REFERENCES Power Point V-LO-PP-21101 Main Steam System (slide # 139 for HS actuations) Power Point V-LO-PP-21201 Steam Dump System VEGP learning objectives: LO-PP-21101-12, Discuss the following concerning the "Main Steam Isolation Valves" (MSIVs) b. Basic description of how they operate

N otes Monday, August 13, 2007 10:00:38 AM

90

Response form

QUESTIONS REPORT for HL-14 NRC RO DRAFT EXAM 35. 037AA2.10 001/1/2/SGTL - TECH SPEC LCO/C/A - 3.2/MODIFIED/R/NRC RO/TNT / RLM Given the following conditions with Unit 2 at 100% power: The RCS Leak Rate surveillance indicates the following: Total RCS leakage is 9.2 gpm Leakage to the PRT is 5.9 gpm Leakage to the RCDT is 2.0 gpm Primary to Secondary Leakage is as follows: S/G # 1 = 0.08 gpm S/G # 2 = 0.09 gpm S/G # 3 = 0.10 gpm S/G # 4 = 0.11 gpm Which ONE of the following would be CORRECT regarding RCS Leakage limits in accordance with Tech Specs? A. No RCS leakage limits have been exceeded. B. The RCS Unidentified leakage limit has been exceeded. C. The Total Primary-to-Secondary leakage limit of 500 gpd has been exceeded. D. The Primary-to-Secondary leakage limit through any ONE S/G has been exceeded.

Monday, August 13, 2007 10:00:38 AM

91

QUESTIONS REPORT for HL-14 NRC RO DRAFT EXAM


F e edback

K/A 037 Steam Generator (S/G) Tube Leak AA2.10 Ability to determine and interpret the following as they apply to the Steam Generator Tube Leak. Tech Spec Limits for RCS Leakage K/A MATCH ANALYSIS Question gives a plausible scenario where an RCS Leak Rate has been performed. The candidate must determine if any RCS Leakage Tech Spec thresholds have been exceeded from the given data. ANSWER / DISTRACTOR ANALYSIS A. Incorrect. > 150 gpd through S/G # 4 exists. Plausible candidate could miscalculate from the given data to determine no leak limits are exceeded. B. Incorrect. Unidentified leakage from given data is 0.92 gpm. Plausible candidate could miscalculate from the given data and choose Unidentified leakage. C. Incorrect. Total from all S/Gs is 547.2 gpd (.38 X 1440 = 547.2). This answer is plausible since Vogtle until recently had a 500 gpd limit for leakage through all S/Gs. Candidates could still recall this number since it is a very recent change to TS. D. Correct. S/G # 4 has 158.4 gpd leakage (1440 X .11 = 158.4) which exceeds the threshold for entering LCO 3.4.13 for RCS Leakage. REFERENCES Technical Specification 3.4.13 for RCS Operational Leakage HL-AU-39000-00-002 from Vogtle LO Active Exam Bank (base for modification) VEGP learning objectives: LO-LP-39208-01, For any given item in section 3.4 of Tech Specs, be able to: a . State the LCO
N otes Response form

Monday, August 13, 2007 10:00:38 AM

92

QUESTIONS REPORT for HL-14 NRC RO DRAFT EXAM


Answers MCS Time: 1 Points: 1.00 Version: 0 1 2 3 4 5 6 7 8 9 Answer: DB CACDCBCC Scramble Range: A- D

Monday, August 13, 2007 10:00:38 AM

93

QUESTIONS REPORT for HL-14 NRC RO DRAFT EXAM 36. 038G2.1.14 001/1/1/SGTR - NOTIFICATIONS/MEM - 2.5/MODIFIED/R/NRC RO/TNT / RLM A Steam Generator Tube Rupture (SGTR) is in progress on Unit 1. - The crew is performing steps of 19030-C, "Steam Generator Tube Rupture" to "minimize secondary system and environmental contamination". In accordance with 19030-C, which ONE of the following is the CORRECT plant personnel to notify prior to performing this procedurally driven evolution? A. HP, notify prior to draining potentially contaminated water to the TB sumps. B. Chemistry, notify prior to resetting CIA and repositioning valves in the plant. C. HP, notify prior to draining potentially contaminated Hotwell water to the CSTs. D. Chemistry, notify upon relief of ruptured steam generator ARV.

Monday, August 13, 2007 10:00:38 AM

94

QUESTIONS REPORT for HL-14 NRC RO DRAFT EXAM


Feedback

K/A 038 Steam Generator Tube Rupture (SGTR): G2.1.14 Knowledge of system status criteria which require the notification of plant personnel. K/A MATCH ANALYSIS Question gives a plausible scenario during a SGTR where alignment / draining of plant systems must be performed to minimize secondary and environmental contamination. The candidate must pick the correct personnel to notify for the evolution as specified in the E-3 procedure. ANSWER / DISTRACTOR ANALYSIS A. Correct. Health Physics personnel must be notified for personnel protection and monitoring purposes. B. Incorrect. Plausible the candidate could confuse and think chemistry notified before resetting CIA, HP would be the proper department to notify. C. Incorrect. Plausible the but Hotwell dump to the CST is isolated during performance of this procedure using LV-4415 and / or manual isolations in the Turbine Building. D. Incorrect. Plausible the candidate may think notification of chemistry appropriate on ARV lifting, HP would be the correct department to notify. REFERENCES Harris February 2006 NRC RO Exam question # 16 19030-C, "Steam Generator Tube Rupture" step # 59 and Attachment "D" step # 1. VEGP learning objectives: LO-LP-37311-07, Using EOP 19030-C as a guide, briefly describe how each step is accomplished.

LO-LP-37311-11, Given a NOTE or CAUTION statement from the EOP, state the bases for that NOTE or CAUTION statement.
Notes

Response form

Monday, August 13, 2007 10:00:38 AM

95

QUESTIONS REPORT for HL-14 NRC RO DRAFT EXAM


Answers MCS Time: 1 Points: 1.00 Version: 0 1 2 3 4 5 6 7 8 9 Answer: ABBCBBBDCB

Scramble Range: A- D

Monday, August 13, 2007 10:00:38 AM

96

QUESTIONS REPORT for HL-14 NRC RO DRAFT EXAM 37. 039A2.01 001/2/1/MAIN RHT - LOCA PATH/C/A - 3.1/NEW/R/NRC RO/TNT / RLM Given the following conditions: Unit 1 at 30% power, Turbine has just been synchronized to the grid. Steam flow suddenly increases on the BOP panel. Plant personnel report a steam leak in the Turbine Building. The RO trips the reactor, the turbine auto trips. The MSIVs and Bypasses are still open. The steam leak appears to have stopped. Which ONE of the following would be CORRECT regarding the indications above? A. Break on the main steam cross-tie header line. B. Break on the steam supply line to the Auxiliary Steam Header. C. Break on the steam supply lines to the Main Feed Pump Turbines. D. Break on the lines between the stop valves and the Main Turbine.

Monday, August 13, 2007 10:00:38 AM

97

QUESTIONS REPORT for HL-14 NRC RO DRAFT EXAM


Feedback

K/ A 039 Main and Reheat Steam System (MRSS): A2.01 Ability to (a) predict the impacts of the following malfunctions or operations on the MRSS; and (b) based on predictions, use procedures to correct, control, or mitigate the consequences of those malfunctions or operations: Flow paths of steam during a LOCA. K/A MATCH ANALYSIS Question gives a plausible scenario with a Turbine Building steam leak which stops with a Reactor-Turbine trip and MSIVs still open. The candidate must determine from the choices the location of the steam leak. ANSWER / DISTRACTOR ANALYSIS A. Incorrect. Plausible candidate could confuse the location of the main steam cross-tie header and think the leak was isolated. B. Incorrect. Plausible the candidate may confuse the location of the Aux. Steam header and think the leak was isolated. C. Incorrect. Plausible the candidate may confuse the location of the MFPT steam supply lines. D. Correct. Turbine trip would isolate a leak between the stop valves and Main Turbine. REFERENCES V-LO-PP-21101, "Main Steam System" slide # 150 VEGP learning objectives: LO-PP-21101-09 Describe the Main Steam flow path to include:

b. Major steam loads downstream of the Main Steam Isolation Valves.


Notes

Monday, August 13, 2007 10:00:38 AM

98

Response form

QUESTIONS REPORT for HL-14 NRC RO DRAFT EXAM


A n swers MCS Time: 1 Points: 1.00 Version: 0 1 2 3 4 5 6 7 8 9 Answer: DDADCCCDDA Scramble Range: A- D

Monday, August 13, 2007 10:00:38 AM

99

QUESTIONS REPORT for HL-14 NRC RO DRAFT EXAM 38. 045G2.1.33 001/2/2/TURB/GEN-TECH SPECS/MEM - 3.4/NEW/R/NRC RO/TNT / RLM Which ONE of the following conditions would require an LCO entry per Technical Specifications? A. Failure of PT-505 while in Mode 1. B. Failure of C-16 in Mode 1 greater than P-9. C. Failure of Main Turbine Vibration Instrumentation. D. Failure of Main Turbine Overspeed Test in Mode 2 with MSIVs closed.

Monday, August 13, 2007 10:00:39 AM

100

QUESTIONS REPORT for HL-14 NRC RO DRAFT EXAM


F e edback

K/A 045 Main Turbine Generator (MT/G) System: G2.1.33 Ability to recognize indications for system operating parameters which are entry-level conditions for Technical Specifications. K/A MATCH ANALYSIS Question asks plausible scenario for an LCO entry per Technical Specifications related to the Main Turbine Generator. Candidate must choose the correct test failure and power level that would require a Tech Spec LCO entry. ANSWER / DISTRACTOR ANALYSIS A. Correct. PT-505 Turbine Impulse Pressure failure would require LCO entry. B. Incorrect. Plausible candidate could consider C-16 but is not Tech Spec. C. Incorrect. Plausible canididate would consider Turbine Vibration instrumentation but is not in Tech Specs. D. Incorrect. Plausible candidate could consider but is not applicable with MSIVs shut and Turbine Overspeed is not Tech Spec. REFERENCES Technical Specification 3.3.1 for Reactor Trip Instrumentation Fu 16f for Turbine Impulse Pressure P-13. VEGP learning objectives: LO-LP-39207-02, Given a set of Tech Specs and bases, determine for a specific set of plant conditions, equipment availability, and operational mode:

a . W hether any Tech Spec LCOs of section 3.3 are exceeded.


Notes Response form Answers MCS Time: 1 Points: 1.00 Version: 0 1 2 3 4 5 6 7 8 9

Answer: ABACBDCB CD

Scramble Range: A- D

Monday, August 13, 2007 10:00:39 AM

101

QUESTIONS REPORT for HL-14 NRC RO DRAFT EXAM 39. 054G2.4.49 001/1/1/LOSS MFW - IOAS/MEM - 4.0/NEW/R/NRC RO/TNT / RLM Given the following: Plant is at 77% with all systems in normal alignment. MFPT "A" trips due to a hydraulic oil pressure leak. MFPT "B" rpms are increasing with discharge pressure reading 0 psig. ALB15 window B05 for MFPT LOW DISHCH HDR PRESSURE is lit. Steam Generator Flow Mismatch Annunciators are present for all S/Gs. S/G levels are rapidly lowering. No actions have yet been taken by the operating crew. Which ONE of the following is the first CORRECT action(s) for the crew to perform? A. Verify rapid insertion of control rods to match Tavg and Tref. B. Trip the reactor and go to 19000-C, "E-0, Reactor Trip or Safety Injection". C. Reduce Turbine Load as necessary, IF SG NR levels cannot be maintained greater than 40%, trip the reactor, go to E-0, Reactor Trip or Safety Injection. D. Press the "Start Setback" pushbutton and ensure the generator output reduces to less than 850 MWe, start the third condensate pump, raise running MFPT speed.

Monday, August 13, 2007 10:00:39 AM

102

QUESTIONS REPORT for HL-14 NRC RO DRAFT EXAM


Feedback

K/A 054 Loss of Main Feedwater: G2.4.49 Ability to perform without reference to procedures those actions that require immediate operation of system components and controls. K/A MATCH ANALYSIS Question gives a plausible scenario with a MFPT trip and the other running MFPT has apparently sheared a shaft or has some other malfunction. The candidate has to pick the correct IOA to perform in accordance with AOP-18016-C, section A. ANSWER / DISTRACTOR ANALYSIS A. Incorrect. Plausible the candidate would consider rapid insertion of control rods to match Tavg and Tref, this is no longer one of the first 10As to perform as a recent procedure change made this 10A step A7 coming late in the 10As. (11-26-2006) B. Correct. With reactor power > 75% and at least one MFPT not providing flow the RNO has the crew perform a reactor trip and go to E-0. C. Incorrect. Plausible the candidate would consider this since it is step A2 RNO, however, with reactor power > 75% this RNO should not be performed and the crew should proceed to step A3. D. Incorrect. Plausible the candidate would consider this since with at least one MFPT providing flow this would be the next IOA step A4. REFERENCES AOP-18016-C, Condensate and Feedwater Malfunction section A for MFP Malfunction 10As steps Al thorugh A7. VEGP learning objectives: LO-LP-60314-02, Describe the operator actions required if during the performance of AOP-18016-C, "Condensate and Feedwater Malfunction" a loss of SG level is imminent.

LO-LP-60314-03, State actions required on loss of both SGFPTs with power > 20% and power < 20%
Notes

Response form

Monday, August 13, 2007 10:00:39 AM

103

QUESTIONS REPORT for HL-14 NRC RO DRAFT EXAM


A n swers MCS Time: 1 Points: 1.00 Version: 0 1 2 3 4 5 6 7 8 9 Answer: B ADAA C A ADD Scramble Range: A- D

Monday, August 13, 2007 10:00:39 AM

104

QUESTIONS REPORT for HL-14 NRC RO DRAFT EXAM 40. 055EK1.01 001/1/1/LOSS AC - BATT CAPAC/MEM - 3.3/BANK/R/NRC RO/TNT / RLM Unit 2 has tripped and a Loss of All AC power event is in progress. The battery chargers to the 125V DC buses have been de-energized for approximately one hour. It is estimated another 4 hours until AC power is restored. DC bus loads are consistent with design bases. From this point on, operators can expect the 125V DC bus voltages to drop _________ at first, then later drop ___________ A. slowly, faster due to high load. B. quickly, more slowly due to battery depletion C. quickly, more slowly due to low load. D. slowly, faster due to battery depletion.

Monday, August 13, 2007 10:00:39 AM

105

QUESTIONS REPORT for HL-14 NRC RO DRAFT EXAM


F e edback

K/A 055 Loss of Offsite and Onsite Power (Station Blackout) EK1.01 Knowledge of the operational implications of the following concepts as they apply to the station blackout: Effect of battery discharge rates on capacity. K/A MATCH ANALYSIS Question gives a plausible during a loss of All AC Power (Station Blackout) where the 1E 125V DC buses are being supplied from the batteries only. The candidate must choose whether the discharge rates of the batteries will get faster or slower over the course of the event and why. ANSWER / DISTRACTOR ANALYSIS A. Incorrect. Rate of decay slow at first then increases due to battery depletion. B. Incorrect. Rate of decay slow at first then increases due to battery depletion. C. Incorrect. Rate of decay slow at first then increases due to battery depletion. D. Correct. Rate of decay slow at first then increases due to battery depletion. REFERENCES 2006 NRC RO Retake Exam. NOTE: This question is a re-use from the May 2006 NRC RO Retake exam and 1 of 4 or less that can be used from last 2 previous exams (May 2006 NRC RO Retake Exam and May 2005 NRC RO Exam) North Anna June 2004 NRC RO Exam question # 50 VEGP learning objectives: N o t a p p l i c a b l e .
N otes Response form Answers

Vogtle May

MCS

Time: 1

Points:

1.00

Version: 0 1 2 3 4 5 6 7 8 9 Answer: DDBBABDDBD

Scramble Range: A- D

Monday, August 13, 2007 10:00:39 AM

106

QUESTIONS REPORT for HL-14 NRC RO DRAFT EXAM 41. 057AA2.19 001/1/1/VITAL AC - AUTO ACT/C/A - 4.0/MODIFIED/RJNRC RO/TNT / RLM The following conditions exist: - A plant startup is in progress. Reactor power is currently 12% and rising. All control systems are in normal alignment. A loss of 120V AC Vital instrument bus 2AY1A occurs. Which ONE (1) of the following describes the effect on the plant? A. Reactor automatically trips due to loss of one Intermediate Range instrument. B. Rapid Control Rod insertion due to loss of Turbine Impulse Pressure instrument. C. Manual reactor trip by Reactor Operator performing Immediate Operator Actions. D. Letdown isolates due to failure of the controlling Pressurizer Level instrumentation.

Monday, August 13, 2007 10:00:39 AM

107

QUESTIONS REPORT for HL-14 NRC RO DRAFT EXAM


Feedback

K/A 057 Loss of Vital AC Instrument Bus: AA2.19 Ability to determine and interpret the following as they apply to the Loss of Vital AC Instrument Bus. The plant automatic actions that will occur on the loss of a vital ac electrical instrument bus. K/A MATCH ANALYSIS Question gives a plausible scenario with failure of a 120V AC Vital Instrument Bus. The candidate has to choose the correct plant response to the failed instrument. ANSWER / DISTRACTOR ANALYSIS A. Incorrect. Reactor will not trip as plant is > P-10 and IR trips should be blocked per the controlling UOP. Plausible candidate may not recall block of IR and a reactor trip would occur if the plant was < P-10. B. Incorrect. Control Rods should not be in auto until reactor power > 15%. Plausible candidate may not recall rods not in auto per controlling UOP or during startup. 10A is to place rods in manual. PT-505 failure would cause rapid insertion at power. C. Incorrect. IOA is to perform manual reactor trip if power < P-10. Plausible candidate could confuse IOA and think manual trip should be performed. D. Correct. Channel 1 failure would cause a failure of LT-459 resulting in a letdown isolation with LV-459 isolation valve closing. REFERENCES Harris February 2006 NRC RO Exam question # 41. AOP-18032-2, section A for Loss of Vital Instrument Bus Panel 2AY1A steps Al - A6. VEGP learning objectives: LO-LP-60324-02, Given that a loss of 120VAC intrument power has occurred to any of the following panels, and given the appropriate plant procedures, describe the operator actions required and why these actions are taken. a. AY1A

Monday, August 13, 2007 10:00:39 AM

108

Notes

QUESTIONS REPORT for HL-14 NRC RO DRAF T EXAM


Response form Answers MCS Time: 1 Points: 1.00 Version: 0 1 2 3 4 5 6 7 8 9 Answer: DDDDA AD D C C Scramble Range: A- D

Monday, August 13, 2007 10:00:39 AM

109

QUESTIONS REPORT for HL-14 NRC RO DRAFT EXAM 42. 058AK3.01 001/1/1/LOSS DC - EDG CONT/C/A - 3.4/BANK/R/NRC RO/TNT / RLM Given the following: DG1B running for the monthly surveillance. A loss of 125V DC 1E bus 1BD1 occurs. - The Shift Supervisor directs you to shutdown DG1B. W hich ONE of the following is the CORRECT action to shutdown DG1B? A. Depress the normal stop pushbutton on the QEAB. B. Depress BOTH Emergency Stop pushbuttons on the QEAB. C. Depress the Emergency Stop pushbutton on the local Diesel Control Panel. D. Depress the Pull to Run / Push to Stop pushbutton at the DG front standard area.

Monday, August 13, 2007 10:00:39 AM

110

QUESTIONS REPORT for HL-14 NRC RO DRAFT EXAM


Feedback

K/A 058 Loss of DC Power: AK3.01 Knowledge of the reasons for the following responses as they apply to the Loss of DC Power. Use of dc control power by D/Gs. K/A MATCH ANALYSIS Question gives a plausible scenario where a DG is running for a monthly surveillance when a loss of DC control power occurs. Candidate must choose the correct method to shutdown the DG. ANSWER / DISTRACTOR ANALYSIS A. Incorrect. Normal starting and stopping circuitry is disabled with a loss of DC control power to BD11 / BD12. Plausible candidate may think normal shutdown from QEAB still available since no SI or LOSP in progress. B. Incorrect. Normal starting and stopping circuitry is disabled with a loss of DC control power to BD11 / BD12. Plausible candidate may think an emergency trip using QEAB pushbuttons may still function. C. Incorrect. Normal starting and stopping circuitry is disablled with a loss of DC control power to BD11 / BD12. Plausible candidate may think the local emergency trip may still function.. D. Correct. None of the QEAB or local pushbuttons will function to stop the DG with a loss of DC control power to BD11 / BD12. The Pull to Run / Push to Stop pushbutton uses pneumatic air to shut the combustion air intake dampers and place the fuel racks to the no fuel position using the shutdown cylinder. REFERENCES V-LO-PP-11101-40-001 from Vogtle LO Active Exam bank was used as the base for this question (slightly modified from bank). AOP-18034-C, "Loss of Class 1 E 125V DC Power" step # B6.b VEGP learning objectives: LO-PP-11101-40, Predict the effect on the following malfunctions on the operation of the diesel engine: b. Loss of engine (A, B, or C) control power.

Monday, August 13, 2007 10:00:39 AM

111

QUESTIONS REPORT for HL-14 NRC RO DRAF T EXA M


Notes Response form Answers MCS Time: 1

Points:

1.00

Version: 0 1 2 3 4 5 6 7 8 9 Answer: DB CCCDCACD

Scramble Range: A- D

Monday, August 13, 2007 10:00:39 AM

112

QUESTIONS REPORT for HL-14 NRC RO DRAFT EXAM 43. 059A3.06 001/2/1/MFW - FWI/MEM - 3.2/NEW/R/NRC RO/TNT / RLM A reactor trip with no safety injection has occurred from 30% power. RCS Tavg is 560 degrees F and stable. Both Reactor Trip breakers are open. The Shift Supervisor directs you to "verify feedwater isolation" (FWI).

Which ONE of the following identifies the locations where the operator would verify FW I? A. MFIVs and BFIVs on the QMCB handswitches. MFRVs and BFRVs on the QMCB Hagen controllers. B. MFIVs and BFIVs on the QMCB RO panel MLBs. MFRVs and BFRVs on the QMCB RO panel ZLBs. C. MFIVs and BFIVs on the QMCB RO panel MLBs. MFRVs and BFRVs on the QMCB RO panel MLBs. D. MFIVs and BFIVs on the QMCB handswitches. MFRVs and BFRVs on the QMCB BOP panel ZLBs.
F e edback

K/A 059 Main Feedwater (MFW) System: A3.06 Ability to monitor automatic operation of the MFW, including: Feedwater Isolation K/A MATCH ANALYSIS Question gives a plausible scenario requiring a FWI. The candidate must determine locations that FWI can be verified from the choices. ANSWER / DISTRACTOR ANALYSIS A. Incorrect. MFIV and BFIV positions can be determined from either the QMCB HS or the RO panel MLBs on TSLB-4. The MFRVs and BFRVs can only be determined from the BOP panel ZLB-04, not the Hagen controllers. Plausible candidate could think the Hagen controllers could be used for verification. B. Incorrect. MFIV and BFIV positions can be determined from either the QMCB HS or the RO panel MLBs on TSLB-4. The MFRVs and BFRVs can only be determined from the BOP panel ZLB-04, not the Hagen controllers. Plausible candidate may
Monday, August 13, 2007 10:00:39 AM

113

QUESTIONS REPORT for HL-14 NRC RO DRAFT EXAM think there is a ZLB on the RO panel for MFRVs and BFRVs. C. Incorrect. MFIV and BFIV positions can be determined from either the QMCB HS or the RO panel MLBs on TSLB-4. The MFRVs and BFRVs can only be determined from the BOP panel ZLB-04, not the Hagen controllers. Plausible candidate may think there are MLB indications on the RO panel for the MFRVs and BFRVs. D. Correct. MFIV and BFIV positions can be determined from either the QMCB HS or the RO panel MLBs on TSLB-4. The MFRVs and BFRVs can only be determined from the BOP panel ZLB-04. REFERENCES V-LO-PP-18101, "Condensate and Feedwater System" slide # 167 Simulator QMCB panels, ZLBs, and MLBs. VEGP learning objectives: LO-PP-18101-18, Discuss the basic operation of the MFRVs. LO-PP-18101-19, Discuss the basic operation of the MFIVs. LO-PP-18101-20, Discuss the basic operation of the BFRVs. LO-PP-18101-21, Discuss the basic operation of the BFIVS.
N o t es R e sponse form Answers MCS Time: 1 Points: 1.00 Version: 0 1 2 3 4 5 6 7 8 9 Answer: DDAAC B C C CA Scramble Range: A- D

Monday, August 13, 2007 10:00:39 AM

114

QUESTIONS REPORT for HL-14 NRC RO DRAFT EXAM 44. 059K3.04 001/2/1/MFW - RCS/C/A - 3.6/BANK/R/NRC RO/TNT / RLM Given the following conditions: The plant is at 100% power with all systems in automatic. The RO has recently performed a small dilution for Tavg control. The following indications are observed in the control room. Power Range NI's are rising. Tavg is lowering. Steam flow and feed flow are stable. Reactor power is 101% and rising slowly. Which ONE of the following is the CORRECT event in progress and required/preferred action in accordance with AOP-18016-C, "Condensate Feedwater Malfunction"? A. Inadvertent RCS dilution, reduce power by reducing turbine load as necessary. B. Inadvertent RCS dilution, reduce power and Tavg by inserting control rods. C. Loss of Feed Water heating, reduce power by reducing turbine load as necessary. D. Loss of Feed Water heating, reduce power and Tavg by inserting control rods.

Monday, August 13, 2007 10:00:40 AM

115

QUESTIONS REPORT

for HL-14 NRC RO DRAFT EXAM


Feedback

K/A 059 Main Feedwater (MFW) System: K3.04 Knowledge of the effect that a loss or malfunction of the MFW will have on the following: RCS K/A MATCH ANALYSIS Question gives a plausible scenario where a reactor over power event is occuring. The candidate must determine the cause of the event and the correct action to take. ANSWER / DISTRACTOR ANALYSIS A. Incorrect. Plausible the candidate may think the dilution caused reactor power to rise, however, the other indications rule this out. Correct actions per AOP 18016-C. B. Incorrect. Plausibe the candidate may think the dilution caused reactor power to rise, however, the other indications rule this out. Actions not those of AOP 18016-C. C. Correct. Loss of Feed Water heating and actions are correct. D. Incorrect. Plausible candidate could identify the loss of Feed Water heating but the actions are not those of AOP 18016-C. REFERENCES AOP-18016-C, "Condensate Feedwater Malfunction" section C for Loss of FW Heaters V-LO-LP-60314-01-001 Vogtle LO Active Exam Bank Beaver Valley March 2005 NRC RO Exam question # 88 VEGP learning objectives: LO-PP-60314-05, Given the entire AOP, describe: a. Purpose of the selected steps. b. How and why the step is performed. c. Expected response of the plant / parameter(s) for the step.
Monday, August 13, 2007 10:00:40 AM

116

Notes

QUESTIONS REPORT

for HL-14 NRC RO DRAFT EXAM


Response form Answers MCS Time: 1 Points: 1.00 Version: 0 1 2 3 4 5 6 7 8 9 Answer: CC CC AAACDC Scramble Range: A- D

Monday, August 13, 2007 10:00:40 AM

117

QUESTIONS REPORT for HL-14 NRC RO DRAFT EXAM 45. 061K5.01 001/2/1/AFW - RCS HEAT XFER/MEM - 3.6/BANK/R/NRC RO/TNT / RLM Following a reactor trip with no SI required, the crew has transitioned to 19001, Reactor Trip Response. Step 3c AER says "Check SGs NR level - ONE GREATER THAN 10%". 1) If yes, AER says "throttle total AFW flow as necessary". 2) If no, RNO says "Verify total AFW flow greater than 570 gpm". Which ONE of the following is CORRECT regarding the basis for the Step 3 actions? A. 1) To limit runout of the AFW pumps. 2) Ensures adequate AFW flow for decay heat removal. 1) To limit runout of the AFW pumps. 2) Ensures enough flow for AFW pump protection. 1) To limit overcooling of the RCS. 2) Ensures adequate AFW flow for decay heat removal. 1) To limit overcooling of the RCS. 2) Ensures enough flow for AFW pump protection.

B.

C.

D.

Monday, August 13, 2007 10:00:40 AM

118

QUESTIONS REPORT for HL-14 NRC RO DRAFT EXAM


Feedback

K/A 061 Auxiliary Feedwater (AFW) System: K5.01 Knowledge of the operational implications of the following concepts as they apply to the AFW: Relationship between AFW flow and RCS heat transfer. K/A MATCH ANALYSIS Question gives a plausible scenario following a plant trip where AFW flow control is directed by the Reactor Trip Recovery procedure. Candidate must choose the correct basis for the throttling / maintaining of AFW flow. ANSWER / DISTRACTOR ANALYSIS A. Incorrect. AFW pumps are protected from runout by flow orifices. Decay heat removal portion is correct. Plausible candidate may think flow is throttled to prevent runout of the pumps. B. Incorrect. AFW pumps are protected from runout by flow orifices. AFW pumps are protected from low flow by miniflow Iine.Plausible candidate may think flow is throttled to prevent runout or kept high enough for pump protection. C. Correct. Limit RCS overcooling and adequate decay heat removal is correct. D. Incorrect. Limit RCS overcooling is correct. AFW pumps are protected from low flow by miniflow line. Plausible candidate may think flow is maintained for pump protection. REFERENCES DC Cook February 2006 NRC RO Exam question # 55 V-LO-PP-20101 Auxiliary Feedwater Power Point slide # 158 VEGP learning objectives: LO-LP-37011-08, State why the control of AFW is so important following a reactor trip.
Notes

Monday, August 13, 2007 10:00:40 AM

119

Response form

QUESTIONS REPORT for HL-14 NRC RO DRAFT EXAM


Answers MCS Time: 1 Points: 1.00 Version: 0 1 2 3 4 5 6 7 8 9 Answer: CDDDBDCCDC Scramble Range: A D

Monday, August 13, 2007 10:00:40 AM

120

QUESTIONS REPORT for HL-14 NRC RO DRAFT EXAM 46. 062AK3.03 001/1/1/LOSS NSCW - EOP/C/A - 4.0/NEW/R/NRC RO/TNT / RLM Given the following sequence of events: Train "A" NSCW is drained and tagged out for leak repair. Train "A" DG has been emergency tripped per the AOP. Train "A" components placed in PTL and / or stop per the AOP. A reactor trip and SI occur due to an RCS LOCA. LOSP occurs on Train "B" and DG1B reloads bus 1BA03. While performing the RO Initial Operator Actions of E-0 the RO points out to the crew that NSCW Train "B' has failed to automatically start. Which ONE of the following is the CORRECT action for the crew to take? A. Trip all reactor coolant pumps and isolate letdown. B. Place all Train "B" components in PTL and / or stop as necessary. C. Manually start two Train "B" NSCW pumps, ensure tower basin return is in auto. D. Emergency trip DG1B using the QEAB pushbuttons, go to Loss of All AC power.

Monday, August 13, 2007 10:00:40 AM

121

QUESTIONS REPORT for HL-14 NRC RO DRAFT EXAM


Feedback

K/ A 062 Loss of Nuclear Service Water: AK3.03 Knowledge of the reasons for the following responses as they apply to the Loss of Nuclear Service Water. Guidance actions contained in EOP for Loss of Nuclear Service Water. K/A MATCH ANALYSIS Question gives a plausible scenario where one NSCW train is unavailable and the other train fails to automatically start to perform it's safety function. The candidate must choose the correct action to take in response. ANSWER / DISTRACTOR ANALYSIS A. Incorrect. Plausible action from AOP-18021 for Loss of NSCW step # 4 RNO. B. Incorrect. Plausible action from AOP-18021 for Loss of NSCW step # 14. C. Correct. Verify means "make it so", crew should start two NSCW pumps and ensure Tower Return Bypass handswitches in auto per Initial Operator Actions of E-0. D. Incorrect. Plausible action from AOP-18021-C for Loss of NSCW step # 3, step # 6 RNO. REFERENCES AOP-18021-C, "Loss of Nuclear Service Cooling Water". EOP-19000-C, "E-0, Reactor Trip or Safety Injection". VEGP learning objectives: LO-LP-37002-19, State how each of the following EOP format elements are used to g u i de the operator in proper performance of the steps of the procedure. g .
No t es Response form

h i g h

l e v e l

t a s k s

s u b t a s k s

Answers MCS Time: 1 Points: 1.00 Version: 0 1 2 3 4 5 6 7 8 9 Answer: CBDDBCBDBD Scramble Range: A- D

Monday, August 13, 2007 10:00:40 AM

122

QUESTIONS REPORT for HL-14 NRC RO DRAFT EXAM 47.


062K1.02 001/2/1/AC DIST - EDG/C/A - 2.5/MODIFIED/R/NRC RO/TNT / RLM

Given the following: An SI has occurred and is NOT reset. An LOSP occurs a few minutes later. 1AA02 is powered from DG 1BA03 is powered from DG1B While the DG's are operating, an electrical perturbation results in the following; DG1A 186A lockout relay energizes (Generator Differential) DG1B 186B lockout relay energizes (Phase Overcurrent) Which ONE of the following is CORRECT with respect to the status of power to the 4160 1 E Emergency Buses at this time? A. B. C. Both 4160 1E emergency buses would be energized. Both 4160 1E emergency buses would be de-energized. 1AA02 would be energized, 1BA03 would be de-energized.

D. 1 B A 0 3 would be energized, 1AA02 would be de-energized.


Feedback

K/A 062 AC Electrical Distribution System: K1.02 Knowledge of the physical connections and / or cause effect relationships between the AC distribution system and the following systems: ED/G K/A MATCH ANALYSIS Question gives a plausbile scenarion with SI occuring and not yet reset. An LOSP results in the EDG's carrying both 4160 1 E emergency buses. Different overcurrent relays energize on each EDG and the candidate must determine the power supply status of both 4160 1 E emergency buses. ANSWER / DISTRACTOR ANALYSIS

A. Incorrect. Generator Differential (186A) lockout will emergency trip DG1A under all
Monday, August 13, 2007 10:00:40 AM

123

QUESTIONS REPORT for HL-14 NRC RO DRAFT EXAM conditions resulting in bus AA02 de-energization. Plausilble candidate may not recognize this as an emergency trip or that it will trip DG during SI condition. B. Incorrect. DG1B or breaker would not trip on Phase Overcurrent (186B) as this trip is not active during SI conditions and output breaker would remain closed. Plausible candidate may think both DG and / or output breakers could trip with the listed conditions since the 186A lockout would trip DG1A. C. Incorrect. As stated above 186A lockout would trip DG1A resulting in AA02 being de-energized. Plausible candidate could invert the malfunctions or not recognize the effect of SI on the DG's and breakers. D. Correct. AA02 would de-energize on DG1A trip, BA03 would remain energized as 186B lockout does not trip DG or output breaker on SI. REFERENCES LO-PP-11101-31-003 from Vogtle LO Active Exam Bank LO-PP-11101-55-001 from Vogtle LO Active Exam Bank VC Summer 2004 NRC RO Exam question # 44 SOP-13145-1/2, "Emergency Diesel Generators", Precaution 2.1.3 VEGP learning objectives: LO-PP-11101-55, Identify the primary relays which will actuate each of the following lockout relays and how the diesel generator will respond to each normal start from SI, UV, Local Emergency Start, and Normal Start. a. 186A lockout relay b. 186B lockout relay ,
N o t es R e sponse form Answers MCS Time: 1 Points: 1.00 Version: 0 1 2 3 4 5 6 7 8 9 Answer: DBDBCDDDBC Scramble Range: A- D

Monday, August 13, 2007 10:00:40 AM

124

QUESTIONS REPORT for HL-14 NRC RO DRAFT EXAM 48.


063A2.01 001/2/1/DC - GROUNDS/MEM - 2.5/BANK/R/NRC RO/TNT / RLM

While at 100% power, the following annunciator illuminates. - ALB35, window DO1 for 125V DC SWGR AD1 TROUBLE - The Control Building Operator (CBO) has been dispatched to investigate and reports a ground. Which ONE of the following is CORRECT regarding indications / actions to take if a GROUND has occurred? A. There are no bus ground detection targets associated with AD1, de-energize all loads on AD1M, AD11, and AD12 one at a time until the alarm clears. B. A bus ground detection target would be dropped on AD1, de-energize selected loads on AD1M, AD11, and AD12 one at a time to locate the source of the ground. C. There are no local ground detection targets associated with AD1, the CBO would have no indications of a ground, maintenance would have to determine the cause. D. A bus ground detection target would be dropped on AD1, de-energize panels AD1M, AD11, and AD12 one at a time to locate the panel with the ground.
F e edback

K/A 063 DC Electrical Distribution System: A2.01 Ability to (a) predict the impacts of the following malfunctions or operations on the DC electrical systems; and (b) based on those predictions, use procedures to correct, control, or mitigate the consequences of those malfunctions or operations. Grounds K/A MATCH ANALYSIS Question gives a plausible scenario where a ground fault has appeared on a 125V DC electrical bus. The operator must choose which of the indications would be available to determine the source of the ground and corrective action to take. Ground fault indication is available on AD1 but not the panels fed from AD1. The panels fed from AD1 should have loads selectively de-energized one at a time to determine the source of the ground. Not all loads can be de-energized. MSIVs and MFIVs in particular are powered from AD11 / AD12 and would result in a plant trip if de-energized. ANSWER / DISTRACTOR ANALYSIS A. Incorrect. There is ground fault indication available for AD1. However, ALL loads on
Monday, August 13, 2007 10:00:40 AM

125

QUESTIONS REPORT for HL-14 NRC RO DRAFT EXAM AD11 or AD12 cannot be de-energized without resulting in a reactor trip. See D explanation below. B. Correct. Ground fault indication is available for AD1. AD1M, AD11, and AD12 should have loads selectively de-energized to try to find the ground. C. Incorrect. There is ground fault indication available on AD1. Even though maintenance may have to be eventually called, selectively de-energizing loads on AD1 and buses fed from AD1 may find the ground. D. Incorrect. Ground fault indication is available for AD1. However, de-enrgizing the AD11 or AD12 panel would result in a reactor trip due to MSIVs / MFIVs shutting when their solenoids de-energized. REFERENCES ALB17 window D01 for 125V DC SWGR AD1 TROUBLE Vogtle 2006 May NRC RO Retake Exam question # 49 NOTE: This is # 3 of 4 questions allowed to be re-used from the last 2 NRC RO exams. VEGP learning objectives: Not applicable.
N o t e s

Response form Answers MCS Time: 1

Points:

1.00

Version: 0 1 2 3 4 5 6 7 8 9 Answer: BDDCBBAABD

Scramble Range: A- D

Monday, August 13, 2007 10:00:40 AM

126

QUESTIONS REPORT for HL-14 NRC RO DRAFT EXAM 49. 064A3.06 001/2/1/EDG - START/STOP/C/A - 3.3/NEW/R/NRC RO/TNT / RLM Given the following conditions: DG1A is running, an extra licensed operator is dispatched to monitor and observes the green "LOSS OF OFFSITE PWR OR SAFETY INJ SIGNAL" lamp is lit. The operator checks the status of the red "SHUTDOWN SYSTEM ACTIVE" lamp. Which ONE of the following is CORRECT regarding the status of the SHUTDOWN SYSTEM ACTIVE lamp ? The SHUTDOWN SYSTEM STATUS lamp should be _______________ A. Extinguished, indicating the DG Normal trips are active. B. Illuminated, indicating the DG Emergency trips are active. C. Extinguished, indicating the DG Normal trips are bypassed. D. Illuminated, indicating the DG Emergency trips are bypassed.

Monday, August 13, 2007 10:00:40 AM

127

QUESTIONS REPORT

for HL-14 NRC RO DRAFT EXAM


Feedback

K/A 064 Emergency Diesel Generator (ED/G) System: A3.06 Ability to monitor automatic operation of the ED/G system, including: Start and stop K/A MATCH ANALYSIS Question gives a plausible scenario with local indications showing an emergency start of the DG. The candidate must determine the correct status of the SHUTDOWN SYSTEM ACTIVE lamp which indicates whether normal trips are active or bypassed. ANSWER / DISTRACTOR ANALYSIS A. Incorrect. During an emergency start, the normal trips should be bypassed and this lamp should be extinguished. Plausible the candidate may invert the purpose of the lamp thinking the normal trips are active. B. Incorrect. Plausible the candidate may think the lamp indicates the Emergency Trips are active by being illuminated. C. Correct. During an emergency start, the normal trips should be bypassed and this lamp should be extinguished. D. Incorrect. Plausible the candidate may think the light should be illuminated indicating the Emergency Trips are bypassed. REFERENCES SOP-13145-1/2, "Diesel Generators" section 4.1.5 for Local Emergency Startup of DG and section 4.4.3 for DG Operation Under Emergency Conditions. VEGP learning objectives: LO-PP-11101-31, List the diesel engine trips and for each: a. State the setpoint and coincidence if applicable. b. Identif y as emergency or non -emergency trip.
Notes

Monday, August 13, 2007 10:00:41 AM

128

Response form

QUESTIONS REPORT for HL-14 NRC RO DRAFT EXAM


A n swers MCS Time: 1

Points:

1.00

Version: 0 1 2 3 4 5 6 7 8 9 Answer: CDACCCCBDC

Scramble Range: A- D

Monday, August 13, 2007 10:00:41 AM

129

QUESTIONS REPORT for HL-14 NRC RO DRAFT EXAM 50. 065AA2.08 001/1/1/LOSS AIR - FAILURES/C/A - 2.9/BANIQR/NRC RO/TNT / RLM Which ONE of the following valves will fail CLOSED on a loss of intrument air? A. Main Feed Pump Miniflow (FV-5201) B. Main Feed Regulating Valve (FV-540) C. RHR Heat Exchanger Outlet (HV-606) D. Charging Flow Control Valve (FV-121)

Monday, August 13, 2007 10:00:41 AM

130

QUESTIONS REPORT for HL-14 NRC RO DRAFT EXAM


Feedback

K/A 065 Loss of Instrument Air: AA2.08 Ability to determine and interpret the following as they apply to the Loss of Instrument Air. Failure modes of air-operated equipment. K/A MATCH ANALYSIS Question asks from a list of air operated valves which one will fail shut on a loss of air. ANSWER / DISTRACTOR ANALYSIS A. Incorrect. Main Feed Pump Mini Flow fails open on loss of air. B. Correct. Main Feed Regulating Valves fail shut on loss of air. C. Incorrect. RHR Heat Exchanger outlets fail open on loss of air. D. Incorrect. Charging flow control valve FV-121 fails open on loss of air. REFERENCES 18028-C, section B for Loss of Instrument Air while in Modes 3, 4, or 5 VEGP learning objectives: LO-LP-60321-02, State the fail position of the following valves on loss of instrument air. f. SGFP mini-flow valves g. FRV i . RHR heat exchanger outlet valve o .
No tes Response form

F V - 1 2 1

( C h a r g i n g

F l o w

C o n t r o l

V a l v e )

Answers MCS Time: 1 Points: 1.00 Version: 0 1 2 3 4 5 6 7 8 9 Answer: BACDADBBDB Scramble Range: A- D

Monday, August 13, 2007 10:00:41 AM

131

QUESTIONS REPORT for HL-14 NRC RO DRAFT EXAM 51. 068AK3.18 001/1/2/EVAC - EOP ACTIONS/MEM - 4.2/BANK/R/NRC RO/TNT / RLM Evacuation of the Control Room is required due to a Control Room fire. - Personnel safety is NOT in jeopardy. The actions of AOP-18038-1, "Operation From Remote Shutdown Panels", prior to evacuating the control room, include which ONE of the following? A. Trip all reactor coolant pumps (RCPs) to minimize heat input. B. Trip both main feedwater pumps (MFPTs) to limit RCS cooldown. C. Place the pressurizer (PZR) pressure control in MANUAL to prevent inadvertent heater and spray actuations. D. Ensure CCP suction aligned to the VCT with makeup in AUTO to ensure adequate NPSH to the charging pumps.

Monday, August 13, 2007 10:00:41 AM

132

QUESTIONS REPORT for HL-14 NRC RO DRAFT EXAM


Feedback

K/A 068 Control Room Evacuation AK3.18 Knowledge of the reasons for the following responses as they apply to the Control Room Evacuation. Actions contained in the EOP for control room evacuation emergency task. K/A MATCH ANALYSIS Question gives a plausible scenario where a fire requires evacuation of the control room but personnel safety is not jeopardized. The candidate must choose which control room action would be performed prior to leaving the control room. ANSWER / DISTRACTOR ANALYSIS A. Incorrect. Plausible since RCPs are tripped to minimize heatup during other procedures such as loss of heat sink. In this procedure they are tripped to prevent an inadvertent depressurization due to open spray valves. B. Correct. Both MFPTs are required to be tripped whether personnel safety is jeopardized or not and reason is to limit RCS cooldown. C. Incorrect. PORV block valves are shut prior to leaving control room eliminating the use of PORVs for pressure control. Placing PRZR pressure control in MANUAL is not directed per the AOP. D. Incorrect. Ensuring CCP suction aligned to VCT is incorrect, should be aligned to the RWST to ensure suction. REFERENCES Vogtle 2002 NRC RO Exam question # 66 AOP-18038-1, "Operation From Remote Shutdown Panels" steps 1 - 6 VEGP learning objectives: LO-LP-60328-07, Given the enter AOP, describe:

b .
Notes

h o w

a n d

w h y

e a c h

s t e p

i s

p e r f o r m e d .

Monday, August 13, 2007 10:00:41 AM

133

Response form

QUESTIONS REPORT for HL-14 NRC RO DRAFT EXAM


Answers MCS Time: 1 Points: 1.00 Version: 0 1 2 3 4 5 6 7 8 9 Answer: BBBBACADBB Scramble Range: A- D

Monday, August 13, 2007 10:00:41 AM

134

QUESTIONS REPORT for HL-14 NRC RO DRAFT EXAM 52. 073K4.01 001/2/1/PROC RAD-TERM RELEAS/C/A - 4.0/MODIFIED/R/NRC RO/TNT / RLM Given the following: Unit 2 refueling outage in progress with several systems draining in progress. The Auxiliary Building Clean Water Sump is being pumped to the Turbine Building Drain system to the Waste Water Retention Basins. A misalignment drains a radioactive system into the Aux. Building Clean Water Sump. A HIGH radiation alarm is received for RE-0848, "Turbine Building Drain" rad monitor. Which ONE of the following describes how the release would automatically terminate? A. B. C. D.
F e edback

The Aux. Building Clean Water sump pumps automatically trip. The Turbine Building Drain System re-aligns to the TB Dirty Drain Tank. The Turbine Building Drain System sump pumps automatically trip. The Aux. Building Clean Water sump pumps re-align to the Floor Drain Tank.

K/A 073 Process Radiation Monitoring (PRM) System: K4.01 Knowledge of PRM system design feature(s) and / or interlock(s) which provide for the following: Release termination when radiation exceeds setpoint. K/A MATCH ANALYSIS Question gives a plausible scenario during an outage with system draining in progress. A misalignment results in radioactive water drainage into a clean water system. The candidate must determine upon receipt of HIGH radiation how the release would be terminated. ANSWER / DISTRACTOR ANALYSIS A. Incorrect. Auxiliary Building Clean Water Sump pumps do not have a High radiation automatic trip function. Plausible candiate could think a clean water sump pump could have a high rad trip interlock. Some sumps such as FHB are interlocked with tank pumps to prevent inadvertent releases.
Monday, August 13, 2007 10:00:41 AM

135

QUESTIONS REPORT for HL-14 NRC RO DRAFT EXAM B. Correct. On High radiation the Turbine Building drain system isolates flow to the WWRBs and routes flow to the Dirty Drain Tank. C. Incorrect. Turbine Building Sump pumps do not have a High radiation automatic trip function. Plausible candiate could think the sump pumps could have a high rad trip interlock. D. Incorrect. Auxiliary building sump pumps do not auto re-align to the Floor Drain Tank, this requires manual valve manipulation. CW sumps can be manually re-aligned to the FDT on if high radiation is detected. Plausible candidate could choose this but release is diverted to the TB Dirty Drain Tank. REFERENCES HL-AU-32000-00-001 Vogtle LO Active Bank question. V-LO-PP-45311, Turbine Building Drain System slide # 15 V-LO-PP-45431, Non-Radioactive Drain System slides # 7 and 9. Annunciator response for ALB05 window CO3 for High Radiation page # 26 VEGP learning objectives: LO-PP-45311-02, Describe the automatic actions that occur when radiation levels exceed the setpoint of the turbine building radiation monitor.
Notes R e s p o nse form Answers MCS Time: 1 Points: 1.00 Version: 0 1 2 3 4 5 6 7 8 9 Answer: BACBDCCBDB Scramble Range: A- D

Monday, August 13, 2007 10:00:41 AM

136

QUESTIONS REPORT for HL-14 NRC RO DRAFT EXAM 53. 076AK2.01 001/1/2/HIGH ACT - PRO RAD/MEM - 2.6/BANK/R/NRC RO/TNT / RLM Which ONE of the following would provide the earliest indication of a gross fuel failure in the control room? A. RE-12442 Main Plant Vent Radiogas Hi Range monitor B. Reactor Coolant Dose Equivalent 1-131 sample C. RE-48000 CVCS Letdown radiation monitor D. Reactor Coolant E-Bar calculation

Monday, August 13, 2007 10:00:41 AM

137

QUESTIONS REPORT for HL-14 NRC RO DRAFT EXAM


Feedback

K/A 076 High Reactor Coolant Activity AK2.01 Knowledge of the interrelations between the High Reactor Coolant Activity and the following: Process radiation monitors. K/A MATCH ANALYSIS Question asks which indication would be the earliest indication of gross failed fuel to the control room operators. Candidiate must pick the correct choice from the list. ANSWER / DISTRACTOR ANALYSIS A. Incorrect. Plausible candidate may pick this since some accidents result in high radiation on the plant vent. Since this is high range, it is least likely for radiation to appear first. While the Plant Vent Radiogas may see some activity, an RCS leak would be required for it to detect gross failed fuel. B. Incorrect. This is a method of obtaining specific activity but it will not be the earliest. Plausible candidate may choose this if not aware of function of letdown rad monitor. C. Correct. This is the function of this radiation monitor and failed fuel is listed as a cause in the annunciator response procedures. D. Incorrect. This is a method that requires an RCS sample and a calculation. While failed fuel could be detected by this method, it would not be the earliest. Plausible candidate could choose this if unaware of function of the letdown rad monitor. REFERENCES VC Summer April 2004 NRC RO Exam question # 53 Annunciator response 17100 for RE-48000 (page 73) VEGP learning objectives:

Notes

Monday, August 13, 2007 10:00:41 AM

138

Response form

QUESTIONS REPORT for HL-14 NRC RO DRAFT EXAM


Answers MCS Time: 1 Points: 1.00 Version: 0 1 2 3 4 5 6 7 8 9 Answer: CDDCBDDAAD Scramble Range: A- D

Monday, August 13, 2007 10:00:41 AM

139

QUESTIONS REPORT for HL-14 NRC RO DRAFT EXAM 54. 076G2.1.2 001/2/1/SERV H20- OPER RESP/MEM - 3.0/NEW/R/NRC RO/TNT / RLM Given the following: Train "A" NSCW is Danger Tagged out under a clearance for maintenance. A catastrophic leak occurs on NSCW Train "B". The crew enters 18021-C, "Loss of Nuclear Service Cooling Water". Which ONE of the following is the first action required by the operating crew? A. Place Train "B" NSCW in single pump operation and initiate a unit shutdown. B. Place all 3 Train "B" NSCW pumps in pull-to-lock (PTL). C. Depress both DG1B QEAB "Emergency Stop" pushbuttons to disable DG1B. D. Place Train "B" equipment cooled by NSCW into PTL for equipment protection.

Monday, August 13, 2007 10:00:41 AM

140

QUESTIONS REPORT for HL-14 NRC RO DRAFT EXAM


F e edback

K/A 076 Service Water System (SWS): G2.1.2 Knowledge of operator responsibilities during all modes of plant operation. K/A MATCH ANALYSIS Question gives a plausible scenario with one train of NSCW tagged out and a catastrophic leak occurs on the other train. The candidate must determine the first course of action he must take once AOP-18021-C for Loss of NSCW is entered. ANSWER / DISTRACTOR ANALYSIS A. Incorrect. Plausible the candidate may think single pump operation would be performed but this is only done after a reactor trip has been performed. A controlled unit shutdown would not be performed. Also, not the first action. B. Correct. Placing the pumps into PTL is the first action required on a leak. C. Incorrect. Plausible the candidate may think Emergency Trip of DG1B is the next action to perform, it is the 2nd action after placing the pumps into PTL. D. Incorrect. Plausible the candidate may think placing running components into PTL for equipment protection but this is directed later in the procedure. REFERENCES 18021-C, "Loss of Nuclear Service Cooling Water" VEGP learning objectives: LO-LP-60317-04, Given the entire AOP, describe:

b . H ow
No tes Response form Answers MCS Time: 1 Points: 1.00 Version: 0 1 2 3 4 5 6 7 8 9 Answer: BDCBDDCB AD Scramble Range: A- D

and

wh y

the

step

is

being

perf ormed.

Monday, August 13, 2007 10:00:41 AM

141

QUESTIONS REPORT for HL-14 NRC RO DRAFT EXAM 55.


076K4.02 001/2/1/SERV H20-AUTO STARTS/C/A - 2.9/BANK/R/NRC RO/TNT / RLM

Given the following: Unit 1 is at 100% power. NSCW pumps 1, 2, 5, and 6 are in service. NSCW pump 3 and 4 handswitches are in AUTO. A fault on RAT 1A result in an LOSP to 1AA02. DG1A starts and re-energizes 1AA02. Two minutes later an SI signal occurs due to low PRZR pressure. Which ONE of the following is the CORRECT response of the NSCW pumps? A. All six NSCW pumps operating. B. Only two NSCW pumps operating per train. C. Two Train A NSCW pumps operating, three Train B NSCW pumps operating. D. Three Train A NSCW pumps operating, two Train B NSCW pumps operating.

Monday, August 13, 2007 10:00:41 AM

142

QUESTIONS REPORT for HL-14 NRC RO DRAFT EXAM


F e edback

K/A 076 Service Water System (SWS) K4.02 Knowledge of the SWS design feature(s) and / or interlocks which provide for the following: Automatic start features associated with SWS pump controls. K/A MATCH ANALYSIS Question gives a plausible scenario with an NSCW pump alignment given followed by a safety injection and LOSP. Candidate must determine the correct status of the NSCW pumps which were in AUTO standby. ANSWER / DISTRACTOR ANALYSIS A. Incorrect, plausible the candidate may think the SI starts NSCW pumps 3 and 4 but forgets the load shed on train A would only re-start 2 Train A pumps. B. Incorrect, plausible the candidate may confuse the load shed and SI sequence and think only 2 pumps started by the SI in each train and forget the Train B standby pump would still be running after SI. C. Correct. Two train A pumps start after the load shed, the 3rd pump on Train B starts on SI and does not load shed. D. Incorrect, Plausible the candidate may invert the sequence of events and get the status of the systems backwards. REFERENCES LO-PP-06101-09-003 Vogtle LO Active Exam Bank VEGP learning objectives: L O P P -06101-09, Describe the NSCW system response to an SI or LOSP signal.
Notes Response form Answers

MCS

Time: 1

Points:

1.00

Version: 0 1 2 3 4 5 6 7 8 9 Answer: CBABDDCAAC

Scramble Range: A- D

Monday, August 13, 2007 10:00:42 AM

143

QUESTIONS REPORT for HL-14 NRC RO DRAFT EXAM 56. 078K2.01 001/2/1/IAS-AIR COMPRESSORS/C/A - 2.7/NEW/R/NRC RO/TNT / RLM Given the following conditions: Unit 1 is at 100% power. RATs 1A and 1B are de-energized due to an LOSP. Both DGs start and re-energize 1AA02 and 1BA03. A reactor trip has NOT occurred. Swing air compressor (AC) # 4 was aligned to Unit 1 Which ONE of the following is CORRECT regarding the Swing AC # 4 power supply ? A. AC # 4 was de-energized and remains de-energized. B. AC # 4 was briefly de-energized and then re-energized. C. AC # 4 is energized and will be de-energized if a Unit 1 reactor trip occurs. D. AC # 4 is energized and will remain energized if a Unit 1 reactor trip occurs.

Monday, August 13, 2007 10:00:42 AM

144

QUESTIONS REPORT for HL-14 NRC RO DRAFT EXAM


Feedback

K/A 078 Instrument Air System (IAS): K2.01 Knowledge of power supplies to the following: Instrument Air Compressor. K/A MATCH ANALYSIS Question gives a plausible scenario where an LOSP occurs with the DGs reenergizing the 4160 1 E buses AA02 and BA03. The candidate has to pick the correct status of Swing AC # 4 power supply. ANSWER / DISTRACTOR ANALYSIS A. Incorrect. The swing AC # 4 is powered from 1NB12 which is powered ultimately from the "B" UAT while at power via 1NA04. Plausible the candidate may think that 1NA04 power is lost on the LOSP and think the AC # 4 is de-energized. RAT 1B would normally power 1NA04 - 1NB12 during plant shutdown and low power conditions. B. Incorrect. This would imply the swing AC # 4 is powered from a switchgear from the stub bus and would have reloaded on the LOSP sequence. Plausible the candidate could think the swing AC # 4 power is from a stub bus switchgear. C. Correct. If a reactor trip occurs, power will swap from the UATs to the RATs which are currently de-energized resulting is a loss of power to AC # 4. D. Incorrect. If a reactor trip occurs, power will swap from the UATs to the RATs which are currently de-energized resulting is a loss of power to AC # 4. Plausible the candidate could think AC # 4 is energized from stub bus and will not de-energize on reactor trip. Also, candidate may not recall swap to UATs on a reactor trip. REFERENCES Simulator HS for AC # 4 and QEAB electrical boards. LO PP-01101, "Electrical Distribution" slide # 20 LO-PP-02101, "Service and Instrument Air", slide # 9 LO -TX-02101, Vogtle Text for Service and Instrument Air, page # 5 VEGP learning objectives: Not applicable

Monday, August 13, 2007 10:00:42 AM

145

QUESTIONS REPORT for HL-14 NRC RO DRAF T EXAM


N otes Response form Answers MCS Time: 1 Points: 1.00 Version: 0 1 2 3 4 5 6 7 8 9 Answer: CABBAADCAA Scramble Range: A- D

Monday, August 13, 2007 10:00:42 AM

146

QUESTIONS REPORT for HL-14 NRC RO DRAFT EXAM 57.


086K4.01 001/2/2/FIRE PROT-H20 SUPPLY/MEM - 3.1/BANK/R/NRC RO/TNT / RLM

The Fire Protection System is in normal alignment with virtually no system leakage. The Pre-action sprinkler system has actuated at the Diesel Generator Building on Unit 1. The Fire Water header system pressure has LOWERED to 88 psig. Which ONE of the following describes the AUTOMATIC response of the Fire Protection System to ensure adequate system pressure is maintained? A. ONLY the Electric Fire Water Pump STARTS. B. ONLY the Standby Jockey Water Pump STARTS. C. The Electric Fire Pump and Diesel Fire Pump # 1 both START. D. The Standby Jockey Water Pump and the Electric Fire Pump both START.

Monday, August 13, 2007 10:00:42 AM

147

QUESTIONS REPORT for HL-14 NRC RO DRAFT EXAM


F e edback

K/A 086 Fire Protection System (FPS) K4.01 Knowledge of design feature(s) and / or interlock(s) which provide for the following: Adequate supply of water for FPS. K/A MATCH ANALYSIS Question gives a plausible scenario with a Pre-action sprinkler system actuation with a corresponding drop in fire header pressure. Candidate must determine the proper fire protection system response to maintain adequate fire water supply. ANSWER / DISTRACTOR ANALYSIS A. Incorrect. Electric Fire Pump will start with system pressure dropping below 110 psig but so would the Diesel Fire Pump # 1. B. Incorrect. The Standby Jockey pumps is normally turned OFF. Plausible candidate may think standby jockey pump starts and is sufficient to maintain header pressure. C. Correct. The Electric Fire Pump would start at this pressure and the Diesel Fire Pump # 1 would start. D. Incorrect. The Electric Fire Pump would start at this pressure but the Standby Jockey Pump is normally OFF and would not have started. REFERENCES Vogtle LO Active Exam bank question # V-LO-LP-43101-05-001. Callaway February 2004 NRC RO Exam question # 65. SOP-13903-C, "Fire Protection System Operation" LO-PP-43101, "Fire Protection System" slides # 7, 8 and 23.

VEGP learning objectives:


LO-PP-43101-04, Discuss normal operation of system jockey pumps including where they are operated from. LO-PP-43101-05, Discuss the system response to an auto sprinkler actuation or fire hose operation which lowers system header pressure.
Monday, August 13, 2007 10:00:42 AM

148

QUESTIONS REPORT for HL-14 NRC RO DRAFT EXAM


N o t e s R esponse form Answers MCS Time: 1

Points:

1.00

Version: 0 1 2 3 4 5 6 7 8 9 Answer: CDB A AB D B AD

Scramble Range: A- D

Monday, August 13, 2007 10:00:42 AM

149

QUESTIONS REPORT for HL-14 NRC RO DRAFT EXAM 58.


103A1.01 001/2/1/CNMT - PRESS TEMP/C/A - 3.7/BANKIR/NRC RO/TNT / RLM

Given the following plant conditions: - The plant is in Mode 1 with all systems in normal alignment. Containment pressure is 1.9 psig and slowly rising due to a small air leak. Containment temperature is 122 degrees F and slowly rising. 4 containment coolers are currently running. Which ONE of the following is CORRECT regarding containment pressure and temperature and the actions to take to comply with Tech Specs? A. Pressure is within Tech Spec limits, use mini-purge to allow containment to vent. B. Pressure exceeds Tech Spec limits, use mini-purge to allow containment to vent. C. Temperature is within Tech Spec limits, start additional coolers in slow speed. D. Temperature exceeds Tech Spec limits, start additional coolers in slow speed.

Monday, August 13, 2007 10:00:42 AM

150

QUESTIONS REPORT for HL-14 NRC RO DRAFT EXAM


Feedback

K/A 103 Containment System: A1.01 Ability to predict and / or monitor changes in parameters (to prevent exceeding design limits) associated with operating the containment system controls including: Containment pressure, temperature, and humidity. K/A MATCH ANALYSIS Question gives a plausible scenario with containment parameters out of Tech Spec limits or approaching Tech Spec limits. Candidate must choose whether in limits and the correct action to take. ANSWER / DISTRACTOR ANALYSIS A. Incorrect. Containment pressure out of limits, mini-purge would be a correct action. B. Correct. Containment pressure exceeds limits and mini-purge is correct action. C. Incorrect. Temperature exceeds limits and coolers should be started high speed. D. Incorrect. Temperature exceeds limits but coolers should be started high speed. REFERENCES Technical Specifications 3.6.4 for Containment Pressure Technical Specifications 3.6.5 for Containment Air Temperature. ARP-17001 window E06 directs start of additional coolers. SOP-13120-1/2, Containment Building Cooling Systems normally has coolers running in high speed except during LOCA events. VEGP learning objectives: LO-PP-29101-08, Describe routine actions taken to adjust Containment pressure and temperature.
Notes Response form
Monday, August 13, 2007 10:00:42 AM

151

QUESTIONS REPORT for HL-14 NRC RO DRAFT EXAM


Answers MCS Time: 1 Points: 1.00 Version: 0 1 2 3 4 5 6 7 8 9 Answer: B C C DB CDB CD Scramble Range: A- D

Monday, August 13, 2007 10:00:42 AM

152

QUESTIONS REPORT for HL-14 NRC RO DRAFT EXAM 59. G2.1.31 001/1/2/CR HS-PROPER LU/MEM - 4.2/NEW/R/NRC RO/TNT / RLM Given the following: - Unit 1 is at 100% power with all systems in normal alignment. Which ONE of the following would provide a method for verifying accumulator valve positions in the main control room? A. Using the QMCB handswitches. B. Using the QMCB monitor light boards (MLBs). C. Using the PSMS - Plant Safety Monitoring System. D. Using the ACCUM TANK VALVE FULL OPEN annunciator

Monday, August 13, 2007 10:00:42 AM

153

QUESTIONS REPORT for HL-14 NRC RO DRAFT EXAM


F e edback

K/A G2.1.31 Ability to locate control room switches, controls and indications and to determine that they are correctly reflecting the desired plant lineup. K/A MATCH ANALYSIS Question gives a plausible scenario where the RO is performing the Daily Tech Spec rounds and need to verify Accumulator Isolation Valve position. The candidate has to choose the method required SR 3.5.1.1 as specified in the surveillance. ANSWER / DISTRACTOR ANALYSIS A. Incorrect. Plausible the candidate could consider the isolation valve handswitches, however, these are normally de-energized at power once the valves are open. B. Correct. These are the indicators specified by the CR Tech Spec rounds. C. Incorrect. Plausible the candidate could consider the PLASMA display but this panel is for PAM system and does not have accumulator valve positions. D. Incorrect. Plausible candidate could confuse the alarm which is for ACCUM VALVE NOT FULL OPEN versus ACCUM VALVE FULL OPEN. REFERENCES Tech Spec 3.5.1 and Bases for ECCS Accumulators OSP-14000-1, "Operations Shift and Daily Surveillance Logs" (CR Tech Spec Rounds) VEGP learning objectives: LO-LP-39209-02, Given a set of Tech Specs and the bases, determine for a set of plant conditions, equipment availability, and operational mode: a. Whether any Tech Spec LCOs of section 3.5 are exceeded. L O L P -39209-03, Describe the bases for any given Tech Spec in section 3.5
No tes Response form Answers

MCS

Time: 1

Points:

1.00

Version: 0 1 2 3 4 5 6 7 8 9 Answer: BBCCDADCCB

Scramble Range: A- D

Monday, August 13, 2007 10:00:42 AM

154

QUESTIONS REPORT for HL-14 NRC RO DRAFT EXAM 60. G2.1.9 001/1/2/DIRECT ACTIVITY CR/MEM - 2.5/NEW/R/NRC RO/TNT / RLM Given the following conditions: - A prolonged loss of All AC power has occurred, 19100-C, "Loss of All AC Power" is in effect. Power is restored to an emergency bus and the crew is preparing to transition to 19102-C, "Loss of All AC Power Recovery With SI Required". The person monitoring the CSFSTs points out an ORANGE path on CORE COOLING exists. The SS announces entry into FRP 19222-C, "Response to Degraded Core Cooling". Which ONE of the following conditions would require the RO to dispute this course of action by the SS (I.E., prevents transition to the ORANGE path)? A. Transition should not be made unless a RED path FRP exists during performance of the Loss of All AC series of procedures. B. Transition should not be made until the 19102-C attachment for performing the first 16 steps of E-0 has been completed. C. Transition should not be made until completion of 19102-C directs return to another optimal recovery procedure. D. Transition should not be made until the step where 19102-C directs implementation of the FRPs is appropriate.

Monday, August 13, 2007 10:00:42 AM

155

QUESTIONS REPORT for HL-14 NRC RO DRAFT EXAM


F e edback

K/A G2.1.9 Ability to direct personnel activities from inside the control room. K/A MATCH ANALYSIS Question gives a plausible scenario with a loss of all AC power in progress and a transition from 19100 when power is recovered. The candidate must determine the correct reason a transition to a CSFST ORANGE path procedure should not be made. In order to correctly direct control room activities, the operator must posses the knowledge of what procedure needs to be utilized. This question tests the knowledge of when FRP procedures should be implemented following a Loss of All AC power. ROs are required to know when FRP procedures should be implemented and entered. ANSWER / DISTRACTOR ANALYSIS A. Incorrect. Plausible since that candidate may know a special rule for implementation exists and think transition only on a RED path is required. B. Incorrect. Plausble since several FRPs such as ATWT and LOHS direct the performance of the first 16 steps of E-0. However, 19102-C has no such attachment to perform this action which also would not prohibit a transition to another FRP if appropriate. C. Incorrect. Plausible since normally transitions to other FRPs can occur when transitioning out of an FRP to another procedure. However, in this instance 19102-C states FRPs are not to be implemented until later in the procedure which would come before a transition to another FRP. D. Correct. FRPs are not to be implemented until step 16 of 19102-C.. REFERENCES 19100-C, "Loss of All AC Power. 19102-C, "Loss of All AC Power Recovery With SI Required. Vogtle May 2005 NRC RO Exam question # 68 VEGP learning objectives: LO-LP-37031-09, Given a NOTE or CAUTION statement from the EOP, state the bases for that NOTE or CAUTION statement. LO-LP-37033-03, State when the CSFSTs should be monitored / implemented during 19102-C.
Monday, August 13, 2007 10:00:42 AM

156

QUESTIONS REPORT for HL-14 NRC RO DRAFT EXAM


N o t e s

Response form Answers MCS Time: 1

Points:

1.00

Version: 0 1 2 3 4 5 6 7 8 9 Answer: DAAABBABDC

Scramble Range: A- D

Monday, August 13, 2007 10:00:42 AM

157

QUESTIONS REPORT for HL-14 NRC RO DRAFT EXAM 61. G2.2.13 001/1/2/C & T PROCEDURES/MEM - 3.6/BANK/R/NRC RO/TNT / RLM Which ONE of the following is CORRECT regarding an OPERATING PERMIT TAG present on a component per NMP-AD-003 "Equipment Clearance and Tagging"? A. Component position can NOT be changed - The equipment is isolated for maintenance or testing. B. Component position can NOT be changed - The equipment is operating for maintenance or testing. C. Component position can be changed - The equipment is under the control of a Tagout Holder for the purpose of position alignment, testing, or maintenance. D. Component position can be changed - The equipment has been Danger Tagged but can be manipulated for the purpose of alignment, testing, or maintenance.

Monday, August 13, 2007 10:00:43 AM

158

QUESTIONS REPORT for HL-14 NRC RO DRAFT EXAM


Feedback

K/A G2.2.13 Knowledge of tagging and clearance procedures. K/A MATCH ANALYSIS Question asks whether or not an OPERATING PERMIT TAG may be manipulated. ANSWER / DISTRACTOR ANALYSIS A. Incorrect. Plausible the candidate may think the operating permit is providing an isolation point during maintenance or testing. B. Incorrect. Plausible the candidate may think the operating permit prevents manipulation while in operation for maintenance or testing. C. Correct. The Tagout Holder may position the component as required for maintenance or testing. D. Incorrect. Plausible the candidate may think the operating permit allows manipulation while in operation for maintenance or testing, however, the procedure specifically prohibits co-existence of Danger Tags and Operating Permit on the components at the same time. REFERENCES NMP-AD-003, "Equipment Clearance and Tagging" page # 15 Farley August 2004 NRC RO Exam question # 63 VEGP learning objectives: L O L P 63304-01, Define the following terms; O p e r a t i n g
Notes Response form Answers

P e r m i t

T a g

MCS

Time: 1

Points:

1.00

Version: 0 1 2 3 4 5 6 7 8 9 Answer: CB CDB CCDAA

Scramble Range: A- D

Monday, August 13, 2007 10:00:43 AM

159

QUESTIONS REPORT for HL-14 NRC RO DRAFT EXAM 62. G2.2.22 001/1/2/LCO AND SAFETY LIMIT/C/A - 3.4/BANKJR/NRC RO/TNT / RLM Which ONE of the following sets of conditions represents a violation of a Technical Specification Safety Limit and the required action? (REFERENCE PROVIDED) A. Power = 10%, Pressure = 2400 psig, Tavg = 655 degrees F Restore to within limits OR be in Mode 3 within 1 hour. B. Power = 80%, Pressure = 2250 psig, Tavg = 640 degrees F Restore to within limits AND be in Mode 3 within 1 hour. C. Power = 10%, Pressure = 2400 psig, Tavg = 655 degrees F Restore to within limits in 2 hours OR be in Mode 3 in 6 hours. D. Power = 80%, Pressure = 2250 psig, Tavg = 640 degrees F Restore to within limits in 2 hours OR be in Mode 3 in 2 hours.

Monday, August 13, 2007 10:00:43 AM

160

QUESTIONS REPORT for HL-14 NRC RO DRAFT EXAM


Feedback

K/A G2.2.22 Knowledge of limiting conditions for operations and safety limits. K/A MATCH ANALYSIS Question asks a straight forward which condition violates Tech Spec Safety Limits and the required action. ANSWER / DISTRACTOR ANALYSIS A. Incorrect. Within limits, Plausible candidate may think a parameter out of limits. Tech Spec action would be incorrect also, restore AND be in Mode 3. B. Correct. Out of limits and the action is correct per Tech Specs. C. Incorrect. Within limits, plausible candidate may think a parameter out of limits. Tech Spec action is incorrect also, this is the action for the DNB Parameters spec. D. Incorrect. Out of limits but the action is incorrect per Tech Specs. Plausible candidate may confuse with the DNB Parameters Tech Spec actions. REFERENCES NOTE: A reference is to be provided of the RCS Core Safety Limit Figure 2.1.1-1. Vogtle Tech Specs for Reactor Core Safety Limit 2.1.1 Vogtle 2002 NRC RO Examination question # 84. VEGP learning objectives: L O L P -39203-02, Given the Safety Limit curve and appropriate plant parameter values, determine if the Safety Limits are being exceeded.
Notes Response form Answers MCS Time: 1 Points: 1.00 Version: 0 1 2 3 4 5 6 7 8 9 Answer: BBABACBBCB Scramble Range: A- D

Monday, August 13, 2007 10:00:43 AM

161

QUESTIONS REPORT for HL-14 NRC RO DRAFT EXAM 63. G2.2.27 001/1/2/REFUELING PROCESS/MEM - 2.6/MODIFIED/R/NRC RO/TNT / RLM Given the following conditions: Unit 1 refueling outage in progress. The SRO Fuel Handling Supervisor (FHS) and refueling crew are in containment. Control rod latching and verification is in progress. The RO notices a significant increase in Source Range (SR) counts during a 10 foot control rod withdrawal for latching verification. Which ONE of the following would be the CORRECT action for the RO to take? A. Allow rod withdrawal to continue, this is an expected response. B. Suspend refueling operations on his own authority due to the SR indications. C. Allow rod withdrawal to continue, the RO should suspend the operation only if the SR Hi Flux at Shutdown Alarm is received. D. Notify the FHS and SS of the SR count indications, FHS or SS authority is required to suspend control rod withdrawal during latching.

Monday, August 13, 2007 10:00:43 AM

162

QUESTIONS REPORT for HL-14 NRC RO DRAFT EXAM


Feedback

K/A G2.2.27 Knowledge of the refueling process. K/A MATCH ANALYSIS Question gives a plausible scenario during a refueling evolution, control rod latching and verification. A significant increase in SR counts occurs during rod withdrawal. The candidate must determine the correct action for the RO to perform. ANSWER / DISTRACTOR ANALYSIS A. Incorrect. A significant increase in SR counts is not expected during this evolution. The RO should suspend the evolution. Plausible the candidate may expect the SR counts to increase significantly during this evolution. B. Correct. The RO should suspend the evolution on his own authority as stated in 10000-C, Conduct of Refueling Operations. C. Incorrect. Plausible the candidate would consider suspension at the SR Hi Flux at Shutdown alarm. However, he should not wait until SR counts increase to this level to suspend the evolution at a point requiring Containment Evacuation. D. Incorrect. Plausible the candidate may consider the notification of the FHS and SS to be appropriate, however, they are not the only ones with authority to suspend the control rod latching evolution. The RO has this authority level. REFERENCES 10000-C, "Conduct of Refueling Operations" section 4.4 for Reactor Operator Responsibilities and Precaution and Limitation 3.1 for Personnel with Authority to Suspend Refueling Operations LO-PP-25101-19-003 Vogtle LO Active Exam Bank VEGP learning objectives:

LO-PP-25101-19, State which members of the refueling team have the authority to suspend refueling operations.
Notes Response form

Answers MCS Time: 1 Points: 1.00 Version: 0 1 2 3 4 5 6 7 8 9 Answer: BBBDCBB CAC Scramble Range: A- D

Monday, August 13, 2007 10:00:43 AM

163

QUESTIONS REPORT for HL-14 NRC RO DRAFT EXAM 64. G2.3.1 001/1/2/10CFR20 RAD CONTROL/C/A - 2.6/BANK/R/NRC RO/TNT / RLM An individual received the following radiation dose at Plant Vogtle during the first half of the calendar year: Committed Dose Equivalent (CDE): Deep Dose Equivalent (DDE): 85 mrem 55 mrem

Shallow Dose Equivalent (SDE), skin of Whole Body: 75 mrem Committed Effective Dose Equivalent (CEDE): 55 mrem

Which ONE of the following CORRECTLY states the individual's total whole body occupational dose for the first half of the calendar year? A. 110 mrem B. 130 mrem C. 140 mrem D. 195 mrem

Monday, August 13, 2007 10:00:43 AM

164

QUESTIONS REPORT for HL14 NRC RO


Feedback

DRAFT EXAM K/A G2.3.1 Radiation Control - Knowledge of 10CFR20 and related facility radiation control requirements. K/A MATCH ANALYSIS Question requires applicant to calculate occupational whole body dose using knowledge of internal and external dose terminology found in 10CFR20. ANSWER / DISTRACTOR ANALYSIS A. Correct. Whole Body (TEDE) = DDE + CEDE = 55 mrem + 55 mrem = 110 mrem. B. Incorrect. Plausible, 130 mrem = 55 mrem (DDE or CEDE) + 75 mrem (SDE). C. Incorrect. Plausible, 140 mrem = 55 mrem (DDE or CEDE) + 85 mrem (CDE). D. Incorrect. Plausible, 195 mrem = 110 mrem (DDE + CEDE) + 85 mrem (CDE). REFERENCES Watts Bar September 2006 NRC RO Exam question # 72. VEGP learning objectives: LO-LP-63920-07, State the VEGP Administrative Exposure Guidelines for any or all of the following workers:

a. Annual limits for TEDE, TODE, LDE, SDE, ME, exposure to the whole body and extremities.
Notes Response form Answers MCS Time: 1 Points: 1.00 Version: 0 1 2 3 4 5 6 7 8 9 Answer: ABCDBBBBCC Scramble Range: A- D

Monday, August 13, 2007 10:00:43 AM

165

QUESTIONS REPORT for HL-14 NRC RO DRAFT EXAM 65.


G2.3.2 001/1/2/ALARA PROGRAM/C/A - 2.5/MODIFIED/R/NRC RO/TNT / RLM

A job must be performed under the following conditions: Dose rate at the job is 80 mrem / hr. Airborne Radioactivity Area due to noble gasses present. Total Internal dose for the job if respirator is worn is 0 mrem. Total Internal dose for the job if no respirator is worn is 72 mrem. Time to complete job while wearing a respirator is 3.25 hours. Time to complete job without wearing a respirator is 2.25 hours. Which ONE of the following describes whether a respirator will be worn, and why? A. No, wearing a respirator will raise total exposure. B. Yes, wearing a respirator will lower total exposure. C. No, wearing a respirator will result in the same exposure. D. Yes, respirators are required anytime airborne radiation is present.

Monday, August 13, 2007 10:00:43 AM

166

QUESTIONS REPORT for HL-14 NRC RO DRAFT EXAM


Feedback

K/A G2.3.2 Knowledge of facility ALARA program. K/A MATCH ANALYSIS Question gives a plausible scenario where a job will be performed at a given dose rate with airborne particulates present. Candidate must determine whether a respirator should be worn and why. (no, raises total dose received) ANSWER / DISTRACTOR ANALYSIS A. Correct. 252 total mrem without respirator, 260 total mrem with respirator. B. Incorrect. 252 total mrem without respirator, 260 total mrem with respirator. C. Incorrect. 252 total mrem without respirator, 260 total mrem with respirator. D. Incorrect. Plausible that candidate may think respirators required anytime airborne radiation is present. REFERENCES Wolf Creek 2006 NRC RO Exam question # 72 Harris February 2006 NRC RO Exam question # 72 V. C. Summer October 2006 NRC RO Exam question # 95 NOTE: This was the exact same question at all three of these plants. 00910-C, VEGP ALARA Program This question was significantly modified from the 3 NRC exam questions above to make the answer different. Choices "C" and "D" were also modified to make different. VEGP learning objectives: LO-LP-63930-03, Given a set of conditions involving radiation and/or contamination control, state the applicable personnel responsibilities, according to procedure 00930.
Notes

Response form

Monday, August 13, 2007 10:00:43 AM

167

QUESTIONS REPORT for HL-14 NRC RO DRAFT EXAM


A n swers MCS Time: 1 Points: 1.00 Version: 0 1 2 3 4 5 6 7 8 9 Answer: ADCBBAB AAA Scramble Range: A- D

Monday, August 13, 2007 10:00:43 AM

168

QUESTIONS REPORT for HL-14 NRC RO DRAFT EXAM 66. G2.4.11 001/1/2/ABNORMAL OPER PROCS/C/A - 3.4/BANK/R/NRC RO/TNT / RLM Given the following plant conditions: - Charging flow at 100 gpm Letdown flow at 75 gpm Total seal leakoff flow at 12 gpm PRZR level and pressure are stable. 2RE-0724 "rate of change" (ROC) has remained stable at 75 gpd / hr for 21 minutes. 2RE-12839, 2RE-0019, and 2RE-0021 exhibit an increasing trend of higher than normal readings. The crew has entered AOP-18009-C, "Steam Generator Tube Leakage" Based on the above indications, which ONE of the following would be the CORRECT response for the crew to take? A. Shutdown the plant to Mode 3 within one (1) hour using AOP-18013-C, Rapid Power Reduction". B. Continue plant operations and monitor the SG tube leak for increasing indications above the administrative limits. C. Shutdown the plant to Mode 3 within six (6) hours using UOP-12004, "Power Operation (Mode 1) section 4.2 for Power Descent. D. Shutdown the plant to < 50% power within one (1) hour using AOP-18013-C, Rapid Power Reduction", be in Mode 3 within the next two (2) hours.

Monday, August 13, 2007 10:00:43 AM

169

QUESTIONS REPORT for HL-14 NRC RO DRAFT EXAM


Feedback

K/A G2.4.11 Knowledge of abnormal condition procedures. K/A MATCH ANALYSIS Question gives a plausible scenario with a SGTL and the candidate must choose whether to continue plant operations or shutdown the plant with time limits. Candidiate will have to perform a rough flow balance to determine shutdown time frame. ANSWER / DISTRACTOR ANALYSIS A. Correct. This is RNO actions for step # 5 RNO for leakage greater than 5 gpm. B. Incorrect. Plausible actions if candidate does not recognize limits are exceeded. C. Incorrect. Plausible, actions for step 8 RNO if candidate determines leakage > 150 gpd but doesn't recall the 5 gpm 1 hour shutdown limit. D. Incorrect. Plausible, actions for step 7 RNO if candidate determines leakage > 75 gpm but doesn't recall the 5 gpm 1 hour shutdown limit. REFERENCES Vogtle 2002 NRC RO Exam question # 90 LO-LP-60309-05-005 Vogtle LO Active Exam Bank Pages 6 and 7 of 18009-C, "Steam Generator Tube Leakage" VEGP learning objectives: LO-LP-60309-12, Discuss the major actions taken for a leaking SG for a leak exceeding shutdown requirements.
Notes

Response form Answers MCS Time: 1 Points: 1.00 Version: 0 1 2 3 4 5 6 7 8 9 Answer: AC B CDCDB AB Scramble Range: A- D

Monday, August 13, 2007 10:00:43 AM

170

QUESTIONS REPORT for HL-14 NRC RO DRAFT EXAM 67.


G2.4.23 001/1/2/EOP PRIORITY BASES/MEM - 2.8/BANK/R/NRC RO/TNT / RLM

Given the following conditions: Large break LOCA has resulted in Unit 1 Reactor trip and Safety Injection. RWST level is 36% and lowering. The crew is performing the actions of 19013-C, "Transfer to Cold Leg Recirculation". Critical Safety Function Status Trees (CSFSTs) are as follows: Subcriticality - Green Core Cooling - Orange Heat Sink - Yellow RCS Integrity - Yellow Containment - Red Inventory - Yellow Which ONE of the following is the CORRECT crew response and why? A. Immediately exit 19013-C and transition to the RED path for Containment to verify containment isolation and heat removal. B. Immediately exit 19013-C and transition to the ORANGE path for Core Cooling to verify Safety Injection is established to the RCS. C. Simultaneously perform 19013-C and the RED path for Containment to ensure suction to ECCS pumps and to verify containment isolation and heat removal. D. Remain in 19013-C and establish a flow path from CNMT sumps to RCS Cold Legs to ensure suction to ECCS pumps and coolant flow to the core is maintained.

Monday, August 13, 2007 10:00:43 AM

171

QUESTIONS REPORT for HL14 NRC RO


Feedback

DRAFT EXAM K/A G2.4.23 Knowledge of the bases for prioritizing emergency procedure implementation during emergency operations. K/A MATCH ANALYSIS Question gives a plausible scenario where 19013-C for Transfer to Cold Leg Recirc is in progress when either an ORANGE or RED path CSFST is received for ICC. Candidate must determine the correct implementation of the procedures and bases. ANSWER / DISTRACTOR ANALYSIS A. Incorrect. Plausible the candidate could consider immediate transition to the RED path FRP to be appropriate. B. Incorrect. Plausible the candidate could consider concurrent implemetation of an ORANGE path FRP not as important as RED and needs to perform the actions of both procedures. C. Incorrect. Plausible the candidate could consider immediate transition to either the RED or ORANGE FRP to be appropriate. D. Correct. Per notes, cautions, continuous actions in various EOPs and FRPs, the operators should remain in or transition to 19013-C to perform CL recirc actions. REFERENCES 19013-C, "Transfer to Cold Leg Recirculation" NOTE on page 4. LO-LP-37114-05-001 Vogtle LO Active Exam Bank. VEGP learning objectives:

LO-LP-37113-05, Given a NOTE or CAUTION statement from the EOP, state the bases for that NOTE or CAUTION statement.
Notes Response form Answers

MCS

Time: 1

Points:

1.00

Version: 0 1 2 3 4 5 6 7 8 9 Answer: DADDBCBBDA

Scramble Range: A- D

Monday, August 13, 2007 10:00:43 AM

172

QUESTIONS REPORT for HL-14 NRC RO DRAFT EXAM 68. G2.4.48 001/1/2/SYS STATUS INDICATIO/C/A - 3.5/BANK/R/NRC RO/TNT / RLM Given the following: Unit 1 is at 100% power. Annunciator ALB13 window DO1 for STM GEN 1 HI / LO LVL DEVIATION alarm is illuminated. Only S/G # 1 level is rising. Both MFPTs speeds are rising. Which ONE of the following describes the (1) cause, (2) required action, (3) direct consequence of an operator failing to take action? A. (1) # 1 S/G FRV is opening, (2) stabilize # 1 S/G at the new level, (3) Turbine Runback Initiated. B. (1) # 1 S/G FRV is opening, (2) return # 1 S/G level to program, (3) Feedwater Isolation Initiated. C. (1) MFP master controller failing high, (2) control MFP speed in manual, (3) Auto Turbine Trip initiated. D. (1) MFP master controller failing high, (2) manually trip the turbine, (3) Feedwater Isolation initiated.

Monday, August 13, 2007 10:00:44 AM

173

QUESTIONS REPORT for HL-14 NRC RO DRAFT EXAM


Feedback

K/ A G2.4.48 Ability to interpret control room indications to verify the status and operation of system, and understand how operator actions and directives affect plant and system conditions. K/A MATCH ANALYSIS Question gives a plausible scenario where a SG # 1 level alarm is received with other indications of a FW perturbation. Candidate must choose the failure, required actions, and consequences of not taking the action. ANSWER / DISTRACTOR ANALYSIS A. Incorrect. Plausible that a single FRV failing open could cause the indications. AOP actions however have the operator return level to program versus stabilizing at the new S/G level. B. Correct. Cause, actions, and consequences are correct for this. C. Incorrect. Plausible with MFP speeds rising the candidate could consider a common failure to MFPs, however with a single S/G level alarm illuminated the other 3 S/Gs would also be affected. Auto turbine trip would also occur on S/G high high level. D. Incorrect. Plausible with MFP speeds rising the candidate would consider a common failure to MFPs, however with a single S/G level alarm illuminated the other 3 S/Gs would also be affected. A manual turbine trip would also be considered prior to exceeding the S/G High High level auto turbine trip. FWI part is correct. REFERENCES Vogtle 2002 NRC RO Exam question # 48 AOP-18016-C, Condensate Feedwater Malfunction section D for Main Feedwater Valve Malfunction. VEGP learning objectives: LO-PP-18101-25, Describe the protective functions generated from Steam Generator Level to include:

a. High level condition including bases.


Notes

Monday, August 13, 2007 10:00:44 AM

174

Response form

QUESTIONS REPORT for HL-14 NRC RO DRAFT EXAM


Answers MCS Time: 1 Points: 1.00 Version: 0 1 2 3 4 5 6 7 8 9 Answer: BB ADAB CB CD Scramble Range: A- D

Monday, August 13, 2007 10:00:44 AM

175

QUESTIONS REPORT for HL-14 NRC RO DRAFT EXAM 69. WEO4G2.4.20 001/1/1/LOCA OUTSIDE CTMT/C/A - 3.3/NEW/R/NRC RO/TNT / RLM Given the following: The crew is performing 19112-C, LOCA Outside Containment". The LOCA is successfully isolated. Crew transitions to 19010-C, "Response to Loss of Primary or Secondary Coolant". Crew then transitions to 19012-C, "Post LOCA Cooldown and Depressurization". The crew is at the step to use the tables to determine if CCPs and SIPs can be stopped based on the subcooling criteria. Subcooling criteria is met. CCP "A" has been stopped. Which ONE of the following is the CORRECT method for stopping the ECCS pumps based on subcooling criteria? A. Subcooling criteria is not required to be checked again before stopping another pump, more subcooling is required if RCPs are running. B. Subcooling criteria is not required to be checked again before stopping another pump, more subcooling is required if RCPs are stopped. C. Subcooling criteria is required to be checked again before stopping another pump, more subcooling is required if RCPs are running. D. Subcooling criteria is required to be checked again before stopping another pump, more subcooling is required if RCPs are stopped.
F e edback

K/A WE04 LOCA Outside Containment: G2.4.20 Knowledge of operational implications of EOP warnings, cautions, and notes. K/A MATCH ANALYSIS Question gives a plausible scenarion where a LOCA Outside Containment occurs and the crew has finally transitioned to 19012-C, Post LOCA Cooldown and Depressurization. The candidate must determine the proper method for stopping the CCPs and SIPs. ANSWER / DISTRACTOR ANALYSIS
Monday, August 13, 2007 10:00:44 AM

176

QUESTIONS REPORT for HL-14 NRC RO DRAFT EXAM A. Incorrect. Plausible the candidate may think the SI termination sequence is like that in SGTR procedure or LOHS where a one time check for subcooling is performed and ECCS pumps stopped on that criteria. Less subcooling is required if RCPs are running and plausible the candidate may invert. B. Incorrect. Plausible the candidate may think the SI termination sequence is like that in SGTR procedure or LOHS where a one time check for subcooling is performed and ECCS pumps stopped on that criteria. .More subcooling is required if RCPs are stopped is correct. C. Incorrect. Subcooling is required to be checked again before stopping another pump, plausible the candidate may invert the subcooling requirement for RCPs and think more required if running. D. Correct. Subcooling is required to be checked again before stopping another pump and more subcooling is required if RCPs are stopped. REFERENCES 19112-C, "LOCA Outside Containment". 19010-C, Loss of Primary or Secondary Coolant". 19012-C, "Post LOCA Cooldown and Depressurization" steps # 25 with associated NOTE prior to the step. VEGP learning objectives: LO-LP-37112-01, Using EOP 19012 as a guide, briefly describe how each step is accomplished. LO-LP-37112-04, Given a NOTE or CAUTION statement from the EOP, state the bases for that NOTE or CAUTION statement.

N otes Monday, August 13, 2007 10:00:44 AM

177

Response form

QUESTIONS REPORT for HL-14 NRC RO DRAFT EXAM 70.


WE05EA2.2 001/1/1/LOCA OUTSIDE CTMT/MEM - 3.7/BANK/R/NRC RO/TNT / RLM

The crew has entered 19231-C, "Response to Loss of Secondary Heat Sink" due to no feedwater capability to the steam generators and have initiated RCS "bleed and feed". Which ONE of the following describes the sequence of actions required to establish an adequate RCS bleed and feed heat removal flow path in accordance with 19231-C? A. "Feed" is established first by initiating SI flow, then "Bleed" is established by opening at least one Pressurizer PORV and a Reactor Vessel Head Vent. B. "Bleed" is established first by opening at least one Pressurizer PORV and opening the Reactor Vessel Head Vents, then "Feed" is established by initiating SI flow. C. "Feed" is established first by initiating SI flow, then "Bleed" is established by opening both Pressurizer PORVs. D. "Bleed" is established first by opening both Pressurizer PORVs, then "Feed" is established by initiating SI flow.

Monday, August 13, 2007 10:00:44 AM

178

QUESTIONS REPORT for HL-14 NRC RO DRAFT EXAM


Feedback

K/A WE05 Inadequate Heat Transfer - Loss of Secondary Heat Sink: EA2.2 Ability to determine and interpret the following as they apply to the Loss of Secondary Heat Sink. Adherence to appropriate procedures and operation within the limitations in the facilities license and amendments. K/A MATCH ANALYSIS Question gives a plausible scenario during a loss of heat sink where RCS bleed and feed is required to be established. Candidate must pick the correct sequence and flow path for adequate heat removal. ANSWER / DISTRACTOR ANALYSIS A. Incorrect. Plausible since RNO in 19231 step # 37 has the Reactor Vessel Head Vents opened if both PORVs cannot be opened. However, a low pressure water source must be aligned in this instance to insure adequate heat removal. Bleed and feed sequence portion is correct. B. Incorrect. Plausible candidate may choose Bleed first since the procedure is worded this way. Plausible since RNO in 19231 step # 37 has the Reactor Vessel Head Vents opened if both PORVs cannot be opened. However, a low pressure water source must be aligned in this instance to insure adequate heat removal. C. Correct. Sequence correct and both PORVs must be open for adequate bleed. D. Incorrect. Both PORVs must be open for adequate bleed but sequence is reversed. Plausible candidate may choose this since procedure is worded this way. REFERENCES 19231-C, "Response to Loss of Secondary Heat Sink" pages 16 and 17. VC Summer April 2004 NRC RO Exam question # 73. VEGP learning objectives: LO-LP-237051-08, Using EOP 19231 as a guide, briefly describe how each major step is accomplished. Describe the bases for each. (Commitment).
Notes

Monday, August 13, 2007 10:00:44 AM

179

QUESTIONS REPORT for HL-14 NRC RO DRAFT EXAM


R e s p o nse form Answers MCS Time: 1 Points: 1.00 Version: 0 1 2 3 4 5 6 7 8 9 Answer: CACDBDBABD Scramble Range: A- D

Monday, August 13, 2007 10:00:44 AM

180

QUESTIONS REPORT for HL-14 NRC RO DRAFT EXAM 71. WEO9G2.4.6 001/1/2/NAT CIRC - EOP MITIG/C/A - 3.1/MODIFIED/R/NRC RO/TNT / RLM A reactor trip has occurred with an LOSP. Power will not be restored for at least 8 hours and a Natural Circulation Cooldown is desired. RCS Tavg is currently 557 degrees F Only one (1) CRDM fan is operable and running. Which ONE of the following are CORRECT procedural actions to mitigate the event in accordance with 19002-C, "Natural Circulation Cooldown"? A. The upper head cannot be prevented from voiding with only 1 CRDM fan, transition to 19003-C, "Natural Circulation Cooldown with Voids with RVLIS". B. Maintain greater minimum subcooling during RCS cooldown and depressurization, and the upper head cooldown rate will be less. C. Raise the RCS cooldown rate from 50 degrees F to 100 degrees F to facilitate a faster cooldown and depressurization of the upper head area. D. Continue at the initial required subcooling margin and cooldown rate, heat removal from a CRDM fan is insignificant compared to steaming the secondary plant.

Monday, August 13, 2007 10:00:44 AM

181

QUESTIONS REPORT for HL-14 NRC RO DRAFT EXAM


Feedback

K/A WE09 Natural Circulation Operations G2.4.6 Knowledge of symptom based EOP mitigation strategies. K/A MATCH ANALYSIS Question gives a plausible scenario during a natural circulation cooldown. CRDM fans trips below minimum level as specified in the procedure and the candidate must determine the correct action to take per 19002-C, Natural Circulation Cooldown. ANSWER / DISTRACTOR ANALYSIS A. Incorrect. Plausible candidate may think a void cannot be prevented with only one CRDM fan and a transition to 19003 required. B. Correct. More subcooling is required and the head will cool off at a slower rate. C. Incorrect. A faster cooldown rate will lead to RCS depressurization with a void formation in the head more likely. Plausible the candidate may think a higher cooldown rate may prevent a void in the head. D. Incorrect. Higher subcooling margin is required. Plausible the candidate may not think higher margin required due to one CRDM fan is not near the secondary steaming rate. REFERENCES 19002-C, "Natural Circulation Cooldown". LO-OR-37012-05-007 Vogtle LO Active Exam Bank VEGP learning objectives: L O L P-37012-15, State the limitations on subcooling and cooldown rate associated with natural circulation cooldown. Include the bases for any variations. (commitment).
Notes Response form Answers

MCS

Time: 1

Points:

1.00

Version: 0 1 2 3 4 5 6 7 8 9 Answer: B BDC CB ACDB

Scramble Range: A- D

Monday, August 13, 2007 10:00:44 AM

182

QUESTIONS REPORT for HL-14 NRC RO DRAFT EXAM 72. WEI lEK2.2 001/1/1/LOSS RECIRC - OPERAT/C/A - 3.9/BANKJR/NRC RO/TNT / RLM Given the following: RCS LOCA is in progress. 19111-C, Loss of Emergency Coolant Recirculation in effect. Containment pressure at 18 psig and rising slowly. - The crew is at the step to initiate a cooldown to Cold Shutdown conditions. What method will the crew use to establish this cooldown? A. Dumping steam to the main condenser using the steam dumps. B. Dumping steam to atmosphere using the Atmospheric Relief Valves (ARVS). C. Reducing RCS pressure using the PRZR PORVs to increase ECCS pump injection. D. Feeding the Steam Generators at maximum rate using Auxiliary Feedwater pumps.

Monday, August 13, 2007 10:00:44 AM

183

QUESTIONS REPORT for HL-14 NRC RO DRAFT EXAM


Feedback

K/A WE11 Loss of Emergency Coolant Recirculation: EK2.2 Knowledge of the interrelations between the Loss of Emergency Coolant Recirculation and the following: Facilities heat removal systems, including primary coolant, emergency coolant, the decay heat removal systems, and relations between the proper operation of these systems to the operation of the facility. K/A MATCH ANALYSIS Question gives a plausible scenario where the crew is initiating a cooldown per the Loss of Emergency Coolant Recirculation procedure. Candidate must choose the correct method for performing the cooldown. ANSWER / DISTRACTOR ANALYSIS A. Incorrect. Plausible candidate would choose this since it is the preferred method. However with CTMT pressure at 18 psig, SLI would have occurred making the steam dumps to condenser unavailable. B. Correct. With steam dumps not available, use of the ARVs is procedurally directed. C. Incorrect. Plausible since RCPs would be stopped and later in the procedure PORVs would be used to depressurize the RCS to limit break flow and increase injection. D. Incorrect. Plausible since increasing AFW flow can result in an RCS cooldown, however filling the SGs for cooldown purposes is not procedurally directed. REFERENCES Wolf Creek 2004 NRC RO Exam question # 17 19111-C, "ECA-1.1 Loss of Emergency Coolant Recirculation" VEGP learning objectives: LO-LP-37114-12, State the internt of E0p-19111, Loss of Emergency Coolant Recirculation
Notes Response form Monday, August 13, 2007 10:00:44 AM

184

QUESTIONS REPORT for HL-14 NRC RO DRAFT EXAM


A n swers MCS Time: 1

Points:

1.00

Version: 0 1 2 3 4 5 6 7 8 9 Answer: BBDCACABCB

Scramble Range: A- D

Monday, August 13, 2007 10:00:44 AM

185

QUESTIONS REPORT for HL-14 NRC RO DRAFT EXAM 73. WEI 2EK2.2 001/1/1/UNCONT DEPRESS - OPE/C/A - 3.6/BANKJR/NRC RO/TNT / REM Given the following plant conditions: Reactor trip and Safety Injection have occurred. 19121-C, ECA-2.1 Uncontrolled Depressurization of All Steam Generators is in effect. - All Steam Generator Pressures (SG) are lowering uncontrollably. Containment pressure is 8 psig and rising. AFW flow to each SG has been throttled to 30 gpm to limit the RCS cooldown. Which ONE of the following conditions would require increasing AFW flow to more than 30 gpm per SG in accordance with 19121-C? A. All SG levels are less than 10% NR. B. SG # 3 level is less than 9% WR. C. RCS hot leg temperatures are increasing. D. Source of SG depressurization has been identified.

Monday, August 13, 2007 10:00:44 AM

186

QUESTIONS REPORT for HL-14 NRC RO DRAFT EXAM


F e edback

K/A WE12 Uncontrolled Depressurization of all Steam Generators: EK2.2 Knowledge of the interrelations between the Uncontrolled Depressurization of all Steam Generators and the following: Facilities heat removal systems, including primary coolant, emergency coolant, the decay heat removal systems, and relations between the proper operation of these systems to the operation of the facility. K/A MATCH ANALYSIS Question gives a plausible scenario during 19121-C where crew has throttled AFW flow to 30 gpm per SG due to excessive cooldown. Crew must pick the choice which would require raising AFW flow to > 30 gpm. ANSWER / DISTRACTOR ANALYSIS A. Incorrect. Plausible since 10% NR corresponds to the SG level for maintaining 570 gpm AFW flow. Procedures step 4d RNO directs to maintain a minimum of 30 gpm flow to each SG with level < 10% NR. B. Incorrect. Plausible since 9% WR corresponds to the SG level for a Dry steam generator in LOHS procedure 19231-C. Candidate may think raising flow correct for a Dry SG. C. Correct. If Hot Leg temperatures rising, procedure directs raising AFW flow to stabilize the Hot Leg temperatures. D. Incorrect. Plausible candidate may confuse this with isolation of the faulted SG. However, until isolated and transition to E-2, AFW should be maintained 30 gpm. REFERENCES VC Summer October 2006 NRC RO Exam question # 81. 19121-C, "Uncontrolled Depressurization of All Steam Generators" page 6 and FOP. VEGP learning objectives:

N o
No tes

Response form

Monday, August 13, 2007 10:00:44 AM

187

QUESTIONS REPORT for HL-14 NRC RO DRAFT EXAM


A n swers MCS Time: 1 Points: 1.00 Version: 0 1 2 3 4 5 6 7 8 9 Answer: CBB ADA AAB B Scramble Range: A- D

Monday, August 13, 2007 10:00:45 AM

188

QUESTIONS REPORT for HL-14 NRC RO DRAFT EXAM 74. WE14EA1.2 001/1/2/CTMT PRESS - OPERATE/C/A - 3.3/BANK/R/NRC RO/TNT / RLM Given the following, conditions: - A large steam break has occurred inside containment. During performance of E-0, the crew discovered containment pressure to be 22 psig. Proper operation of the Containment Spray System was verified. The crew has transitioned to 19020-C, "E-2 Faulted Steam Generator Isolation". Containment pressure is now 26 psig and slowly lowering. The crew should immediately transition to 19251-C, "FR-Z.1 Response to High Containment Pressure" A. if pressure rises to B. since pressure has remained > 21.5 psig C. to verify proper alignment of the NSCW system. D. to verify proper operation of the Containment Coolers.
F e edback

> 52 psig.

K/A WE14 High Containment Pressure EA1.2 Operating Characteristics of the Facility. K/A MATCH ANALYSIS Question gives a plausbile scenario where a steam line break has occured resulting in a high containment pressure above C. Spray actuation setpoint. Crew has to pick the correct procedural action to take from the given conditions. ANSWER / DISTRACTOR ANALYSIS A. Correct. With spray pump proper operation a transition to 19251-C is not required since this is a yellow path. Pressure > 52psig would result in a RED path and would require a transition. B. Incorrect. Plausible since > 21.5 psig at one time was an ORANGE path, with spray pumps running this is a YELLOW path. Spray pumps should eventually reduce pressure and the liner will act as a gas membrane to maintain leakage within limits for up to 24 hours at nearly design pressure. Transition not immediately required
Monday, August 13, 2007 10:00:45 AM

189

QUESTIONS REPORT for HL-14 NRC RO DRAFT EXAM unless pressure > 52 psig (RED path). C. Incorrect. Plausible since containment coolers assist spray system in reducing containment pressure, however, this is a YELLOW path only. Candidate could think the condition is ORANGE or RED with the listed conditions requiring an immediate transition to address the challenge to a barrier. D. Incorrect. Plausible since NSCW is checked in 19251-C to ensure cooling, but the listed conditions are YELLOW path only. Crew should focus on the Faulted SG isolation. Candidate could think the condition is ORANGE or RED with the listed conditions requiring an immediate transition to address the challenge to a barrier. REFERENCES V-LO-PP-37002-08-005 from Vogtle LO Initial Active Exam Bank question. Harris 2004 March NRC RO Exam question # 5 was referenced by our bank question. Both are attached for a reference. NOTE: Reworded stem and distractors to make question more efficient and less verbose by eliminating verbage repeated in each choice and moved this to the stem. VEGP learning objectives: LO-LP-37002-07, State how the Critical Safety Functions (CSFs) are prioritized. Be able to demonstrate how the highest priority CSF is selected and addressed.

N otes Monday, August 13, 2007 10:00:45 AM

190

Response form

QUESTIONS REPORT for HL-14 NRC RO DRAFT EXAM 75. WE16EA2.1 001/1/2/CTMT PRESS - OPERATE/C/A - 2.9/MODIFIED/R/NRC RO/TNT / RLM Given the following conditions: A small break LOCA has occurred on Unit 1. 19010-C, "Loss of Reactor or Secondary Coolant" is in progress. RCS pressure is 1635 psig with CETCs reading 540 degrees F. Containment Emergency Sump levels are 17" and slowly rising. Containment Low Range Radiation Monitors RE-002 and 003 read 795 MR / HR. SG # 3 pressure is 1190 psig and stable, the ARV is non-functional and one (1) code safety appears to be open. The person monitoring Critical Safety Functions announces an entry criteria for a YELLOW path procedure. Based on the indications above, which ONE of the following procedures should be implemented at SS discretion? A. 19252-C, "Response to Containment Flooding". B. 19223-C, "Response to Saturated Core Cooling". C. 19232-C, "Response to Steam Generator Overpressure". D. 19253-C, "Response to High Containment Radiation Level".

Monday, August 13, 2007 10:00:45 AM

191

QUESTIONS REPORT for HL-14 NRC RO DRAFT EXAM


F e edback

K/A WE16 High Containment Radiation EA2.1 Ability to determine and interpret the following as they apply to the High Containment Radiation. Facility conditions and selection of appropriate procedures during abnormal and emergency operations. K/A MATCH ANALYSIS Question gives a plausible scenario with an RCS LOCA in progress and associated plant parameters. The candidate must choose the correct YELLOW path FRP to enter. ANSWER / DISTRACTOR ANALYSIS A. Incorrect. Plausible the candidate may consider Containment Flooding due to emergency sump level. However, Containment Flooding is an ORANGE path and the threshold (> 105 inches) has not been exceeded. B. Incorrect. Plausible the candidate may think the RCS saturated during a LOCA. However, with conditions stated in the stem RCS is 69 degress subcooled. C. Incorrect. Plausible candidate could consider SG Overpressure since riding on the Code Safety. However, to threshold for entry not met since SG pressure has to be higher than the highest Code Safety valve setpoint. D. Correct. Containment radiation levels exceed 750 MR / HR on RE-002 and 003. REFERENCES 19200-C, "Critical Safety Function Status Trees" 19223-C, "Response to Saturated Core Cooling" 19232-C, "Response to Steam Generator Overpressure" 19252-C, "Response to Containment Flooding" 19253-C, "Response to Containment High Radiation Level" Farley December 2003 NRC RO Exam question # 75. VEGP learning objectives: Not applicable.
Monday, August 13, 2007 10:00:45 AM

192

QUESTIONS REPORT for HL-14 NRC RO DRAF T EXAM


N o tes Response form Answers MCS Time: 1 Points: 1.00 Version: 0 1 2 3 4 5 6 7 8 9 Answer: DC B C AB ADDA Scramble Range: A- D

Monday, August 13, 2007 10:00:45 AM

193

QUESTIONS REPORT for HL-14 NRC SRO DRAFT Exam 1. 001AA2.01 001/1/2/RODS- TRIP BREAKERS/C/A - 4.2/NEW/S/NRC SRO/TNT / RLM The RO withdraws control rods 3 steps, upon release of the .N-HOLD-OUT switch, rods continue to withdraw and cannot be stopped. Which ONE of the following reactor trips is based on this event and would result in the reactor trip breakers opening to mitigate the event without any operator action? (Assume all appropriate procedural actions have been performed to this point) A. SR Hi Flux reactor trip, power at the point for taking critical data. BY IR Hi Flux reactor trip, power at 4% just prior to entering Mode 1. C. D. PR Low Setpoint Hi Flux reactor trip, power at 18% power during swap to MFRVs. PR Positive Rate Trip, power near 100% power performing rod operability testing.

Monday, August 13, 2007 9:45:21 AM

QUESTIONS REPORT for HL-14 NRC SRO DRAFT Exam


Fee dback

K/ A 001 Continuous Rod Withdrawal AA2.01 Ability to determine and interpret the following as they apply to the Continuous Rod Withdrawal. Reactor tripped breaker indicator K/A MATCH ANALYSIS Question gives a plausible scenario where an uncontrolled rod motion occurs after the RO releases the reactor trip breakers. The candidate has to pick which Reactor Trip setpoint is based on this event and power level. Question meets 10CFR55.43(b) criteria item 2 - Facility operating limits in Tech Specs and their bases. ANSWER / DISTRACTOR ANALYSIS A. Incorrect. Plausible the candidate may know the bases for SR Hi Flux is uncontrolled rod withdrawal but the trip is blocked above P-6 when critical data is taken. B. Correct. IR Hi Flux based on uncontrolled rod withdrawal and would not be blocked at this power level. C. Incorrect. Plausible the candidate may know the bases for PR Low Setpoint Hi Flux is for an uncontrolled rod withdrawal but this trip should be blocked per UOP actions when P-10 is received, far below the current power level stated in the question. D. Incorrect. Plausible the candidate may confuse the bases for PR High Positive Rate reactor trip which is an ejected rod event with uncontrolled rod motion. REFERENCES Technical Specifications 3.3.1 and Bases for Reactor Trip Instrumentation. VEGP learning objectives: LO-LP-39207-02, Given a set of Tech Specs and the bases, determine for a specific set of plant conditions, equipment availability, and operational mode:

a. Whether any Tech Spec LCOs of section 3.3 are exceeded.


Notes Monday, August 13, 2007 9:45:21 AM

QUESTIONS REPORT for HL-14 NRC SRO DRAFT Exam


Response form

Answers

MCS

Time: 1

Points:

1.00

Version: 0 1 2 3 4 5 6 7 8 9 Answer: BBDABCDBDC

Scramble Range: A- D

Monday, August 13, 2007 9:45:22 AM

QUESTIONS REPORT for HL-14 NRC SRO DRAFT Exam 2. 002A2.04 001/2/2/RCS - LOHS/C/A - 4.6/NEW/S/NRC SRO/TNT / RLM Given the following conditions: Crew is performing the actions of 19231-C, "Loss of Heat Sink" due to a prolonged loss of feedwater. RCS Bleed and Feed has been initiated. SG WR levels are all approximately 15%. Containment pressure is 0.2 psig RE-002 and RE-003 are in INTERMEDIATE alarm AFW flow has just been restored to a single Steam Generator Based on the above conditions, which ONE of the following describes the plant response and the actions the SS should take? A. Containment pressure will remain stable, immediately transition to 19010-C, Loss of Reactor or Secondary Coolant. B. Containment pressure will rise over time, immediately transition to 19010-C, Loss of Reactor or Secondary Coolant. C. Containment pressure will remain stable, remain in 19231-C, Loss of Secondary Heat Sink to perform further actions.

D? Containment pressure will rise over time, remain in 19231-C, Loss of Secondary Heat Sink to perform further actions.
Feedback

K/A 002 Reactor Coolant System (RCS). A2.04 Ability to (a) predict the impacts of the following malfunctions or operations on the containment system and (b) based on those predictions, use procedures to correct, control, or mitigate the consequences of those malfunctions or operations. Loss of heat sinks. K/A MATCH ANALYSIS

Questions gives a scenario during a Loss of Heat Sink where the crew has just established AFW flow to a SG. Conditions show the PRT has ruptured and the crew has to determine whether to remain in the LOHS FRP and impact on containment.
Monday, August 13, 2007 9:45:22 AM

QUESTIONS REPORT for HL-14 NRC SRO DRAFT Exam Question meets 10CFR55.43(b) criteria item # 5 - Assessment of facility conditions and selection of procedures during normal, abnormal, and emergency conditions. Therefore, the question is SRO only. ANSWER / DISTRACTOR ANALYSIS A. Incorrect. Plausible the candidate may not recall PORVs open causing containment pressure to rise but rad monitors indicate PRT ruptured. 19010 is improper due to PORVs must be sequentially closed before the transition. B. Incorrect. Plausible the candidate may recognize PORVs cause containment pressure to rise and think transition to 19010 is appropriate and not think of terminating feed and bleed. C. Incorrect. Plausible the candidate may not recall PORVs open causing containment pressure to rise but rad monitors indicate PRT ruptured. Remaining in 19231 to terminate SI is correct. D. Correct. Containment pressure should rise and remain in 19231 is correct. REFERENCES 19231-C, Loss of Secondary Heat Sink pages 21 - 24, and 31 - 36. VEGP learning objectives: LO-LP-37051-08, Using EOP 19231 as a guide, briefly describe how each major step is accomplished. Describe the bases for each. (commitment) LO-LP-37051-10, State all conditions when the procedure 19231, Response to Loss of Secondary Heat Sink, would be terminated.

N otes Monday, August 13, 2007 9:45:22 AM

Response form

QUESTIONS REPORT for HL-14 NRC SRO DRAFT Exam 3. 007G2.4.6 001/1/1/RX TRIP - EOP STRAT/C/A - 4.0/NEW/S/NRC SRO/TNT / RLM The unit trips from 100% power when a lightning strike causes a fault on 13.8 Kv bus 1NAB. The other 13.8Kv bus, 1NAA fails to fast bus transfer but is capable of being re-energized. 4160 non-1E buses 1NA01, 1NA04, and 1NA05 are still energized. The crew is performing the actions of 19001-C, "Reactor Trip Response. Control bank D (CDB) rod M12 indicates 18 steps on DRPI. CST # 1 level is 52% and slowly lowering. CST # 2 level is 74% and stable. - Letdown isolates post Rx. trip, attempts to re-establish are unsuccessful because LV-460 will not re-open. Which ONE of the following would be the CORRECT actions to take ? Pe Re-energize 1NAA per 13420-1, "13.8Kv Electrical Distribution System" and start RCP # 1 per 19001-C Attachment "A", "Starting a Reactor Coolant Pump". B. Perform an Emergency Boration of the RCS in accordance with SOP-13009-1/2, CVCS Reactor Makeup Control System and perform a Shutdown Margin. C. Perform the actions of ADP-18007-C, section A for Loss of Letdown and place Safety Grade Letdown in service per 13006, "Chemical Volume Control System". D. Immediately swap AFW pump suctions to CST # 2 per 13610-1, "Auxiliary Feedwater System" and transition to 19002-C, "Natural Circulation Cooldown".

Monday, August 13, 2007 9:45:22 AM

QUESTIONS REPORT for HL-14 NRC SRO DRAFT Exam


Feedback

K/A 007 Reactor Trip - Stabilization - Recovery G2.4.6 Knowledge of Symptom based EOP mitigation strategies K/A MATCH ANALYSIS Question gives a plausible scenario post reactor trip with several malfunctions, the candidate must choose the correct action / procedure to respond to the symptoms. Question meets 10CFR55.43(b) item # 5 - Assessment of facility conditions and selection of procedures during normal, abnormal, and emergency conditions. ANSWER / DISTRACTOR ANALYSIS A. Correct. 13.8 Kv bus should be re-energized and RCP # 1 started. B. Incorrect. Plausible the candidate may consider this action but the emergency boration is only required for 2 or more stuck rods. Shutdown margin is required. C. Incorrect. Plausible the candidate would consider performing the actions for the Loss of Letdown AOP in parallel, however, Excess Letdown would be placed in service versus Safety Grade Letdown which is used on a loss of instrument air. Safety Grade Letdown is mentioned in notes and cautions of 19001-C. D. Incorrect. AFW suctions should only be swapped when one CST level lowers to less than 15%, there would be no reason to go to 19002 for Natural Circ cooldown when it is possible to start an RCP and CST levels are not a problem. REFERENCES 19001-C, "Reactor Trip Response". 18007-C, "CVCS Malfunction section A for Loss of Letdown." VEGP learning objectives: LO-LP-37011-03, State the bases for the "Reactor Trip Recovery" procedure. LO-LP-37011-04, State and describe the major action categories of 19001, "Reactor Trip Recovery".
Notes

Response form
Monday, August 13, 2007 9:45:22 AM

QUESTIONS REPORT for HL-14 NRC SRO DRAFT Exam


Answers MCS Time: 1 Points: 1.00 Version: 0 1 2 3 4 5 6 7 8 9 Answer: ACDDAADADA Scramble Range: A- D

Monday, August 13, 2007 9:45:22 AM

QUESTIONS REPORT for HL-14 NRC SRO DRAFT Exam 4. 008AA2.15 001/1/1/PRZR VAPOR SPACE ACC/C/A - 4.2/NEW/S/NRC SRO/TNT / RLM Given the following plant conditions with both units at 100% power: Unit 1 has one Block valve closed and de-energized to isolate a PORV that is partially stuck open. Unit 2 has both Block valves closed and still energized to isolate both PORVs which have excessive seat leakage. Which ONE of the units would be required to shutdown due to INOPERABLE PORV / Block valve status per Tech Specs and what is the CORRECT bases? A. Unit 1 - both PORVs are required to be capable of automatically cycling to limit RCS pressure following the blowdown of a faulted Steam Generator. B. Unit 2 - both PORVs are required to be capable of automatically cycling to mitigate events such as a Steam Generator Tube Rupture or Loss of Heat Sink. O., Unit 1 - both PORVs are required to be capable of being manually cycled to mitigate events such as a Steam Generator Tube Rupture or Loss of Heat Sink. D. Unit 2 - both PORVs are required to be capable of being manually cycled to limit RCS pressure following the blowdown of a faulted Steam Generator.

Monday, August 13, 2007 9:45:22 AM

QUESTIONS REPORT for HL-14 NRC SRO DRAFT Exam


Feedback

K/A 008 Pressurizer Vapor Space Accident AA2.15 Ability to determine and interpret the following as they apply to the Pressurizer Vapor Space Accident. ESF control board, valve controls, and indicators K/A MATCH ANALYSIS Question gives a plausible scenario where both units have PORVs closed to due either excessive seat leakage or stuck open PORVs. The candidate must determine which unit is required to shutdown per Tech Specs and the Tech Spec bases. Question meets 10CFR55.43(b) criteria for item # 2 - Facility operating limits in Tech Specs and their bases. ANSWER / DISTRACTOR ANALYSIS A. Incorrect. Plausible the candidate may think both PORVs have to automatically cycle and this is not a correct bases. B. Incorrect. Plausible the candidate may think both PORVs have to automatically cycle and this is the correct bases. C. Correct. Both PORVs must be capable of being manually cycled and this is a correct bases. D. Incorrect. Plausible the candidate may know both capable of manually cycling but this is an incorrect bases. REFERENCES Technical Specification 3.4.11 and bases for PORVs and Block Valves VEGP learning objectives: LO-LP-39208-02, Given a set of Tech Specs and the bases, determine for a specific set of plant conditions, equipment availability, and operational mode:

a. W hether any Tech Spec LCOs of section 3.4 are exceeded.


Notes

Monday, August 13, 2007 9:45:22 AM

10

Response form

QUESTIONS REPORT for HL-14 NRC SRO DRAFT Exam


Answers MCS Time: 1 Points: 1.00 Version: 0 1 2 3 4 5 6 7 8 9 Answer: CB C C C CAACB Scramble Range: A- D

11
Monday, August 13, 2007 9:45:22 AM

QUESTIONS REPORT for HL-14 NRC SRO DRAFT Exam 5. 015/017G2.4.4 001/1/1/RCP MALF - EOP ENTRY/C/A - 4.3/MODIFIED/S/NRC SRO/TNT / RLM Unit 2 is at 25% power when the following annunciators are received. ALB08 window B05 for RCP # 3 CONTROLLED LKG HI / LO FLOW ALB08 window B04 for RCP # 3 NO. 2 SEAL LKOF HI FLOW The RO reports the following indications: RCP # 3 seal leakoff flow Hi Range meter is 6.0 gpm. RCP # 3 seal injection flow is 7.9 gpm. RCP # 3 Seal Water Inlet temperature is 223 degrees F and stable. Which ONE of the the following is the CORRECT procedurally directed action(s) for the SS to take? A. Per UOP-12004-C, "Power Operations (Mode 1)", commence a unit shutdown to be in Mode 3 in 8 hours. BY' Trip the reactor and enter E-0, "Reactor Trip and Safety Injection", per SOP-13003, "RCP Operation", stop RCP # 3 and close seal leakoff valve HV-8141C. C. Per SOP-13003, stop RCP # 3, close seal leakoff valve HV-8141C, enter AOP-18005, "Partial Loss of RCS Flow", commence unit shutdown per UOP-12004. D. Per UOP-12004, maintain reactor power at 25%, monitor the RCP per SOP-13003 s e c t ion 4.2.1 "Pump Operation With A Seal Abnormality", contact Duty Engineering.
Feedback

K/A 015 / 017 RCP Malfunctions G2.4.4 Ability to recognize abnormal indications for system operating parameters which are entry-level conditions for emergency and abnormal operating procedures. K/A MATCH ANALYSIS Question gives a plausible scenario with indications of an RCP # 3 seal failure. Candidate must choose the correct procedural actions to address the failure. Question meets 10CFR55.43(b) criteria item # 5 - Assessment of facility conditions and selection of procedures during normal, abnormal, and emergency situations.

ANSWER / DISTRACTOR ANALYSIS


Monday, August 13, 2007 9:45:22 AM

12

QUESTIONS REPORT for HL-14 NRC SRO DRAFT Exam A. Incorrect. Plausible the candidate may not recognize the seal leakoff indications of >5.5 gpm require an immediate RCP shutdown and a normal shutdown desired. B. Correct. per SOP-13003 and AOP-18005 if reactor power > 15% and an RCP is to be tripped, correct action is to trip reactor and go to E-0. RCP should be stopped after lOAs of E-0 and seal leakoff valve closed. C. Incorrect. Plausible the candidate may recognize the plant is < P-8 and recognize an auto reactor trip would not occur and think AOP-18005-C Partial Loss of Flow entry is appropriate. SOP13003 and 18005 both state if reactor power > 15% trip the reactor and enter E-0. D. Incorrect. Immediate trip criteria for RCP is exceeded due to excessive seal leakoff. Plausible the candidate may not recognize the immediate trip criteria and know that Duty Engineering would be contacted. REFERENCES AOP-18005-C, "Partial Loss of RCS Flow". SOP-13003-1/2, "Reactor Coolant Pump Operation" Precautions and Limitations, and section 4.2.1 for Operation With a Seal Abnormality with the appropriate Decision Tree Flow Chart (figure). ARP-17008-1/2 windows C05 and C04 for RCP # 3 Controlled Leakage Hi / Lo Flow and # 3 Seal Leakoff Hi Flow. Farley December 2004 NRC SRO Exam question # 4. VEGP learning objectives: LO-PP16401-03, Describe the control room indications for a failure of an RCP seal.
Notes Response form Answers MCS Time: 1 Points: 1.00 Version: 0 1 2 3 4 5 6 7 8 9 A n s wer: BCCDCBDBBC

13
Scramble Range: A- D

Monday, August 13, 2007 9:45:22 AM

QUESTIONS REPORT for HL-14 NRC SRO DRAFT Exam 6. 025AA2.03 001/1/1/LOSS RHR - CTMT SUMP/C/A - 3.8/MODIFIED/S/NRC SRO/TNT / RLM Given the following conditions: Unit 1 RCS drain down for refueling outage in progress. RCS temperature is 185 degrees F. Charging and letdown flows are balanced. RHR pump 1B is operating in shutdown cooling mode, RHR pump 1A is in standby. The following occurs: Containment sump levels begin to rise with radiation alarms received on RE002 and RE-003. RCS level decreases to 189' 6" and lowering. The Unit SS should implement which ONE of the following: A. 18004-C, RCS Leakage, section C for RCS Leakage (Mode 5). B. 18004-C, RCS Leakage, section B for RCS Leakage (Mode 3 < 1000 psig and 4). C. 18019-C, Loss of RHR, section A for Loss of RHR capability in Mode 4 or Mode 5 with PRZR Level in the IR. Ds.' 18019-C, Loss of RHR section B for Loss of RHR in Mode 5 or 6 Below PRZR IR or SG Nozzle Dams Installed.

Monday, August 13, 2007 9:45:22 AM

14

QUESTIONS REPORT for HL-14 NRC SRO DRAFT Exam


F e edback

K/A 025 Loss of Residual Heat Removal (RHR) System AA2.03 Ability to determine and interpret the following as they apply to the Loss of Residual Heat Removal System. Increasing reactor building sump level. K/A MATCH ANALYSIS Question gives a plausible scenario where Containment sump levels, radiation, and PRZR level indicate an RCS leak in Containment. The candidate must determine the plant Mode from the given conditions and select the apppriate section of 18004 for RCS Leakage or 18019 for Loss of RHR to enter. Question meets 10CFR55.43(b) criteria for item # 5 - Assessment of facility conditions and selection of procedures during normal, abnormal, or emergency conditions. ANSWER / DISTRACTOR ANALYSIS A. Incorrect. 18019-C, section B should be used for this condition. B. Incorrect. Plausible the candidate may not properly recognize the plant mode and think section B of 18004 is appropriate. C. Incorrect. Plausible the candidate may select this section of 18019-C since the description fits the plant conditions, section A is just for loss of capability, not leakage. D. Correct. With PRZR level below the indicating range, 18019-C section B should be the section to use. REFERENCES 18004-C, RCS Leakage 18019-C, Loss of Residual Heat Removal (RHR) VEGP learning objectives: LO-LP-60304-08, Given conditions and/or indications, determine the required AOP to enter (including subsections, as applicable). (18004 objective) LO-LP-60314-04, Given conditions and/or indications, determine the required AOP to enter (including subsections, as applicable). (18019 objective)
Monday, August 13, 2007 9:45:22 AM

15

QUESTIONS REPORT for HL-14 NRC SRO DRAFT Exam


Notes

Response form Answers MCS Time: 1 Points: 1.00 Version: 0 1 2 3 4 5 6 7 8 9 Answer: DB B BDAB AB A Scramble Range: A- D

16
Monday, August 13, 2007 9:45:23 AM

QUESTIONS REPORT for HL-14 NRC SRO DRAFT Exam 7. 026G2.2.25 001/2/1/C. SPRAY - TS & BASE/MEM - 3.7/MODIFIED/S/NRC SRO/TNT / RLM The Containment Spray System is designed to reduce containment pressure during a LOCA or a_____ (1) ____ . In addition, the Containment Spray System is designed to retain ____ (2) _____ in water solution. Pe (1) Main Steamline Break IRC, (2) Iodine B. (1) Main Steamline Break IRC, (2) Cesium C. (1) Main Feedwater Line Rupture IRC, (2) Iodine D. (1) Main Feedwater Line Rupture IRC (2) Cesium

Monday, August 13, 2007 9:45:23 AM

17

QUESTIONS REPORT for HL-14 NRC SRO DRAFT Exam


F e edback

K/A 026 Containment Spray. G2.2.25 Knowledge of bases in Technical Specifications for limiting conditions for operations and safety limits. K/A MATCH ANALYSIS Question asks which DBA event will produce the highest peak containment temperature and the specific isotope C. Spray enhances retaining in solution. Question meets 10CFR55.43(b) criteria for item # 2 - Facility operating limits in Tech Specs and their bases. ANSWER / DISTRACTOR ANALYSIS A. Correct. DBA event is correct and iodine is the proper isotope to remove. B. Incorrect. Plausible the candidate may think a SLB is the proper event and Cesium is to be removed as it is one of the most common radioisotopes in radwaste. C. Incorrect. Plausible the candidate may think that a FLB produces a high pressure and iodine is the correct isotope. D. Incorrect. Plausible the candidate may think a FLB produces high pressure and Cesium is to be removed as it is one of the most common radioisotopes in radwaste. REFERENCES Technical Specifications 3.6.6 and bases for Containment Spray and Cooling Systems. LO-PP-15101-01-001, Vogtle LO Active Exam Bank. VEGP learning objectives: LO-PP-15101-08, State the LCO, applicability, and the bases o f a l l Containment Spray related technical specifications.
Notes Response form Answers

MCS

Time: 1

Points:

1.00

Version: 0 1 2 3 4 5 6 7 8 9 Answer: ADAC ADAB CA

Scramble Range: A- D

Monday, August 13, 2007 9:45:23 AM

18

QUESTIONS REPORT for HL-14 NRC SRO DRAFT Exam 8.


034G2.1.32 002/2/2/FH EQUIP - P & L/MEM - 3.8/MODIFIED/S/NRC SRO/TNT / RLM

Which ONE of the following describes a condition that would require the SS to direct suspension of fuel movement? At, Both trains of FHB Post Accident Ventilation are delcared INOPERABLE. B. SR channel N31 High Flux At Shutdown Alarm is declared INOPERABLE. C. RHR flow is reduced from 3200 gpm to 3100 gpm. D. Spent Fuel Pool level drops to 217' 4".
F e edback

K/A 034 Fuel Handling Equipment. G2.1.32 Ability to explain and apply all system limits and precautions. K/A MATCH ANALYSIS Question gives a plausible scenario where a Question meets 10CFR55.43(b) criteria item # 7 - Fuel handling facilites and procedures. ANSWER / DISTRACTOR ANALYSIS A. Correct. Both FHB Post Accident Ventilation units INOPERABLE require an immediate suspension of fuel movement. B. Incorrect. HFASA is only required in Mode 5 but plausible the candidate may think the Containment Evacuation alarm is affected during fuel movement. There is no Tech Spec action with this alarm to stop fuel movement either. C. Incorrect. RHR flow is required to be > 3000 gpm. Plausible candidate may confuse with the Tech Spec round requirement to maintain 3200 gpm in higher modes. D. Incorrect. Spent Fuel Pool level is still > Tech Spec limit of 217 feet. Plausible the candiate may confuse with the normal level of 218' 6". REFERENCES TRM 13.9.5 for Fuel Handling Building Post Accident Ventillation System Tech Spec 3.3.8 for High Flux At Shutdown Alarm (HFASA) Tech Spec 3.9.5 for RHR and Coolant Circulation (High Water Level)
Monday, August 13, 2007 9:45:23 AM

19

QUESTIONS REPORT for HL-14 NRC SRO DRAFT Exam Tech Spec 3.9.6 for RHR and Coolant Circulation (Low Water Level) Tech Spec 3.7.15 for Spent Fuel Storage Pool Level Annunciator Response ALB05 window E02 for Spent Fuel Pit Low Level V-LO-PP-25102, Spent Fuel Pool Cooling System, slide # 19 in particular. VEGP learning objectives: V-LO-PP-25102-11, State the Tech Spec LCO, Applicability, and one hour actions for the Spent Fuel Pool. LO-LP-39207-01, For any given item in section 3.3 of Tech Specs, be able to: a. State the LCO b. State any one hour or less required actions. LO-LP-39211-01, For any given item in section 3.7 of Tech Specs, be able to: a. State the LCO b. State any one hour or less actions. LO-LP-39213-03, For any given item in section 13.9 of the TRM, be able to: a. State the TR for operation b. S t a t e a n y one hour or less required actions.
Notes Response form Answers MCS Time: 1 Points: 1.00 Version: 0 1 2 3 4 5 6 7 8 9 Answer: ACCBBDCCCB Scramble Range: A- D

Monday, August 13, 2007 9:45:23 AM

20

QUESTIONS REPORT for HL-14 NRC SRO DRAFT Exam 9. 039A2.03 001/2/1/MSL RAD MNTR - SGTR/MEM - 3.7/MODIFIED/S/NRC SRO/TNT / RLM The plant has experienced a SGTR with the following radiation monitors in alarm: RE-12839, Condenser Air Ejector and Steam Packing Exhaust Effluent Monitor RE-13121, Main Steam Line Radiation Monitor for SG # 2 read on the SRDC. The crew entered and is currently implementing the steps of E-0, "Reactor Trip or Safety Injection". Both MDAFW pumps are UNAVAILABLE. - The TDAFW pump is RUNNING. The BOP requests permission to isolate the steam supply to the TDAFW pump from the ruptured steam generator. Which ONE of the following actions should the Unit SS take regarding the requested operator action? A. Allow isolation of steam to the TDAFW pump only if S/G # 2 wide range level is > 29%. B. Allow the BOP to shut the Trip and Throttle Valve to the TDAFW pump to isolate the steam supply. C. Do Not allow isolation of steam to the TDAFW pump because the ruptured S/G is not positively identified. D.' Do Not allow isolation of steam to the TDAFW pump, this action will be performed a f t e r transition to 19030-C.
Feedback

K/A 039 Main and Reheat Steam. A2.03 Ability to (a) predict the impacts of the following malfunctions or operations on the MRSS; and (b) based on predictions, use procedures to correct, control, or mitigate the consequences of those malfunctions or operations: Indications and alarms for main steam and area radiation. K/A MATCH ANALYSIS Question gives a plausible scenario where a SGTR has taken place on SG # 2. The

BOP asks for permission to isolated the steam supply to the TDAFW pump. The candidate must determine the correct operator action.
Monday, August 13, 2007 9:45:23 AM

21

QUESTIONS REPORT for HL-14 NRC SRO DRAFT Exam This question meets 10CFR55.43(b) criteria for item # 4 - Radiation hazards that may arise during normal, abnormal, and situations, including maintenance activites and various contamination conditions. The SS has to make a decision on whether to perform early isolatation of the radiation release to limit contamination. This question meets 10CFR55.43(b) criteria for item # 5 - Assessment of facility conditions and selections of procedures during normal, abnormal, and emergency conditions. Per Vogtle Rules of EOP Usage procedure 10020-C, isolation would not be allowed until a transition to 19030-C has occurred. This implies a selection or transition of procedures must take place before the isolation is allowed. ANSWER / DISTRACTOR ANALYSIS A. Incorrect. Plausible the candidate may confuse this with the isolation of AFW / feed flow to the S/G which should not occur until S/G level is > 10% NR. B. Incorrect. Plausible the candidate may choose this as it is method to shut down the TDAFW pump,however, it is an RNO action if the individual steam supply is not able to be closed. Early actions are not allowed per 10020-C, Vogtle's EOP Rules of Usage Procedure. C. Incorrect. Plausible the candidate may not feel there is enough information to identify the ruptured S/G, but with the given indications and the BOP requesting permission to isolate implies he has identified the SG that is ruptured. However, per our EOP Rules of Usage this action would not be allowed early and is the reason it cannot be perfomred. D. Correct. S/G # 2 should NOT be isolated until procedurally directed per 10020-C, "EOP Rules of Usage". REFERENCES 19030-C, "Steam Generator Tube Rupture" steps # 7 and # 8 in particular. 10020-C, "EOP Rules of Usage" steps 3.1.2 and 3.1.3 for early operator actions. V. C. Summer October 2006 NRC SRO Retake Exam question # 40. VEGP learning objectives:

Not applicable.
Notes

Monday, August 13, 2007 9:45:23 AM

22

Response form

QUESTIONS REPORT for HL-14 NRC SRO DRAFT Exam


Answers MCS Time: 1 Points: 1.00 Version: 0 1 2 3 4 5 6 7 8 9 Answer: DADABCCCDB Scramble Range: A- D

23
Monday, August 13, 2007 9:45:23 AM

QUESTIONS REPORT for HL-14 NRC SRO DRAFT Exam 10. 040G2.2.22 001/1/1/SLB - TECH SPECS LCO/MEM - 4.1a3ANK/S/NRC SRO/TNT / RLM Which ONE of the following describes the MOST restrictive condition assumed to ensure that the minimum shutdown reactivity of accident analysis is met during a guillotine break of a main steam line inside containment? A. At the beginning of core life, with Tavg at full load operating temperature. B. At the beginning of core life, with Tavg at no load operating temperature. C. At the end of core life, with Tavg at full load operating temperature. De At the end of core life, with Tavg at no load operating temperature.

Monday, August 13, 2007 9:45:23 AM

24

QUESTIONS REPORT for HL-14 NRC SRO DRAFT Exam


F e edback

K/A 040 Steam Line Break - Excessive Heat Transfer G2.2.22 Knowledge of limiting conditions for operations and safety limits. K/A MATCH ANALYSIS Question asks the basis for the minimum shutdown reactivity accident analysis for Shutdown Margin during a main steam line break IRC. Candidate must choose the correct bases. Question meets 10CFR55.43(b) criteria item # 2 - Facility operating limits in Tech Specs and their bases. ANSWER / DISTRACTOR ANALYSIS A. Incorrect. EOL, no load Tavg is bases. B. Incorrect. EOL, no load Tavg is bases. C. Incorrect. EOL, no load Tavg is bases. D. Correct. Per bases of Tech Spec REFERENCES Technical Specifications and Bases for 3.1.1, "Shutdown Margin". Farley December 2003 NRC SRO Exam question # 8. VEGP learning objectives: LO-LP-39205-02, Given a set of Tech Specs and the Bases, determine for a specific set of plant conditions, equipment availability, and operational mode:

a . Whether any Tech Spec LCOs of section 3.1 are exceeded.


Notes Response form Answers

MCS

Time: 1

Points:

1.00

Version: 0 1 2 3 4 5 6 7 8 9 Answer: DADCBCDCBC

Scramble Range: A- D

Monday, August 13, 2007 9:45:23 AM

25

QUESTIONS REPORT for HL-14 NRC SRO DRAFT Exam 11. 056AA2.24 001/1/1/LOSP - CCW PUMP AMPS/C/A - 3.1/NEW/S/NRC SRO/TNT / RLM Given the following conditions: The crew is in 18031-C due to an LOSP on 2BA03 with DG2B tying to the bus. Various CCW Train B low flow and pressure alarms annunciate, then clear. The crew notes 3 CCW train B pump red lights illuminated on the QMCB. Which ONE of the following is CORRECT regarding CCW Train B and the actions the SS should take? A. CCW pump locked rotor has occurred. Monitor pump amps on the QEAB to determine which pump to stop. Enter LCO 3.7.7 for CCW. B. CCW pump locked rotor has occurred. Monitor pump amps on the QEAB to determine which pump to stop. Enter INFO LCO 3.7.7 for CCW. C. CCW pump shaft shear has occurred. Monitor pump amps locally at 2BA03 to determine which pump to stop. Enter LCO 3.7.7 for CCW.
DY

CCW pump shaft shear has occurred. Monitor pump amps locally at 2BA03 to determine which pump to stop. Enter INFO LCO 3.7.7 for CCW.

Fee dba ck

K/A 056 Loss of Off-site Power AA2.24 Ability to determine and interpret the following as they apply to the Loss of Offsite Power. CCW pump ammeter, flowmeter and run indicator K/A MATCH ANALYSIS Question gives a plausible scenario during an LOSP on a class 1 E electrical bus which starts the DG and the bus is re-energized. Three CCW pumps will be running due to low pressure. The candidate must determine which pump to stop and if an LCO entry is required. Question meets 10CFR55.43(b) criteria for item # 2 - Facility operating limits in Tech Specs and their bases. ANSWER / DISTRACTOR ANALYSIS A. Incorrect. Locked rotor would result in a breaker trip. Pump amps not available on the QEAB. Plausible the candidate may think amps available on QEAB or an LCO
Monday, August 13, 2007 9:45:23 AM

26

QUESTIONS REPORT for HL-14 NRC SRO DRAFT Exam entry is required. B. Incorrect. Locked rotor would result in a breaker trip. Pump amps not available on the QEAB. Plausible the candidate may think amps available on QEAB and know an INFO LCO entry is required. C. Incorrect. Plausible the candidate may know pump amps monitored at swgr and think an LCO entry is required. D. Correct. REFERENCES Tech Spec 3.7.7 for CCW and the Bases. AOP-18031-C, "Loss of Class lE Electrical Systems" section B for Loss of Power With DG Tying to Bus. VEGP learning objectives: LO-LP-39211-02, Given a set of Tech Specs and the bases, determine for a specific set of plant conditions, equipment availability, and operational mode. a. Whether any Tech Spec LCOs of section 3.7 are exceeded. LO-LO39211-04, Describe the bases for any given Tech Spec in section 3.7
N o t e s R esponse form Answers MCS Time: 1 Points: 1.00 Version: 0 1 2 3 4 5 6 7 8 9 Answer: DADCBBBDAD Scramble Range: A- D

Monday, August 13, 2007 9:45:23 AM

27

QUESTIONS REPORT for HL-14 NRC SRO DRAFT Exam 12.


063A2.02 001/2/1/ELEC - VENT BATTERY/MEM - 3.1/MODIFIED/S/NRC SRO/TNT / RLM

The SO reports the 125V DC class 1 E battery room fans have tripped while the batteries were being charged. Which ONE of the following describes the primary concern and actions the SS should direct the SO to take based on the above conditions? A. Battery room temperatures exceeding Tech Spec limits. Direct the SO to perform 13405, "125V DC Electrical Distribution System" to stop the battery chargers and prop open the battery room doors per 00310-C, "Standards for Use of Doors". By' Explosive Hydrogen gas could accumulate in the battery rooms. Direct the SO to prop the doors open per 00310-C, "Standards for Use of Doors" and establish portable ventillation per 13302, "Control Building ESF Ventillation Systems". C. Smoke purge function may be lost in the event of a battery room fire. Direct the BOP to place the battery rooms in the smoke purge mode per 13302, "Control Building ESF Ventillation Systems". D. Toxic fume buildup could affect battery room habitability / personnel safety. Direct the SO to prop doors open per 00310-C, "Standards for Use of Doors" and establish portable ventillation per 13302, "Control Building ESF Ventillation S y s t ems".
Feedback

K/A 063 DC Electrical Distribution. A2.02 Ability to (a) predict the impacts of the following malfuncitons or operations on the DC electrical systems; and (b) based on those predictions, use procedures to correct, control, or mitigate the consequences of those malfunctions or operations. Loss of ventilation during battery charging. K/A MATCH ANALYSIS Question gives a plausible scenario where the lE battery room fans are lost while the batteries are charging. The candidate must determine the primary concern and the correct actions to mitigate. Question meets 10CFR55.43(b) criteria item # 5 - assessment of facility conditions and selection of procedures during normal, abnormal, and emergency situations. Question is also SRO only by KA Catalog # for SRO giving an importance factor rating of 3.1 while the RO importance factor rating is only 2.3.

Monday, August 13, 2007 9:45:23 AM

28

QUESTIONS REPORT for HL-14 NRC SRO DRAFT Exam ANSWER / DISTRACTOR ANALYSIS A. Incorrect. Plausbile the candidate may recall Tech Spec limits on room temperature and consider stopping charging of the battery chargers but is not the primary concern , propping open the doors is a correct action. B. Correct. Explosive Hydrogen gas is the main concern and 13302 gives direction to establish portable ventillation and to prop open room doors per 00310-C. C. Incorrect. Battery room ventillation fans do not perform a smoke purge function but plausible the candidate may consider the use of Smoke Purge Mode to remove hydrogen or fumes, smoke purge would not be used until after a fire is out to prevent feeding 02 to the fire. D. Incorrect. Plausible the candidate may consider toxic fumes to be a personnel hazard and actions same as that for "B" above but not the primary concern. REFERENCES SOP-13302-1/2, CB ESF Ventillation Systems, Precautions 2.1.1 and 2.1.2 13404-1/2, "125V DC Electrical Distribution Systems" Precaution 2.1.3. 00310-C, "Standard For Use of Doors". Vogtle October 2005 SRO Audit Exam question # 21 modified to meet 10CFR55.43(b). VEGP learning objectives:

N
Notes

Response form Answers MCS Time: 1 Points: 1.00 Version: 0 1 2 3 4 5 6 7 8 9 Answer: BCCADAAACC Scramble Range: A- D

Monday, August 13, 2007 9:45:23 AM

29

QUESTIONS REPORT for HL-14 NRC SRO DRAFT Exam 13. 067G2.1.33 001/1/2/FIRE - TECH SPEC S/MEM - 4.0/NEW/S/NRC SRO/TNT / RLM Given the following: The Unit is at 100% Rated Thermal Power (RTP). SIP "A" inoperable, Tech Spec LCO 3.5.2 for ECCS - Operating in Effect A fire occurs at DG1B - At 0400 LCO 3.8.1, A. C. Sources Operating is entered due to DG1B inoperable. Which ONE of the following is REQUIRED in accordance with Tech Specs? A. Immediately take actions of LCO 3.0.3 (Motherhood) B. No other LCO entry is required, LCO 3.8.1 addresses this condition. CY By 0800 take actions of LCO 3.5.2 for both trains of ECCS inoperable. D. Immediately take actions of LCO 3.5.2 for both trains of ECCS inoperable.

Monday, August 13, 2007 9:45:24 AM

30

QUESTIONS REPORT for HL-14 NRC SRO DRAFT Exam


F e edback

K/A 067 Plant Fire on Site G2.1.33 Ability to recognize indications for system operating parameters which are entry level conditions for Technical Specifications. K/A MATCH ANALYSIS Question gives a plausible scenario where a fire at a DG has resulted in inoperable Question meets 10CFR55.43(b) criteria for item # 2 - Facility operating limits in Tech Specs and their bases. ANSWER / DISTRACTOR ANALYSIS A. Incorrect. Plausible candidate may think motherhood is immediately required to be entered. B. Incorrect. Plausible candidate may think no other LCO required. SIP "A" has to be declared inoperable at 0800. C. Correct. At 0800 SIP "B" would have to be declared inoperable. D. Incorrect. Plausible candidate may think actions required for LCO 3.5.2 but there is 4 hours to try to restore per 3.8.1. REFERENCES Technical Specification 3.8.1 for A. C. Sources - Operating and the bases. Technical Specification 3.5.2 for ECCS - Operating VEGP learning objectives: LO-LP-39209-02, Given a set of Tech Specs and the Bases, determine for a specific set of plant conditions, equipment availability, and operational mode: a. Whether any Tech Spec LCOs of section 3.5 are exceeded. LO-LP-39212-02, Given a set of Tech Specs and the Bases, determine for a specific set of plant conditions, equipment availability, and operational mode: a . Whether any Tech Spec LCOs of section 3.8 are exceeded.
Notes

Monday, August 13, 2007 9:45:24 AM

31

QUESTIONS REPORT for HL-14 NRC SRO DRAF T Exam


Response form Answers MCS Time: 1 Points: 1.00 Version: 0 1 2 3 4 5 6 7 8 9 Answer: C AB AAAC C B C Scramble Range: A- D

Monday, August 13, 2007 9:45:24 AM

32

QUESTIONS REPORT for HL-14 NRC SRO DRAFT Exam 14. 078G2.1.32 001/2/1/IAS - PREC & LIMITS/MEM - 2.9/BANK/S/NRC SRO/TNT / RLM The following annunciators / indications are present in the Unit 1 Control Room: - ALB01 window B05 for "SERVICE AIR CMPSR TROUBLE". ALB01 window C06 for "SERVICE AIR HDR LO PRESS". Instrument air pressure is slowly decreasing as read on QMCB meter PI-9361. - All available compressors are running and the Turbine Building Operator (TBO) reports all compressors are loading and unloading properly. The TBO reports an air dryer is malfunctioning. Which ONE of the following is CORRECT actions for the Unit SS to take / direct? A. Direct the TBO to implement SOP-13710-1, "Instrument Air System", if an instrument air dryer malfunction, bypass the instrument air dryer by opening the air dryer bypass, then slowly close the air dryer outlet, then close the air dryer inlet. B. Enter AOP-18028-C, "Loss of Instrument Air", if a service air dryer malfunction, isolate control air to the service air dryer by manually closing the petcock valve located on the air regulator at the dryer inlet. C. Direct the TBO to implement SOP-13711-1, "Service Air System", if a service air dryer malfunction, bypass the service air dryer by bleeding off control air to the dryer by depressing the Sullicon controller pushbutton at the dryer inlet. ao Enter AOP-18028-C, "Loss of Instrument Air", if an instrument air dryer malfunction, place the instrument air dryer in the two chamber full flow mode by depressing the p u s h button at the front end of the dryer.
Feedback

K/A 078 Instrument Air System. G2.1.32 Ability to explain and apply all system limits and precautions. K/A MATCH ANALYSIS Question gives a plausible scenario with either an instrument or service air dryer malfunction. The candidate must pick the appropriate procedure and actions to mitigate the event. ANSWER / DISTRACTOR ANALYSIS

A. Incorrect. Plausible, action is a correct action for a service air dryer.


Monday, August 13, 2007 9:45:24 AM

33

QUESTIONS REPORT for HL-14 NRC SRO DRAFT Exam Instrument air dryers do not have a bypass. 13710-1/2 is the correct procedure for normal instrument air evolutions. B. Incorrect. Plausible, this is the old method for placing a service air dryer in 2 flow full chamber mode but no longer directed per procedure. Note in procedure says NOT to isolate control air to a dryer, an incorrect action. AOP-18028-C, is the proper procedure to use. C. Incorrect. Procedure directs placing in 2 chamber full flow mode by depressing button at front of dryer, not by bleeding off control air. This is method for causing a rotary air compressor to fully load. 13711-1/2 is the correct procedure for normal service air evolutions. D. Correct. Placing air dryer in two chamber full flow mode by depressing the pushbutton at the front of the dryer is the proper response. AOP-18028-C is the proper procedure to use. REFERENCES NOTE: This is a re-use question from Vogtle May 2006 NRC SRO Exam question # 15 with KA # (078A2.01). This is the only question re-used from the last 2 previous SRO exams given at Plant Vogtle. This question will also fit KA # 078G2.1.32 as it tests system limits and precautions for instrument air dryers. AOP-18028-C, "Loss of Instrument Air" SOP-13710-1/2, "Instrument Air System". SOP-13711-1/2, "Service Air System". VEGP learning objectives: LO-LP-60321-06 Describe the operator actions required during normal full power operation when instrument air header pressure falls to < 80 psig or < 70 psig.
Not es Res pon se for m Answers MCS Time: 1 Points: 1.00 Version: 0 1 2 3 4 5 6 7 8 9 Answer: DCACB CAAC C Scramble Range: A- D

Monday, August 13, 2007 9:45:24 AM

34

QUESTIONS REPORT for HL-14 NRC SRO DRAFT Exam 15. 079A2.01 001/2/2/SERVICE AIR - XTIE/C/A - 3.2/NEW/S/NRC SRO/TNT / RLM Given the following conditions: - Unit 1 at 100% RTP and Unit 2 at 55% RTP. All Unit 1 Air Compressors are available. The "Swing' Air compressor has been aligned to Unit 2 and the air headers crosstied. No Unit 2 air compressors are available. Unit 2 air pressure is 73 psig and slowly lowering, service air is isolated. Unit 1 air pressure is 78 psig and slowly lowering for both instrument and service air. - The Unit 1 SERVICE AIR LO PRESS annunciator is illuminated. AOP-18028-C, "Loss of Instrument Air" section A for Mode 1 is in effect. Which ONE of the following are the CORRECT actions that should be performed? Ay' Per AOP-18028-C, isolate / separate the unit air headers and continue with actions of 18028-C, section A for Mode 1. B. Per AOP-18028-C", verify Unit 1 service air isolates, maintain the unit air headers crosstied and continue with actions of 18028-C, section A for Mode 1. C. Per AOP-18028-C, isolate / separate the unit air headers, trip the Unit 2 Main Turbine and implement 18011-C, Turbine Trip Below P-9, continue with 18028-C. D. Per AOP-18028-C, verify Unit 1 service air isolates, maintain the air headers crosstied, trip Unit 2 enter and E-0, perform AOP-18028-C section B for Mode 3.

Monday, August 13, 2007 9:45:24 AM

35

QUESTIONS REPORT for HL-14 NRC SRO DRAFT Exam


F e edback

K/A 079 Station Air. A2.01 Ability to (a) predict the impacts of the following malfunctions or operations on the SAS; and (b) based on those predictions, use procedures to correct, control, or mitigate the consequences of those malfunctions or operations: Cross connection with IAS. K/A MATCH ANALYSIS Question gives a plausible scenario where one unit has lost all available air compressors, the candidate has to determine if crosstie of headers is allowed and at what point to isolate the headers and trip the affected unit. Question meets 10CFR55.43(b) criteria item # 5 - Assessment of facility conditions and selection of procedures during normal, abnormal, and emergency situations. ANSWER / DISTRACTOR ANALYSIS A. Correct. Air headers should be separated, Unit 2 trip criteria of 70 psig not met yet. B. Incorrect. Plausible the candidate may think verification of Unit 1 service air isolation should be verified to stabilize pressure. AOP directs isolation of air headers if Unit 1 pressure drops below 80 psig. C. Incorrect. Plausible the candidate may feel a Unit 2 Turbine trip is imminent and entry into 18012 appropriate. Air headers isolated part is correct. D. Incorrect. Plausible the candidate may feel reactor trip appropriate but Unit trip criteria of 70 psig not reached yet. Also, air headers should have been separated at 80 psig. REFERENCES 18028-C, "Loss of Instrument Air" section A for Mode 1 operations. VEGP learning objectives: LO-LP-60321-06 Describe the operator actions required during normal full power o p eration when instrument air header pressure falls to < 80 psig or < 70 psig.
Notes Response form

Monday, August 13, 2007 9:45:24 AM

36

QUESTIONS REPORT

for HL-14 NRC SRO DRAFT Exam


Answers MCS Time: 1 Points: 1.00 Version: 0 1 2 3 4 5 6 7 8 9 Answer: ABDCDBCDAB

Scramble Range: A- D

Monday, August 13, 2007 9:45:24 AM

37

QUESTIONS REPORT for HL-14 NRC SRO DRAFT Exam 16. 103A2.01 001/2/1/CTMT - ILRT/C/A - 2.6/NEW/S/NRC SRO/TNT / RLM The following conditions exist on Unit 2 while at 100% power. The Shift Manager receives word that the containment air lock has failed the leakage rate surveillance test due to excessive air lock leakage. The Containment overall leak rate is now being exceeded. Which ONE of the following is the CORRECT required Tech Spec action(s)? A. Apply Tech Spec 3.6.1, "Containment", restore Containment to operable status within 7 days or be in Mode 3 in the following 6 hours and in Mode 5 in 36 hours. B. Apply Tech Spec 3.6.2, "Containment Air Locks", within 1 hour close and lock at least 1 Air Lock door. Restore leakage within limits within 7 days or apply Tech Spec 3.6.1 "Containment" actions. Oe Apply Tech Spec 3.6.1, "Containment", restore Containment to operable status within 1 hour or be in Mode 3 in the following 6 hours and in Mode 5 in 36 hours. D. Apply Tech Spec 3.6.2, "Containment Air Locks", immediately close and lock both Air Lock doors. Restore leakage within limits within 7 days or apply Tech Spec 3 . 6 . 1 "Containment" actions.
Feedback

K/A 103 Containment. A2.01 Ability to (a) predict the impacts of the following malfunctions or operations on the containment system and (b) based on those predictions, use procedures to correct, control, or mitigate the consequences of those malfunctions or operations. Integrated Leak Rate Test K/A MATCH ANALYSIS Question gives a plausible scenario where Containment Air Lock doors cause Containment overall leak rate to exceed leakage limits. Note 3 of Tech Spec 3.6.2 states if Air Lock Leakage causes Containment Leakage to exceed limits, apply Tech Spec 3.6.1, "Containment". Candidate must determine the correct Tech Spec actions. Question is also SRO due to KA # for SRO has an importance factor of 2.6 where the KA # for RO only has an importance factor of 2.0.
Monday, August 13, 2007 9:45:24 AM

38

Question meets 10CFR55.43(b) criteria item # 2 - Facility Operating limits in Tech Specs and their bases.

QUESTIONS REPORT for HL-14 NRC SRO DRAFT Exam ANSWER / DISTRACTOR ANALYSIS A. Incorrect. Plausible since Condition A actions of 3.6.2 Air Locks, says to perform actions of 3.6.1 for Containment. Plausible could be a 7 day action. B. Incorrect. Plausible since it is Condition B actions of 3.6.2 Air Locks, but is for interlock mechanisms and implies the plant can run in this condition for 7 days before applying LCO 3.6.1. C. Correct. 3.6.1 for Containment must be applied and 1 hour to bring within limits or shutdown to Mode 3 in 6 hours and be in Mode 5 in 36 hours. D. Incorrect. Plausible since it is close to Condition C actions of 3.6.2 Air Locks but implies Tech Spec 3.6.1 does not have to be applied for 7 days. REFERENCES Tech Spec 3.6.1, "Containment" and bases. Tech Spec 3.6.2 "Containment Air Locks" and bases. VEGP learning objectives: LO-LP-39210-02, Given a set of Tech Specs and the bases, determine for a specific set of plant conditions, equipment availability, and operational mode: a. Whether any Tech Spec LCOs of section 3.6 are exceeded. b. T h e r e q u ired actions for all sections 3.6 LCOs.
Notes Response form Answers MCS Time: 1 Points: 1.00 Version: 0 1 2 3 4 5 6 7 8 9 Answer: CB B C ADB ACD Scramble Range: A- D

Monday, August 13, 2007 9:45:24 AM

39

QUESTIONS REPORT for HL-14 NRC SRO DRAFT Exam 17. G2.1.34 001/3/N/A/CHEMISTY LIMITS/MEM - 2.9/BANK/S/NRC SRO/TNT / RLM Given the following conditions: Unit 1 has just tripped from 100% power and is at no-load temperature and pressure. Chemistry has sampled the primary and secondary plants.

Which ONE of the following CORRECTLY states the Tech Spec limits for RCS Dose Equivalent 1-131 and Secondary Specific Activity? RCS Dose Equivalent 1-131 A. B. C. 0.15 micro curies per gram 1.5 micro curies per gram 0.10 micro curies per gram Secondary Specific Activity 1.5 micro curies per gram 0.15 micro curies per gram 1.0 micro curies per gram 0.10 micro curies per gram

DY 1.0 micro curies per gram

Monday, August 13, 2007 9:45:24 AM

40

QUESTIONS REPORT for HL-14 NRC SRO DRAFT Exam


Feedback

K/A G2.1.34 Ability to maintain primary and secondary plant chemistry within allowable limits. K/A MATCH ANALYSIS Question gives a plausible scenario . Question is SRO and meets 10CFR55.43(b) criteria item # 2 - Facility operating limits in Tech Specs and their bases. Question is also SRO due to the KA Catalog importance factor for this KA # is only a 2.3 for RO and is 3.4 for SRO. Therefore, it is an SRO level question. ANSWER / DISTRACTOR ANALYSIS A. Incorrect. Plausible the candidate may confuse the limit with the values for Chlorides or Fluorides from the TRM and invert the numbers. B. Incorrect. Plausible the candidate may confuse the limit with the values for Chlorides or Fluorides from the TRM. C. Incorrect. Plausible the candidate may invert the correct values for primary and secondary 1-131. D. Correct.

REFERENCES TRM 13.4.1 Reactor Coolant System - Chemistry. HL-AW-39000-00-016 Vogtle SRO Audit question from HL-13. VEGP learning objectives: LO-LP-64101-03, State the LCO applicability and any action statement required within one hour for the following Technical Specifications: a. T.R.M. 13.4.1 Chemistry

b .
Notes

T . S .

3 . 4 . 1 6

S p e c i f i c

A c t i v i t y

Monday, August 13, 2007 9:45:24 AM

41

QUESTIONS REPORT for HL-14 NRC SRO DRAFT Exam


Response form Answers MCS Time: 1 Points: 1.00 Version: 0 1 2 3 4 5 6 7 8 9 Answer: DB D C DA ADDB Scramble Range: A- D

Monday, August 13, 2007 9:45:24 AM

42

QUESTIONS REPORT for HL-14 NRC SRO DRAFT Exam 18. G2.1.5 001/3/N/A/SHIFT STAFFING/C/A - 3.4/BANK/S/NRC SRO/TNT / RLM The shift supervisor is confronted with four personnel scenarios. Which ONE of the following would be acceptable per 10003-C, "Manning the Shift"? A. An SO has called in just prior to shift turnover and will be about an hour late due to a flat tire. This would put staffing 1 below the minimum. It will not be necessary to hold anyone over since he will arrive at the plant within the next hour. B. An SO has unexpectedly failed an FFD test and is not allowed back on site. This puts staffing 1 below the minimum. This would be acceptable for a time period not to exceed 1 shift. De 2 licensed control room operators have become extremely sick due to something they ate and have to leave work. This puts staffing 1 below the minimum. This would be acceptable for a time period not to exceed 2 hours. D. An RO has called in sick just prior to shift turnover. Another RO has been called in and can arrive within 2 hours. This puts staffing 1 below minimum until he arrives. It would not be necessary to hold over the day shift RO.

Monday, August 13, 2007 9:45:24 AM

43

QUESTIONS REPORT for HL-14 NRC SRO DRAFT Exam


Feedback

K/A G2.1.5 Ability to locate and use procedures and directives related to shift staffing and activities. K/A MATCH ANALYSIS Questions gives several plausible scenarios the candidate may encounter as an SRO regarding minimum shift staffing and the candidate has to determine the correct action. Question meets 10CFR55.43(b) criteria item # 2 - Facility operating limits in Tech Specs and their bases. There is no bases for the Shift Mannin Admin requirements. Question also is SRO due to KA importance factor for this topic is only 2.3 for the RO level and 3.4 for the SRO level. Therefore, it is an SRO only topic. ANSWER / DISTRACTOR ANALYSIS A. Incorrect. You can only go below minimum staffing for an exepected absence of duty. In this instance, there would be time to hold shift personnel over to maintain the minimum staffing. Plausible the candidate could think it would be OK to allow the personnel to leave as long as the off duty person arrives within 2 hours or not think of minimun requirements pertaining to non-licensed personnel. B. Incorrect. Miminum for unexpected on duty person is limited to 2 hours. Would need a replacement within 2 hours. Plausible the candidate could think there is one shift to replace the persons since a lot of other admin requirements are within one shift. C. Correct. Could go below minimum due to unexpected absense but would be required to replace the personnel within 2 hours. D. Incorrect. There is a 2 hour time limit for unexpected absence of on duty personnel. Would have to hold someone over. Plausible the candidate would think it is OK to allow the person to leave since a replacement should arrive within 2 hours. REFERENCES 10003-C, "Manning the Shift", REQUIREMENT # 3.8 G2.1.5 from Vogtle October 2005 SRO Audit Exam VEGP learning objectives:

N o t
Notes

a p p l i c a b l e .

Monday, August 13, 2007 9:45:24 AM

44

QUESTIONS REPORT for HL-14 NRC SRO DRAFT Exam


Response form Answers MCS Time: 1 Points: 1.00 Version: 0 1 2 3 4 5 6 7 8 9 Answer: CDDBCBDABC Scramble Range: A- D

45
Monday, August 13, 2007 9:45:25 AM

QUESTIONS REPORT for HL-14 NRC SRO DRAFT Exam 19. G2.2.14 001/3/N/A/CONFIGURATION CHANGE/MEM - 3.0/MODIFIED/S/NRC SRO/TNT / RLM Which ONE of the following situations would require a piece of equipment to be added to the CAUTION Tag log? A. A normally locked open valve is tagged out in accordance with a clearance, the clearance is expected to be released on the following shift. B. Opening a normally closed valve in accordance with a troubleshooting work document that addresses promptly returning the valve to normal alignment. C. Placing a safety related pump handswitch in PTL in accordance to a surveillance procedure and will be placed back to normal by the end of shift. DY A pump breaker is opened per SS direction due to a low oil level, the pump will be restored to normal on the next shift.

Monday, August 13, 2007 9:45:25 AM

46

QUESTIONS REPORT for HL-14 NRC SRO DRAFT Exam


Feedback

K/A G2.2.14 Knowledge of the process for making configuration changes. K/A MATCH ANALYSIS Question gives a plausible scenario where the candidate has to determine which component will require an entry to be made into the CAUTION Tag Log. Question meets SRO only criteria by 10CFR55.43(b) item # 3 - Facility licensee procedures required to obtain authority for design and operating changes in the facility. Positions off normal condition require entry into logs such as OOPL and CAUTION Tag Log and this requires SRO authorization. Also, G2.2.14 in KA Catalog only has an importance factor of 2.1 for the RO level and an importance factor of 3.0 for the SRO level. Therefore, this is an SRO only level question. ANSWER / DISTRACTOR ANALYSIS A. Incorrect. Plausible, however configuration would be controlled by the clearance and would not require a CAUTION Tag Log entry. B. Incorrect. Plausible, not required since procedurally driven by the MWO with specific controls in place to return the equipment to normal configuration. C. Incorrect. Plausible, not required since the surveillance procedure drives this and the component will be placed back in normal by the end of shift. D. Correct. A component is added to the CAUTION TAG Log if it will not be restored prior to the end of shift. REFERENCES 10000-C, "Conduct of Operations" section 4.6 "Tracking Out of Position Components". Vogtle May 2006 SRO Retake Exam question # 18 Watts Bar July 2004 NRC SRO Exam question # 21. VEGP learning objectives: LO-LP-63500-03, Describe General work practices associated with Conduct of Operations. i. Equipment return to service.

Monday, August 13, 2007 9:45:25 AM

47

QUESTIONS REPORT for HL-14 NRC SRO DRAFT Exam


N o t e s

Response form Answers MCS Time: 1 Points: 1.00 Version: 0 1 2 3 4 5 6 7 8 9 Answer: DAC C CB CABC Scramble Range: A- D

Monday, August 13, 2007 9:45:25 AM

48

QUESTIONS REPORT for HL-14 NRC SRO DRAFT Exam 20. G2.2.7 001/3/N/A/TESTS NOT IN FSAR/MEM - 3.2/BANK/S/NRC SRO/TNT / RLM A new system engineer has requested that CCP 1A be started with the discharge valve throttled to determine current and flow rate data under these conditions. Which ONE of the following describes the process for evaluating this test and who can perform this evaluation ? The 10CFR50.59 process is required to determine if ......... A. NRC review of the results is required after conducting the test and the 50.59 evaluation can be performed by any currently licensed SRO. By" NRC approval is required prior to conducting the test and the 50.59 evaluation can only be performed by a Qualified Reviewer. C. PRB approval is required prior to conducting the test and the 50.59 evaluation can be performed by any currently licensed SRO. D. PRB review of the results is required after conducting the test and the 50.59 evaluation can only be performed by a Qualified Reviewer.

Monday, August 13, 2007 9:45:25 AM

49

QUESTIONS REPORT for HL-14 NRC SRO DRAFT Exam


F e edback

K/A G2.2.7 Knowledge of the process for conducting tests or experiments not described in the safety analysis report. K/A MATCH ANALYSIS Question gives a plausible scenario where engineering wants to perform a test on a safety related MOV. The candidate must determine what is required to allow performance of the test. Questions meets 10CFR55.43(b) criteria for item # 3 - Facility license procedures required to obtain authority for design and operating changes in the facility. Question also is SRO only due to the KA catalog # for RO only rates a 2.0 importance factor while the rating is 3.2 for SRO. Therefore, question is an SRO only question. ANSWER / DISTRACTOR ANALYSIS A. Incorrect. Plausible candidate may think an NRC review of the test results is required and any qualified SRO may perform. B. Correct. Determines if NRC approval is required and a Qualified Reviewer to review. C. Incorrect. Plausible the candidate may think the PRB is highest level of approval but 10CFR50.59 is to determine if NRC approval required. Also, plausible the candidate may think the SRO level is all that is required. D. Incorrect. Plausible the candidate may think a PRB review of the test results is required and must be performed by a qualified reviewer. REFERENCES NMP-AD-010, "10CFR50.59 Screening and Evaluations" Oconee June 2004 NRC SRO Exam question # 21 VEGP learning objectives:

L O-LP-63400-01 Ex lain the urose of slant design control SRO onl


Notes Response form

Monday, August 13, 2007 9:45:25 AM

50

QUESTIONS REPORT

for HL-14 NRC SRO DRAFT Exam


Answers MCS Time: 1 Points: 1.00 Version: 0 1 2 3 4 5 6 7 8 9 Answer: BAAC CADC CB

Scramble Range: A- D

Monday, August 13, 2007 9:45:25 AM

51

QUESTIONS REPORT for HL-14 NRC SRO DRAFT Exam 21. G2.3.6 001/3/N/A/REVIEW RLEASE PERMIT/C/A - 3.1/NEW/S/NRC SRO/TNT / RLM Given the following: The Shift Supervisor has declared A-RE-0014 Inoperable. The radiation monitor will not come off the low end of scale. Which ONE of the following statements is CORRECT regarding approving a permit for the release of Waste Gas Decay Tank # 3? A. Approve. As long as the Shift Manager and the Chemistry Manager concurrently give permission to perform the release. B. Disapprove. The release CANNOT proceed because the discharge flow path cannot be aligned with A-RE-0014 failed offscale low. C. Disapprove. The release CANNOT proceed until A-RE-0014 has been returned to Operable status in accordance with ODCM requirements. DY Approve. As long as independent samples of tank contents are analyzed and the discharge valve alignment and release rate calculations are independently verified.

Monday, August 13, 2007 9:45:25 AM

52

QUESTIONS REPORT for HL-14 NRC SRO DRAFT Exam


Feedback

K/A G2.3.6 Knowledge of the requirements for reviewing and approving release permits. K/A MATCH ANALYSIS Question gives a plausible scenario where A-RE-0014 in Inoperable and a release permit needs to be approved. The candidate must determine which requirement must be met to approve the release permit. Question meets 10CFR55.43(b) SRO criteria item # 4 - Radiation hazards that may arise during normal and abnormal situations, including maintenance actitivies and various contamination conditions. Question meets 10CFR55.43(b) SRO criteria item # 2 - Facility operating limits in Tech Specs and their bases. Question is also SRO only from KA Catalog since RO importance factor is only 2.1 and SRO importance factor is 3.1 for this topic. Therefore, question is SRO only. ANSWER / DISTRACTOR ANALYSIS A. Incorrect. Plausible the candidate may think a higher level of authority from the two departments who normally approve releases is required. B. Incorrect. Plausible the candidate may think that the RE failure prevents opening RV-0014, this would be true on high failure, but not low. C. Incorrect. Plausible the candidate may think RE-0014 inoperable would prevent the release, ODCM actions allow. D. Correct. ODCM action # 45 requirements. REFERENCES

ODCM Table 3-1 for Radioactive Gaseous Effluent Monitoring Instrumentation for ARE-0014, action # 45. Similar to Vogtle 2005 SRO Retake but for a different system and ODCM action. VEGP learning objectives: LO-PP-46101-10, Describe the major steps involved in releasing a gas decay tanks contents to the environment. LO-PP-46101-15, State the LCO, ODCM LCO, or TR, applicability, and any one hour or less actions for the GWPS.
Monday, August 13, 2007 9:45:25 AM

53

QUESTIONS REPORT for HL-14 NRC SRO DRAFT Exam


N o t es Response form Answers MCS Time: 1 Points: 1.00 Version: 0 1 2 3 4 5 6 7 8 9 Answer: DAAAACDB CD Scramble Range: A- D

Monday, August 13, 2007 9:45:25 AM

54

QUESTIONS REPORT for HL-14 NRC SRO DRAFT Exam 22.


G2.3.8 001/3/N/A/GAS RELEASE PROCESS/C/A - 3.2/BANK/S/NRC SRO/TNT / RLM

A release of Waste Gas Decay Tank # 1 is in progress when the following occurs during the tank release. RE-0013, Waste Gas Process Monitor fails and is declared Inoperable. Auxiliary Building Continuous Exhaust Unit # 1 has just tripped on low flow. Waste Gas Decay Tank # 2 pressure is noted to be lowering by the ABO. - The inlet Oxygen analyzer on the recombiner panel is declared Inoperable. Which ONE of the conditions listed above would REQUIRE release termination? A. RE-0013 monitor Inoperable. B. Auxiliary Building Continuous Exhaust trip.

0'

Waste Gas Decay Tank # 2 pressure lowering. D. Inlet Oxygen analyzer for recombiner inoperable.

Monday, August 13, 2007 9:45:25 AM

55

Feedback

QUESTIONS REPORT for HL-14 NRC SRO DRAFT Exam

K/A G2.3.8 Knowledge of the process for performing a planned gaseous radioactive release. K/A MATCH ANALYSIS Question gives a plausible scenario where a Waste Gas Decay Tank release is in progress with several malfunctions occurring. The candidate must determine which of the malfunctions would require termination of the release. Question meets 10CFR55.43(b) criteria item # 4 - Radiation hazards that may arise during normal and abnormal situations, including maintenance activities and various contamination conditions. Question is also SRO only since RO importance factor is 2.3 for this KA #, the SRO importance factor for this KA # is 3.2. Therefore, this is an SRO only question. ANSWER / DISTRACTOR ANALYSIS A. Incorrect. RE-0013 not required per ODCM for Waste Gas Decay Tank release. Plausible the candidate may consider this since it is a gaseous radiation monitor but would only indicate radiation during processing, not releases. Not required to terminate the release. B. Incorrect. Plausible the candidate may consider an Aux. Building Exhaust unit trip as requiring to terminate the release as it could affect Aux. Building Pressure. C. Correct. Procedure specifically states to stop release if more than 1 tank lowering. D. Incorrect. Plausible but no procedure requirement to terminate due to this. Candidate may consider possible 02 or H2 concentrations a reason to stop the release. REFERENCES SOP-13202-1/2, "Gaseous Releases"step 4.2.13 and preceeding CAUTION. Vogtle 2002 NRC SRo Exam question # 85 VEGP learning objectives: LO-PP-46101-11, State the events that require immediate termination of a gaseous release.
Notes

Monday, August 13, 2007 9:45:25 AM

QUESTIONS REPORT for rotest


1. 001AK1.05 001

The following conditions exist: - Reactor power = 100% - Control Bank D is at 137 steps withdrawn - Rod control is in AUTO If PT-505 fails LOW, how will the rods in Control Bank D respond? A. Move inward at 48 steps per minute. B.' Move inward at 72 steps per minute. C. Move outward at 72 steps per minute. D. Move outward at 48 steps per minute. 2. 001K1.05 001 Given the following: - 75% power - Channel N-41 - 74% - Channel N-42 - 73% - Channel N-43 - 75% - Channel N-44 - 74% - Rod control is in Automatic Which of the following describes Rod Control system response to channel N-41 failing low? A. Control Rods drive in at a maximum rate until C-5 blocks rod motion or a reactor trip on low prz press occurs. B. Control Rods drive in until the temperature mismatch equals the power mismatch and Tavg stablizes at a lower temperature. C' Control Rods remain in present position until power mismatch causes a signal to move. D. Cont ro l Rods d r ive out unt il the temperatu re m is m atch equa ls the power m ism at ch and Tavg stab li zes at a lower tem perature .

Tuesday, January 07, 2003 02:21:26 PM

QUESTIONS REPORT
3. 0021(5.10 002
Given the following conditions: - Tavg is on program - Unit 1 is at 94% power and ramping up. - Rods are in automatic with Bank D at 200 steps - Turbine load set is raised to 1220 MWe using the increase pushbutton - Turbine control valves are opening and megawatts are increasing for rotest

Which ONE of the following describes Tavg behavior assuming no operator action? A. Tavg and Tref will increase and continue to be matched until the turbine reaches set load. B. Tref will increase until the turbine reaches set load, but Tavg will remain constant. C. Tavg and Tref will remain constant and matched as the turbine load increases. D' Tavg will decrease and Tref will increase until the turbine reaches set load.

4. 003A2.02R 002 Given the following: - Unit 1 is at 39% power - Annunciator ALB08 A05 "RCP 1 Controlled LKG HI/LO FLOW" is illuminated - RCP 1 Seal Inlet Temperature is at 240F and increasing. Which ONE of the following describes the correct sequence of actions for the control room crew? A. Trip RCP 1, close #1 Seal Return valve, trip the Rx and initiate 19000-C, "E-0, Rx Trip or Safety Injection" 13:' Trip the Rx and initiate 19000-C, "E-0, Rx Trip or Safety Injection," trip RCP 1, and close #1 Seal Return Valve C. Trip RCP 1, close #1 Seal Return valve and continue in 12004-C, "Power Operation (MODE 1)" D. Trip RCP 1, close #1 Seal Return valve and initiate 18005-C, "Partial Loss of Flow"

Tuesday, January 07, 2003 02:21:26 PM

5. 00302.4.49 002 At 11:00 you are notified that RCP #1 ACCW inlet line is leaking badly. Maintenance was notified and is in the process of determining what type of repairs are needed. At 11:05 via plant computer and other control room indications, you determine that the following conditions exist:
Motor bearing temperature is 195 of and rising at 1 of / min Motor stator winding temperature is 315 OF and steady Seal water Inlet temperature is 190 OF and steady Based on the above conditions, what action(s) should be taken? Immediately trip the RCP B. Trip the RCP if ACCW to the pump is not re-established by 11:16 with total #1 seal flow greater than 5 gpm C. Trip the RCP if ACCW flow is not re-established by 11:10 D. Trip the RCP, then the reactor if seal water temperature is not returned to 230 of by 11:16 6. 004A2.12 001 Given the following: - Unit 2 is at 100% power - CCP "A" is in service, providing normal charging flow - An inadvertent "B" train SI was generated by l&C - SI "A" train is NOT present - No operator action takes place Which of the following is correct? A. Normal mini-flow paths for both CCPs are isolated, alternate flow-paths for both CCPs are available. B.' Normal mini-flow paths for both CCPs are isolated, CCP "A" alternate miniflow path is isolated, CCP "B" alternate mini-flow path is available. C. CCP "A" normal mini-flow path is available, CCP "A" alternate miniflow path is isolated, CCP "B" alternate mini-flow path is available. D. Normal mini-flow paths for both CCPs are isolated, alternate flow-paths for both CCPs are isolated.

QUESTIONS REPORT for rotest

Tuesday, January 07, 2003 02:21:26 PM

QUESTIONS REPORT for rotest 7. 004A4.07 001 Which of the following describes the proper reactor makeup control system valve positions after the Makeup Control Switch is placed to the START position with the Mode selector switch in DILUTE? A!' Boric Acid to Blender Valve (FV-0110A) - CLOSED Blender Outlet to Charging Pumps Suction Valve (FV-0110B)CLOSED Blender Outlet to VCT Valve (FV-0111B)- OPEN RX MU WTR to BA Blender Valve (FV-0111A)- MODULATED B. Boric Acid to Blender Valve (FV-0110A) - CLOSED Blender Outlet to Charging Pumps Suction Valve (FV-0110B)- OPEN Blender Outlet to VCT Valve (FV-0111B)- MODULATED RX MU WTR to BA Blender Valve (FV-0111A)- CLOSED C. Boric Acid to Blender Valve (FV-0110A) - OPEN Blender Outlet to Charging Pumps Suction Valve (FV-0110B)OPEN Blender Outlet to VCT Valve (FV-0111B)- CLOSED RX MU WTR to BA Blender Valve (FV-0111A)- MODULATED D. Boric Acid to Blender Valve (FV-0110A) - MODULATED Blender Outlet to Charging Pumps Suction Valve (RI-0110B)MODULATED Blender Outlet to VCT Valve (FV-0111B)- OPEN RX MU WTR to BA Blender Valve (FV-0111A)- OPEN 8. 004K4.04 002 Unit 1 is recovering from a remote shutdown outside the control room. The Auxiliary Building SO is currently locally controlling CVCS charging flow using 1FHC-0121 outside the charging pump rooms. Which of the following is the correct method of transfering charging flow control back to the control room? A. Place both the local (FHC-0121) and control room (FIC-0121) controllers at 0% demand, then place the charging pump transfer switch in the control room position. Match the control room demand (on FIC-0121) with the local controller (FHC-0121), then place transfer switch in the control room position. C. Place the local controller in "REMOTE", the control room demand automatically tracks the local demand. D. Place the control room controller (FIC-0121) in "AUTO", no local action is required.

Tuesday, January 07, 2003 02:21:26 PM

QUESTIONS REPORT for rotest 9. 004K5.04 001 Which one of the following is the basis for maintaining a hydrogen cover gas in the VCT during normal at power operations? A. To assure N-16 concentrations are ALARA B. To maintain RCS pH within limits C. To maintain conductivity below .017 micro Mhos D!' To maintain oxygen concentration within limits
10. 005AK2.02R 001

Unit 1 was at 25% power and ramping up when the RO noticed that one of the Control Bank C control rods is 13 steps below the other rods in Control Bank C which were at 215 steps and moving as designed. Power accension was halted. The rod had an electrical problem which was repaired and the operations management staff has concurred with realignment of the misaligned rod in accordance with 18003-C, Rod Control System Malfunction. Which ONE of the following outlines the method of realigment in accordance with AOP 18003-C? A. Record information from Bank Overlap Unit, step counters, and P/A converter. Disconnect lift coil for the affected rod, reset step counters, select Bank C and insert Bank C control rods. Log the final rod position. B. Reset Master Cycler, record information from Bank Overlap Unit, step counters, and P/A converter. Disconnect lift coil of the affected rod, select Bank C and insert Bank C. Log final position of affected rod. C' Position Rod Bank Selector Switch to affected bank, disconnect all lift coils in Bank C except the affected rod, and withdraw affected control rod, reset the affected group step counter to recorded position. Log the final rod position. D. Reset master cycler, record information for P/A converter, and step counters. Disconnect all lift coils in Bank C except the affected rod, select Bank C and withdraw the affected rod.

Tuesday, January 07, 2003 02:21:27 PM

QUESTIONS REPORT
for rotest 11. 005K5.03R 001 Unit 1 is in Mode 4 with #1 Reactor Coolant Pump running. "A" train RHR has JUST been placed in service when the Reactor Operator notes source range counts suddenly increasing on both channels. ALB10 C01 "SOURCE RANGE HI FLUX LEVEL AT SHUTDOWN" is illuminated. Which ONE of the following describes the correct action? A!' Commence an emergency boration per 13009-1, CVCS Reactor Makeup Control System. B. Start the #2 Reactor Coolant Pump and secure the #1 Reactor Coolant Pump. C. Start "B" Train RHR and secure "A" train RHR per 13011-1, Residual Heat Remaoval System. D. Commence a normal boration until the source range counts begin to decrease.

12. 006A1.11 002 Given the following plant conditions:


-

SGTR has occurred on SG #1. 19031, "Post-SGTR Cooldown Using Backfill", is in

progress. Ruptured SG level is 25% NR. Crew is cooling down using steam dumps to condenser. RCP #4 in service.

19031, "Post-SGTR Cooldown Using Backfill" - Step 14 requires a return to step 3 if RCS temperature is greater than 200F. - Step 3 requires the operator to ensure adequate shutdown margin. Why is it necessary to re-verify shutdown margin at this point in the procedure? A. The RCS temperature change during cooldown will cause significant boron concentration changes due to PZR outsurge. B. Charging to maintain PZR level during cooldown will cause significant boron concentration changes. C:' The secondary fluid in the ruptured SG will cause a significant decrease in RCS boron concentration. D. Using auxiliary spray will cause a significant decrease in RCS boron concentration changes.

Tuesday, January 07, 2003 02:21:27 PM

QUESTIONS REPORT for rotest


13. 006K1.07 002

Given the following sequence of events: The plant is operating at 100% power. Safety Injection actuated Reactor tripped due to the SI signal Controlling #1 SG level transmitter fails low after the trip #4 SG ARV opened momentarily and developed a large packing leak. Which ONE of the following would cause the INITIAL main feedwater isolation during this transient? A. The #1 SG level reached 86%. 13:" The safety injection actuation signal. C. Tavg dropping to 564F following the reactor trip. D. Main Steam Valve Room level had risen due to the ARV packing leak.

14. 008AK1.01 001 Given the following: - Unit 1 is stable at 100% power - A pressurizer safety valve opens and fails to reseat, remaining 25% open and the Unit trips - RCS pressure stabilizes at 1600 psig - SI acuates Which of the following indications would the operator expect to see as a result of this event in the next 30 minutes? A. Safety tailpipe temperature would increase to greater than 600 OF and then decrease to 450 OF. B. Safety tailpipe temperature would increase to greater than 600 OF and then slowly increase. C." Safety tailpipe temperature would increase to between 220 and 340 OF and then decrease and stablize. D. Safety tailpipe temperature would increase to between 220 and 340 of and then slowly increase and stablize at 600 OF

Tuesday, January 07, 2003 02:21:27 PM

QUESTIONS REPORT for rotest 15. 008K1.02R OW Which ONE of the following correctly describes the uses of ACCW in the containment building. A. B. C. RCP motor coolers, Seal Water HX and RCP thermal barriers. RCP motor coolers, RCP thermal barriers and CVCS Letdown Regen HX. RCP motor coolers, RCP thermal barriers and Normal Letdown HX. 13:'

RCP motor coolers, RCP thermal barriers and CVCS Excess Letdown HX.

16. 009BA2.39R 002 Given the following conditions: Small break LOCA in progress - RCS pressure is at 1100 psig No SI pumps - No Charging pumps Which one of the following explains why RCPs are NOT tripped under these conditions? IV To provide heat removal via the Steam Generators. B. To maintain two phase mixture level above the break longer. C. To limit single phase inventory loss out of the break. D. To prevent boron stratification in the core.

17. 010K3.01 001


Given the following: Pressurizer pressure is 2230 psig and increasing. The proportional heaters are energized. The spray valves are closed. The Master Pressure Controller fails to a constant output equivilent to 2219 psig. Which ONE of the following describes the response of the pressure control system if the operator takes no further action? A. Pressure will rise until the spray valves open to control pressure. B.' Pressure will rise until PORV 456 opens to control pressure. C. D. Pressure will rise until PORV 455 opens to control pressure. Pressure will cycle on the variable heaters at a higher setpoint.

Tuesday, January 07, 2003 02:21:27 PM

QUESTIONS REPORT for rotest 18. 011A3.03 001 Given the following: Unit is at 50% power. All automatic control systems are in their normal lineup. Pressurizer program level sticks at constant output for 50% power. Assume no operator action is taken. Which of the following describes the effect on charging flow and PZR level as the plant load is increased to 100%? A. B. Actual PZR level remains constant and charging flow increases. Actual PZR level decreases and charging flow decreases.

C:' Actual PZR level increases and charging flow decreases. D. Actual PZR level increases and charging flow remains constant. 19. 011EK2.02 001 Given the following conditions: A large break LOCA occurred Operators have just completed 19013-C, "Transfer to Cold Leg Recirculation" A loss of offsite power occurs Which ONE of the following describes the actions required for this condition? A. Pull to lock SIPs and CCPs until the RHR pumps are started by the blackout sequencer afte the diesel generators start and load the 4160 vital buses. B..' Ensure the RHR pumps are manually started after the diesel generators start and load the 4160 vital buses, then manually start SIPs as needed. C. Ensure both RHR pumps are started by the blackout sequencer after the diesel generators start and load, then manually start CCPs and SIPs as needed D. Ensure all ECCS pumps are started by the blackout sequencer when the diesel generators reenergize 4160 vital buses.

Tuesday, January 07, 2003 02:21:27 PM

QUESTIONS REPORT for rotest 20. 012K6.10 001 Given the following conditions - Unit shutdown in progress - Power at 9% - Permissive "P-7 LO POWER TRIPS BLOCKED" illuminates Which ONE of the following describes the effects on RPS? A. The reactor will not trip on Pressurizer High Pressure. B. The reactor will not trip on high positive rate. C:' The reactor will not trip on Pressurizer Low Pressure. D. The reactor will not trip on low steam generator level. 21. 013K2.01 002 Given the following: - SIP 1A & 1B are in Auto - Control power is lost to SIP 1A - Safety injection (SI) occurs Which of the following describes the response of the SI pumps to the Safety Injection signal? A!' SIP 1B will start, but SIP 1A will not auto start or manually start from the QMCB handswitch. B. SIP 1B will start, but SIP 1A will not auto start and must be started from QMCB handswitch. C. Both SI pumps will auto start, but the SIP 1A can not be stopped from the QMCB. D. Both SI pumps will auto start, but the SIP 1A can not be stopped except mechanically at the breaker. 22. 013K6.01R 002 An upscale failure of which one of the following will directly result in an automatic ESFAS actuation? A. Containment pressure instrument, PT-936. B. Control Room Area monitor, RE-0001. C. Pressurizer Pressure instrument, PT-455. D:' Control Room Air Intake monitor, RE-12116.

Tuesday, January 07, 2003 02:21:28 PM

10

QUESTIONS REPORT for rotest 23. 014A4.01 002 Given the following: - Operators are conducting a reactor startup - ALL shutdown banks are withdrawn - Control banks are being withdrawn in 50 step increments - The SS notices that Control Bank A is at 200 steps and Control Bank B is at 0 steps Which of the following explains these indications? A. Rod bank selector switch is in the manual position. 13:' Rod bank selector switch is in the Control Bank A postion. C. D. The rod control startup switch is stuck in the reset postion. DRPI Data B failure has occurred.

24. 015G2.2.12R 002 NIS channel check surveillance is required every 12 hours by technical specfications. The last channel check was completed at 0615 on 10/31/02. Outage activities are hindering completion of this surveillance. The NIS channel would be declared INOPERABLE if the next channel check is not completed by: AO' 2115 on 10/31/02 B. 2215 on 10/31/02 C. 0615 on 11/01/02 D. 1215 on 11/01/02

25. 015K4.06 002 Given the following conditions: - Reactor power is at 18% with a startup in progress - N41 was placed in bypass (BTI) following a failed surveillance How does this affect the coincidence for a power range High Flux Reactor Trip? A. 2 out of 4 power range channels at 25% reactor power will generate a reactor trip B. 2 out 3 power range channels at 25% reactor power will generate a reactor trip C! 2 out 3 power range channels at 109% reactor power will generate a reactor trip D. 2 out of 4 power range channels at 109% reactor power will generate a reactor trip
Tuesday, January 07, 2003 02:21:28 PM

11

QUESTIONS REPORT for rotest 26. 015K5.04 001 Manual calibration of the NIS is being performed in accordance with procedure 14030-2, "Nuclear Instrument Calorimetric Calibration." Feedwater average temperature is incorrectly calculated to a value 17 degrees less than actual. For these conditions, which of the following is correct? Calculated reactor thermal power will be ... A. lower than actual and a gain adjustment of the NI channels using the calculated value would be non-conservative such that the indicated power is farther from the setpoints. B. higher than actual and a gain adjustment of the NI channels using the calculated value would be non-conservative such that the indicated power is farther from the setpoints. C. lower than actual and a gain adjustment of the NI channels using the calculated value would be non-conservative such that the indicated power is closer to the setpoints. a' higher than actual and a gain adjustment of the NI channels using the calculated value would be conservative such that the indicated power is closer to the setpoints.

27. 016K3.02 001 With Pressurizer Level Control Selector Switch LS-459D selected to 459/460, the following SEQUENTIAL plant events occur due to a failure without operator action.
- Charging flow increases to maximum - Pressurizer level begins to rise - Letdown isolates and heaters turn off - Pressurizer level eventually rises to the high level reactor trip Which ONE of the following failures occurred? A. B. Level channel LT-469 failed high. Level channel LT-460 failed

high. C" Level channel LT-459 failed low. D. Level channel LT-460 failed low.

Tuesday, January 07, 2003 02:21:28 PM

12

QUESTIONS REPORT for rotest

28. 017AK1.02 002 IF RCP #1 trips when the plant is at 10% power during a load increase to 100%, which one of the following statements is correct?
A. A reactor trip will occur and operators should implement 19000-C. B.' The affected pressurizer spray valve should be shut to prevent spray flow from bypassing the pressurizer. C. The affected SIG blowdown rate may be isolated to facilitate level control. D. Reactor trip breakers should be immediately opened to comply with the action statement for LCO 3.4.4, "RCS Loops-Modes 1 and 2".

29. 017K3.01 002

Given the following conditions: Reactor trip .- All RCPs are tripped All core-exit thermocouples are inoperable - 19001-C is in progress Which one of the following describes the indication that could be used to verify that natural circulation cooling is in progress A. B. RCS loop Tave's stable or lowering. RCS loop loop Tave's increasing to full power values.

C:' Steam Generator Pressures Stable or lowering. D. Steam Generator Levels Stable or lowering. 30. 022AG2.1.32 001 During water solid operations with letdown from RHR, procedure 13011-1, "Residual Heat Removal System" requires that 1-HV-0128, Letdown From RHR, be full open. Which ONE of the following describes the basis for this precaution? A. To ensure maximum letdown flow rate for purification. B. To ensure VCT level can be maintained under all charging flow conditions. C:" To ensure 1-PIC-0131, Low Pressure Letdown Controller can control pressure transients. D. To ensure RCS to RHR Supply Line Relief Valves PSV-8708B and PSV-8708A are not challenged.

Tuesday, January 07, 2003 02:21:28 PM

13

QUESTIONS REPORT for rotest 31. 024AK2.01 002 Given the following conditions: - 2 rods stick out on a reactor trip The RO initiates emergency boration using HV-8104 Boric flow on FI-0183A is 23 gpm and charging flow is 42 gpm Which of the following is the correct response to this condition? The RO can correct this condition by ... A. B. Placing charging flow controller FIC-0121 in manual and increasing the set point to >42 gpm, ensuring charging flow increases and FI-0183A indicates >30 gpm. Opening FV-110A, closing FV-110B, and ensuring FI-0183A indicates >30 gpm.

C:' Opening HV-112D and HV-112E, then closing HV-11213 and HV112C, ensuring charging flow is >87.5 gpm. D. Opening HV-112B and FIV-112C, then closing HV-112D and HV112E, ensuring charging flow is >87.5 gpm.

32. 025AK2.02 001 Given the following: - RCS temperature is 118 of - Reactor Vessel head is removed - Reactor Upper Internals are installed in the reactor vessel Refueling Level is 187.9 ft RCS draining is in process at 10 gpm RHR pump A is running with indicated flow of 3800 gpm RHR pump A is exhibiting indications of cavitation The cavitation and resulting loss of RHR is occurring due to .. A. draining with the upper internals in place, which reduced the RHR suction pressure. B. steam binding of the RHR pump, caused by low recirculation flow. C' air entrapment at the RHR suction inlet, caused by the high flow conditions. D. draining with the upper internals in place, which reduced the RHR discharge pressure.

Tuesday, January 07, 2003 02:21:28 PM

14

QUESTIONS REPORT for rotest 33. 026A1.01 002 Given the following conditions on Unit 2: - Following a LOCA, containment pressure is rising. - Containment pressure has reached 20 psig on three channels and 22 psig on one channel. Which one of the following are the MINIMUM actions that would result in containment spray actuation? A. When 1 more containment pressure channel indicates 22 psig, or when 1 manual handswitch is actuated. 131' When 1 more containment pressure channel indicates 22 psig, or when 2 manual handswitches are actuated. C. When 2 more containment pressure channels indicate 22 psig, or when 1 manual handswitch is actuated. D. When 2 more containment pressure channels indicate 22 psig, or when 2 manual handswitches are actuated.

34. 026AK3.01 002 Given the following plant conditions: - Unit 2 is in Mode 3 - ALB 04, window A2, "ACCW LO HDR PRESS" is illuminated - ALB 07, window D3, "LTDN HX OUT HI TEMP" is illuminated Which one of the following events would cause both of these alarms? A. Letdown Hx Tube Rupture 13! ACCW Supply Header Rupture C. D. Loss of Seal Injection Loss of Charging Flow

Tuesday, January 07, 2003 02:21:28 PM

15

QUESTIONS REPORT for rotest 35. 027AK3.03 002 Given the following conditions: - Unit 1 is at 100% power. - ALB12 E01 "PRZR RELIEF DISCH HI TEMP" illuminates - Both PORVs indicate closed. - PORV-455 tailpipe temperature is reading 220 degrees F. - PORV-456 tailpipe temperature is reading 187 degrees F. - Pressurizer pressure is lowering. Which one of the following is the correct action in accordance to ARP 17012-1? A. Close the associated block valve for PORV-456 because a vapor-space leak causes PZR level to increase. B. Close both block valves because a vapor-space leak causes PZR level to increase. C:' Close the associated block valve for PORV-455 to stop leakage to the PRT. D. Close both block valves to stop leakage to the PRT. 36. 029EA1.13 001 Which ONE of the following is the NEXT action the operator is required to take if the main turbine does NOT trip automatically and CANNOT be tripped from the QMCB per 19211-C, FR-S.1, "RESPONSE TO NUCLEAR POWER GENERATION/ATWT"? A. Place both EHC pumps control switches in P-T-L. B. Trip the turbine locally at the front standard. C:' Manually Runback the turbine. D. Shut the MSIV's. 37.029K1.02 001 RE-2565 samples the _________ and initiates a Containment alarm. A. Cntmt Purge Exhaust; Purge trip 13:' Cntmt Purge Exhaust; Vent Isolation C. Cntmt Purge Supply; Vent Isolation D. Cntmt Purge Supply; Purge trip

___________upon E.

Tuesday, January 07, 2003 02:21:28 PM

16

QUESTIONS REPORT for rotest 38. 032AK1.01 002 Given the following conditions: - Reactor Startup in progress - Startup is on hold due to a problem with SR N-31 - SR N-32 indicates 1000 cps - SR N-31 is in bypass Which ONE of the following will occur if the control power fuse for SR N-31 blows? A. Lose indication for SR N-31 on Main Control Board and MS cabinets. B. Both SR drawers deenergize and the "non-operate" alarm acuates. C' Reactor will trip. D. Rod withdrawal is blocked. 39. 033G.2.4.18R 002 Given that CCW has been lost on Unit 1. Actions in 18030-C "Loss of SFP Cooling" direct the operators to feed & bleed the SFP. Which of the following is the correct action to take? A. Feed the SFP with RWST and bleed with SFP pump "A" simultaneously. B:" Feed the SFP with RWST and bleed with SFP pump "B" simultaneously. C. Feed the SFP with RMWST and then bleed with SFP pump "A" D. Feed the SFP with RMWST and then bleed with SFP pump "B" 40. 034K4.01 001 Which one of the following describes a feature of the Refueling Machine designed to prevent the accidental release of a fuel assembly? A. The Gripper is mechanically engaged and disengaged by a remote operating handle on the bridge and requires no power or air to operate. B:' The gripper requires air to disengage, however, a mechanical latch prevents gripper release under load even if air is supplied. C. The gripper will disengage upon loss of air, however, a mechanical latch prevents gripper release under load even if air is removed. D. When the gripper is engaged, fuel handlers mechanically lock gripper in place with extension shaft which must be unlocked before the gripper can release.

Tuesday, January 07, 2003 02:21:28 PM

17

QUESTIONS REPORT for rotest 41. 035K1.09 002 Given the following plant conditions: The reactor is operating at 50% power. Rod control is in MANUAL. Turbine control is at set load. #3 SIG ARV fails OPEN. Which ONE of the following describes the resulting steady-state conditions? (Assume no reactor trip, no operator action and turbine power remains constant) A!' Final Tavg < initial Tavg and final power > initial power. B. C. Final Tavg < initial Tavg and final power = initial power. Final Tavg = initial Tavg and final power > initial power.

D. Final Tavg = initial Tavg and final power = initial power. 42. 037AK3.07 001 Given the following: - A SG #3 has a 30 gpm leak. - AOP 18009-C, Steam Generator Tube Leak is being implemented. - SIG #3 is maintained greater than 10%. Which ONE of the following is a bases for ensuring the affected SG level greater than 10%? A. To ensure that the pressure and temperature limits of the SG shell are maintained. 13:' To prevent RCS cooldown from causing depressurization of the affected SG. C. To prevent SG overfill.

D. To prevent thermal shock to the tubes during RCS cooldown.

Tuesday, January 07, 2003 02:21:28 PM

18

QUESTIONS REPORT for rotest 43. 039A1.03 002 Given the following: - Unit 2 is in mode 3 at NOPT for a post refueling start up - MSIVs are shut and the main steamlines need to warmed - SG ARVs are in manual, controlling loop Taves at 557 OF Which of the following correctly describes the method used and the indications observed while warming the main steamlines? A. Slowly open MSIVs one at a time. RCS pressure will remain constant while temperature decreases. B. Open steam dumps to 1% to 2% after placing both bypass switches in the bypass position. RCS temperature and pressure will decrease. C:' Open MSIV bypass valve(s) and maintain MSIVs shut. RCS temperature and pressure will decrease. D. Open MSIV bypass valve(s) after placing both steam dump bypass switches in the bypass position. RCS temperature and pressure will decrease.

44. 040AG2.4.4 002 Given the following plant conditions: - Unit was at 100% power - A main steam line break occurred in the Turbine Building - Operators were unable to close the MSIVs and transitioned to procedure 19121-C, ECA-2.1, "Uncontrolled Depressurization of All Steam Generators." SI termination steps of ECA-2.1 are in progress - Loop 3 MSIV is closed locally The CRO observes the # 3 S/G pressure rising slowly Which of the following actions should be performed? A. B. C. Immediately transition to E-2, "Faulted S/G Isolation" Immediately transition to ES-1.1, "SI Termination" Remain in ECA-2.1 until RHR is in

service D:' Remain in ECA-2.1 until SI is terminated

Tuesday, January 07, 2003 02:21:28 PM

19

QUESTIONS REPORT for rotest 45. 04110.04 001 Given the following conditions: Unit at 100% power, EOL conditions. Turbine operating at set point A steam dump valve inadvertently comes full open. All other control systems normal. Which ONE of the following correctly describes the plant conditions, when the plant stabilizes, assuming NO reactor trip and NO operator action? A. Megawatts electrical same as initial; reactor power increases. B. Megawatts electrical same as initial; reactor power decreases. e' Megawatts electrical decreases; reactor power increases. D. Megawatts electrical decreases; reactor power decreases. 46. 045A3.05 001 The following plant conditions exist: - Unit 1 is at 90% power - Main Turbine is in STANDBY to repair a failed speed sensor Which ONE of the following correctly describes the status of the Turbine Control System? A. All overspeed protection has been defeated except for the Mechanical Trip and the Backup Overspeed Trip. B. Overspeed protection from the speed control circuits and PLU has been defeated. IV Fast Closure, Mechanical Trip, and Backup Overspeed Trip are still operable. C' The Power Load Unbalance circuit is still active and will allow fast closure of the Control Valves and the Intercept Valves if a sudden load rejection of more than 40% occurs. D. The Power Load Unbalance circuit is defeated and the Backup Overspeed Trip setpoint is reduced to 105%.

Tuesday, January 07, 2003 02:21:28 PM

20

QUESTIONS REPORT for rotest


47. 051AA2.02R 001 Given the following: Condenser pressure rising; operators dropping turbine load in attempt to maintain vacuum. Condenser vacuum is decreasing, currently reading 25.20 inches Hg Which ONE of the following would be the FIRST to automatically occur or be procedurally required if condenser pressure continues to rise? A. B. Auto main turbine trip on low vacuum. Manual reactor trip.

C.`' Loss of steam dump capability. D. Manual turbine trip.

48. 054EG2.4.48 002 Given the following: - Unit 1 is at 100% power. - Annunciator ALB13-D01 STM GEN 1 HI/LO LVL DEVIATION alarm is illuminated - Only S/G #1 level is rising - Both MFPs speed are rising Which ONE of the follow describes the (1) cause, (2) required action and (3) direct consequence of an operator failing to take action? A. (1) #1 S/G FRV is opening, (2) stabailize #1 SIG level at new level, (3) Turbine Runback Initiated (1) #1 S/G FRV is opening, (2) return #1 S/G level to program, (3) Feedwater Isolation initiated C. (1) MFP master controller failing high, (2) control MFP speed using manual, (3) Auto Turbine Trip initiated D. (1) MFP master controller failing high, (2) manually trip turbine, (3) Feedwater Isolation initiated

Tuesday, January 07, 2003 02:21:28 PM

21

QUESTIONS REPORT for rotest 49. 055EK3.02 002 Which ONE of the following is a purpose of depressurizing all intact SGs to 300 psig during the performance of 19100-C, "ECA-0.0, Loss of All AC Power"? A. Reduces DP across SG U-tubes to minimize possibility of tube rupture. Reduces DP across RCP seals to minimize leakage and loss of RCS inventory.

C. Maximizes Natural Circulation flow before Reflux cooling begins as the RCS becomes saturated. D. Maximizes Natural Circulation flow to allow reactor vessel head to cool since CRDM cooling fans are unavailable.

50. 055K3.01the following conditions: Given 002 Reactor power is steady-state at 100%. Rod control is in automatic. Sealing steam pressure drops to 0 psig due to a malfunction

Which ONE of the following conditions will result if NO operator action is taken in response to this condition? A. Rising megawatt output and rising steam seal header pressure. B. Rising megawatt output and rising condenser hotwell level. C. Dropping megawatt output and dropping condenser pressure. Dropping megawatt output and rising condenser pressure.

Tuesday, January 07, 2003 02:21:29 PM

22

QUESTIONS REPORT
for rotest 51. 056AK1.01R 002 Given the following plant conditions: - Unit 1 has experienced a Loss of Offsite Power - The operating crew is currently performing a cooldown, in accordance with procedure 19002-C, "ES-0.2, Natural Circulation Cooldown" - Prior to initiating the cooldown, two CRDM fans trip Which ONE of the following describes the maximum allowable cooldown rate? A. B. <100 F in any one hour <75F in any one hour

C:' <50F in any one hour D. <25F in any one hour

52. 056K1.03 002 Unit 1 is at 70% power. A limit switch problem causes MFP 1A turbine exhaust valve to close. Which ONE of the following describes the effect on continued plant operation?

A. 1A MFP rolls to idle, the Standby condensate pump starts and Rx power can remain at 70% B." 1A MFP trips, the standby condensate pump remains in standby, and Rx power can remain at 70%. C. 1A MFP trips, the standby condensate pump starts and Rx power must be reduced to 56%. D. 1A MFP rolls to idle, the standby condensate pump does not start and Rx power must be reduced to below 56%

Tuesday, January 07, 2003 02:21:29 PM

23

QUESTIONS REPORT for rotest 53. 057AA2.18R 002 Given the following: - Unit 1 at 100% power Alarms received indicate a failed electrical bus - All channel 1 trip status lights illuminated MSIV's remain open and the unit does not trip All channel 1 instruments fail down scale Which ONE of the following identifies which electrical bus is de-energized? A. 125 V DC Vital Bus 1AD1 B:' 120 VAC Vital Instrument Bus 1AY1A C. 125 V DC Vital Bus 1BD1 D. 120 VAC Vital Instrument Bus 1BY1B 54. 059A4.11 001 After which one of the following events can feedwater isolation be reset by operating the feedwater isolation reset handswitches without performing any other actions? A. A spray valve fails open causing pressurizer pressure to drop to 1725 psig. The spray valve is closed and pressure returns to 2235. W At 75% Rx power, the operator overfeeds a single steam generator to the High-High Level setpoint and clears the High-High Level. C. Turbine high vibrations causes a turbine trip from 65% Rx power. Steam dumps open to control Tavg at 557F. D. A high steam line flow causes a low Tavg and an SI. Main Steam isolation terminates high flow condition and allows Tavg to return to 557F.

Tuesday, January 07, 2003 02:21:29 PM

24

QUESTIONS REPORT for rotest 55. 059AA1.01 001 Given the following plant conditions: Plant is operating a 100% power. Plant systems aligned for normal at power operations. RE-1950, Auxiliary Component Cooling Water radiation monitor, is in alarm Which ONE of the following lists the type and process flows that are sensed by the alarming radiation monitor? A!' Gamma; Thermal Barrier leakage. B. Beta; Excess Letdown Hx leakage C. Gamma; RHR Hx leakage D. Beta; RCP Motor Cooler leakage 56. 059K4.19 001 Which ONE of the following describes the automatic functions which take place on receipt of a feedwater isolation signal due to a reactor trip with T-ave less than 564 degrees F? A. Main feed pumps trip, main feedwater isolation valves close, main feed regulating and bypass feed regulating valves close, bypass feed isolation valves close B:' Main feedwater isolation valves close, main feed regulating and bypass feed regulating valves close, bypass feed isolation valves close C. Main feedwater isolation valves close, bypass feed isolation valves close, auxiliary feedwater isolation valves close D. Main feed pumps trip, main feedwater isolation valves close, auxiliary feedwater isolation valves close

Tuesday, January 07, 2003 02:21:29 PM

25

QUESTIONS REPORT for rotest 57. 059K6.09 001 Given the following: - Unit 1 is at 60% power with both MFPs operating in AUTO. - PT-507, Steam Header Pressure, output begins to slowly drift low. Which ONE of the following describes the initial effect on the Main Feed Water System, assuming no operator action? A. Both MFP's discharge pressure begins to increase and all Feed Water Reg valves begin to close. B. Both MFP's discharge pressure begins to increase and all Feed Water Reg valves begin to open. C.' Both MFP's discharge pressure begins to decrease and all Feed Water Reg valves begin to open. D. Both MFP's discharge pressure begins to decrease and all Feed Water Reg valves begin to close. 58. 061A1.04 002 Given the following plant conditions: - The Unit 1 is at 100% power - A loss of both RATs occurs due to a switchyard fault - The unit is manually tripped - 1AA02 and 1BA03 are energized by their D/Gs Which of the following correctly describes the effect this will have on CST level and the actions that will be necessary? CST #1 level will: )6%.:' continuously lower requiring manual swap to CST #2. B. C. D. be maintained by automatic makeup. continuously lower and automatically swaps to CST #2. remain full until CST #2 level reaches 66% .

Tuesday, January 07, 2003 02:21:29 PM

26

QUESTIONS REPORT for rotest 59. 061AA1.01 002 Which of the following area radiation monitors intiate a CVI? A. ARE-014, Waste Gas Effluent Monitor B:' RE003, Containment Low Range C. ARE-2533A, Fuel Handling Building D. RE-12117, Control Room Ventilation 60. 061K5.01 002 Given the following plant conditions: - The Unit 1 reactor is in mode 3 - RCS heatup to 557 degrees F in progress The heat transfer rate between RCS and the steam generators will A!' increase as RCS temperature increases and AFW flow increases. B. decrease as RCS temperature decreases and AFW flow increases. C. increase as AFW temperature increases and AFW flow increases. D. decrease as AFW temperature decreases and AFW flow increases. 61. 062A3.05 002 Given the following sequence: A small break LOCA occurred resulting in a reactor trip and SI. The SI signal was reset during the performance of 19010-C, "E-1, Loss of Reactor or Secondary Coolant." A loss of offsite power occurred and the diesel generators loaded as designed. Assuming no operator actions, which ONE of the following would be the status of the loads on the 4160Vac 1E buses? A. All equipment powered from the 4160Vac 1E buses with the control board switch in automatic will be restarted. B. No 4160Vac 1 E bus loads are automatically restarted. C' Equipment normally started during a LOSP will be automatically restarted; SI and RHR pumps remain OFF. D. All equipment that was operating prior to the LOSP will be automatically restarted; All running ESF equipment will be reenergized.

Tuesday, January 07, 2003 02:21:29 PM

27

QUESTIONS REPORT for rotest


62. 062AG2.4.24 002 The crew is in 19100-C, "Reactor Trip or Safety Injection". Prior to the step that the crew places equipment in PTL, the procedure cautions that 2 NSCW pumps should be available to load on each AC Emergency Bus. These pumps are required to provide cooling for the ...... A. SI pump B. MDAFW pump C. ACCW pump EDG

63. 064K2.03 002 Unit 2 is at 100% power All Diesel Generators are currently operable Annunciator ALB38-B09, "DG2B ENGINE CNTL POWER B FAILURE" just illuminated Which ONE of the following describes the status of the Diesel Generator 2B with this annunciator in alarm? A. DG2B still has capability to start and load once and is operable. B. If DG2B was running when this occurred it will continue to operate and can be shutdown from the control room. DG2B must be declared inoperable. C. DG2B can be started and loaded manually but is inoperable. If DG2B was running it will continue to operate and can only be shutdown from the front standard. DG2B must be declared inoperable.

64. 065AA1.03 002 Unit 1 just experienced a loss of air and entered AOP 18028-C, "Loss of Instrument Air" The header pressure is 63 psig. Given the current conditions, what action, per procedure, should be taken by the operator to prevent inadvertent operation of equipment repositioned due to the loss of air? ?O' Verify the cask loading pit gate seal assemblies are supplied with bottled nitrogen > 50 psig. B. C. Ensure 1PV-9385 "Service Air Isolation Valve" is open. If MS1Vs closed due to low air pressure, place the MSIV hand switches to closed.

D. If normal letdown had not isolated due to low pressure, place the normal letdown isolation switches in open.

Tuesday, January 07, 2003 02:21:29 PM

28

QUESTIONS REPORT for rotest 65. 067AK1.02 001 There is a fire in the generator hydrogen cooling system. Fire fighting efforts have caused the fire main header pressure to lower. Which one of the following is the design pressure that will cause the FIRST diesel driven fire pump to start automatically? A fire header pressure of ..... A. 85 psig. 13' 95 psig. C. D. 105 psig. 110 psig.

66. 068AK3.18 001 Evacuation of the Control Room is required due to a Control Room fire. The actions of AOP 18038-1, "Operation From Remote Shutdown Panels," prior to evacuating the control room, include which of the following? A' Trip both main feedwater pumps. B. Ensure SIG pressure control in AUTO C. Place the PZR pressure control in AUTO D. Trip all RCPs 67. 068K1.07 002 Which one of the following describes how the incore instrumentation seal table leakage enters the Liquid Rad Waste system? idV Drains to Reactor Cavity Sump, and then is pumped to the Floor Drain Tank. B. Drains to the floor drain, overflows into the Reactor Cavity Sump, and then is pumped to the Waste Monitor Tank C. Drains to the floor drain, overflows into the Reactor Cavity Sump, and then is pumped to the Floor Drain Tank D. Drains to Reactor Cavity Sump, and then is pumped to the Waste Monitor Tank.

Tuesday, January 07, 2003 02:21:29 PM

29

QUESTIONS REPORT
for rotest 68. 068K6.10 002 Which ONE of the following would require termination of the release of Unit 1 Waste Monitor Tank # 10 ?

A' 1-RE-0018 fails high B. C. D. 1-RE-039A fails high ARE-0014 fails low 1-RE-039A fails low

69. 069AK2.03 002 Given the following conditions: - The unit 1 RCS is at 190 F and 350 psig - The Aux Bldg. SO reported that both containment airlock doors are open Which one of the following is the correct action? A. Close the outer door within 1 hour or reduce RCS temperature to 140 oF within 24 hours B:" Close the outer door prior to RCS heatup above 200 oF C. Close the outer door prior to RCS heatup above 350 oF D. Close the outer door within 1 hour and lock the outer door within 24 hours.

70. 071K3.05 001 Chemistry has taken a grab sample of the release in progress from the gaseous radwaste system. The results indicate that the release is above the release permit setpoints. Which one of the following caused this?

A:' Waste Gas effluent monitor, RE-14, failed low. B. C. D. Waste Gas effluent monitor, RE-14, failed high. A loss of 125 vDC power to radiation trip valve, RV-0014. A loss of instrument air to radiation trip valve, RV-0014

Tuesday, January 07, 2003 02:21:29 PM

30

QUESTIONS REPORT for rotest 71. 072A1.01 002 ARE-2532A and ARE-2533A are indicating increasing levels of radiation. If this trend continues which of the following should occur? A. Only the Train A FHB isolation dampers close, the supply unit trips but the postaccident filter unit does not start. B. The AREs will alarm locally and in the control room, initiating an isolation and a direct trip of the normal FHB HVAC units. C. Only the Train A supply and exhaust dampers isolate, supply and exhaust units continue to run and the post-accident filter units start. D" Train A and B supply and exhaust dampers isolate, the supply and exhaust units trip on low flow, and the post-accident filter units start.

72. 072G2.1.28 001 RE-0019, 0020 and 0021 are in alarm. Waste Water Retention Basin RV-0021 and FV1150 have lust closed. Which equipment caused the closure? A. RE-0019 Only B. RE-0020 Only RE-0021 Only D. The Combination of both RE-0019 and RE-0020

Tuesday, January 07, 2003 02:21:29 PM

31

QUESTIONS REPORT
for rotest

73. 073A2.01R 001


Given the following: - Unit 1 is at 100% power - a liquid waste release is in progress. - Annuciator ALB 05 CO3, HIGH RADIATION ALARM, is illuminated. - 1-RE-0018 spiked to high alarm due to an erractic power supply, and is now decreasing to less than backgound. Which ONE of the following describes the initial required actions by the control room operators? A. B. Have the detector assembly cleaned. Notify RADCON to take radiation readings in the detector area.

C:' Verify liquid rad waste release valve 1-RV-0018 is shut. D. Initiate a work order to repair 1-RE-0018.

74. 075A4.01 001 An operator making rounds reports that 1-TI-1712, which measures the NSCW temperature at the outlet of the CCW Hx is reading 204 degrees F. Which one of the following describes how this can be verified in the control room? A. Use DIG jacket water inlet temperature since this is essentially the same temperature as the CCW Hx outlet temperature. B. ACCW Hx outlet temperature can be used, a table is available to convert the indicated temperature. C.' Use the IPC since redundant information is available on the IPC. D. CCW Hx and ACCW Hx flow can be used since a nomograph is available to convert the indicated temperature.

75. 0761(2.08 001 Which one of the following describes the power supply for containment coolers 7 & 8 NSCW Supply Valve (HV-1809)? A. 480 Vac switchgear 1AB16 B. 480 Vac switchgear 1NB21 C! 480 Vac MCC 1BBD D. 480 Vac MCC 1NBM

Tuesday, January 07, 2003 02:21:29 PM

32

QUESTIONS REPORT for rotest 76. 07802.4.11 002 Given the following conditions: - Plant in Mode 3. - Total Loss of Instrument Air has occurred. - The RO reports that Pressurizer level is 92% and slowly rising. Which ONE of the following actions should be taken to control Pressurizer level per 18028-C, "Loss of Instrument Air"? A. Locally isolate charging and letdown and place excess letdown in service. B. Stop CCPs until PZR level is less than 70% then cycle a CCP on and off to maintain PZR level between 65 and 70%. C. Locally isolate charging until PZR level is less than 70% then locally operate 1FHC0121 to control charging flow. Place the reactor Head Vent System in service, adjust head vent flow valves to desired letdown rate. 77. 0791(4.01 002 If Station Instrument and Service Air System air pressure is dropping, the Service Air System is designed to automatically isolate from the Instrument Air System when pressure drops below which ONE of the following? A. 70 psig. B. 78 psig. C:' 80 psig. D. 100 psig. 78. 08602.4.25 002 During the response to a fire in the turbine building, an injured person must be transported off site. The only safe way to transport this person is via the Turbine Building elevator. The elevator is locked out. According to procedure 92005-C, which one of the following persons can authorize the use of the elevator during the fire? A:' Fire Team Captain, only B. The Unit Shift Supervisor of the affected unit, only C. Any Fire alarm response Team Member D. The Burke County Emergency Management Agency

Tuesday, January 07, 2003 02:21:29 PM

33

QUESTIONS REPORT for rotest 79. 103A2.03R 001 Given the following conditions for Unit 1: - Safety Injection has actuated - The crew is performing 19000-C, "E-0, Reactor Trip or Safety Injection", when the RO notes that Containment Pressure is 23.2 psig and that neither the Containment Spray NOR CIA have actuated. Which ONE of the following describes the minimum action(s) required? A:' Must manually actuate either CIA manual handswitch and manually actuate both containment spray handswitches. B. Must manually actuate both CIA manual handswitches and manually start the Containment Spray pumps. C. Manually actuate either CIA manual handswitch and manually shut the MSIV's and bypasses. D. Must manually actuate both CIA manual handswitches and manually shut the MSIV's and bypasses. 80. G2.1.2 002 Given the following indications on Unit 1: - 19030-C, SGTR is being implemented - Both CCPs and both SIPs are running - RCS pressure 1350 psig and falling slowly - Level in SIG 1 is 80% NR rising slowly - Level in other SIGs is 5% NR rising slowly MSIVs are open - RCS temperature is 558 degrees F and rising slowly Which ONE of the following describes the action to be taken and the basis for that action? A. B. Dump steam at the maximum rate to cooldown the RCS. Isolate SG#1 to minimize RCS cooldown. Stop all RCPs because RCP trip Criteria have been met prior to initiation of RCS cooldown. D. Stabilize the level in intact S/Gs to preserve a heat sink for cooldown.

Tuesday, January 07, 2003 02:21:30 PM

34

QUESTIONS REPORT for rotest 81. G2.1.3 002 A procedure writer with an active RO license is in the control room to review a procedure revision. The RO is called for a random drug test. The RO requests that the procedure writer relieve him for about 15 minutes for the drug test. Which one of the following describe the shift relief requirements for this situation? The procedure writer may relieve the RO provided... A. ...the procedure writer reviews the narrative logs, rounds sheets, and checklists for his station. The review shall include narrative logs since the last shift worked or the preceding 3 days, which ever is longer. B. ...the procedure writer and the on-shift RO independently walk-down their assigned control boards to verify checklists items and discuss equipment status. C. ...the relief is 45 minutes or less and a) the relieving operator is knowledgeable of plant conditions, b) they perform a joint walkdown of applicable control panels, and c) the Unit Shift Supervisor acknowledges the relief. D:' .....a full turnover is conducted as described in procedure 10004-C, Shift Relief. 82. G2.1.32 001 Which ONE of the following describes the normal configuration of the Component Cooling Water system and the reason, respectively? A. 2 pumps running, one pump in pull-to-lock, to prevent CCW Hx tube vibration damage from excessive flow rates. B. 2 pumps running, one pump in pull-to-lock, to avoid system pressure exceeding relief setpoints C:' 2 pumps running, one pump on standby, to avoid system pressure exceeding relief setpoints D. 2 pumps running, one pump on standby, to prevent CCW Hx tube vibration damage from excessive flow rates.

Tuesday, January 07, 2003 02:21:30 PM

35

QUESTIONS REPORT for rotest 83. 62.2.12 002 Unit 1 is at 100% Surveillance 14905-1, RCS Leakage Calculation (Inventory Balance) is in progress Which ONE of the following would invalidate the leak rate calculation? A:' RCS diluted 50 gallons to raise RCS temperature back to program. B. Main turbine load reduced 5 MWe to prevent exceeding allowed power limits. C. Control rods inserted 5 steps for AFD control. D. ECCS accumulator filled with an SI pump due to a slow leak. 84. G2.2.22 001 Which one of the following sets of conditions represents a violation of a technical specification safety limit and required action? A. Power = 10%, Pressure = 2400 psig, Tavg = 655F, restore to within limits OR be in Mode 3 in 1 hour 13:' Power = 80%, Pressure = 2250 psig, Tavg = 640F, restore to within limits AND be in Mode 3 in 1 hour C. Power = 10%, Pressure = 2400 psig, Tavg = 655F, restore to within limits in 2 hrs or be in Mode 3 in 6 hrs D. Power = 80%, Pressure = 2250 psig, Tavg = 640F, restore to within limits in 2 hrs or be in Mode 3 in 2 hrs 85. G2.2.3 002 Which one of the following decribes the Plant Integrated Computer (1PC) terminals? They are in mirror image locations. The IPC terminals are identical except that A. the common radiation monitors go to Unit 2 only and weather data goes to Unit 1. B. the common radiation monitors go to Unit 1 only and weather data goes to Unit 2. both the common radiation monitors and weather data go to Unit 1 only. D. both the common radiation monitors and weather data go to Unit 2 only.

Tuesday, January 07, 2003 02:21:30 PM

36

QUESTIONS REPORT for rotest

86. G2.3.1 001


Which one of the following dose components are combined in a Radiation Worker's Occupational Dose? A. Total Effective Dose Equivalent and Planned Special Exposures. B. Planned Special Exposures and Committed Effective Dose Equivalent. C. Total Effective Dose Equivalent and Committed Effective Dose Equivalent D:' Deep Dose Equivalent and Committed Effective Dose Equivalent.

87. G2.3.4 002


Given the following plant conditions: A LOCA occurred and a Site Area Emergency was declared. The TSC and OSC have been activated. It is recommended that entry be made into the Safety Injection Pump Room 1A to determine why the pump will not start. - Projected dose rate in the pump room is 1.16x105 mr/hr. Duration of the exposure is expected to be 3 minutes. Which ONE of the following may authorize this exposure? A. EOF Manager B. Operations Support Center Manager C. Health Physics Supervisor W Emergency Director

Tuesday, January 07, 2003 02:21:30 PM

37

QUESTIONS REPORT for rotest 88. G2.3.9 001 Given the following conditions: Unit 1 is in Mode 2 following a refueling outage Containment Mini-Purge System was placed in operation in preparation for maintenance personnel to make a containment entry. Maintenance has requested the Mini-purge system be shutdown to reduce noise levels while they perform their activities. Which ONE of the following should be considered prior to securing Containment Purge? A. Outside air temperature and pressure B. Containment humidity C." ALARA conditions D. Containment Purge HEPA and Charcoal filter DP 89. G2.4.1 002 Given the following events: Unit 1 reactor trip Operators observe the following conditons: - 1AA02 is deenergized due to a bus fault - 1B-DG supplying 1BA03 - 1BB06 feeder breaker trips during load sequencing Which of the following would describe the correct actions to take? A. B. C. Remain in 19000-C on step 3 until power is restored to 181306 Transition to 19100-C, "ECA-0.0, Loss of All AC Power" Transition to 18031-C, "Loss of Class 1 E Electrical Systems" D:`

Continue in 19000-C while trying to restore power to 1AA02 and 1BB06

Tuesday, January 07, 2003 02:21:30 PM

38

QUESTIONS REPORT for rotest 90. G2.4.11 002 Given the following conditions: - Unit 1 has been operating at 100% power with a 32 GPD tube leak for the last 3 months on SIG #2 - RE-724 & RE-810 became inoperable 4 hours ago, chemistry is investigating Chemisty sampling 2 hours ago indicated the leak rate was 37 GPD - Chemisty sampling indicates the leak rate is currently 48 GPD Which ONE of the following should be performed per procedure 18009-C? A:" Be in Mode 3 within 24 hours. B. C. Reduce load to hot standby within 2 hrs, then cooldown and depressurize the RCS. Trip the reactor; enter E-0, then transition to E-3.

D. Convene PRB to evaluate continued operation. 91. 02.4.16 001 While in the Emergency Response procedures the team is directed to "Go To" another procedure, which one of the following is the correct implementation of this action? A. The "GO TO" implies the procedure in use is not applicable, and therefore any tasks in progress need not be completed. B. The original procedure remains applicable because tasks still in progress must be completed prior to the transition directed by the "GO TO" step. The "GO TO" implies the procedure in use is no longer applicable, transition to the new procedure but any tasks in progress should be completed. D. Tasks still in progress need not be completed prior to the transition directed by the "GO TO" step, unless preceded by a note stating otherwise.

Tuesday, January 07, 2003 02:21:30 PM

39

QUESTIONS REPORT for rotest 92. G2.4.8 002 Unit 1 has the following symptoms: - Reactor is tripped - core exit TC temperatures greater than 1200 F - RVLIS full range indication is 25% Which ONE of the following are your required actions? Enter 19221-C FR-C.1, Response to Inadequate Core Cooling, from .... A. Orange path on Core Cooling, and 1) 2) 3) Reinitiate high pressure safety injection Rapidly depressurize the steam generators Restart RCPs and/or open PRZR PORVs

B. Orange path on Core Cooling, and 1) 2) 3) Reinitiate high pressure safety injection Slowly depressurize the steam generators Stop all running RCPs and open PRZR PORVs

C:' Red path on Core Cooling, and 1) 2) 3) Reinitiate high pressure safety injection Rapidly depressurize the steam generators Restart RCPs and/or open PRZR PORVs

D. Red path on Core Cooling, and 1) 2) 3) Reinitiate high pressure safety injection Slowly depressurize the steam generators Stop all running RCPs and open PRZR PORVs

93. WE02EK1.2R 001 19012-C, "Post LOCA Cooldown and Depressurization," is used following a LOCA event in which the RCS remains at pressure, and it is determined that A. B. C. D. SI termination criteria cannot be met or maintained. SI termination criteria have been met and can be maintained. the possibility of core voiding needs to be prevented. a pressurized thermal shock condition is imminent.

Tuesday, January 07, 2003 02:21:30 PM

40

QUESTIONS REPORT for rotest 94. WE03EK2.2 002 Given the following plant conditions: A small break LOCA has occurred. RCPs have been tripped. Appropriate actions in accordance with 19010-C, E-1, Loss of Reactor or Secondary Coolant have been completed. RCS pressure is stable at 1525 psig. ECCS is operating in cold leg injection mode.

Which ONE of the following statements describes the primary method of decay heat removal at this time? A!' Heat transfer between the RCS and the S/Gs due to natural circulation flow. B. Heat transfer between the RCS and CCW via the RHR Heat Exchangers. C. Heat transfer from the injection of water from the RWST and the removal of steam/water out of the break. D. Heat transfer from Reflux boiling in the S/Gs. 95. WE05EA1.3 002 Given the following plant conditions: - Operators implement 19231-C, "FR-H.1, Response to Loss of Secondary Heat Sink". - RCS feed and bleed criteria were met. - Manual Safety Injection was initiated and when the operator attempted to open the pressurizer PORVs, PORV 455 failed to open. Which ONE of the following describes the correct operator mitigation strategy to respond to this problem? A. Stop one Centrifugal Charging Pump to reduce loss of inventory through PORVs. B. Close any open Pzr PORV to conserve RCS inventory and return to the steps to re-establish Main Feedwater. C:' Open the reactor head vents to reduce RCS pressure since one pressurizer PORV may not provide sufficient heat removal capacity. D. Verify PORV 456 and its block valve open to reduce RCS pressure since 1 Pzr PORV provides adequate heat removal capacity for a loss of heat sink.

Tuesday, January 07, 2003 02:21:30 PM

41

QUESTIONS REPORT for rotest

96. WE07A2.02R 001


Given the following plant conditions: - The Unit has tripped from 100% power with a LOCA in progress - Pzr pressure is 1700 psig - RCPs are tripped - Core Exit thermocouples indicate 720 degrees F - RVLIS full range indicates 36% Which of the following describes the conditions existing in the core as applicable to the EOPs? A. Saturated conditions, which do not present an extreme challenge to the fuel matrix and fuel cladding as long as the hot leg temperatures remain at saturated conditions. B. Super heated conditions, which do not present an extreme challenge to the fuel matrix and fuel cladding as long as the cold leg temperatures remain at saturated conditions. C. Saturated conditions, which present an extreme challenge to the fuel matrix and fuel cladding. D:' Super heated conditions, which present an extreme challenge to the fuel matrix and fuel cladding. 97. WE08EA2.1R 001 Step 1 of 19241-C, "FRP-1, Response Imminent Pressurized Thermal Shock Condition", has the operator check: - RCS pressure < 300 psig - RHR flow > 500 gpm This step is based on: A!' preventing implementation of 19241-C if a large break LOCA is in progress. B. ensuring adequate low head safety injection cooling prior to isolating ECCS accumulators. C. preventing core exit temperatures from exceeding the required temperature to place RHR in service. D. ensuring RHR system is in service to provide adequate mixing in the cold leg downcomer region.

Tuesday, January 07, 2003 02:21:30 PM

42

QUESTIONS REPORT for rotest 98. WEI 0EA2.2 002 Given the following plant conditions: Shutdown is in progress to comply with Tech Specs The unit must be in mode 4 within 60 minutes Reactor trip occurred with subsequent loss of RCPs. Operators have implemented 19002, "ES-0.2, Natural Circulation Cooldown". A cooldown rate of 50 F/hour has been established. Current RCS temperature is 450F Operators are monitoring PZR level and RVLIS for void formation. Which ONE of the following describes the appropriate procedural actions. A. STOP the cooldown and remain in 19002, "ES-0.2, Natural Circulation Cooldown". B. RAISE the cooldown rate and remain in 19002, "ES-0.2, Natural Circulation Cooldown" . C. Transition to 19003-C, "ES-0.3, Natural Circulation Cooldown With Steam Voids in Vessel (With RVLIS)" and LOWER the cooldown rate. D. Transition to 19003-C, "ES-0.3, Natural Circulation Cooldown With Steam Voids in Vessel (With RVLIS)" and RAISE the cooldown rate.

99. WEI lEA1.3 002 Given the following plant conditions: Reactor trip and SI occurred on Unit 1 due to a LOCA. Crew is performing 19111-C "ECA-1.1, Loss of Emergency Coolant Recirculation", due to the failure of both RHR sump suction valves to open. Crew has reduced ECCS flow to one train per step 11 of 19111-C Crew is performing Step 19 of 19111-C to check ECCS flow adequate and observes the following indications: - No RCP running - RVLIS Full Range = 60% and slowly dropping Which one of the following lists the correct operator action for this condition? A. Maintain the current ECCS alignment. B.' Raise ECCS flow to increase RVLIS indication. C. D. Place RHR shutdown cooling in service. Isolate the ECCS accumulators.

Tuesday, January 07, 2003 02:21:30 PM

43

QUESTIONS REPORT for rotest 100. WE13EK3.2R 002 Given the following condtions: - FRP 19232-C, "Response to SG Overpressure" is being implemented - SG #1 pressure is 1250 psig - The MSIVs are shut - SG #1 ARV and code safety valves are failed shut Which one of the following actions is required to correct this situation?

A!' Dump steam from SG #1 by opening the MSIV bypass valves. B. Stop RCP #1 to reduce the heat input to SG #1. C. Initiate maximum AFW flow to SG#1 to inject cold water to help lower SG #1 pressure. D. Actuate SI to inject cold water into the RCS to help lower SG #1 pressure.

56

Tuesday, January 07, 2003 02:21:30 PM

44

QUESTIONS REPORT

f o r HL-14 NRC SRO DRAFT Exam


Response form Answers MCS Time: 1 Points: 1.00 Version: 0 1 2 3 4 5 6 7 8 9 Answer: CACBDBBDDC Scramble Range: A- D

Monday, August 13, 2007 9:45:25 AM

57

QUESTIONS REPORT for HL-14 NRC SRO DRAFT Exam 23. G2.4.46 001/3/N/A/ALARMS & CONDITIONS/C/A - 3.6/NEW/S/NRC SRO/TNT / RLM Given the following conditions: Unit 1 at 100% power. A reactor trip and SI occur due to a large LOCA, on the reactor trip, an LOSP occurs to both RAT 1A and RAT 1B. Both DGs re-energize 4160 1E busses 1AA02 and 1BA03. The following alarms are noted by the RO during the Initial Operator Actions of E-0. -ACCW LO HDR PRESSURE - ACCW RPC 1 (2, 3, 4) CLR LO FLOW ACCW RCP 1 (2, 3, 4) CLR OUTLET HI TEMP ACCW RX COOLANT DRN TK HX LO FLOW ACCW EXCESS LTDN HX LO FLOW ACCW RTN HDR FROM RCP LO FLOW Which ONE of the following is CORRECT regarding the alarms and the actions the SS should direct the crew to perform? At' Direct the RO to start an ACCW pump per direction of E-0 RO Initial Operator Actions. B. Direct the RO to trip the RCPs, direct an operator to perform actions of 18022C, "Loss of ACCW" in parallel with E-0. C. Direct the RO to trip the RCPs, direct an operator to perform actions of 18022-C in parallel with E-0, if ACCW is restored, re-start the RCPs. D. Take no action at this time, the EOP network will address the loss of ACCW pumps in later steps of E-0 or upon transition to another EOP.
F e edback

K/A G2.4.46 Ability to verify that the alarms are consistent with the plant conditions. K/A MATCH ANALYSIS Question gives a plausible scenario during a simultaneous LOSP / SI where the ACCW pumps aren't running due to being locked out by the SI sequence. The candidate must
Monday, August 13, 2007 9:45:25 AM

58

QUESTIONS REPORT for HL-14 NRC SRO DRAFT Exam determine the correct course of action / procedure to use to address the condition. Question meets 10CFR55.43(b) criteria item # 5 - Assessment of facility conditions and selection of procedures during normal, abnormal, and emergency situations. ANSWER / DISTRACTOR ANALYSIS A. Correct. ACCW pumps would not be running on a simultaneous LOSP / SI as SI sequence prevails and locks out the pumps. Initial Operator Actions of E-0 would have the RO notify the SS of the condition and request permission to start the ACCW pumps, if they can't be started, trip the RCPs. B. Incorrect. Plausible if the candidate does not realize E-0 initial actions address restoring ACCW or recognize why the pumps are stopped. An ACCW pump should be started and the RCPs not tripped. C. Incorrect. Plausible that candidate may realize why the ACCW pumps are stopped but not realize E-0 Initial Operator Actions addresses. Stopping RCPs to protect them is a logical choice on loss of ACCW and AOP could be run in parallel by another operator. Restart of RCPs when available is also directed in many EOPs. D. Incorrect. Plausible the candidate may not recognize why the pumps are stopped or that E-0 Immediate Operator Actions will address the condition. It is plausible that E-0 would address the situation later, other EOPs such as the LOCA procedure which is the most likely transition in this situation would address the loss of ACCW pumps. REFERENCES 19000-C, "E-0 Reactor Trip or Safety Injection", RO Initial Operator Actions step # 11. 18022-C, "Loss of ACCW", in particular symptoms and entry conditions for low flow annunciators which would occur on a loss of ACCW. LO-PP-04101, ACCW Power Point slides 14 - 19, 14 & 15 included here. VEGP learning objectives: LO-PP-04101-04, From memory describe the expected system response and operator corrective actions for each of the following:

c SI followed by LOSP
Notes

Monday, August 13, 2007 9:45:26 AM

59

Response form

QUESTIONS REPORT for HL-14 NRC SRO DRAFT Exam


Answers MCS Time: 1 Points: 1.00 Version: 0 1 2 3 4 5 6 7 8 9 Answer: ADCDBCBDDC Scramble Range: A- D

Monday, August 13, 2007 9:45:26 AM

60

QUESTIONS REPORT for HL-14 NRC SRO DRAFT Exam 24. WE03EA2.1 001/1/2/POST LOCA CD - PROCS/C/A - 4.2/NEW/S/NRC SRO/TNT / RLM Given the following conditions: The crew is performing the steps of 19010-C, "Loss of Reactor or Secondary Coolant". RCS pressure is 450 psig and relatively stable. RHR flow is reading 0 gpm on both trains. PRZR level is offscale low. Which ONE of the following procedures would be the CORRECT procedure to perform with the given conditions? A. Transition to 19011-C, "SI Termination". B. Continue with subsequent steps of 19010-C. C. Transition to 19111-C, "Loss of Emergency Coolant Recirculation". D." Transition to 19012-C, "Post LOCA Cooldown and Depressurization".

Monday, August 13, 2007 9:45:26 AM

61

QUESTIONS REPORT for HL-14 NRC SRO DRAFT Exam


F e edback

K/A WE03 Facility conditions and selection of appropriate procedures during abnormal and emergency operations. EA2.1 Facility conditions and selection of appropriate procedures during abnormal and emergency operations. K/A MATCH ANALYSIS Question gives a plausible scenario where the crew is performing19010-C. A transition point has been reached and the candidate must determine the proper procedure transition to make. Question meets 10CFR55.43(b) criteria item # 5 - Assessment of facility conditons and selection of procedures during normal, abnormal, and emergency conditions. ANSWER / DISTRACTOR ANALYSIS A. Incorrect. Plausible the candidate may think 19011-C would be appropriate since RCS pressure is stable. B. Incorrect. Plausible the candidate may think continuing with 19010-C is appropriate. C. Incorrect. Plausible the candidate may think a transition to 19111-C, Loss of Emergency Coolant Recirculation is appropriate due to no RHR flow. D. Correct. Transition to 19012-C, Post LOCA Cooldown and Depressurization is required per the 19010-C step. REFERENCES 19011-C, "SI Termination. 19012-C, "Post LOCA Cooldown and Depressurization". VEGP learning objectives: LO-LP-37111-08, Using EOP 19010 as a guide, briefly d e scribe how each step is accomplished.
Notes Response form

Monday, August 13, 2007 9:45:26 AM

62

QUESTIONS REPORT for HL -14 NRC SRO DRAFT Exam


Answers MCS Time: 1 Points: 1.00 Version: 0 1 2 3 4 5 6 7 8 9 Answer: DCDAACBCCD Scramble Range: A- D

63
Monday, August 13, 2007 9:45:26 AM

QUESTIONS REPORT for HL-14 NRC SRO DRAFT Exam 25. WEO8G2.1.14 001/1/2/PTS - NOTIFICATIONS/C/A - 3.3/MODIFIED/S/NRC SRO/TNT / RLM Given the following plant conditions: A LOCA has just occurred at 0742 hours. RHR flow is approximately 4500 gpm per train. RCS lowest Cold leg temperature is 282 degrees F and stable. The Shift Manager (SM) makes an emergency classification at 0750 hours. Which ONE of the following is CORRECT regarding the emergency classification and required notifications? A. The SM was required to declare an NOUE and was expected to notify plant personnel by 0747 hours. BY The SM was required to declare an ALERT and was expected to notify plant personnel by 0755 hours. C. The SM was required to declare an NOUE and was expected to notify plant personnel by 0757 hours. D. The SM was required to declare an ALERT and was expected to notify plant personnel by 0805 hours.
F e edback

K/A WE08 RCS Overcooling - PTS. G2.1.14 Knowledge of system status criteria which require the notification of plant personnel. K/A MATCH ANALYSIS Question gives a plausble scenario where an RCS LOCA has occurred resulting in two differeent reasons the RCS Barrier is challenged (Potential Loss) and requires an emergency declaration. The candidate must determine if an NOUE is required or an ALERT emergency and the time frame for notifying plant personnel. Questions meets SRO only threshold by requirement of a classification in addition to the notification requirement. Per the KA Catalog Knowledge of Emergency action level thresholds and classification is an SRO importance factor of 4.1 with the RO importance factor level being only a 2.3. Therefore, this is an SRO only level question. Vogtle also has specific objectives for classification which are SRO only objectives.
Monday, August 13, 2007 9:45:26 AM

64

QUESTIONS REPORT for HL-14 NRC SRO DRAFT Exam ANSWER / DISTRACTOR ANALYSIS A. Incorrect. Plausible the candidate may not recognize the threshold for RCS Integrity barrier challenged yet due to Cold Leg Temperature > 265 degrees F. However, with RHR flow the break is greater than the capacity of one charging pump in the normal mode of operation. Also, plausible the candidate may think the time for plant personnel notification is 5 minutes from the time of the event. B. Correct. Alert emergency required and time is 5 minutes from declaration. C. Incorrect. Plausible the candidate may not recognize the threshold for RCS Integrity barrier challenged yet due to Cold Leg Temperature > 265 degrees F. However, with RHR flow the break is greater than the capacity of one charging pump in the normal mode of operation. Also, plausible the candidate may think the time for plant personnel notification is 15 minutes from the time of the event by confusing the time limit with that of notification of state and local authorities. D. Incorrect. Plausible the candidate may the threshold for RCS Integrity barrier challenged with RHR flow the break is greater than the capacity of one charging pump in the normal mode of operation. Also, plausible the candidate may think the time for plant personnel notification is 15 minutes from the time of the event by confusing the time limit with that of notification of state and local authorities. REFERENCES 91001-C, Emergency Classification and Implementation Instructions. Vogtle May 2005 NRC SRO Exam question # 94 used as base for modification. VEGP learning objectives: LO-LP-40101-02, State the four emergency classifications in order of severity from least to worst.
Notes R e s p o nse form Answers MCS Time: 1 Points: 1.00 Version: 0 1 2 3 4 5 6 7 8 9 Answer: BBCBDBDBDC Scramble Range: A- D

Monday, August 13, 2007 9:45:26 AM

65

08/13/2007 09:26 FAX 706 826 3953

VEGP TRAINING CENTER

ZI002

gEC'EXE-Ice fba- Iry 670-esno,,,)


COLR for VEGP UNIT 1 CYCLE 14

(Fully Withdrawn*)

MESOMMOMMUMMWHIMOAVEMSEUMMEMMEMMEOWEEPLIMM TUOM OMMOMMOMMIUMUMEMIMMOMMENNIUMMIUMMUMOMMUINI M 200 OMMINUOMWAIMMOMMIMMEMEMMEIMMOMIT2MMUMMENME MO OMMIlmmimmAMMOMOMMOMMOONOMMIMMOMPAMMOMOU NOLUN 1 180 IMMOMMOMPAMOMMEMEMUMMEMOMMIMMINUMMUMIMMOM MEMIN Mompurimummummimmoussimmummmummummul ro OMMERAMMUMEMMIMMOMMMOMMIMMAMMIMIMMEMIUMEME A NI MMINIKAMr- imummummiummummummumm ,4160 BANK 8 MEMMKOMML- -JEMMOMMEMMUMMOMMEMOMMIUmmomMOMMOMMO mmummulaimmommismismimmumnimummummerna
6

220

211WOMPUBBINIMMEMOMMOMMO2MOMOMMOOMMOMMMUSIW WW1 MMEMMUMMOMMIUMUMMOMMOMMEMMOMMIUMEMEMMEMMOM MOMPA 0 1 2 0 MINIMMEMMOMMUMMUMMEMMEMMINIMMEMMEMMEMMEMUMW AS MOMMOMORMENIMMOMMIRMAMMIUMMOMMOMIMMOMMUURIP AU IIMMIMMOMMUMMUMEEMEMOOMMOMMINIMMOMMEMIMMINFA 1 3 , 1 0 0 MME


80

140

millummullammotammusellum

mmmmmimggggggmmrdmugmmmmmmmmmmmgmmmim
MMEMMEMUMIumumMEMMAMUMMEMMEMMOOMOMMEMMEMORM EMOMM MUMOMMIIMMOMMIMMEMOWTANCOMMOMMOMOMMOMPAIMMEM MM

C1 0: 60

ImminlOMOOMMOMMUMML- 11111ORMOMMOMMIUMOMOMMEMS muommummmumnimminalumnimmummummommommo mrsimmonmommommummummummummisimmemm pmemummommonmumminuisimmiummumiummum immommummummummummommisimmumummumummum 40 omminsommumummummilmummummiummummommumme mommummormummorimmignommorAmmuummomm mummunrimumms IIMUMPAIMMUMMIUMMUM
MOIMMUMFAMMUMEES 2 0 MIMOVAUMMEMMEMMEMMUMME MMENOWWOOMMMMNNMMMMMMMMRMMMMMROMMMMMMMEPMMSM OMMN MEMMERAMMIONNOMOMMIMMOMMIMMOKIMMUMMUMMINIMMIN amumummumummommommummumanimmr---mmommomms
BANK D

MIUM

WM

MOMFAMOMMNIMMEMMEMMOMMEMOMMEMIL-JIMMINUMMINI mimmummoommummum linglnummumilmommumumma pirmommummulum mmuumMinumummummummium 2 mummummummis Wingalimm MOMMEMMEMMINIMMO WL M O M M I M M I O M M O MM O M O M E II R O M M O O MMM M U M MSO M M O M M M E E MNM O M M O M M I M M A MM E I M E E M M I M I I M M O M M O M M O M M I M MU M M M E N M M M O M M O M MM M A M O M M U M M O M U M O MO O MM U E O PM OM O MM O MO MM WIRMINOMMMONMENNWAMPEMMOMMOMEMITTOMMOMMOIN EMIUMMOMOMOMMORMOOMMIUMMOOMMI III mummommulmaskarmommummumm mommummem
80 90

nommommummilimminmimmEmmminummilMONN
0 0 1 0 2
*

mummenom
11 0

0 3 0 4 5 70 POWER (% of Rated Thermal Power)

Fully withdrawn shall be the condition where control rods are at a position within the interval c 225 and 231 steps withdrawn.

NOM The Rod Bank Insertion Limits are based on the control bank withdrawal Eequenne A, B, C, D and a control bank tip-to-tip distance of 115 steps.

FIGURE 3

PAGE 9 of 11

ROD BANK INSERTION LIMITS VERSUS % OF RATED THERMAL POWER

08/13/2007 09:27 FAX 706 826 3953 VEGP TRAINING CENTER

a 003

OED

rod- ko acre 5 7 - r Q -1=t. 6


SL$ 2.0

nvemigimm INTHISAREA
1111111111111
M
i l

oa

NOT

OEAE PRT

1111 i l

6 1611111 IN

a4oMpg

l i a l l a i r M I N I N I N E illire,V4111111111111=114

0 4

610

1111 E l l

19"

14111111111111111 1 IIINEM 1 111

680 ski

10

ZO

E t.r 'F A T . r I
40 . 50

50

70

so

10

100 11.0

PERCENT OF RATED THERMAL POWER

1. 0
120

Figure 2.1.11 Reactor Core Safety Limits

Vogtle Units 1 and 2

2.0-2

Amendment No. 128 (Unit 1)


Amendment No. 106 (Unit 2)

08/13/2007 09:27 FAX 706 826 3953

VEGP TRAINING CENTER

U004

GENE RIC ALS EXAMINATION EQUATIONS AND CONVERSIONS HANDOUT SHEET

EQUATIONS = rbepAT Q= MAh. = UAAT


I 1 4at

P = PolOsuK9 P = P oe ) A = Age4t = S/(1 Ic)

Circ

CR.,(1 - Ken) = CR2(1 - 1cff2) 1/M = CR,/eRx A =la' F = PA ria= pAV Wu =th,A.Pu Pmp E IR k f ir Eff_ = Net Work Out/Energy in
1)(p2 pi) + (722 Vi 2) + g(z2 0

AT IN2at Circ
CK K e f f I

p)

P (Kra 1)/Ktff SUR = 26.06k


=

P kff. P
e

* +

P =+

e' =

10' sec

IQs= 0.1 sec' (for small positive p) DRW x 44. 2 vg = 32.2 Ibm-ft/lbf-see CONVERSIONS

1 M.w = 3.41 x 106Btu/hr 1 hp = 2.54 x 103 Btu/hr 1 Btu = 778 ft-lbf QC F = (5/9)(F - 32) = (9/5)(QC) + 32

1 Curie = 3.7 x 101 dps 1 kg = 2.21 lbm

1 gal,,, = 8.35 Ibm 1 = 7.48 gal

INITIAL SUBMITTAL
CATAWBA EXAM 50-413, 414/2001-301 APRIL 2 - 6 & 16 - 20, 2001

INITIAL SUBMITTAL RO/a/te WRITTEN EXAMINATION

NRC Official Use Only

Nuclear Regulatory Commission Reactor Operator Licensing Examination

Catawba Nuclear Station RO Exam With ANSWER KEY


This document is removed from Official Use Only category on Date of examination

NRC Official Use Only

Catawba Sample Plan

Facility: Catawba K 1 1 1 0 K 2 5 2 1 K 3 3 4 0 Tier Totals 1 2 Plant Systems 2 3 Tier Totals 3 3 3 6 2 1 1 1 8 1 1 1 7 1 3 2 ,

Date of Exam: 5/19/00 K/A Catego K K 5 4 , ., .


44

Exam Level: RO Point A 2 2 1 1 A 3 r. y F . , A 4 G 1 4 0 Total 16 17 3 Target 16 17 3

Tier 1 Emergency & Abnormal Plant Evolutions

Group 1 2 3

Points K A 6 1 . 4 ,. , 5 1

' 1 4 1 3 1 0 1 1 0

10 1 3 1

4 5 3 0 2 2 0

r 3 1 1

5 4 0 1

36 23 20 8

36 23 20 8

6 Cat 1 3

2 Cat 2 3

8 Cat 3 4

5 Cat 4 3

51

51

Generic Knowledge and Abilities

13

13 Totals: 100

Note:

* Attempt to distribute topics among all K/A categories; select at least one topic from every K/A category within each tier. * Actual point totals must match those specified in the table. * Select topics from many systems; avoid selecting more than two or three K/A topics from a system unless they relate to plant-specific priorities. * Systems !evolutions within each group are identified on the associated outline. * The shaded areas are not applicable to the category/tier. ** Denotes plant specific, high priority K/As

Summary

For Official Use Only

2/8/01

Catawba Semple Plan

ElePE a / Name) Safety Function III

l il il ni l
1.02

KJA Toplc(e).

Imp.

Points

Question

000005 Inoperable/Stuck Control Rod ft

Ability to operate and/or monitor ...rod selection switches

N I

2.07

IIIIIIMIIIIIII
4.1 Knowl . - of EOP ant

1 779
11,786

2.9/2.9

000024 Emergency BoratIon11

conditions and Immediate action steps

4.3/4.6 2.9/3.0 2.6/2.8 4.2/4.2

1 764 1 782 III 783

000026 Loss of Component Cooling Water 1 VIII 000027 Pressuizer Pressure Control System Cont Malfunction / IIIr 000040 Steam Line Rupture - Exceestve Heat Transfer /N

107 2.03 112

Ability to operate andfor monitor rates to the components and systems that are serviced by the CCWS, interactions emo components Knowledge of the interrelationships betweencontrollers and posiboners 11111

1 791

000051 Loss of Condenser Vacuum 1 IV

000055 Station Blackout / VI

I
M

1 11 1

2.02 1.01

Ability to determine and interpret.. conditions requirin a reactor or turbine trip Ability to operate and/or monitor ...10-core thermocouple temperatures randomly deselected
Ability to determine and Interpret...the cause of possible SWS loss

3.9/4.1 3 7/3 9

1 53a 1

000062 Loss of Nuclear Service Water / IV

2.02

2.9/3.6

1 757

!!!!

3.12

Knowledge of the reasons for the following responses ... Required Sequence of actions for emergency evacuation of the control mom

4.1/4.5

5968

111
e C A / K
0

291

111 M

U Knowled = of the interralationshi . a between.... . ss radiation monitors ! I

2.6/3.0

1 380
16i 15

: s l a t o T

EAPEs 11 G1

For Official Use Only

2/6/01

Catawba Sample Plan

Bank EJAPE 61 Name 1 Safety Function K 1 K 2 K A 3 1 A Topic(*) Imp. Points Question

000001 Continuous Rod Withdrawal 1 I

1.32 Ability to amtain all system limits and precautions

3.4/3.8

1 503a

000003 Dropped Control Rod /I

4.1 Knowledge of EOP entry conditions and immediate action steps

4.3/4.8

1 768

000007 Reactor Trip Stabilization - Recovery / I

randomly deselected Knowledge of the operational implications of the following concepts ...thermodynamics end flow charactedstica of open or leaking valves

000008 Pressurizer Vapor Space Accident! lll

1.01

3.2/3.7

1 311

000009 Small Break LOCA / III

3,9 Knowledge of process for performing a containment purge

2.5/3.9

1 787

000011 Large Break LOCA nit

3.10

/ W/E04 LOCA Outside Containmentill

2.1

Knowledge Mite reasons for the following responses ... PTS limits on RCS pressure and temperature 3.7/3.9 Knowledge of the interrelationships between...components and functions of control and safety systems including instrumentation, signals, interlocks, failure modes, and automatic and manual features 3.513.9

1 080a

1,388

W/E.11 Loss of Emergency Coolant Racine i IV

randomly deselected Ability to determine and Interpret the following... facility conditions and selection of appropriate procedures during abnormal and emergency operations Ability to operate and/or monitor ...RCP seal flows, temperatures, pressures and vibrations

W/E02 SI Termination/III

2.1

3.3/4.2

1 185b

000022 Loss of Reactor Coolant Makeup / II

1.09

3.2/3.3

1 647a

000025 Lose of RHR System / W

randomly desedeCted

000029 Anticipated Transient wlo Scram 1 I

randomly deselected

000032 Loss of Source Range NI / VII

1.01

Ability to operate and/or monitor ...manual restoration of power

3.113.4"

1 393

000033 Loss of intermediate Range Ni 1 VII

randomly deselected Knowledge of RO tasks performed outside the mein control room during emergency operations Including system geography 434 and system implications

000037 Steam Generator Tube Leak 1111

3.8/3.8

1 300b

000038 Steam Generator Tube Rupture 1111

1.38

Ability to operate and/or monitor ...PZR heaters Knowledge of the reasons for the following responses ... manual control of AFW flow control valves Components and functions of control and safety systems, Including Instrumentation, signals, Interlocks, failure modes and automatic and manual features Ability to operate and/or monitor...cross-be of the affected DC bus with the alternate supply

3.3/3.3

1 193a

000054 Loss of Mein Feechvater 1 IV W1E05 Inadequate Heat Transfer - Loa of Secondary Heat Sink / N

3.03

3.8/4.1

1 768

1.1

4.114.0

1 769

000058 Lose of DC Power 1 VI

1.01

3.4'/3.5

1 770

008059 Accidental Liquid Radwasts Rel. 1 IX

randomly deselected Knowledge of the reasons for the following responses ... Isolation of auxiliary building ventilation

000000 Accidental Gaseous Midwest* Rel. / IX

3.02

3.3'/3.5'

1 771

000061 ARM System Mums /VII

randomly deselected Knowledge of the Interrelationships between...components and functions of control and safety systems including instrumentation, signals, interlocks, failure modes, and automatic and manual features 4 5 1 4 Group Point Total:

W1E16 High Containment Radiation ; IX ItiA Category Totals: 1

2.1 2

3.0/3.3 17

1 797 17
i

17

EAPEs T1 G2

For Official Use Only

2001

Catawba Sample Plan

s i! EPAPE #

SftFntn a uc e i y o .

K 1

K 2

K 3

A 2

K/A Toplc(al

Imp.

Points

Question

000025 Pressurizer Laval Malfunction 1 II

1.02

Ability to operate and/or monitor ...CVOS

3.4/3.4

1 104a

000035 Fuel Handling Accident! VIII

2.02

Ability to determine and interpret...occurrence of a fuel handling accident

3.414.1

1 33

000056 Loss of Off-sits Power! VI

randomly deselected

000063 Loss of Instrument Alr 1 VIII

randomly deselected

W/E 13 Steam Generator Overpressure / IV

randomly deselected Knowledge of the Interrelationships between...components and functions of control and safety systems Including instrumentation. signals. Interlocks, failure modes, and automatic and manual features

W/E 15 Containment Flooding / V

2.1

2.8/2.9

1 518

, -. .

...

K/A Category Totals;

Group Point Total:

EAPEs T1 G3

For Official Use Only

2/8/01

Catawba Sample Plan

System a/ Name

K 1

K 2 I

KK 4 II

KA H 5

A 2

A A 3 4

:Filn g

imp

Points Question

001 Control Rod Drive

Knowledge of the bales for prioritizing safely fundiOns during abnormal and 42 emergency operations 2 i I KnovAedge of the following operational implicatlons ..effects of RCP shutdown on secondary parameters eielh as elearti pressure, steam flow and feed Sow

3 0/4 0

1 714

003 Reactor Coolant Pump

50 .4

3 2/3 5

1 715

004 Chemical Volume Control

20 2 111.2 113 11 10

1111 11/ 1 1

Knowledge of bus power supplies to...makeup pumps

1 725

013 Engineered Safely Features Actuation

Ability to monitor automatic operation Of theOperation of actuated equipment 4.1/4.2

015 Nuclear Inatrumentetion

bt t r a p t t t e m eo to r q i l a r 2 2 A iyon n uaeh = o c nr l se ue I

t eaiy ew e s ud w a d ei na h f c b t e nh t io n n d s t i l t g

4.0/3.5

1 '

017 incont Temperature Monitor

20 .1 i

Abilityp w re e ds n t dto predict dm impede of the following malfunction or operation.. and ei ae o es v l g l s do p r t t o eae based on that* preddions, use procedures to correct, control or mitigate the consequences of...thermocouple open and short circuits 3.1/3.5 Kn wle o th e tas . O made O o th a a lo o malfu c will h on .. o dge f e ffec rd* f pe s ss r n tion ave in no tio itiatio f n I II MI IM Abelly to predict the impacts of the following malfunction or operation.. and based on those predictions, use procedures to correct control or II mitigate the diniequerldit Of...loss of oordensate pumps 1 Ability tOmanumit/ operate and/or monitor In the control mom...initiation of automaticlearNaiter Isolation
,

1 793

022 Containment Cooling

4.1/4.3

1 720

1111 111 11 1 111 11 1

20 4

l l

722

059 Main Feedwatiw

3 4/3 5

1 723 I N

11 11

50 1

Knowledge of the fdlowing operational Implicalons...relationship between II I AFVO Sow end RCS heat Varied IM IM 3.5/3 9

1 1 1 1 . 404

1111 111 1111 1/1 /11

OfKnoww6111led,quaofreleasetanksdesignMature(s)and/orinterlock(*)whichprovidefar. Isolskons 2.9/3.4

1 728

072 Area Radiation Monitoring 1 1 1 M 1 1 1 1 1

22 Knowledge of new and spent fuel movement procedures .8 Ability to manually operate and/or monitor In the control roorn...RHR temperature. PZR heaters and 80w, anal N , Ability to manually operate and/or monitor in the control room.. alarm and Interlocks eetpcint checke ended elinents

2.8/3.5

1 283

40 3

2.8'12.7'

1 728

4Of

3.0113

1 789

11 11

Ell

23

23

SYSTEMS 12 01

For Official Use Only

2/8101

10182

100

02

matito iod

Z0 Z.L. 2013.1.5AS

Or

00

00

:1010111110d0043

"

Z8Z 1111

0 FA Z 9 US Z

Ey'tux Impasuu00-M0101-10 amen. ow aka 1100ap jo wawa 'Daum. 01 sampsexud inn '1004.10000d 0004 uo pang pus ucuu0do00uorpundowBulmonimp00toodtuiottippord01Auocry

104 .

in, imam en

i l

111 111 1

84/ 1

PEWS

Illi
1 E l i

Ad - MAW 0 0 0 0 1 0 M 1 0P I1 0 1 1 0 0' 1 101 110 01 10P0 0 00C1 M0,L M0B i3s n st a a I 1 0 0 0 1 a 1 O 1 1 U 1 1 - 1 11 11n1 1 / I1P01 O 01 A P1up1 0 u n d 0 M es 0 2 0 , 1 2 2 0 2 1 0 4 .0 1 J i U1 1 0d 1A r 0 1 10 0 1 4 1 0 1 2 0 1 00 : 1 . wy 1CM0 0 01 0 101 10 PI IM O, 4 1 41 1 1 1 M M * Ismip 'mummy, 'amour limp mum& 008100010 .001004.0 01 4010
1

80'1 1 10 II
1/11 11 11 1

1 1 1 LS O, 10 I

M I N T AIWA'0110100J10sla 100111301304:1090

4Si'VE uonnauisia imouPet3 3V no nmewall .610 1N1111103 040

Amapa. I I
0411 4
0111 1

ME
0 0/9.0
0'011 0

urw... 1115054Him ualpunmewJO0.0010Oldpeal0010000001000001

J1001 0011 00001us'-unusti41W0060undauJO22012421.111021.111101.11.40Bal: 0.1)1 411.1.


1 1,210 0, 2 1114 ioodprypods '.110011o4utra ait01 icdutuu mtputmock. Ammuew 01410:00 -

xnususio Inoue gee


111 11 1 1

1. 01

1
11 1

0 4/9 0 eZ1111
141. .1:949E

1 On 1 KG 1 KC 1
LEL l

(404:smdui) uouns icy dums ilm i ummuo0 ui lug 0110001: JOI291101d1PIWAIIN10010010110/P20(2)0J01201004000000040.0101 10"' m00110420000100021212J2140SINN 0 4 B l e r'145 10 41 M()10 0 11 0 110( ) , 1 0 O / 23 0 W 0 uP a 0 9 01 0 4 21 00 1, 11 :0 500 2 1 W0 1 0 1 0 >1 MN110 3 0 I 14 1 40.001-'10,12PV,Old1P2.1.,0710001001ulumusMaims%06u.p00sBps0.000 W000101100030Jotsar'40swauenbesuct0Mo10000a4010.0un0VOA=01 Uunp000.81sonsuodnpEudasap110POMOPU11-LtODIRIed040 uon0undeut00000101510Jospedumusmiplud01 As0q0
1 1 S d 1 d 0 00 1 1'11 011 000111)1151 110 400X O 0 Z 0 0 4 0" 0 110 11410 0043408 0114 . 8

.6 ZIA Z
.1. V S Z

Leg 0

L0 O

1 1 1 1 1
1 0 I I ill 1
11/ 11 11 1

amid youlunnuo3 60
I

L4 O II 104 I 004

Amide uswiennuon 6.80 uonuumundsul Jesi0m000N 010 00112010010011120dP01:010 I uon0104110101uN no 11 11 C0 a 004 1041100 WWI Jorunsuud 110 100003 iunssud Hann sum 010

i MT 4'108t

9L

I Ni

d
wilg" 14u0d

" Et SVAIS2-000100080111. Imo pug emulows suormuuna 00011/610) eepommouN 'VC V0 V114 E41, 101 N E

l I

autos 10winsAs

yew OldunISInInnt103

C t w aS m l Pa aa b e pe l n

SystemalN.m. 0 5R sd nH a R m v l 0 ei u i e t e o a

KC 1I I K 10 1

31 14

51 11 11 :: 10 M a

W T pcs A o lf)

I p m.

Pons it MIL

Q eb n uso 71 5

"

3
1

0 1

3 .

1 1

1 M

1 I

1 I

1 I

1 M

1 I 41 42 ./. I I 2 ' 28 51 . ' 17 2 5 1 M

0 8C m o e tC oi gW t r 0 o p n n o ln ae 0 7C nan e tI dn R m v l 2 o ti mn o i e e oa 023 dtenRacomblnerandP. Control 0 4F eH n ln E up e t 3 u l a di g q i m n 0 1Se mD m l ubn B p s C nr l 4 t a u pT r i s y a s o to 0 3M i T r i eG n r t r 4 en ubn e eao


111 111 111 11

. 1 43 0

1 1 1 1 1 1 1 1 1 1 1 M M I E
1 1 1 1 1 1 1 , 1 , 1 1 1

17 8 9 M E I n 174 5 I I I

r n o l d s l ce a d my e ee t d M ryt p rompes okppo e uef rt er aiyi cu i go eai gtoe at o ef r r - h r i r c d r . o h e lt ,n l dn p r tn hs 2 c nr l a s d t dwt p ne up e tM tc udaf cr a t i 1 o tos s o ee ih e t q i m n e o l fe t e cn 3/ a 73 r n O ld itet d a d m etl ie

1 3C nan e t 0 o ti mn

I I I I I I I I I I I I I I

K/A Co o

Toull*:

1 1 1 1 1 1 1 1 1 1 1 1 1 1 1 11111111111 1 1111111 1 11111111111 1 11111 1111111 1


1 111 11
1 1 1 1 1 1 111111 1 1 1 2 1 0 E 1 0 0 1

.
norxicelly doulected 1 1 1 1 . 1 1 1 1 1 1 1 1 1 1 1 1 1 . IIIII I I I

1 1 1 1 1 1 1 1 1 1

I I I E l l

I I

I I I E l l

1 1 1 1 1 1 1 1 1 1 1 1 1 1 1 1 1 1 1 1 1 1 1 1 1 1

~~ ~ t
MEM I / 1 1 I I I I

I I 1 Grey. Paint Total: I

SYSTEM* T203

For Official Use Only

2/8101

Catawba Sample Plan

M E N E M

Conduct of Operations

E l l
. 1 . 1.31

Topic .11 o eva ua a p an pe ormance an. ma a opera ions j .gmen s 'ass, on 1.7 operating characteristics, reactor behavior, and instrument interpretation to obtain and interpret station electrical and mechanical drawings

im. 3.7/4.4 2.8/3.1

Points 1 1 1 M E =

Question M M r r 792

Ability to locate control room switches, controls and indications to determine if 4.2/3.9 A b y t l ithey are correctly reflecting the desired plant lineup

Knowledge of process for determining the internal and external effects on core Equipment 2.34 reactivity Control M I El E l l 189a

IIIII 3 3.2 Knowledge of facility ALARA program Knowledge of radiation exposure limits and contamination control 3.4 including permissible levels in excess of those authorized 1.11 o pe orm proce. ures o re. uce excessive eve s o raia ion an. guar. 3.10 against personnel exposure 3.11 Ability to control radiation releases M I 2.5/3.1 2.9/3.3 2.7/3.2

ME

N
124

3 1 1 661a 111M1 1 671a M i l l 775

Radiation Control

IIIII

M E

M E
Emergency Procedures and Plan

4 3.1/3.8 2.7/3.7 3.0/3.5 111. 1111. 1 1

776

w o n k g d e l of EOP terms and definitions

4.19 Knowledge of EOP layout, symbols and icons 4.15 Knowledge of communications procedures associated with EOP implementation

13a 39 I I M

3 l a T t n i o P 3 r e i T 1 3

3 1 3

Generics

For Official Use Only

2/8/01

Catawba NRC Operator Licensing Examination The following table summarizes the changes made to the randomly selected K/As. All changes were discussed and approved with the NRC Chief Examiner. EAPE or System T1G1 000024 Emergency Boration T1G2 000003 Dropped Control Rod G2.4.1 K/A Tested G2.4.1 Summary of Changes Randomly selected 2.3.9 (Radiation Control knowledge of process for performing a containment purge). This K/A was not applicable for the selected EAPE. Selected next applicable K/A in sequence. Randomly selected 2.3.1 (Knowledge of 10CFR20 and related facility radiation requirements) and 2.3.4 (Knowledge of radiation exposure limits, contamination control including permissible limits in excess of those authorized). These K/As are PWGs that are not applicable to this EAPE. Randomly selected next K/A in sequence. Randomly selected 2.3.4 (Knowledge of radiation exposure limits, contamination control including permissible limits in excess of those authorized). This K/A was a PWG that is not applicable to this EAPE. Randomly selected next K/A in sequence. Randomly selected K3.11 (Knowledge of the reasons for the following responses ...NC and PC). The team could not determine what this K/A was testing and did not know the terminology "PC". Randomly reselected K3.10. Randomly selected G2.4.39 (Knowledge of ROs responsibilities in emergency plan implementation). This was a PWG that did not test any EAPE specific knowledge. Randomly reselected G2.4.34. The original sample plan contained a typographical error. The stem statement was incorrect. Corrected in the attached sample plan. Randomly selected 2.2.6 (Knowledge of process for making changes in procedures as described in the SAR) and 2.2.29 (Knowledge of SRO fuel handling responsibilities). These K/As are PWGs that are not applicable to this system. Randomly selected G2.2.2.

T1G2 000009 Small Break LOCA

G2.3.9

T1G2 000011 Large Break LOCA

K3.10

T1G2 000037 Steam Generator Tube Leak T1G2 000058 Loss of DC Power T2G1 015 Nuclear Instrumentation

G2.4.34

A1.10

G2.2.2

For Official Use Only

Page 1

2/8/01

Catawba NRC Operator Licensing Examination T2G1 068 Liquid Rad Waste A2.04 Shifted from K/A 2.03 with NRC approval (Ability to predict the impacts of the following malfunction or operation...and based on those predictions, use procedures to correct, control or mitigate the consequences of... insufficient sampling frequency of boric acid in the evaporator bottoms) because we did not know what this K/A was testing. Randomly selected A2.04. Originally randomly sampled the Condensate System (0056). Decided to resample system to RCPs because the condensate system had already been sampled once and there were very few K/As that were < 2.5 in this system. Randomly sampled G2.1.16 (paging system operation) and G2.1.26 (non-nuclear safety high temp, high pres, etc). These K/As were PWGs and were not directly applicable to this system. Randomly selected G2.1.32. Randomly sampled K4.02. However, there was only 1 K1 that had been randomly sampled in all Tier 2 systems on the SRO exam. The ES-1021 requires a minimum of 2 questions in each category to assure that all categories are represented (assuming one question is deleted during exam review). Randomly selected this question (from all systems that had K4s selected) and randomly reselected K1.02. This increased the number of Kls from I to 2 in Tier 2 and decreased the number of K4s from 7 to 6 on the SRO exam. Randomly selected A1.02 (Ability to predict and/or monitor changes in parameters (to prevent exceeding design limits) associated with operating the...fuel consumption rate with load). However, this K/A was judged not to be appropriate RO level of knowledge at Catawba. Randomly selected K/A G2.3.6 (Radiation control: Knowledge of requirements for preparing a radiation work permit), K/A G2.3.10 (Radiation Control: Ability to perform procedures to reduce excessive levels of radiation and guard against

T2G1 003 Reactor Coolant Pump

A4.03

T2G1 013 Engineered Safety Features Actuation T2G2 010 Pressurizer Pressure Control

G2.1.32

K1.02

T2G2 064 Emergency Diesel Generator

A1.08

T2G3 041 Steam Dump/Turbine Bypass Control

G2.2.1

For Official Use Only

Page 2

2/8/01

Catawba NRC Operator Licensing Examination personnel exposure) and G2.3.4 (Knowledge of radiation exposure limits, contamination control including permissible limits in excess of those authorized) these 1C/As were determined to be inappropriate for this system. Selected G2.2.1 at the direction of the Chief Examiner. Randomly selected G2.1.27 (Knowledge of system purpose or function), which was not a PWG. Randomly resampled. Randomly sampled G2.2.4 (Ability to explain the variations in control board layouts, systems, instrumentation and procedural actions between units at a facility). This K/A had initially already been used in systems questions and was removed as a duplicate but this K/A was later deselected during the sample plan review process. Randomly selected G2.4.22 (Knowledge of bases for prioritizing safety functions during abnormal and emergency operations). This K/A had initially already been used in systems questions and was removed as a duplicate from Tier 2 Group 1 Control Rod Drive system. Randomly selected G2.4.15 as replacement KJA.

Tier 3 Group I Conduct of Operations Tier 3 Group 2 Equipment Control

G2.1.24 G2.2.22

Tier 3 Group 4 Emergency Procedures and Plans

G2.4.15

For Official Use Only

Page 3

2/8/01

RO Exam References: Steam Tables question 311 Tech Specs: Tech Spec 3.4.13 - question 480A Tech Spec 3.8.4 - question 770 Tech Spec 3.8.7 - question 770 Tech Spec 3.8.8- question 770 Tech Spec 3.9.2 - question 774 SLCs:

SLC 16.11-2

EOPs E-1 page 5, Steam Tables question 185b ES-0.1 End 3 page 21 question 780 AOPs AP/1/A/5500/17 (Loss of Control Room) question 189a Other Reactor Operating Databook, Sections 4.1 & 4.2 question 189a

NRC Official Use Only

Nuclear Regulatory Commission Reactor Operator Licensing Examination

Catawba Nuclear Station RO Exam


Answer Key

This document is removed from Official Use Only category on Date of examination

NRC Official Use Only

For Official Use Only

Catawba Nuclear Station RO Exam Question #1

Bank Question: 033 1 Pt(s)

Answer: B

Unit 1 is in a refueling outage. Given the following events and conditions: A full core off-load is in progress One spent fuel assembly is in the fuel transfer tube being transported to the spent fuel pool The following annunciators alarm: SPENT FUEL POOL LEVEL HMO 1EMF-17 REACTOR BLDG REFUEL BRIDGE

Which one of the following correctly describes the type of event and the required operator actions that should be performed first? A. Loss of refueling cavity or spent fuel pool level. Install the weir gate and inflate the seals. Loss of refueling cavity or spent fuel pool level Move the fuel transfer cart to the spent fuel side (pit). Loss of spent fuel pool level only Move the fuel transfer cart to the reactor side.

B.

C.

D.

Loss of refueling cavity level only Close 11(F-122 (KF Fuel Transfer Canal Isolation).

Distracter Analysis: A. B. C. Incorrect: using the weir gate is optional. Plausible: this is one of the operator follow-up actions. Correct: Incorrect: the cart must be moved to the SFP side. Plausible: if the problem is on the SFP side it might be reasonable to not to add more fuel to that side. Incorrect: level is dropping on both sides, and you can't close valve with cart in the tube. Plausible: the candidate may choose this answer due to the EMF alarm and closing the valve is the next action. Level: RO&SRO

D.

Ques_033

For Official Use Only

Page 1

KA: APE 036AA.2.02(3.2/3.9) Lesson Plan Objective: KF LPRO 15

For Official Use Only

Catawba Nuclear Station RO Exam Question #1

Source: Mod Ques_033e Level of knowledge: memory References: 1. OP-CN-FH-FHS pages 7, 17 2. AP/1/A/5500/26 pages 1-2

Ques_033

For Official Use Only

Page 2

For Official Use Only

Catawba Nuclear Station RO Exam Question #2

Bank Question: 080a 1 Pt(s)

Answer: D

Which one of the following accidents has the highest severity for pressurized thermal shock (PTS) in the NCS? A. B. Small break LOCA, NCPs running Large break LOCA, NCPs NOT running

C. D.

Large break LOCA, NCPs running Small break LOCA, NCPs NOT running

Distracter Analysis: A. B. Incorrect: PTS is worse with NCPs not running. Plausible: small LOCAs have PTS potential. Incorrect: large break LOCA has little concern for PTS. Plausible: If candidate misunderstands significance of no pressure versus NCPs not running. Incorrect: large break LOCA has little concern for PTS Plausible: large break LOCAs with pumps running are more limiting in other respects. Correct: Level: RO&SRO KA: EPE 011EK3.10(3.7/3.9) Lesson Plan Objective: PTS SEQ 13, 14 Source: Mod Ques_080 Level of knowledge: comprehension References: 1. OP-CN-TA-PTS page 12-18

C. D.

Ques_080a

For Official Use Only

Page 3

For Official Use Only Catawba Nuclear Station

RO Exam Question #3

Bank Question: 104a 1 Pt(s)

Answer: C

Unit 1 is operating at 100% power. Given the following events and conditions: Pressurizer pressure and level controls are in the 1-2 position. NCS pressure is 2200 psig NCS temperature is 560 F. Charging flow increases Pressurizer level increases PZR low level deviation alarms. All pressurizer heaters remain energized throughout the event

What is the cause of these indications? A. B. C. D. Pressurizer level master controller output fails high Pressurizer level channel I fails low NCS Loop C narrow range T, channel fails high Pressurizer level channel II fails low

Distracter Analysis: Tcold failing high causes reference level to fail high. This causes the master level controller output to go high on reference level. This causes increased charging and increased PZR level. A. Incorrect: would not cause DEV-LO alarm not controlled by the master level output. Plausible: would cause all other indications Incorrect: charging flow decreases and heaters would trip off Plausible: will cause other symptoms. Correct: Heaters on due to current NCS pressure. Incorrect: heaters would trip off Plausible: will cause other symptoms Level: RO&SRO KA: APE 028AA1.02(3.4/3.4) Lesson Plan Objective: ILE LPSO 6 Source: Mod Ques_104 Level of knowledge: comprehension

B. C. D.

Ques_104a

For Official Use Only

Page 4

For Official Use Only

Catawba Nuclear Station RO Exam Question #3

References: 1. OP-CN-PS-ILE page 15-17, 21

Ques_104a

For Official Use Only

Page 5

For Official Use Only

Catawba Nuclear Station RO Exam Question #4

Bank Question: 124 1 Pt(s)

Answer: B

A team of workers must repack the seats on a pump in a 1500 mrem/hr high radiation area. Which one of the following work teams and estimated repair times would maintain worker exposure ALARA? A. B. C. D. 10 people working for 20 minutes 6 people working for 30 minutes 4 people working for 1 hour 2 people working for 2 hours

Distracter Analysis: A. B. C. D. Incorrect: six people can accomplish the job with 4 1/2 Rem. Plausible: Each individual would have the least exposure. Correct: Incorrect: six people can accomplish the job with 4 1/2 Rem. Plausible: fewest individuals not exceeding the admin dose limit. Incorrect: six people can accomplish the job with 4 1/2 Rem. Plausible: Exposes the fewest individuals. Level: RO&SRO KA: G2.3.2 (2.5/2.9) Lesson Plan Objective: HP LPRO 10 Source: Mod Ques_124e Level of knowledge: comprehension References: 1. OP-CN-RAD-HP page 19

Ques_124

For Official Use Only

Page 6

For Official Use Only

Catawba Nuclear Station RO Exam Question #5

Bank Question: 162a 1 Pt(s)

Answer: B

Unit 1 is purging containment while in mode 5. Which one of the following instruments will prevent the release of radioactivity outside containment by completing the corresponding sequence of actions? A. EMF-36 (UNIT VENT GAS) will secure VP and VQ, and stops any waste gas release in progress. EMF-39 (CONTAINMENT GAS) will sound the containment evacuation alarm, secure VP and initiate containment ventilation isolation. EMF-40 (CONTAINMENT IODINE) will initiate containment ventilation isolation, and shutoff containment sump pump and ventilation drain headers.

B.

C.

D.

EMF-53A/B (CONTAINMENT TRN A(B) HI RANGE) will secure VP and VQ, and shutoff containment sump pump and ventilation drain headers.

Distracter Analysis: A. Incorrect: EMF-36 does not secure VP. Plausible: EMF-36 monitors the final VP release and secures the others. Correct answer Incorrect: EMF-40 does not isolate containment drains. Plausible: EMF-40 will isolate the VP release. Incorrect: EMF-53 will not secure VP. Plausible: EMF-53 is a containment radiation monitor, and isolates the containment drains. Level: RO&SRO KA: SYS 029A3.01(3.8/4.0) Lesson Plan Objective: EMF LPRO 2 Source: Mod Ques_162 Ques_162a

B. C. D.

For Official Use Only

Page 7

Level of knowledge: memory

For Official Use Only

Catawba Nuclear Station RO Exam Question #5

References: 1. OP-CN-WE-EMF pages 17-18

Ques_162a

For Official Use Only

Page 8

For Official Use Only

Catawba Nuclear Station

RO Exam Question #6

Bank Question: 185b 1 Pt(s)

Answer: C

Unit 1 was operating at 100% power when a steam line rupture occurred inside containment. The operators responded by entering: E-0 (Reactor Trip or Safety Injection) followed by E-2 (Faulted Steam Generator Isolation) where they isolated the rupture and then transitioned to E-1 (Loss of Reactor or Secondary Coolant).

Given and the following conditions at the following times: Time Subcooling [F] S/G A (NR) [%] S/G B (NR) [%] S/G C (NR) [%] S/G D (NR) [%] Feed Flow A S/G [GPM] Feed Flow B S/G [GPM] Feed Flow C S/G [GPM] Feed Flow D S/G [GPM] NC pressure [psig] Pzr Level [%] Containment pressure [psig] 0200 +9 5 8 0 9 135 150 0 170 1710 15 3.2 0205 +6 7 10 0 15 135 130 0 180 1725 18 2.9 0210 +5 9 16 0 21 145 160 0 150 1750 21 2.5 0215 +2 11 20 0 30 130 160 0 155 1765 25 2.4

At 0200, the operators are at step 12 of E-1. What is the earliest time that the operators can transition to ES-1.1 (Safety Injection Termination)? REFERENCES PROVIDED E-1 page 5, Steam Tables A . B . C . 0200 0205 0210

D .

0215 Distracter Analysis:

--

A.

Incorrect: PZR level does not meet the ACC of 20%.

Ques_185b

For Official Use Only

Page 9

For Official Use Only

Catawba Nuclear Station RO Exam Question #6

B.

C. D.

Plausible: If the candidate uses non-ACC value of 11%. Incorrect: PZR and SG levels do not meet the ACC of 20%and 29% Plausible: If the candidate uses non-ACC values due to Cont Press <3.0 Correct: AFW flow >450gpm and PZR level and subcooling above limits. Incorrect: Meets the criteria for termination later in time. Plausible: Based on miscalculating the AFW flow for C. Level: RO&SRO KA: WE 02AA2.1 (3.3/4.2) Lesson Plan Objective: EP2 LPRO 8, 9 Source: Bank 185 Level of knowledge: analysis References: 1. OP-CN-EP-EP2 page 8 2. E-1 page 5 - PROVIDED 3. OMP 1-7 page 7

Ques_185b

For Official Use Only

Page 10

For Official Use Only

Catawba Nuclear Station RO Exam Question #7

1 Pt(s) Bank Question: 189a Answer: C

Unit 1 was operating at 100% power when a toxic gas accident caused the operators to evacuate the control room and take control at the Auxiliary Shutdown Panel (ASP) Given the following conditions:

When the reactor tripped, 2 control rod bottom lights failed to light Pressurizer pressure stabilized at 2235 psig Safety injection has not occurred T a d d = 500 F Core bum up = 90 EFPD Boron concentration = 1200 ppm The OAC is out of service

How much boric acid must be added in accordance with AP/1/A/5500/17 (Loss of Control Room) to maintain shutdown margin? REFERENCES PROVIDED Reactor Operating Databook, Sections 4.1 & 4.2 AP/1/A/5500/17 (Loss of Control Room) A. B. 285028 gallons 15460154 gallons

C. D.

20550205 gallons 24025240 gallons Distracter Analysis: AP/17 Enclosure 7, step 1.b requires boration to 2850 ppm. This will require 20553 gallons of boric acid.

A. B. C.
Ques_189a

Incorrect: 20553 gallons of boric acid required Plausible: if the candidate misreads the need add 2850ga1 instead of getting to 2850 graph Incorrect: 20553 gallons of boric acid required Plausible: if the candidate does not calculate the addition properly (various plausible errors). Correct Answer: borate to 2850 ppm which requires 20553 gallons to go from 1200 ppm to 2850 ppm
For Official Use Only Page 11

D.

Incorrect: 20553 gallons of boric acid required Plausible: if the candidate uses the wrong table, or other miscalculations.

For Official Use Only Catawba Nuclear Station

RO Exam Question #7

Level: RO&SRO KA: G2.2.34 (2.8/3.2) Lesson Plan Objective: RT-RB LPRO 4 Source: Mod Ques_189 Level of knowledge: analysis References: 1. OP-CN-RT-RB page 9, 10 2. AP/1W5500/17 - PROVIDED 3. Reactor Operating Databook Sections 4.1 & 4.2 - PROVIDED

Ques_189a

For Official Use Only

Page 12

For Official Use Only

Catawba Nuclear Station RO Exam Question #8

Bank Question: 190a 1 Pt(s)

Answer: A

Unit 1 is in a refueling outage with the following plant conditions: NC system is in mid-loop ND is in operation Refueling has been completed

If there are no other exceptions to the containment closure verification, which one of the following statements correctly describes the required status of the containment equipment hatch? A. Must be kept closed at all times while the NC system is in mid-loop. May be open provided a Watch is stationed to immediately close the hatch in the event of a loss of ND. Must be fastened by at least 4 bolts in mode 6.

B.

C.

D.

May be open provided contingency plans are in place and equipment is staged to close the hatch prior to core boiling in the event of a loss of ND.

Distracter Analysis: A. B. Correct: Incorrect: it takes more than one person to close this hatch Plausible: based on misconstruing equipment vs. personnel hatches In co rr ec t: the e qui pme nt hat ch ma y be ope n i n mo de 6 i f co re al te ra tio ns a re not i n pro g ress Pl au sibl e: t his i s t he T S re qui rem e nt fo r co re a l te ra t io ns i n mo de 6 . Incorrect: exceptions may NOT be authorized in mid-loop per SD 3.1.30. Plausible: if the candidate misunderstands the containment closure program. Level: RO Only KA: APE 069AK2.03(2.8 / 2.9)

C.

D.

Ques_190a

For Official Use Only

Page 13

Lesson Plan Objective: CNT-CNT SEQ 23 Source: Mod Ques_190

For Official Use Only

Catawba Nuclear Station RO Exam Question #8

Level of knowledge: memory References: 1. OP-CN-CNT-CNT page 22 2. Site Directive 3.1.30 page 19 - 20

Ques_190a

For Official Use Only

Page 14

For Official Use Only

Catawba Nuclear Station RO Exam Question #9

1 Pt(s)
Bank Question: 193a Answer: A

E-3 (Steam Generator Tube Rupture), enclosure 5 (NC Pressure and Makeup Control to Minimize Leakage) directs operators to energize pressurizer heaters if the ruptured SIG level is decreasing and pressurizer level is greater than 25%. What is the basis for this action in E-3? A. B. Maintain pressurizer saturation temperature equal to ruptured SIG pressure. Maintain pressurizer saturation temperature below the S/G PORV setpoint.

C. D.

Maintain pressurizer saturation temperature above ruptured S/G pressure. Maintain NCS pressure above ruptured S/G pressure.

Distracter Analysis: The purpose of this question is to determine if the candidate understands that thermal hydraulic equilibrium that is established between the NCS and the ruptured S/G. No references are provided because the candidate should be able to answer the question by simply comprehending the pressures and reasons for this equilibrium. A. B. Correct Answer: Incorrect: energizing heaters will not reduce (or hold down) pressure. Plausible: it is another requirement to maintain NCS pressure below the PORV setpoint. Incorrect: required to maintain NCS pressure equal to ruptured S/G pressure Plausible: condition will address decreasing S/G level but overcompensate. Incorrect: required to maintain NCS pressure equal to ruptured SIG pressure. Plausible: condition will address decreasing S/G level but overcompensate. Level: RO&SRO KA: EPE 038EA1.38(3.3/3.3)
Ques_193a For Official Use Only Page 15

C.

D.

Lesson Plan Objective: EP-EP4 Obj: 7

For Official Use Only

Catawba Nuclear Station RO Exam Question #9

Source: Bank McGuire Exam 1997 Level of knowledge: comprehension References: 1. OP-CN-EP-EP4 page 8 2. E-3 page 61 4. ERG Background document E-3 pages 48-49

Ques_193a

For Official Use Only

Page 16

For Official Use Only

Catawba Nuclear Station RO Exam Question #10

Bank Question: 263 1 Pt(s)

Answer: A

Unit 1 is shutdown in mode 6 with fuel movement in progress. Given the following events and conditions: The new fuel elevator fails to operate in the up direction Which

one of the following statements describes the cause of this problem? A. B. C. D. 1EMF-15 (SPENT FUEL BLDG REFUEL BRIDGE) has failed high. IEMF-20 (NEW FUEL STOR 1A) has failed high. The load in the new fuel elevator weighs 1100 lbs. The spent fuel bridge crane is NOT indexed over the new fuel elevator. Distracter Analysis: A. B. Correct answer Incorrect: does not have an interlock with the new fuel elevator Plausible: new fuel vault monitor sounds like it "fits" with new fuel monitor if candidate does not know answer Incorrect: If load exceeds 1200 lbs., will prevent movement Plausible: this is a valid interlock but the weight is insufficient to actuate it Incorrect: there is no interlock to prevent moving the new fuel elevator Plausible: there is an interlock to prevent moving the spent fuel pool crane Level: RO&SRO KA: SYS 072G2.28 (2.6/3.5) Lesson Plan Objective: FHS LPRO 8 Source: NRC Catawba Exam 1999 Ques 263 Level of knowledge: memory References: 1. OP-1/B/6100/010Z C/5

C.

D.

Ques_263

For Official Use Only

Page 17

For Official Use Only

Catawba Nuclear Station RO Exam Question #11

Bank Question: 282a 1 Pt(s)

Answer: B

Which of the following describes the plant response to decreasing VI system pressure? A. 86 psig - Standby Compressor starts 80 psig - VI-78 (VS AUTO BACKUP TO VI) opens. 76 psig - VI-500 (VI COMPRESSOR D TO VS HEADER BACKPRESSURE CONTROL) closes. B. 86 psig - Standby Compressor starts 80 psig - VI-500 closes. 76 psig - VI-78 opens. C. 86 psig- Backup Temporary/ Diesel VI Compressor starts 80 psig - VI-500 closes. 76 psig - VI-78 opens. D. 86 psigBackup Temporary/ Diesel VI Compressor starts 80 psig - VI-78 opens. 76 psig -. VI-500 closes Distracter Analysis: Loss of VI (Obj. #5, 8) Automatic actions: 86 psig - Standby Compressor starts 80 psig - 'LO VI PRESS' Alarm in Control Room. 80 psig VI 670 'VI Dryer Auto Bypass' opens 80 psig - VI500 'VI supply to VS' closes. (Tag label in answers) 76 psig - VS78 'VS supply to VI' opens - VS provides instrument air via oil removal filters. (Tag label in answsers) Incorrect: VI 500 and VI 78 actions are in reverse order Plausible: If candidate reverses the order Correct: Incorrect: Plausible: backup compressor does not automatically start Incorrect: Plausible: backup compressor does not automatically start

A. B. C. D.

Ques_282a

For Official Use Only

Page 18

Level: RO&SRO KA: SYS 079A2.01(2.9/3.2)

For Official Use Only Catawba Nuclear Station RO Exam Question #11

Lesson Plan Objective: VI SEQ 5, 8 Source: NRC Catawba Exam 1999 Ques_282 Level of knowledge: memory References: 1. VI lesson plan page 19 of 36

Ques_282a

For Official Use Only

Page 19

For Official Use Only

Catawba Nuclear Station RO Exam Question #12

Bank Question: 300b


1 Pt(s)

Answer: A

Unit 1 was operating at 100% power when a 5 gpm S/G tube leak occurred in the B S/G. Given the following events and conditions: The operators implement AP/10 (REACTOR COOLANT LEAK). The steam supply to the turbine driven CA pump must be isolated

Which one of the following statements describes the correct method for isolating B S/G steam supply to the turbine drive CA pump? A. B. C. Manually close the isolation valve (1SA-1) in the doghouse. Manually close the stop-check valve (1SA-3) in the doghouse. Manually close the isolation valve (1SA-4) in the mechanical penetration room. Manually close the stop-check valve (1SA-6) in the mechanical penetration room.

D.

Distracter Analysis: A. B. Correct: Incorrect: wrong location 1SA-3 is located in the aux building mechanical penetration area - not the doghouse. Plausible: 1SA-3 is the RNO action if 1SA-1 does not close. Incorrect: 1SA-4 isolates the steam supply from the C S/G - not located in the mechanical penetration area. Plausible: if the candidate does not recognize the wrong valve Incorrect: will not isolate steam to the B S/G Plausible: 1SA-6 is the RNO action if 1SA-4 does not close. Valve is located in the mechanical penetration area. Level: RO&SRO KA: APE 037G2.4.34 (3.8/3.6) Lesson Plan Objective: none Source: Mod Catawba Exam 1997 Ques_300 Level of knowledge: memory References:

C.

D.

Ques_300b

For Official Use Only

Page 20

For Official Use Only Catawba Nuclear Station RO Exam Question #12 1. AP-10 page 21 1. OP-CN-STM-SM page 13, 18

Ques_300b

For Official Use Only

Page 21

For Official Use Only

Catawba Nuclear Station RO Exam Question #13

Bank Question: 311

Answer: A

1 Pt(s) Unit 1 is operating at 50% power. Given the following conditions: Pressurizer pressure is 2235 psig Pressurizer Relief Tank (PRT) pressure is 21 psig PRT temperature is 125 F PRT level is 81% A pressurizer code safety valve is suspected of leaking by it's seat

What temperature would be indicated on the associated safety valve discharge RTD if the code safety were leaking by? REFERENCES PROVIDED: Steam Tables A. B. C. D. 258-262 F 227-231 F 161-165F 123 -127F

Distracter Analysis: A. B. Correct answer Incorrect: Temp is too low - the correct temp is 260 F Plausible: If the candidate makes the mistake of not correcting for atmospheric pressure by failing to adding 14.6 psi to the PRT pressure and uses 20 psia. Incorrect: Temp is too low - the correct temp is 260 F Plausible: If the candidate reverses the correction for atmospheric pressure by subtracting 14.6 psi from PRT pressure of 20 psig to get 5 psia. Incorrect: Temp is too low - the correct temp is 260 F Plausible: If the candidate thinks that the discharge temperature will be at the same temperature as the PRT fluid. Level: RO&SRO KA: APE 008AK1.01(3.2/3.7) Lesson Plan Objective: FLO SEQ 8

C.

D.

Ques_311

For Official Use Only

Page 22

For Official Use Only Catawba Nuclear Station RO Exam Question #13 Source: NRC Catawba Exam 1999 Ques_311 Level of knowledge: analysis References: 1. OP-CN-THF-FLO pages 17-18 2. Steam Tables - PROVIDED

Ques_311

For Official Use Only

Page 23

For Official Use Only

Catawba Nuclear Station RO Exam Question #14

Bank Question: 339a 1 Pt(s)

Answer: D

E-3, (Steam Generator Tube Rupture), step #21.b reads as follows: "IF AT ANY TIME ruptured S/G(s) pressure is decreasing..., THEN perform Step 21." Which one of the following statements is correct with regards to this step? A. The step is applicable continuously unless it is determined not to be pertinent to the recovery effort. The step is applicable while in E-3 and after transition to subsequent procedures until alternative guidance is provided.

B.

C.

The step is only applicable until another continuous action step is reached in E-3. The step is only applicable while in E-3.

D.

Distracter Analysis: A. B. C. Incorrect: step applicable only inE-3 Plausible: this is the construct for When ... then actions Incorrect: step applicable only inE-3 Plausible: this is an alternate construct for If at any time actions Incorrect: step applicable only inE-3 Plausible: this is the construct for generalized continuous action steps. Correct: Level: RO&SRO KA: G2.4.19 (2.7 / 3.7) Lesson Plan Objective: ADM-OP SEQ 21 Source: NRC Catawba Exam 97 Ques_339 Level of knowledge: memory

D.

Ques_339a

For Official Use Only

Page 24

References: 1. OP-CN-ADM-OP page 10 2. OMP 1-7 page 7 3. EP/1/A/5000/E-3 page 22

For Official Use Only

Catawba Nuclear Station RO Exam Question #15

1 Pt(s) Bank Question: 343 Answer: B

Unit 1 is responding to a LOCA inside containment. Which one of the following situations describes the case where the operators should use ES1.2 (Post LOCA Cooldown and Depressurization) to bring the plant into cold shutdown? A. A small break LOCA with a loss of containment sump level due to a ruptured FWST. A small break LOCA where NC system pressure is above shutoff head for the ND pumps.

B.

C.

A large break LOCA where NC system pressure is below shutoff head for the ND pumps. A large break LOCA with a loss of both NI pumps.

D.

Distracter Analysis: A. Incorrect: would use ECA-1.1 (Loss of Emergency Coolant Recirc) Plausible: if the candidates do not recognize that a loss of containment sump level causes a loss of recirc capability Correct Answer: ES-1.2 is used for small or intermediate break LOCAs where the plant can be repressurized Incorrect: remain in E-1 Plausible: if the candidates do not understand the major actions categories for E-1 Incorrect: ES-1.2 is entered from ES-1.1 when NC pressure is less than shutoff head for NI pumps requires NI pumps to be injecting Plausible: if the candidates do not recall that they remain in E-1 for large break LOCAs Level: RO Only KA: WE 03EK3.4(3.5 / 3.9) Lesson Plan Objective: EP-EP2 SEQ 3 Source: NRC Catawba Exam 97, Ques_343 Level of knowledge: memory

B. C.

D.

Ques_343

For Official Use Only

Page 25

References: 1. OP-CN-EP-EP2 page 10

For Official Use Only

Catawba Nuclear Station RO Exam Question #16

Bank Question: 371 1 Pt(s)

Answer: C

In the event of a steamline rupture that cannot be isolated, FR-P.2 (Response to Anticipated Pressurized Thermal Shock Condition), could be implemented due to an excessive cooldown. What is the limiting component for this PTS event and what is the best indication of the temperature of this component? Limiting Component A. B. C. D. Steam generator tube sheet Pressurizer spray nozzle Reactor vessel wall Reactor vessel downcomer Best Indication
Tcoid

PZR

ed('

hot

Distracter Analysis: A. Incorrect: - the steam generator tube sheet is not the limiting component for a steam leak event Plausible: - if the candidate corresponds the steam leak to the steam generator and thinks that this is limiting due to the large pressure difference across the steam generator from the rupture (PTS) Incorrect: - the pressurizer spray nozzle is not limiting for this event as there is no spray flow Plausible: - if the candidate confuses this event with other events where the pressurizer delta Temp is limiting - or does not recognize that NC pumps have been stopped Correct answer Incorrect: - the reactor vessel downcomer is not generally limiting Plausible: - if the candidate does not recognize that Thot is not the most limiting temperature. The Rx vessel wall and the Rx vessel downcomer are essentially synonymous for the same region. Level: RO Only KA: WE 08EK1.1(3.5 / 3.8) Lesson Plan Objective: TA-PTS SEQ 2, 13 Source: NRC; McGuire Exam 99, Ques 371 Level of knowledge: memory

B.

C. D.

Ques_371

For Official Use Only

Page 26

For Official Use Only Catawba Nuclear Station RO Exam Question #16 References: 1. OP-CN-TA-PTS pages 7, 12-13, 20

Ques_371

For Official Use Only

Page 27

For Official Use Only Catawba Nuclear Station RO Exam Question #17

Bank Question: 380 1 Pt(s)

Answer: C

Which one of the following statements is a correct description of the capabilities of EMF-48 (Reactor Coolant) at 100% power? A. Detects beta flux from the NC system coolant. This prevents the detector falsely responding to N16 gamma radiation, which would mask a failed fuel event. Detects the N16 gamma flux from the NC system coolant, which is proportional to the amount of failed fuel cladding. Detects total gamma flux from NC system coolant after a one minute sample delay time to allow N16 gamma radiation to decay away.

B.

C.

D.

Detects total gamma flux from NC system coolant. The gamma source term from a clad failure would be much greater than the N16 gamma flux at power.

Distracter Analysis: A. Incorrect: - EMF-48 does not detect beta radiation Plausible: - the detector could function if designed this way because N16 gamma would mask the failed fuel problems. B. Incorrect: - N16 gamma is proportional to reactor power level and does not correlate to failed cladding. Plausible: - if the candidate was confused over the correlation between N16 gamma levels and power levels. C. Correct answer D. Incorrect: - The gamma flux from N16 is >> failed fuel at power Plausible: - if the candidate did not know that N16 gamma was >> than failed fuel source term levels. Level: RO&SRO KA: APE 076AK2.01(2.6/3.0) Lesson Plan Objective: none Source: NRC McGuire Exam 1999 Ques_380 Level of knowledge: memory

Ques_380

For Official Use Only

Page 28

References: 1. OP-CN-WE-EMF page 11

For Official Use Only Catawba Nuclear Station RO Exam Question #18

1 Pt(s) Bank Question: 387a Answer: B

Enclosure 1 to E-1 (Loss of Reactor or Secondary Coolant) provides foldout page actions to close NV-202B and NV-203A (NV PUMPS A&B RECIRC ISOL) when NC pressure is less than 1500 psig. Which one of the following statements correctly describes the operator response and reason for this response when pressurizer pressure is 1495 psig? A. Notify the SRO of the need to close the valves to prevent NV pump runout at low pressures. Close the valves to prevent a reduction of full SI flow to the core. Close the valves to prevent overheating the NV pumps. Notify the SRO of the need to close the valves to prevent a reduction of full SI flow to the core.

B. C. D.

Distracter Analysis: A. Incorrect: - pump runout is not a concern and the actions are automatic. Plausible: if the candidate does not know that foldout actions are independent - pump runout can be a concern for situations where the pump discharge pressure is very low. Correct answer - maximizes flow into the core. Incorrect: - pump runout is not a concern. Plausible: pump runout can be a concern for situations where the pump discharge pressure is very low, approximately 600 psig for NV pumps. Incorrect: - the actions are automatic. Plausible: - if the candidate does not know that foldout actions are independent. Level: RO&SRO KA: G2.4.15 (3.0/3.5) Lesson Plan Objective: EP-INTRO LPRO 12 Source: Mod McGuire Exam 1999 Ques_387

B. C.

D.

Ques_387a

For Official Use Only

Page 29

Level of knowledge: memory

For Official Use Only Catawba Nuclear Station RO Exam Question #18 References: 1. OP-CN-EP-INTRO page 8 2. E-I Foldout page 2 3. OMP 1-7 page 20-21 4. Background Document E-1 page 30

Ques_387a

For Official Use Only

Page 30

For Official Use Only

Catawba Nuclear Station RO Exam Question #19

Bank Question: 388

Answer: A

1 Pt(s)

Unit 1 is responding to a LOCA in the auxiliary building. The operators have implemented ECA-1.2, (LOCA Outside of Containment). Step 2 guides the operators to attempt to find and isolate the leak. Step 2C requires the following sequence: 1 ) 2 ) 3 ) Verify following NI pump miniflow valves OPEN 1NI-115A (NI Pump IA Miniflow Isol) 1NI144A (NI Pump IB Miniflow Isol) 1N1-147B (NI Pump 1C Miniflow Isol) Place the "PWR DISCON FOR 1NI-162A" in "ENABLE" Close INI-162A (NI To C- Legs Inj Hdr Isol)

What is the reason for verifying the mini flow valves are open? A. B. C. D. Protect the NI pumps from operating against shutoff head. Isolation of a potential LOCA path to the FWST. Protect the NI pumps from runout conditions upon restart. Provide a diversion path to prevent high pressure water from over-pressurizing the ND system during leak isolation procedures.

Distracter Analysis: A. B. Correct answer Incorrect: - the mini flow valves will not isolate any potential path in the injection mode. Plausible: - in ES-1.3 (and some other EOPs), the NI mini flow valves are closed to prevent pumps from recirculating radioactive water back to the FWST - right reason, wrong procedure Incorrect: - Opening mini flow valves will not protect against runout Plausible: - if the candidate is confused over the difference between runout conditions and shutoff head conditions Incorrect: - this will not create a diversion path - it will align the NI recirc line to the FWST. The ND system is protected by check valves and in this case by the closed 1NI-162A. Plausible: - the ND system has a design pressure of 600 psig and if NC system pressure was applied, it would rupture. Setting up a For Official Use Only Page 31

C.

D.

Ques_388

For Official Use Only Catawba Nuclear Station RO Exam Question #19

diversion path would be a reasonable thing to do. This alignment does not accomplish this goal. Level: RO&SRO KA: WE04EK2.1(3.5/3.9) Lesson Plan Objective: EP-E2 SEQ 14 Source: NRC McGuire Exam 1999 Ques_388 Level of knowledge: comprehension References: 1. OP-CN-EP-E2 pages 13-14 2. ECA-1.2 page 6 3. OP-CN-CN-NI pages 7-8, 18

Ques_388

For Official Use Only

Page 32

For Official Use Only

Catawba Nuclear Station RO Exam Question #20

I Pt(s)
Bank Question: 393 Answer: C

Unit 1 was in mode 3 with the shutdown banks fully withdrawn, preparing to conduct a reactor startup when source range channel N-31 failed. Given the following conditions and events: No reactor trip has occurred prior to this point AP/16 (Malfunction of Nuclear Instrumentation System) has been completed N-31 repairs have been made N-31 is being returned to service N-32 101 CPS Immediately upon taking the "level trip" switch to the "normal" position a reactor trip occurred

Which of the following operator errors would explain this event? A. B. C. The "Operation selector" switch was left in "level adj." position with level potentiometer set at a level of 106 CPS after retesting The "High-flux at shutdown" switch left in the "normal" position The instrument power fuse blew when the N-31 channel was reenergized. D. The source range detector instrument discriminator voltage was set too high.

Distracter Analysis: A. Incorrect: - the operation selector switch is taken out of the circuit when the level trip switch is taken to normal Plausible: - if the candidate thinks that a test signal can be inserted with level trip switch in the normal position Incorrect: - no effect this is a normal switch alignment - only blocks out high flux alarm at 105 CPS - trip is separate from level trip function Plausible: - if the candidate does not understand that the level trip switch in bypass does not effect the high flux at shutdown trip the high flux at shutdown switch will be in the blocked position under these circumstances. Correct answer For Official Use Only Page 33

B.

C. Ques_393

D.

Incorrect: - if discriminator voltage is too high, less neutrons will be passed, signal will be lower.

For Official Use Only Catawba Nuclear Station RO Exam Question #20
Plausible: - if the candidate does not understand how the pulse height discriminator circuit operates. Level: RO&SRO KA: APE 032AA,101(3.1/3.4) Lesson Plan Objective: ENB LPRO 9 Source: NRC McGuire Exam 1999 Ques_393 Level of knowledge: comprehension References: 1. OP-CN-IC-ENB page 5, 9-10, 26

Ques_393

For Official Use Only

Page 34

For Official Use Only

Catawba Nuclear Station RO Exam Question #21

1 Pt(s) Bank Question: 437a Answer: B

The operator is investigating a suspected ground on the negative leg of a 125VDC bus. The Battery to Ground Volt Meter Selector Switch is in the "NEG" position. Which one of the following indications is correct for the existence of a substantial ground on the negative leg of the 125 VDC electrical system? A. Battery ground negative leg light burns dimly Battery to Ground Volt Meter reads bus voltage minus the ground voltage. Battery ground negative leg light burns brightly Battery to Ground Volt Meter reads the ground voltage. Battery ground negative leg light burns dimly. Battery to Ground Volt Meter reads the ground voltage.

B.

C.

D.

Battery ground negative leg light burns brightly Battery to Ground Volt Meter reads bus voltage minus the ground voltage.

Distracter Analysis: A. B. Incorrect: - a brightly burning lamp = grounded condition Plausible: - believes dim light = ground Correct: - the negative leg light is brightly lit. If a ground exists, the Volt Meter will indicate the amount of volts to ground when this switch is taken to the NEG (measures volts to ground on the negative leg) Incorrect Answer: Plausible if candidate reverses the correct answer in his/her mind a grounded condition often leads to a reduction in voltage and lamps glow dimly Incorrect: Plausible: - candidate believes voltmeter reads bus voltage, less the ground. Level: RO&SRO

C.

D.

Ques_437a

For Official Use Only

Page 35

KA: SYS 063A3.01 (2.7/3.1) Lesson Plan Objective: EPL SEQ 15, 16

For Official Use Only Catawba Nuclear Station RO Exam Question #21

Source: Mod - McGuire Exam 1999 Ques_437 Level of knowledge: memory References: 1. Lesson plan pages 16 and 17

Ques_437a

For Official Use Only

Page 36

For Official Use Only

Catawba Nuclear Station RO Exam Question #22

1 Pt(s) Bank Question: 439 Answer: B

What automatic actions will occur if radioactive particulate levels exceed the 1EMF-35(L) (Unit Vent PART HI RAD) trip 2 alarm set points in the unit vent exhaust flow stream? A. B. C. D. Stops containment purge (VP) supply fans Stops unit-related unfiltered exhaust (VA) fans Stops spent fuel pool ventilation exhaust (VF) fans Stops containment annulus ventilation (VE) fans

Distracter Analysis: A. Incorrect: - does not stop VP supply fans Plausible: - trip 2 alarm on EMF-39 (L) (containment gas monitor) causes this action Correct answer - lEMF-35(L) monitors this exhaust stream Incorrect: - does not stop these fans shifts VF to filter mode VF exhaust fans continue to run but filter train is shifted into the exhaust line. Plausible: - EMF-35 monitors exhaust from Spent Fuel Pool area Incorrect: - does not stop VE fans Plausible: - monitors VE exhaust Level: RO Only KA: SYS 073A1.01(3.2/3.5) Lesson Plan Objective: WL-EMF LPRO 2 Source: NRC McGuire Exam 1999 Ques 439 _ Level of knowledge: memory References: 1. OP-CN-WL-EMF pages 9

B. C.

D.

Ques_439

For Official Use Only

Page 37

For Official Use Only

Catawba Nuclear Station RO Exam Question #23

I Pt(s) Bank Question: 444a Answer: B

Which one of the following statements describes the operation of the Containment Annulus Ventilation System (VE) during a large break LOCA into containment? A. VE fans start in the exhaust mode at +3.0 psig in containment VE fans stop running when annulus pressure reaches -1.5 in. H2O VE fans cycle on and off between -1.5 in H2O and +3 psig YE fans start in the exhaust mode at +1.2 psig in containment VE shifts into recirc mode when annulus pressure reaches -1.5 in. H2O VE modulates dampers between recirc and exhaust modes to maintain -1.5 in. H2O in the annulus YE fans start at on an EMF-38, 39, 40 (containment monitors) VE fans stop running when annulus pressure reaches -1.5 in. H2O VE fans cycle on and off between -1.5 in Hg and +3.0 psig D. V E fans start at on an EMF-38, 39, 40 (containment monitors) VE shifts into recirc mode when annulus pressure reaches -1.5 in. H2O VE modulates dampers between recirc and exhaust modes to maintain -1.5 in. H2O in the annulus Distracter Analysis: A. Incorrect: - VE Fans are not designed to cycle on and off to maintain annulus pressure. They would not recirc through the filter trains if they tripped off to maintain pressure and Iodine removal would not be as effective Plausible: - tripping at 3 psig would be consistent with the initiation point. This is a plausible safety trip to ensure annulus pressure is not dropped too low Correct answer Incorrect: - EMF monitors do not start VE fans; VE Fans are not designed to cycle on and off to maintain annulus Plausible: VP and VQ systems are controlled by EMF 38, 39, 40 Incorrect: - EMF monitors do not start VE fans Plausible: EMF 38, 39, 40, controls VP and VQ systems Level: RO&SRO KA: SYS 027A4.03(3.3/3.2)

B.

C.

B. C.

D.

Ques_444a

For Official Use Only

Page 38

Lesson Plan Objective: VE LPRO 9

For Official Use Only Catawba Nuclear Station RO Exam Question #23

Source: NRC McGuire Exam 1999 Ques 444 Level of knowledge: memory References: 1. OP-CN-CNT-VE pages 8-9

Ques_444a

For Official Use Only

Page 39

For Official Use Only Catawba Nuclear Station RO Exam Question #24

Bank Question: 453a

Answer: C

1 Pt(s)

Unit 1 was operating at 70% power when a loss of condenser vacuum occurred. Given the following events and conditions: Reactor power, 68% Turbine load, 66% based on turbine impulse pressure Turbine exhaust hood temperature is 225 F The operators are rapidly decreasing turbine load The operator reports that condenser vacuum is 23.2 in Hg and is continuing to decrease.

Which one of the following statements correctly describes the required action(s)? A. B. C. D. Immediately manually trip the reactor. Immediately manually trip the reactor and then manually trip the turbine in anticipation of reaching the trip setpoint. Continue to monitor condenser vacuum, if vacuum decreases to 21.8 inches Hg, manually trip turbine. Continue to monitor condenser vacuum, if vacuum decreases to 21.8 inches Hg, first manually trip the reactor then manually trip the turbine.

Distracter Analysis: Turbine Trip Criteria: IF condenser vacuum decreases to less than 21.8 in. Hg, THEN: a. IF reactor power is greater than or equal to 69%, THEN manually trip reactor. b. Ensure turbine - TRIPPED. c. IF reactor is tripped, THEN GO TO EP/1/A/5000/E-0 (Reactor Trip Or Safety Injection). d. REFER TO AP/1/A/5500/02 (Turbine Generator Trip). IF turbine exhaust hood temperature is greater than 2250 F AND turbine load is less than 60% --based on impulse pressure equal to 370 PSIG), THEN: a. Ensure turbine - TRIPPED. A. B. Incorrect: Tripping the reactor is not a required action below 69% Plausible: If unfamiliar with the 69% reactor trip criteria. Incorrect: Do not trip the reactor before tripping the turbine

Ques_453a

For Official Use Only

Page 40

For Official Use Only Catawba Nuclear Station RO Exam Question #24
Plausible: tripping the turbine is not required until the set point is reached but is allowed by OMP 1-8 - if the candidate does not recognize that power is below 65% Correct: Incorrect: should not immediately trip the reactor. Plausible: If the candidate does not recognize that power is below 65% Level: RO&SRO KA: APE 051 AA2.02 (3.914.1) Lesson Plan Objective: ZM SEQ 17 Source: mod Catawba Exam 1999 Ques_453 Level of knowledge: memory References: 1. AP/1/5500/23 Loss of Condenser Vacuum 2. OMP 1-8 page 3

C. D.

Ques_453a

For Official Use Only

Page 41

For Official Use Only

Catawba Nuclear Station RO Exam Question #25

Bank Question: 480a 1 Pt(s)

Answer: D

Unit 1 is operating at 100% power preparing for a reactor startup. Given the following events and conditions: Isotopic analysis of SIG samples at 1200 hours on 3/19 indicated the following primary to secondary leak rates: S/G A = 0.097 gpm S/G B = 0.11 gpm S/G C = 0.08 gpm S/G D = 0.103 gpm 1EMF-71-74 (S/G A-D Leakage HI RAD) has not changed over the last 72 hours.

Which one of the following statements correctly describes the actions required (if any) based on these values? REFERENCES PROVIDED - Tech Spec 3.4.13 A. B. C. No action required. Isolate S/G B by 1600 on 3/19 Be in mode 3 by 1800 on 3/19

D.

Be in mode 5 by 0400 on 3/21

Distracter Analysis: Leakage from the B S/G = 0.11 gpm x 60 min/hr x 24 hr/day = 158.4 gpd Combined leakage from all 4 S/Gs S/G A = 0.097 gpm x 60 min/hr x 24 hr/day = 139.7 gpd S/G B = 0.11 gpm x 60 min/hr x 24 hr/day = 158.4 gpd S/G C = 0.08 gpm x 60 min/hr x 24 hr/day = 115.2 gpd S/G D = 0.103 gpm x 60 hr/min x 24 hr/day = 148.3_ gpd 561.6 gpd total leakage Each S/G must be < 150 gpd and (Tech Spec 3.4.13) Combined leakage through all 4 S/Gs < 576 gpd (Tech Spec 3.4.13) A. B. Incorrect: must shutdown within 10 hours Plausible: if candidate miscalculates S/G leakage Incorrect: isolation of S/G B in 4 hrs will not stop the leak can't isolate a S/G U-tube leak. Plausible: action statement A allows 4 hours to isolate the leakage For Official Use Only Page 42

Ques_480a

C.

if the candidate does not understand that the leak cannot be isolated. Incorrect: must be in mode 3 by 2200

For Official Use Only Catawba Nuclear Station RO Exam Question #25
Plausible: action statement B (6 hours) to mode 3 if disregard the 4 hrs available to stop the leak before reducing power. D. Correct answer: 4 hours to isolate then 36 hours to be in mode 5 40 hours from 1200 3/19 = 0400 on 3/21 Level: RO Only KA: SYS 002A2.01 (4.3 / 4.6) Lesson Plan Objective: PS-NC SEQ 10 Source: Mod McGuire Exam 1999 Ques_480 Level of knowledge: comprehension References: 1. Tech Spec 3.4.13 - PROVIDED

Ques_480a

For Official Use Only

Page 43

For Official Use Only

Catawba Nuclear Station RO Exam Question #26

Bank Question: 503a 1 Pt(s)

Answer: A

Unit 2 was operating at 80% power. Given the following events and conditions: Turbine impulse pressure instrument Channel I failed low Operators perform all actions in AP-15 (Rod Control Malfunctions) Case II (Continuous Rod Movement). Tave is maintained by adjusting turbine load

Which one of the following statements correctly describes the consequences of the Reactor Operator returning the CRD Bank Select switch to the Automatic position 10 minutes later? A. B. Rods will move in because Tref is less than Tave. Rods will move in because impulse pressure is less than nuclear power. Rods will not move in because low impulse pressure blocks rod

C.

movement. D. Rods will not move in because the impulse pressure input to power mismatch is not changing.

Distracter Analysis: A. B. Correct: Incorrect: The derivative of the Imp Press change has timed out. Plausible: based on misunderstanding the derivative function of the power mismatch circuit. Incorrect: C-5 only blocks rod withdrawal. Plausible: based on misunderstanding the C-5 rod stop function. Incorrect: The temperature mismatch function has an error signal at this time. Plausible: If the candidate does not recognize the cumulative nature of both error signals. Level: RO&SRO KA: APE 001G2.1.32(3.4/3.8)

C. D.

Ques_503a

For Official Use Only

Page 44

Lesson Plan Objective: IC-IRX LPRO 5 Source: Mod Catawba exam 1999

For Official Use Only Catawba Nuckar Station RO Exam Question #26

Level of knowledge: analysis References: 1. OP-CN-IC-IRX pages 9-10, 13-15 2. OP-CN-IC-IPX page 20

Ques_503a

For Official Use Only

Page 45

For Official Use Only Catawba Nuclear Station RO Exam Question #27

1 Pt(s) Bank Question: 518 Answer: C

Which one of the following design features prevents the containment isolation valves inside containment (KC, NC, NI and NM systems) from spuriously opening during a containment flooding event following a containment isolation phase A signal? A. The valves are all located above the equipment qualification (EQ) flooding level and should never be submerged by design. These valves were designed to use air operators, which are not subject to spurious actuation when submerged. These valves have motor operators that have their OPEN controls disabled and must be manually reset from the control room.

B.

C.

D.

These valves have all power removed from their motor operators and must be manually enabled from the control room.

Distracter Analysis: A. B. Incorrect: Valves located below flooding plane Plausible: This is one way of preventing the problem Incorrect: Valves have MOVs not AOVs Plausible: Some plants have mainly AOVs in containment for this reason Correct: Incorrect: Valves do not have closing power removed or this would prevent actuation in response to a valid ESF signal Plausible: This would prevent the valves from spuriously opening. Some ECCS valves are protected this way.: Level: RO&SRO KA: WE15EK2.1(2.8/2.9) Lesson Plan Objective: CNT LPRO 13 Source: NRC Catawba Exam 1999 Ques_518 Level of knowledge: memory

C. D.

Ques_518

For Official Use Only

Page 46

References: 1. OP-CN-CNT-CNT page 16

For Official Use Only Catawba Nuclear Station RO Exam Question #27 2. OP-CN-EP-FRZ pages 5, 7

Ques_518

For Official Use Only

Page 47

For Official Use Only

Catawba Nuclear Station RO Exam Question #28

Bank Question: 547 1 Pt(s)

Answer: C

Unit 1 was operating at 100% power when main condenser vacuum dropped from 25 in Hg to 23 in Hg. Which one of the following statements correctly describes the cause of this problem? A. B. C. Condenser water boxes are full RC system flow has increased CM flow to CSAE inter-cooler has been obstructed

D.

Condensate depression has increased.

Distracter Analysis: A. Incorrect: Will not reduce condenser vacuum - full is a normal condition for the water boxes. Plausible: If the ZP water boxes are NOT full, air can accumulate in the upper tubes and interfere with the heat transfer across the tubes Incorrect: Increased RC flow will improve vacuum Plausible: Decreased RC flow will degrade vacuum Correct answer - reduces effectiveness of steam jets Incorrect: - if condensate depression increases, the condensate temperature becomes lower than saturation temperature for the condenser pressure - vacuum would increase Plausible: Condenser thermodynamic efficiency decreases Level: RO&SRO KA: SYS 055K3.01 (2.5 / 2.7) Lesson Plan Objective: MT-ZM SEQ 9, 14 Source: NRC Catawba Exam 99 Ques_547 Level of knowledge: comprehension References: 1. OP-CN-MT-ZM pages 5-8 Ques_547 For Official Use Only Page 48

B. C. D.

For Official Use Only Catawba Nuclear Station RO Exam Question #29

Bank Question: 556


1 Pt(s)

Answer: D

Unit 2 is operating at 100% power when containment parameters vary as shown below: Containment Temperature (F) Upper Lower Humidity (% rel) Upper Lower Containment pressure (psig) Aux. Bldg. pressure (in Hg) 1200 85 105 25 15 0.11 29.1 1500 86 107 26 15 0.13 29.2 1800 87 106 25 18 0.15 29.3 2100 86 105 26 14 0.18 29.4 2400 85 106 25 15 0.19 29.5

Which one of the following statements correctly describes the cause of the trends in the containment atmospheric parameters? A. B. Normal external heating from the sun. Auxiliary Building pressure has increased.

C. D.

A packing leak on the letdown backpressure control valve. An air leak on the AOV for PORV 2NC-36.

Distracter Analysis: A. Incorrect: pressure not following temperature Plausible: temperature increases them decreases Incorrect: An increase in Auxiliary Building pressure would cause a drop in containment pressure as the containment pressure instrument is referenced to external Aux. Bldg. pressure Plausible: pressure is increasing - if the candidate reverses the effect of barometric pressure on containment pressure Incorrect: temperature and humidity are not increasing Plausible: pressure is increasing but not following temperature - the difference between upper and lower containment is normal. Correct answer: cont. pressure increasing without other cont. parameters increasing

B.

C.

D.

Ques_556

For Official Use Only

Page 49

Level: RO&SRO KA: G2.1.7 (3.7/4.4)

For Official Use Only Catawba Nuclear Station RO Exam Question #29

Lesson Plan Objective: VQ LPRO 2 Source: NRC Catawba 1999 Ques_556 Level of knowledge: analysis References: 1. OP-CN-CNT-VQ page 6

Ques_556

For Official Use Only

Page 50

For Official Use Only

Catawba Nuclear Station RO Exam Question #30

Bank Question: 580 1 Pt(s)

Answer: D

Which one of the following statements correctly describes the biological hazard of Tritium in the liquid RadWaste system? A. Tritium emits a low-energy gamma that can cause whole body dose as well as an internal hazard with a 12-year biological half-life if ingested. Tritium emits a low-energy alpha that is an internal hazard with a 12-day biological half-life if ingested. Tritium emits a low-energy beta that can cause both a skin dose as well as an internal hazard with a 12-day biological half-life if

B.

C.

ingested. D. Tritium emits a low-energy beta that is an internal hazard with a 12-day biological half-life if ingested.

Distracter Analysis: A. Incorrect: emits low energy beta only, not gamma - radiological half life is 12 years Plausible: Tritium is an internal hazard with a 12-day biological half-life Incorrect: emits low energy beta only - radiological half life is 12 years Plausible: Tritium is an internal hazard with a 12-day biological halflife Incorrect: It is not hazardous to the skin as the beta energy is too low Plausible: Tritium is an internal hazard with a 12-day biological halflife Correct answer Level: RO Only KA: SYS 068K5.04 (3.2/3.5) Lesson Plan Objective: WL LPRO 13 Source: NRC Catawba Exam 1999 Ques_580

B.

C.

D.

Ques_580

For Official Use Only

Page 51

Level of knowledge: memory

For Official Use Only Catawba Nuclear Station RO Exam Question #30 References: 1. OP-CN-CH-PC pages 11-13

Ques_580

For Official Use Only

Page 52

For Official Use Only

Catawba Nuclear Station RO Exam Question #31

Bank Question: 596a


1 Pt(s)

Answer: B

Units 1 and 2 were operating at 100% power when a fire broke out in the back of the control room. Given the following conditions: The fire has not yet affected or degraded any control systems Heavy black smoke is throughout the control room The SRO implements AP/17 (Loss of Control Room)

Which one of the following statements correctly describes the operator response to this event? A. Swap control to the auxiliary shutdown panels, then trip both unit reactors and turbines, and evacuate the control room. Dispatch RO's to the auxiliary shutdown panels, trip both unit reactors, turbines and feed pumps, then evacuate the control room. Evacuate the control room; trip both unit turbines and reactors

B.

C.

on the way to the auxiliary shutdown panel. D. Immediately trip both unit reactors and turbines and evacuate the control room to the standby shutdown facility.

Distracter Analysis: A. B. C. Incorrect: control is not swapped until after the CR is evacuated. Plausible: this is a more controlled approach to CR evacuation? Correct: Incorrect: Reactor and turbines tripped from the CR. Plausible: This is a reasonable approach assuming operators can no longer function in the CR. Incorrect: Evacuate to the ASP not the SSF Plausible: if the candidate confuses the SSF with ASP. Level: RO&SRO KA: APE 068AK3.12(4.1 / 4.5)

D.

Ques_596a

For Official Use Only

Page 53

Lesson Plan Objective: CP-RSS SEQ 9 Source: Mod McGuire Exam 2000 Ques_596

For Official Use Only Catawba Nuclear Station RO Exam Question #31

Level of knowledge: memory References: 1. OP-CN-CP-RSS page 13 2. AP/ I /A/5500/17 pages 2-3

Ques_596a

For Official Use Only

Page 54

For Official Use Only

Catawba Nuclear Station RO Exam Question #32

Bank Question: 647a 1 Pt(s)

Answer: D

Unit 1 was operating at 100% power. The followingtrends were noted:


0200 100 3.0 >400 10 8.0 50% 55% 87 75 0205 130 3.5 >400 2.0 1.5 49% 56% 88 75 0210 145 4.0 >400 1.0 <1.0 50% 55% 87 75

IA-1D NCP#1 seal outlet temp (F) 1A-1D NCP #1 seal leakoff flow (gpm) 1A-ID NCP #1 seal dip (psid) IA-1D NCP seal water inj filter dip (psid) I A-1D NCP seal injection (gpm) VCT Level Pressurizer level Charging Flow (gpm) Letdown Flow (gpm)

Which one of the following conditions would cause these parameter trends? A. B. C. D. #1 seal injection filter clogged 1NV-294, (NV PMPS A&B DISCH FLOW CTRL), failed closed The VCT depressurized 1NV-309, (SEAL WATER INJECTION FLOW), failed open

Distracter Analysis: Seal injection filter d/p and seal injection flow are decreasing which indicates loss of seal injection. Seal leakoff is increasing as indicated by leakoff flow and leakoff temp increasing. Pressurizer level stays about the same as seal injection flow is diverted into the NC system total charging flow remains stable as corrected by pressurizer level control. A. Incorrect: seal injection filter d/p would increase Plausible: most other parameters would trend as indicated except seal filter d/p would increase. Incorrect: closing 1NV-294 would stop charging and result in opposite trends in PZR & VCT Plausible: if the candidate focuses only on NCP seal parameters. Incorrect: VCT depressurization would result in increased seal injection flow. Plausible: if the candidate focuses only on seal leakoff flow and temperature. Correct Answer: seal injection flow and d/p decreasing

B.

C.

D.

Level: RO&SRO Ques_647a For Official Use Only Page 55

For Official Use Only Catawba Nuclear Station RO Exam Question #32

KA: APE 022AA1.09(3.2/3.3) Lesson Plan Objective: NCP LPRO 3; NV LPRO 6, 7 Source: Mod McGuire Exam 2000 Ques_647 Level of knowledge: analysis References: 1. OP-CN-PS-NV pages 26-27, 32- 34, 59, 61 2. OP-CN-PS-NCP pages 13, 22

Ques_647a

For Official Use Only

Page 56

For Official Use Only

Catawba Nuclear Station RO Exam Question #33

Bank Question: 661a 1 Pt(s)

Answer: B

Units 1 and 2 are at 100% power. Given the following conditions: Unit 2 has experienced 2 fuel pin failures. The mechanical seal has failed on NI pump 2B. The NI-2B pump room general area is 400 mrem/hr. In order to reach the NI-2B pump room the worker must transit through a 6 Rem/hr high radiation area for 2 minutes and return. The worker has an accumulated annual dose of 400 mrem, respectively.

What is the maximum allowable time that the worker can participate in the seal repair on NI Pump 2B without exceeding the exclusion flag exposure limit for external exposure? A. B. C. No longer than 2 hours No longer than 2.5 hours No longer than 3 hours

D.

No longer than 3.5 hours

Distracter Analysis: The candidate should determine that the exclusion flag exposure limit is 90% of 2000 mrem admin limit 1800 mrem Transient exposure is 400 mrem (6000mrem/hr x 4/60hr). (During transit to and from the job). 400 mrem + 400 mrem = 800 mrem 1800 mrem 800 mrem = 1000 mrem allowable before reaching exclusion flag exposure admin limit 1000 mrem / 400 mrem/hr = 2.5 hours A. B. C. D. Incorrect: The answer is 2.5 hours. Plausible: based on using alert flag limit (1600) versus exclude flag. Correct: Incorrect: The answer is 2.5 hours. Plausible: based on calculating a one-way transit dose. Incorrect: The answer is 2.5 hours. Plausible: based on using admin limit (2000) and a one-way transit dose.

Ques_661a

For Official Use Only

Page 57

For Official Use Only

Catawba Nuclear Station RO Exam Question #33

Level: RO&SRO KA: G2.3.4 (2.5/3.1) Lesson Plan Objective: HP LPRO 2, 11 Source: Mod McGuire Exam 2000 Ques_661 Level of knowledge: comprehension References: 1. NSD 507.6 page 10 2. GET Rad Worker Training pages 33, 34, 45

Ques_661a

For Official Use Only

Page 58

For Official Use Only Catawba Nuckar Station RO Exam Question #34

1 Pt(s)

Bank Question: 671a

Answer: D

Unit 1 was releasing the contents of a waste gas decay tank in accordance with an approved release permit. 1EMF-50(L) (Waste Gas Disch) failed high during the release, and will not be repaired for 3 days. Which one of the following actions must be taken to release the waste gas decay tank today? A. The release cannot be restarted until the repairs on IEMF-50(L) (Waste Gas Disch) are completed. Recalculate the trip set points using IEMF-50(H) (Waste Gas Disch) as the release path monitor, then restart the release after a new GWR form has been approved. Continue the release using lEMF-36(L) (Unit Vent Gas) as the backup release path monitor.

B.

C.

D.

Recalculate the trip set points using IEMF-36(L) (Unit Vent Gas) as the release path monitor, then restart the release after a new GWR form has been approved.

Distracter Analysis: A. Incorrect: not required to use lEMF-50(L) as the only qualified release path monitor Plausible: if the candidate does not recognize that IEMF-36(L) can be used to monitor the release path Incorrect: lEMF-50(H) does not automatically trip WG-160 and cannot be used as a waste gas release path monitor Plausible: if the candidate thinks that substituting the high range of 1 EMF-50(L) provides the same automatic protection Incorrect: the release would be terminated when IEMF-50(L) tripped Plausible: if the candidate did not recognize that lEMF-50(L) provided an automatic trip of WG-160 to terminate the release. Correct answer Level: RO&SRO KA: G2.3.11 (2.7/3.2)

B.

C.

D.

Ques_671a

For Official Use Only

Page 59

Lesson Plan Objective: WE-WG LPRO 4

For Official Use Only Catawba Nuclear Station RO Exam Question #34 Source: NRC McGuire Exam 2000 Ques_671 Level of knowledge: memory References: 1. OP-CN-WE-EMF pages 9, 11 2. OP-CN-WE-WG page 9

Ques_671a

For Official Use Only

Page 60

For Official Use Only

Catawba Nuclear Station RO Exam Question #35

Bank Question: 714

Answer: C

1 Pt(s)

Unit 1 was in mode 2 preparing for a plant startup when an electrical transient occurred. Given the following events and conditions: Steam dumps are in steam pressure control Reactor power is 2% Tave decreases Rods have sequenced normally during the startup. Bank D group step counter indicates 200 steps DRPI indication for Bank D rods reads 198 steps Rod bottom light H-8 is illuminated DRPI indication for rod H-8 reads 0 steps DRPI indication for rod D-2 reads 192 steps

Which one of the following actions best describes the correct action that should be taken by the crew and the reason for this action? A. B. C. D. Trip the plant because a dropped rod below mode one is not an analyzed condition. Trip the plant because greater than 2 rods are misaligned. Ensure CRD BANK SELECT switch is selected to MANUAL to prevent rod motion. Ensure that CRD BANK SELECT switch is in AUTOMATIC to allow Tave to recover to Tref. Distracter Analysis: A. Incorrect: Not required to trip the reactor unless > 2 rods are dropped or misaligned one dropped rod is an analyzed condition Plausible: If the candidate thinks that the AP14 case II immediate actions for a single dropped rod only applies to mode 1 conditions Incorrect: Rod H-8 is not Plausible: Plant trip is the preferred method of shutting the reactor and ensuring the reactor is subcritical. Correct: Immediate action per AP/14 step 2 Incorrect: Switch must be in manual Plausible: AP/14 step 3 requires the operator to match Tave to Tref Level: RO&SRO KA: 001G4.22(3.014.0) Ques_714 For Official Use Only Page 61

B. C. D.

For Official Use Only Catawba Nuclear Station RO Exam Question #35

Lesson Plan Objective: IRE SEQ 20 Source: New Level of knowledge: memory References: 1. AP/14 Case II page 5

Ques_714

For Official Use Only

Page 62

For Official Use Only

Catawba Nuclear Station RO Exam Question #36

Bank Question: 715 I Pt(s)

Answer: D

Unit 1 was operating at 30% power. Given the following events and conditions: "A" NC pump trips No operator action has been taken All safety systems operate as designed

While the plant is still at power, which one of the following parameters will initially INCREASE? A. B. C. D. "A" steam generator level. Loop cold leg temperatures in the B, C and D loops. Steam generator pressures in the B, C and D loops. Steam generator steam flows in the B, C and D loops.

Distracter Analysis: The reactor will not trip below P-8. A. Incorrect: A SIG level will decrease Plausible: when the pump trips, steam flow in the steam generator will decrease, level will "shrink" the candidate may reverse this cause and effect. Incorrect: Unaffected Tcolds will decrease Plausible: Steam flow in the unaffected loops will increase, heat removal from the RCS will increase, and cold leg temperature will decrease the candidate may reverse this cause and effect or become confused with the reverse flow in the A loop Incorrect: Unaffected SIG pressures will decrease Plausible: Unaffected steam generators will increase steaming, pressure will decrease the candidate may reverse this cause and effect. Correct: Steam flow in the affected steam generator will decrease, load does not change, the remaining steam generators will increase their steam rates, steam flow in those steam generators will increase. Level: RO&SRO KA: SYS003K5.04(3.2/3.5) Lesson Plan Objective: none

B.

C.

D.

Ques_715

For Official Use Only

Page 63

For Official Use Only Catawba Nuclear Station RO Exam Question #36 Source: New Level of knowledge: analysis References: 1. OP-CN- THF-FF page 15 2. OP-CN-CF-IFE page 6

Ques_715

For Official Use Only

Page 64

For Official Use Only

Catawba Nuclear Station RO Exam Question #37

1 Pt(s) Bank Question: 716 Answer: A

Unit 1 is responding to a large break LOCA inside containment. Given the following condition: The "C-Leg Recirc FWST To CONT SUMP SWAP ENABLE TRN A" light is lit on MC-11,

This light will light when swapover is not defeated and which one of the following events occurs? A. Safety Injection. B. FWST level reaches the swapover setpoint. C. Safety Injection occurs and FWST level reaches the swapover setpoint. D. 1 N1-185A (ND PUMP DI CONT SUMP SUCT) opens. Distracter Analysis: A. B. Correct: The light is lit IF Safety Injection has occurred and the DEFEAT pushbutton has not been depressed Incorrect: FWST level does not input into the logic for the light Plausible: FWST level does input into the logic for opening the sump isolation valves. Incorrect: Does not indicate that FWST level has reached t he swapover setpoint Plausible: Both of these conditions must exist to open the sump isolation valves. Incorrect: The valves open when SI has occurred, (even if reset) and FWST reaches 37%, they are not part of the light. Plausible: If the candidate confuses the enable feature with the actuation. Level: RO&SRO KA: 013A3.02(4.1/4.2) Lesson Plan Objective: ISE SEQ 4 Source: New

C.

D.

Ques_716

Level of knowledge: comprehension For Official Use Only

Page 65

References: 1. OP-CN-ECCS-ISE page 25

For Official Use Only

Catawba Nuclear Station RO Exam Question #38

Bank Question: 717 1 Pt(s)

Answer: C

Which of the following describes the expected response of AFD over core life? Assume ARO conditions are maintained throughout core life. A. Becomes more negative throughout core life. B. Remains relatively unchanged throughout core life. C. Initially, (within the first month), moves negative, becomes less negative over the remainder of core life. D. Initially, (within the first month), moves less negative, becomes more

negative over the remainder of core life. Distracter Analysis: A. Incorrect: becomes less negative after first month Plausible: Operator fails to understand the effect over core life of fuel bumup Incorrect: changes substantially over core life Plausible: operator feels that the competing effects of boron, MTC and bumup offset each other to lead to no net effect. Correct: Boron depletion and MTC becoming negative will push flux to the bottom of the core early in core life. After that, because some fuel is already depleted in the bottom of the core, flux will tend to shift to the top and AFD will become less negative. Incorrect: opposite to the actual conditions Plausible: Operator confuses the effect of MTC on AFD. Level: RO Only KA: SYS 015A1.06(2.5/2.9) Lesson Plan Objective: PD Obj 9 Source: Mod PD-18-D Level of knowledge: comprehension References: 1. OP-CN-CTH-PD pages 14-15

B.

C.

D.

Ques_717

For Official Use Only

Page 66

For Official Use Only

Catawba Nuclear Station RO Exam Question #39

Bank Question: 718

Answer: B

1 Pt(s) Unit 2 is conducting a plant shutdown from 100% power. Given the following events and conditions: Reactor power is 6% All manual actions have been taken as required in the procedures Intermediate Range channel N-36 fails HIGH.

Which of the following statements correctly describes how this failure affects the reactor shutdown and subsequent operation of the Nuclear Instrumentation System? A. The reactor will trip; the source range detectors will reenergize when N-35 decreases to the proper setpoint. B. The reactor will trip; the source range detectors will have to be manually reenergized. C. The reactor will not trip; the source ranges will reenergize when N-35 decreases to the proper setpoint. D. The reactor will not trip; the source ranges will have to be manually reenergized. Distracter Analysis: A. Incorrect: The source range instruments will not automatically reenergize Plausible: If the operator believes the remaining IR energizes the Source Ranges, but knows the reactor trips. Correct: The IR trip will occur when either IR channel increases to > 25% equivalent. However this trip is blocked manually when P-10 is satisfied. Since reactor power is given as 6%, when N-36 fails high, the IR high flux trip will occur. The Source Range instruments will automatically reenergize when: 1. P-10 is not satisfied, 3/4 NIS PR < 10% and 2. P-6, both IR < 10' amps Otherwise they will need to be MANUALLY reenergized. With a high failure of IR N-36, they will not automatically reenergize. Incorrect: The reactor will trip and the source range instruments will not automatically reenergize

B.

C.

Ques_718

For Official Use Only

Page 67

Plausible: Operator believes the IR trip is blocked and only one IR is necessary to energize the SR

For Official Use Only Catawba Nuclear Station RO Exam Question #39
D. Incorrect: The reactor will trip and the source range instruments will not automatically reenergize Plausible: Operator believes the IR trip is blocked but knows both IR are necessary to energize the SR. Level: RO&SRO KA: SYS 015KG2.2(4.0/3.5) Lesson Plan Objective: ENB SEQ 9 Source: New Level of knowledge: analysis References: 1. OP-CN-IC-ENB page 10, 13

Ques_718

For Official Use Only

Page 68

For Official Use Only

Catawba Nuclear Station RO Exam Question #40

Bank Question: 720

Answer: C

1 Pt(s)

Unit 2 was operating at 100% power. Given the following events and conditions: The transmitter for channel I of Containment pressure has failed. The appropriate bistables have been tripped. Subsequently, the vital 120VAC power supply for instrument bus channel IV, (ERPD) fails All instruments powered from the bus are de-energized.

Which one of the following correctly describes the status of the Containment Spray System if containment pressure subsequently increases to 5 psig? A. Bistable for channel IV has tripped; containment spray has actuated when power was lost. B. Bistable for channel IV has tripped; containment spray will actuate when containment pressure exceeds 0.4 psig. C. Bistable for channel IV has not tripped; containment spray will actuate if containment pressure exceeds 3.0 psig. D. Bistable for channel IV has not tripped; containment spray auto actuation is prevented from occurring. Distracter Analysis: Containment Spray actuation is normally 2 of 4 channel coincidence. One channel was tripped when it failed earlier making the coincidence 1/3. The loss of power to the second channel does not trip a second bistable because they are energize-to-actuate. Of the two remaining channels, one tripping will result in auto actuation at the setpoint of 1 psig. A. Incorrect: The bistable for channel IV has not tripped since the bistables are energize-to-actuate Plausible: If the candidate thinks that the bistables are tripped when they are deenergized like most other protection bistables and forgets about the CCPS enable signal. Incorrect: Bistables are energize-to-actuate. Plausible: If the candidate reverses the energize-to-actuate logic, the For Official Use Only Page 69

B.
Ques_720

C.

CCPS enables containment spray actuation at 0.4 psig. Correct: One bistable is tripped; only one more needs to trip above the 0.4 psig CCPS interlock trips at 3.0 psig in containment.

For Official Use Only Catawba Nuclear Station RO Exam Question #40 D. Incorrect: The loss of channel IV does not preclude containment spray actuation. Plausible: One bistable is tripped only one additional channel needs to trip.

Level: RO&SRO KA: SYS 022A3.01(4.1/4.3) Lesson Plan Objective: ISE Obj 4 Source: New Level of knowledge: analysis References: 1. OP-CN-ECCS-ISE pages 10 and 20

Ques_720

For Official Use Only

Page 70

For Official Use Only

Catawba Nuclear Station RO Exam Question #41

1 Pt(s) Bank Question: 721 Answer: C

Unit 1 was shutdown in mode 5 following refueling operations preparing to commence a plant heat up to mode 4. Given the following events and conditions: Alarm lAD-13 B17 (ICE COND ACCESS DOOR OPEN) annunciates Containment divider barrier integrity has been established Which one of the following conditions could cause this alarm? REFERENCE PROVIDED: Tech Spec 3.6.14 and bases page 1 A. B. Malfunction of the containment air return system. Malfunction of the containment pressure control system.

C. D.

The door's inflatable rubber boot develops a leak. Increasing containment pressure.

Distracter Analysis: containment divider barrier integrity would require that the NF access doors be closed and the seal inflated. A. B. C. D. Incorrect: would not effect the access doors. Plausible: this is a cause for ice condenser lower inlet doors to open. Incorrect: would not effect the access doors Plausible: this is a cause for ice condenser lower inlet doors to open. Correct: For the door to indicate closed, it must be latched and the inflatable rubber boot must be inflated. Incorrect: containment pressure would not cause the access door to open Plausible: increasing containment pressure causes other ice condenser doors to open if a DIP develops between upper and lower containment. Level: RO Only KA: SYS 025 K6.01 (3.4/3.6) Lesson Plan Objective: none

Ques_721

For Official Use Only

Page 71

Source: New Level of knowledge: comprehension

For Official Use Only Catawba Nuclear Station RO Exam Question #41

References: 1. OP/1/B/6100/010N B/7 2. NF lesson plan page 10 3. Tech Spec 3.6.14 page 1-3 4. Tech Spec 3.6.14 bases page 1

Ques_721

For Official Use Only

Page 72

For Official Use Only

Catawba Nuclear Station RO Exam Question #42

Bank Question: 722

Answer: B

1 Pt(s)

Unit 2 is operating at 100% power. Given the following events and conditions: A turbine control valve closes Reactor power decreases to 65%.

What action should the crew take to prevent possibly deadheading the condensate booster pumps? A. Start the standby hotwell pump. B. Stop the "C" heater drain tank pumps.

C. Open the "C" heater bypass valve. D. Ensure 2CM-83 is modulating to 150 psig. Distracter Analysis: An engineering evaluation of CNS Unit 1 and 2 reactor trips showed that the Heater Drain Tank pumps develop sufficient discharge pressure to deadhead the condensate system. This phenomenon lasts until the drain flow to the tanks decreases and level control closes places the pumps in recirculation. This was observed to last up to 2.5 minutes. (PIP 98-1726). A. Incorrect: This action while possibly appropriate does not address the deadheading issue. Plausible: This will change the condensate flow and the candidate may think that this will prevent deadheading. Correct: To avoid the problem of deadheading, heater drain tank pumps are shutdown by procedure anytime the unit operates below 70% power. Incorrect: Opening the heater bypass valve would not prevent deadheading. Plausible: This action would be taken if there was a problem with the heater, it does not address deadheading the condensate system. Incorrect: This has no effect on deadheading the condensate For Official Use Only Page 73

B. C.

D. Ques_722

system Plausible: This was the correct answer to a previous NRC question on load rejection. Level: RO&SRO

For Official Use Only Catawba Nuclear Station RO Exam Question #42
KA: SYS 056A2.04(2.6/2.8) Lesson Plan Objective: CM Obj 4 Source: New Level of knowledge: comprehension References: 1. OP-CN-lesson plan CM page 14

Ques_722

For Official Use Only

Page 74

For Official Use Only

Catawba Nuclear Station RO Exam Question #43

Bank Question: 723 1 Pt(s)

Answer: C

Unit 2 was operating at 45% power. Given the following events and conditions: Problems with feedwater control result in overfeeding all steam generators SIG NR levels increased to 80% No operator action has been taken.

Which one of the following statements correctly describes the automatic actions (if any) that should have already occurred? A. No automatic actions should have occurred at this time. B. Turbine trip, feedwater isolation. C. Turbine trip, feedwater isolation, CF pumps tripped. D. Reactor trip, turbine trip, feedwater isolation, CF pumps tripped.

Distracter Analysis: A. Incorrect: turbine tripped, feedwater isolated and CF pumps tripped Plausible: possible if operator believes P14 setpoint is 84% (unit 1 setpoint is 84% - unit 2 setpoint is 77%) Incorrect: CF pumps are tripped Plausible: if operator realizes CF isolation occurs but does not realize the CF pumps also trip. Correct: P-14 causes CF isolation, main turbine trip and CF pump trip. Incorrect: reactor would not trip power below 48% P8 Plausible: operator believes turbine trip will cause a reactor trip. Level: RO&SRO KA: SYS 059A4.12(3.413.5) Lesson Plan Objective: CF Obj 10 Source: New Level of knowledge: analysis References:

B.

C. D.

Ques_723

For Official Use Only

Page 75

For Official Use Only Catawba Nuclear Station RO Exam Question #43 1. OP-CN-MC-CF page 24 2. OP-CN-IC-IPX page 19

Ques_723

For Official Use Only

Page 76

For Official Use Only Catawba Nuclear Station RO Exam Question #44

Bank Question: 724

Answer: D

1 Pt(s)

The crew is performing a cooldown in accordance with EP/1/A/5000/ES-0.2 (Natural Circulation Cooldown). Which of the following actions will cause both the cooldown rate and the NC system flow rate to increase? A. B. Starting more CRDM fans. Increasing the setpoint on the steam dumps, if in automatic.

C. D.

Decreasing the output of the steam dump controller, if in manual. Increasing auxiliary feedwater flow to the steam generators.

Distracter Analysis: The NC system flow rate increases as the temperature difference between the S/G and reactor vessel increases causing the thermal driving head to increase. A. Incorrect: while starting more fans enhances head cooling, it will reduce the temperature difference between the SIG and reactor vessel and thereby reduce the thermal driving head and decreasing the NC system flow rate. Plausible: Starting more CRDM fans will increase the cooldown rate. Incorrect: increasing the setpoint will raise the pressure setpoint, if in auto; the dumps will close, decreasing the cooldown rate and NC system flow rate. Plausible: if the candidate reverses the effects that steam pressure has on the thermal driving head. Incorrect: decreasing the output closes steam dumps and reduces the cooldown rate on the SIG, which causes the delta temp to reduce, and decreases NC system flow. Plausible: if the candidate reverses the effect of the reduction in S/G cooldown rate. Correct: increased CA flow will have the effect of increased steam flow, a decrease in NC temperature and will increase the cooldown rate. For Official Use Only Page 77

B.

C.

D.

Ques_724

Level: RO&SRO KA: SYS 061K5.01(3.6/3.9)

For Official Use Only Catawba Nuclear Station RO Exam Question #44

Lesson Plan Objective: EP1 Obj 21 Source: New Level of knowledge: comprehension References: 1. OP-CN-THF-HT pages 10 and 11

Ques_724

For Official Use Only

Page 78

For Official Use Only

Catawba Nuclear Station RO Exam Question #45

Bank Question: 725 1 Pt(s)

Answer: B

Unit 1 was operating at 100% power. Given the following events and conditions: The feeder breaker to 600VAC MCC-1EMXA opens and the MCC is deenergized.

Which of the following NV loads have been lost? A. Reactor makeup pump lA and boric acid pump IA. B. Boric acid pump lA only.

C. Reactor makeup pump 1A only. D. Centrifugal charging pump A auxiliary lube oil pump. Distracter Analysis: A. B. C. D. Incorrect: power supply is MXW to the reactor makeup pump Plausible: 600 V load Correct: power supply is EMXA Incorrect: power supply is MXW to the reactor makeup pump Plausible: 600 V load Incorrect: power supply is MXK Plausible: 600 V load Level: RO&SRO KA: 004 K2.02 (2.9/3.1) Lesson Plan Objective: NV SEQ 17 Source: New Level of knowledge: memory References: 1. OP-CN-0157-01.03 (Not provided - large drawing size) 2. NV lesson plan pages 28, 37 & 38

Ques 725

For Official Use Only

Page 79

For Official Use Only Catawba Nuclear Station RO Exam Question #46

Bank Question: 726 1 Pt(s)

Answer: A

Unit 2 is operating at 100%. Given the following events and conditions: A planned release is in progress from the waste gas system. Plant Vent monitor, EMF-35 (L) (Unit Vent Part) reaches the trip 2 setpoint.

Which one of the following automatic actions should occur? A. If open, IWG-160, (WG Decay Tank Outlet to Unit Vent Control), will close. B. Containment Ventilation Isolation signal will be generated. C. If open, 1WL-124, (Waste Monit Tnk Pmps Disch), will close. D. Fuel Pool Ventilation (VF) Filter Train will be tripped.

Distracter Analysis: EMF-35, 36, and 37 Automatic functions: Trips unit related unfiltered exhaust fans. Aligns VF filter train to the filtered mode. Secures WG release (shuts 1WG-160). Secures cont air release (shuts VQ-10). A. Correct: B. Incorrect: not generated by EMF-35(L) Plausible: this is generated by emf-38, 39 or 40 C. Incorrect: not generated by EMF-35(L) Plausible: closed by emf-49 D. Incorrect: not generated by EMF-35(L) Plausible: VF not tripped, it is aligned to filtered mode Level: RO&SRO KA: SYS 071K4.04(2.9/3.4) Lesson Plan Objective: EMF SEQ 2 Source: New

Ques_726

For Official Use Only

Page 80

For Official Use Only Catawba Nuclear Station RO Exam Question #46
Level of knowledge: memory References: 1. OP-CN-MC-EMF page 9 and 11 2. OP/1/B/6100/010Y All 3. OP/1/B/6100/010X C/5

Ques_726

For Official Use Only

Page 81

For Official Use Only

Catawba Nuclear Station RO Exam Question #47

Bank Question: 728 1 Pt(s)

Answer: B

Unit 2 is in mode 4. The crew is preparing to start all the NCPs. Which one of the following would satisfy the oil lift system interlock and allow the starting of the NCPs? A. Start both oil lift pumps, when oil lift pressure is greater than 500 psig, start the NCP. B. Start one oil lift pump, when oil lift pressure is greater than 500 psig, start the NCP. C. Start both oil lift pumps, when oil lift pressure is greater than 200 psig, start the NCP. D. Start one oil lift pump, when oil lift pressure is greater than 200 psig, start the NCP.

Distracter Analysis: A. B. Incorrect: Plausible: Only one lift pump can be started at a time. Correct: Plausible: One lift pump started, when pressure is greater than 500 psig, the NCP can be started. Incorrect: Plausible: Only one lift pump can be started and the pressure is 500 psig. Incorrect: Plausible: Pressure setpoint is 500 psig.

C.

D.

Level: RO&SRO KA: SYS 003A4.03(2.8/2.5) Lesson Plan Objective: NCP SEQ 10 Source: New Level of knowledge: memory References: 1. OP-CN-PS-NCP page 11

Ques_728

For Official Use Only

Page 82

For Official Use Only Catawba Nuclear Station RO Exam Question #47

Ques_728

For Official Use Only

Page 83

For Official Use Only

Catawba Nuclear Station RO Exam Question #48

Bank Question: 729 1 Pt(s)

Answer: D

Unit 1 was operating at 100% when a control rod M-4 became misaligned. Given the following events and conditions: The crew has entered AP/14, Case I, (Control Rod Misalignment). Data B Failure lights are lit on the DRPI panel. The DRPI position for Rod M-4 in Control Bank D is 204 steps The group step counter for Control Bank D is at 218 steps There is a General Warning alarm for Rod M-4 There are no Urgent Failure lights lit No operator action has been taken at this time

What is the position of the misaligned rod? A. Between 216 and 192 steps. B. Between 214 and 194 steps. C. Between 214 and 200 steps D. Between 208 and 194 steps

Distracter Analysis: Due to the calculation method in half accuracy mode (one Data Cabinet not supplying accurate data) the system accuracy is reduced. With a Data A Failure, the system accuracy is +10, -4 steps. With a Date B Failure, the system accuracy is -10, +4 steps. Note in AP states: If either Data A or Data B Failure lights are flashing and the Urgent Failure lights are dark, then the individual rod position indication will be in the "half accuracy" mode providing 12 step increment position indication instead of 6. Individual rod position indication may differ by as much as 10 steps from group step counter indication. Incorrect: Plausible: Accuracy assumed is +1- 12 Incorrect: Plausible: Accuracy assumed is +1-10 Incorrect: Plausible: Accuracy assumed is +101-4 Correct: Correct accuracy of 101+4

A. B. C. D.

Ques_729

For Official Use Only

Page 84

For Official Use Only Catawba Nuclear Station RO Exam Question #48

Level: RO Only KA: SYS 001A2.17(3.3/3.8) Lesson Plan Objective: EDA SEQ 3 Source: New Level of knowledge: comprehension References: 1. OP-CN-MC-EDA page 10 2. AP/14 case I page 2-3

Ques_729

For Official Use Only

Page 85

For Official Use Only

Catawba Nuclear Station RO Exam Question #49

Bank Question: 730


1 Pt(s)

Answer: B

Unit 1 is operating at 100% power, with the makeup system in automatic control. Given the following events and conditions: A small (8 gpm) leak develops downstream of 1NV-294 (NV PMPS A &B FLO CTRL)

Which one of the following statements correctly describes the plant indications several hours after the leak starts? A. Increased VCT makeup and pressurizer level decreasing. B. Increased VCT makeup and pressurizer level constant. C. VCT level constant and pressurizer level decreasing. D. VCT level decreasing to the FWST swapover setpoint and

pressurizer level constant. Distracter Analysis: A. Incorrect: pressurizer level will not decrease Plausible operator believes that due to the leak, level will continue to decrease. Correct: Initially pressurizer level will decrease, which will cause charging to increase and return level to program. Pressurizer level will remain on program since the leak is within the capacity of the charging pumps. Because charging is greater than letdown, but within the capability of the makeup system, VCT will decrease to the makeup setpoint and be returned to program (this will repeat for as long as the leak exists). Incorrect: VCT level does not remain constant Plausible: operator fails to realize that a constant mass loss out of the VCT exists. Incorrect: VCT level will not decrease to the swapover setpoint Plausible: operator believes that makeup will not keep pace with the leak. Level: RO Only KA: SYS 004K3.05(3.7/4.1) Lesson Plan Objective: none

B.

C.

D.

Ques_730

For Official Use Only

Page 86

For Official Use Only Catawba Nuclear Station RO Exam Question #49 Source: New Level of knowledge: comprehension References: 1. OP-CN-PS-ILE pages 11, 15-17

Ques_730

For Official Use Only

Page 87

For Official Use Only

Catawba Nuclear Station RO Exam Question #50

Bank Question: 731 1 Pt(s)

Answer: A

Federal Regulations require the emergency core cooling system to be designed to maintain peak cladding temperature below 2200 F. Which one of the following statements correctly describes the basis for this design criterion? A. To prevent acceleration of the zircalloy-water reaction. B. To prevent exceeding the zircalloy clad melting point.

C. To prevent exceeding the fuel melting point. D. To prevent the onset of full film boiling and DNB. Distracter Analysis: A. Correct: The zirconium-water reaction is described by the following chemical equation: Zr + 2H20 --> ZrO2 + 21-12 + HEAT. The rate of this reaction is highly dependent upon clad temperature, such that above approximately 1800F the reaction becomes significant. It becomes accelerated at 2200F and auto-catalytic (self-sustaining) at 4800F. Incorrect: Zircalloy will melt at approximately 3400F. Plausible: its approximately 900 degrees below the melting point. Incorrect: fuel melt is a much higher temperature. 5000 F. Plausible: a logical answer if the candidate doesn't know the answer Incorrect: these are event specific mechanisms rather analysis criteria. Plausible: If the candidate confuses heat transfer mechanisms with ECCS criteria. Level: RO Only KA: SYS 013G2.1.32(3.4/3.8) Lesson Plan Objective: TA-AM SEQ 10 Source: New Level of knowledge: memory

B. C. D.

Ques_731

For Official Use Only

Page 88

References: 1. OP-CN-IC-ISE page 5

For Official Use Only Catawba Nuclear Station RO Exam Question #50 2. OP-CN-TA-AM pages 7-9 3. 10CFR50.46

Ques_731

For Official Use Only

Page 89

For Official Use Only

Catawba Nuclear Station RO Exam Question #51

Bank Question: 732


1 Pt(s)

Answer: B

Unit 1 is performing a plant shutdown. Given the following events and conditions in the following sequence: Turbine load is currently at 300Mwe. IA CF pump is running. 1B CF pump has been shutdown per procedure. A problem with the feed system causes the "B" S/G CF control valve to close. The running CF pump trips. The operator manually trips the reactor.

Assuming systems operate as designed, when did the CA system receive a start signal? A. B. C. When the CF control valve was closed for more than 30 seconds. When the CF pump tripped. When the B S/G NR level decreased below 37%.

D. When the manual reactor trip occurs.

Distracter Analysis: A. B. C. D. Incorrect: trips upon loss of CF pumps Plausible: This signal is in effect only when load is greater than 40% Correct: The AMSAC signal to start CA on loss of both MFPs is always in service Incorrect: trips upon loss of CF pumps Plausible: 37% is the unit 2 setpoint Incorrect: trips upon loss of CF pumps Plausible: a reactor trip causes a CA start signal. Level: RO Only KA: SYS 059A2.01(3.4*/3.6*) Lesson Plan Objective: CF SEQ 13 Source: New

Ques_732

For Official Use Only

Page 90

For Official Use Only Catawba Nuclear Station RO Exam Question #51 Level of knowledge: analysis References: 1. OP-CN-MC-CF page 9-10, 27-28

Ques_732

For Official Use Only

Page 91

For Official Use Only

Catawba Nuclear Station RO Exam Question #52

Bank Question: 733 1 Pt(s)

Answer: A

Unit 2 is responding to a large break LOCA with a failure of safety injection. Given the following events and conditions: The reactor vessel level drops below the top of active fuel and continues to decrease in a uniform manner,

Which one of the following statements correctly describes the expected response of the Source Range instruments as core voiding occurs? A. Initially increase with increased voiding fraction and then decrease when the effects of loss of moderator override the effects of leakage. B. Initially decrease due to the effect of the loss of moderator and then increase due to the increased voiding fraction. C. Continually increase due to the increase in fast neutron leakage.

D. Continually decrease due to the effects of the loss of moderator. Distracter Analysis: A. B. C. Correct: Incorrect: The SR count rate increases then decreases Plausible: If the candidate reverses the effects Incorrect: The SR count rate will decrease after loss of moderator overrides voiding fraction effects Plausible: Initially, the SR response will increase half correct Incorrect: The SR will initially increase Plausible: The SR response will decrease after a certain point half correct Level: RO Only KA: SYS 015K1.04(3.5/3.5) Lesson Plan Objective: ENB SEQ 13 Source: New Level of knowledge: memory References: 1. OP-CN-IC-ENB page 28-29

D.

Ques_733

For Official Use Only

Page 92

For Official Use Only

Catawba Nuclear Station RO Exam Question #53

Bank Question: 735 1 Pt(s)

Answer: C

Which of the following is a characteristic of both NI pumps running as compared to just one NI pump running? A. Pump head loss is approximately halved for each pump. B. Pump running amps are approximately halved for each pump. C. Injection flow is approximately doubled. D. Discharge pressure is approximately doubled.

Distracter Analysis: A. B. C. D. Incorrect: Total system head loss is increased. Plausible: System head loss increases as total flow rate increases Incorrect: Running amps are slightly reduced but not by one half. Plausible: If the candidate does not know pump laws. Correct: The flow is additive for parallel pumps. Incorrect: Discharge pressure will be increased but not doubled Plausible: if the candidate reverses the pump laws for flow and pressure. Level: RO Only KA: SYS 006K5.08(2.9*/3.1*) Lesson Plan Objective: FF Obj 16, 17 Source: New Level of knowledge: memory References: 1. OP-CN-TFIF-FF page 9-12

Ques_735

For Official Use Only

Page 93

For Official Use Only

Catawba Nuclear Station RO Exam Question #54

Bank Question: 736 l Pt(s)

Answer: C

Unit 2 is in the process of conducting a plant startup. Given the following events and conditions: Power range channels indicate the following: PR N41 = 8A) PR N42 = 8% PR N43 = 10% PR N44 = 8%

Which of the following conditions would result in an Automatic Reactor Trip? A. All four RCPs trip. B. Pressurizer level increases to 94%. C. RCS pressure decreases to 1840 psig. D. One turbine impulse pressure channel fails high.

Distracter Analysis: A. Incorrect: all loop flow trips are automatically blocked below P-7 Plausible: if the candidate does not recall that the NCP trip is blocked by P-7 B. Incorrect: Pressurizer High Level, Pressurizer Low Pressure, blocked by P -7 Plausible: C. Correct: As pressure decreases to 1845 psig, SI is actuated. The SI signal generates a Reactor Trip Signal D. Incorrect: P-7 would be enabled, but this does not cause a trip Plausible: if the candidate is confused over the effect of turbine impulse on main generator trip Level: RO&SRO KA: SYS 010K1.02(3.9/4.1) Lesson Plan Objective: ISE SEQ 4 Source: New Level of knowledge: comprehension References: 1. OP-CN-ECCS-ISE page 14

Ques_736

For Official Use Only

Page 94

For Official Use Only

Catawba Nuclear Station RO Exam Question #55

Bank Question: 737 1 Pt(s)

Answer: B

Unit 2 is at 100% power. PZR level and pressure control are selected to channels 1 and 2. Which of the following failures will result in the pressurizer backup heaters immediately de-energizing? A. Controlling pressurizer pressure channel fails low. B. Backup pressurizer level channel fails low. C. Controlling pressurizer level channel fails high. D. Backup pressurizer pressure channel fails high.

Distracter Analysis: A. Incorrect: result is the heaters energize Plausible: a low pressure condition may raise concerns of saturation margin Correct: letdown isolates, heaters de-energize Incorrect: Heaters energize (on deviation) Plausible: high level may elicit concern regarding over-pressurizing a solid pressurizer due to heat up. Incorrect: only input to PORV Plausible: high level may elicit concern regarding over-pressurizing a solid pressurizer due to heat up. Level: RO&SRO KA:SYS 011K3.03(3.2/3.7) Lesson Plan Objective: ILE SEQ 6 Source: New Level of knowledge: comprehension References: 1. OP-CN-PS-ILE pages 15-16

B. C.

D.

Ques_737

For Official Use Only

Page 95

For Official Use Only

Catawba Nuclear Station RO Exam Question #56

Bank Question: 738

Answer: A

1 Pt(s)

Unit 2 is at full power. Reactor Trip breakers A (RTA) and B (RTB) are closed, bypass breakers (RYA, RYB) are open. Which one of the following statements correctly describes how a loss of EPA would effect the operation of Reactor Trip breaker A from the control room? A. RTA would still open from either a manual or automatic signal. B. RTA would not open in response to a manual reactor trip signal; an automatic trip would still open the breaker. C. RTA would not open in response to either an automatic or manual reactor trip signal. D. RTA would immediately trip open because the shunt trip coil would deenergize.

Distracter Analysis: A. B. Correct: EPA powers the shunt trip coil on RTA. This coil energizes to trip RTA. This function would be lost. However, a manual reactor trip signal will also trip the UV coils from SSPS. Incorrect: A manual reactor trip signal will trip the UV coils from SSPS. Plausible: if the candidate does not recognize that SSPS train A provides power to the UV coils which deenergize form the manual trip signal to trip RTA. SSPA train A is powered from an auctioneered circuit from ERPA and ERPB. Incorrect: Both Rx trips would still function the UV coil is unaffected. Plausible: if the candidate does not recognize that a manual Rx trip sends a trip signal to the UV coils which are powered from SSPS train A which receives power from an auctioneering circuit from ERPA and ERPB. Incorrect: The ST coil energizes to trip. Plausible: if the candidate confuses the UV coil and the ST functions ST coil is normally energized Level: RO&SRO KA: SYS 012A2.07(3.2/3.7)

C.

D.

Ques_738

For Official Use Only

Page 96

For Official Use Only Catawba Nuclear Station RO Exam Question #56

Lesson Plan Objective: IPX SEQ 2, 4, 6 Source: New Level of knowledge: comprehension References: I. OP-CN-MC-IPX pages 7, 8 23, 24, 25, 27 2. OP-CN-EL-EPL page 27

Ques_738

For Official Use Only

Page 97

For Official Use Only

Catawba Nuclear Station RO Exam Question #57

Bank Question: 739 1 Pt(s)

Answer: C

During a reactor startup, when should the "RPI at Bottom Rod Drop" (annunciator D/9 on 1AD-2) clear? A. When the rod position startup push button is depressed. B. Once all the shutdown banks are fully withdrawn. C. Once shutdown banks are withdrawn and control bank A is greater than 6 steps off the bottom. D. Once control bank D rods are off the bottom Distracter Analysis: The "RPI at Bottom Rod Drop" annunciator will be activated if any of the following conditions are met: o o Any Shutdown or Control Bank A rod has a calculated position which gives a rod bottom LED.

Any Control Bank B, C, or D has a calculated position which gives a rod bottom LED and the other rods in the same bank and group do not have rod bottom LED's. Any Control Bank B, C and D rod has a calculated position, which gives a rod bottom LED, and rods that should be sequenced out after that rod, are off the bottom. (this is not the case during a startup, so the alarm should remain clear) A. Incorrect: the pushbutton resets all the alarms associated with rod control but not alarms associated with DRPI Plausible: if the candidate does not recognize that DRPI alarms are not reset. Incorrect: Also requires control bank A to be withdrawn Plausible: If the candidate thinks that only the SD banks need to be withdrawn. Correct: Alarm should clear once Bank A is off the bottom. Incorrect: Annunciator will clear when Bank A lifts off the bottom. Plausible: At full accuracy, the rod bottom LED for control bank D rods will extinguish and the six-step position LED will light when the rods are approximately three steps from the bottom.

B.

C. D.

Ques_739

For Official Use Only

Page 98

Level: RO&SRO KA: SYS 014K4.02(2.5*/2.7*)

For Official Use Only Catawba Nuclear Station RO Exam Question #57

-. . . " --.. . .. . . .,

Lesson Plan Objective: EDA SEQ 4, 5 Source: New Level of knowledge: memory References: 1. OP-CN-IC-EDA page 12

Ques_739

For Official Use Only

Page 99

For Official Use Only

Catawba Nuclear St3tion RO Exam Question #58

Bank Question: 740 1 Pt(s)

Answer: D

Unit I is in mode 3 and the operators are controlling the plant from the SSF. What is the coincidence, setpoint and auto action for starting the CAPT? A. 1 of 4 SIG WR level at 45%, SA-2 fails open. B. 2 of 4 SIG WR level at 45%, SA-2 fails open.

C.

1 of 4 SIG WR level at 45%, SA-5 fails open.

D. 2 of 4 SIG WR level at 45%, SA-5 fails open. Distracter Analysis: A. B. C. D. Incorrect: requires 2 of 4 SA-2 does not open Plausible: knows one valve opens, not sure which one. Incorrect: SA-2 does not open Plausible: coincidence correct, auto action incorrect. Incorrect: requires 2 of 4 SIG channels Plausible: incorrect coincidence, correct action. Correct: Level: RO&SRO KA: SYS 016K4.01(2.8*/2.9*) Lesson Plan Objective: CA obj 4 Source: New Level of knowledge: memory References: 1. OP-CN-CA page 9

Ques_740

For Official Use Only

Page 100

For Official Use Only

Catawba Nuclear Station RO Exam Question #59

Bank Question: 741 1 Pt(s)

Answer: D

A LOCA has occurred on Unit 2. The crew has determined that train A of the ND system must be used to supply containment spray. Which one of the following statements correctly describes the electrical interlock that must be satisfied to allow an operator to open 2NS-43A (ND PMP IA TO CONT SPRAY HDR)? A. Both 2 ND-1B, and 2 ND-2A (ND PUMP lA SUCT FRM LOOP B) must be closed. B. Either 2 ND-36B or 2ND-37A (ND PUMP lB SUCT FROM LOOP C) must be closed. C. Train B of the ND system must be operating in the Cold Leg Recirculation mode. D. CPCS signal must be present.

Distracter Analysis: Interlocks to open 2NS-43A 1. ND-1B or ND-2A closed 2. NI-185A open 3. CPCS > 0.4 psig A. Incorrect: Only one valve of these 2 valves must be closed. Plausible: if the operator does not recognize that only 1 of these 2 valves is necessary to satisfy the interlock. Incorrect: These are the train B interlocks for 2ND-36B/37A. Plausible: operator reverses the trains Incorrect: Does not electrically interlock containment spray. Either train may be aligned for cold leg recirc. Plausible: This is an administrative requirement prior to opening 2NS-43A. Correct: Level: RO&SRO KA: SYS 026K4.07(3.8*/4.1*) Lesson Plan Objective: NS SEQ 8, 9 ND SEQ 8 Source: New

B. C.

D.

Ques_741

For Official Use Only

Page 101

For Official Use Only Catawba Nuclear Station RO Exam Question #59

Level of knowledge: memory References: 1. ND system page 14 2. FR-Z.1

Ques_741

For Official Use Only

Page 102

For Official Use Only

Catawba Nuclear Station RO Exam Question #60

Bank Question: 743

Answer: B

1 Pt(s)

Unit 1 is in the process of offloading the core to the spent fuel pool. Spent fuel pool temperature is 120F. Which of the following could cause indicated spent fuel pool level to suddenly decrease? A. The containment purge exhaust flow rate is set less than supply flow rate. B. The spent fuel building running filter exhaust fan trips but the supply fan continues to run. C. After shutting down the containment purge system, the operator places the "MODE SELECTOR" switch on 1RB-CP-1 in the "NORM" position. D. The running spent fuel pool cooling pump trips.

Distracter Analysis: With core offload in progress, containment integrity must be established per Tech Specs. While refueling is in progress, the refueling canal is open and any changes in pressure between containment and the spent fuel pool will result in a manometer effect between the refueling cavity and the SFP. A. Incorrect: exhaust flow < supply flow will cause pressure in containment to increase, causing SFP level to increase. Plausible: if the candidate reverses the effects of supply and exhaust flows on level. Correct: Continuing to supply outside air will increase pressure in the SFP building, increasing pressure, forcing level to shift to containment. Incorrect: This switch realignment will have no effect on containment pressure after the containment purge system has been shut down, Plausible: If this switch was realigned as described when the purge system was in operation, it would cause pressure in upper containment to increase. There a precaution dealing with the mode selector switch, however, placing it in NORM at this time will have zero effect on containment pressure, and therefore no effect on levels. Incorrect: this will not suddenly change level in the spent fuel pool Plausible: It takes hours to reach boiling conditions in the SFP and then it would be a very gradual decrease. For Official Use Only Page 103

B. C.

D.

Ques_743

For Official Use Only Catawba Nuclear Station RO Exam Question #60
Level: RO&SRO KA: SYS 033A1.01(2.7/3.3) Lesson Plan Objective: VF SEQ 3, 4 VP SEQ 3 Source: New Level of knowledge: comprehension References: 1. OP/1/6450/015 enclosure 4.1 and 4.2 2. Lesson plan VF page 5 3. Lesson plan VP page 15

Ques_743

For Official Use Only

Page 104

For Official Use Only

Catawba Nuclear Station RO Exam Question #61

Bank Question: 744a 1 Pt(s)

Answer: C

Unit 1 was operating at 100% power when a tube rupture occurred in the A S/G. Current conditions: A SIG pressure is 1000 psig. A S/G NR level is 100% A S/G WR level is approximately 74% and steady NC pressure is 1500 psig Which one of the following statements correctly explains why the narrow and wide range level indications are not the same? A. The narrow range upper level taps are at a higher elevation inside the S/G than the wide range taps; therefore the wide range instrument will always read lower than the narrow range instrument has reached the upper level tap. B. The wide range upper level taps are at a higher elevation inside the SIG than the narrow range taps; therefore the wide range instrument will continue to show level indication after the narrow range instrument has reached the upper level tap. C. The wide range and narrow range upper level taps are at the same elevation inside the S/G but the wide range level instrument is cold calibrated; therefore it will always read less than narrow range level until the steam generator is cooled down. D. The wide range and narrow range upper level taps are at the same elevation inside the S/G but the wide range level instrument has conservative instrument errors to prevent feeding a hot dry S/G; therefore the WR instrument will always read less than narrow range level until the steam generator is cooled down.

Distracter Analysis: A. Incorrect: NR and WR upper level taps are at the same elevation Plausible: The candidate may think that placing the upper level tap higher causes the WR S/G level to read lower than NR Incorrect: NR and WR upper level taps are at the same elevation Plausible: operator believes that the WR level instruments are at a different location.

B.

Ques_744a

For Official Use Only

Page 105

For Official Use Only Catawba Nuclear Station RO Exam Question #61
C. Correct: Since the WR and NR Level Instruments upper taps are located at the same elevation, a NR indication of greater than 100% indicates both the NR and WR upper taps are submerged. ACTUAL Wide Range Level must therefore be 100%, an indicated level of 74% is expected. As ACTUAL Level in the ruptured S/G continues to increase, the Wide Range Level will continue to indicate 74%. Wide Range Level indication will only increase further if the S/G is cooled down. Incorrect: There is no conservative instrument error inserted in the WR SG level instrument to prevent feeding a hot dry SIG Plausible: partially correct the instrument taps are at the same elevation using conservative instrument error offset would be a plausible way of preventing feeding a hot dry SIG Level: RO&SRO KA: SYS 035K6.03(2.613.0) Lesson Plan Objective: SG SEQ 7 Source: New Level of knowledge: memory References: 1. data book 2. SG lesson page 12&13

D.

Ques_744a

For Official Use Only

Page 106

For Official Use Only Catawba Nuclear Station RO Exam Question #62

Bank Question: 745 1 Pt(s)

Answer: B

Unit 1 was operating at 100% power when a sudden loss of VI supply pressure to the MSIVs occurred. Which one of the following statements correctly describes the MSIV response? A. B. MSIVs will close immediately without operator action. MSIVs will close without operator action in approximately 15 minutes. MSIVs will not close until after the operator manually presses the closed pushbutton.

C.

D.

MSIVs will not close because they require VI pressure to actuate.

Distracter Analysis: A. B. C. Incorrect: there is a backup tank that supplies air pressure. Plausible: if the candidate does not recall that the back up tank will Correct: Incorrect: The valves will close without operator action Plausible: if the candidate thinks that operator action is required to initiate closure with spring pressure Incorrect: MSIVs will close after 15 minutes Plausible: if the candidate thinks that they fail open and require VI pressure to close. Level: RO Only KA: SYS 039A4.01(2.9/2.8) Lesson Plan Objective: SM Obj 12 Source: New Level of knowledge: memory References: 1. OP-CN-STM-SM page 12

D.

Ques_745

For Official Use Only

Page 107

For Official Use Only Catawba Nuclear Station RO Exam Question #63

Bank Question: 747 1 Pt(s)

Answer: D

Unit 1 is operating at 10% power conducting a plant shutdown. Given the following events and conditions: No 6900V tiebreakers are closed. The main turbine is not synchronized to the grid The automatic fast transfer switch is in the DEFEAT position RC pumps A and C are running. A fault occurs on transformer 1T2B and the transformer is deenergized.

Which one of the following statements correctly describes the RC pump response?

A.
B. C. D.

No RC pumps would trip.


Only RC pump A would trip. Only RC pump C would trip. RC pumps A and C would trip. Distracter

Analysis: The long sides of 1TA and 1TC both are fed from 1T2B. Loss of 1T2B would result in a loss of RC pumps A and C. The fast or hot transfer would not occur because the fast transfer is defeated a slow or dead bus transfer would occur. A. Incorrect: RC pumps A and C would trip Plausible: - if candidate thinks that a slow bus transfer would prevent loss of RC pumps B. Incorrect: RC pump C will also trip Plausible: RC pump A will trip if the candidate believes 1T2B feeds only 1TA long side and forgets about 1TC long side C. Incorrect: RC pump A will also trip Plausible: if the candidate believes 1T2B feeds 1TC long side and forgets about 1TA long side. D. Correct: Level: RO&SRO KA: SYS 062K2.01(3.3/3.4) Lesson Plan Objective: EP SEQ 11

Ques_747

For Official Use Only

Page 108

For Official Use Only Catawba Nuclear Station RO Exam Question #63 Source: New Level of knowledge: comprehension References: 1. OP-CN-EL-EP pages 23, 27-30 2. OP-CN-MT-RC page 6

Ques_747

For Official Use Only

Page 109

For Official Use Only

Catawba Nuclear Station RO Exam Question #64

Bank Question: 748 1 Pt(s)

Answer: B

An Emergency Diesel Generator is running in parallel with offsite power. The operator is preparing to shutdown the diesel. In accordance with OP/1(2)/A/6350/002, ENC 4.10, when manually unloading the Emergency Diesel Generator, the output breaker should be opened after load is reduced to 200 KW while maintaining a lagging power factor. Which one of the following statements correctly describes the basis for this requirement? A. Diesel overspeed when the breaker is opened. B. Breaker trip on reverse power. C. Loss of power to the bus.

-_-_-_-_-_-_-_-_-_-_-__
Distracter Analysis: A. Incorrect: While possible if breaker is opened under greater load, this is not the reason for minimum load and lagging pf Plausible: the DG will pick up speed if suddenly unloaded overspeed is a valid concern to protect the diesel against. Correct: leading pf and low load could motor the machine leading to reverse power condition. Incorrect: Unlikely since a breaker failure would have to occur. Plausible: if there was no reverse power trip, this condition could cause a loss of power to the bus if the DG acted as a motor and tripped the offsite power breaker on over-current. Incorrect: An unbalance does not exist, Gen Diff should not occur. Plausible: This is a valid DG trip Level: RO&SRO KA: SYS 064A1.08(3.1/3.4) Lesson Plan Objective: DG3 SEQ 17 Source: bank DG 3-027-D Level of knowledge: memory References: 1. OP-A/A/6350/002 End 4.10

D. Breaker trip on Generator Differential.

B. C.

D.

Ques_748

For Official Use Only

Page 110

For Official Use Only

Catawba Nuclear Station RO Exam Question #65

Bank Question: 749 1 Pt(s)

Answer: C

Which of the following is performed when starting the RC system to prevent pump runout? A. B. C. D. Limiting the number of pumps that can be started based on the number of cooling towers available. Limiting the number of pumps that can be started based on the number of condensers available. Not operating a single pump with the discharge valve fully open. Not allowing the first pump to be started unless the main

condenser is isolated. Distracter Analysis: A. B. C. Incorrect: Plausible: confuses start interlocks with runout protection. Incorrect: Plausible: knows that this is a requirement for pump starts. Correct: Plausible: "Norm"-"T/V" key switch and valve selector switch is provided for the discharge valves of the RC pumps to limit discharge valve to 52 degrees open following the start of the pump, for the valve selected, to prevent runout of the first pump started System procedure cautions that the pump cannot operate with its discharge valve fully open to prevent runout. Incorrect: Plausible: confuses caution, (no pump can be started if the condenser is isolated). Level: RO&SRO KA: SYS 0751(4.01(3.2/3.5) Lesson Plan Objective: RC SEQ 14, 18 Source: New Level of knowledge: memory References: 1. OP-CN-RC 6400/001A

D.

Ques_749

For Official Use Only

Page 111

2. RC system lesson plan, page 7

For Official Use Only Catawba Nuclear Station RO Exam Question #66

Bank Question: 751 I Pt(s)

Answer: B

Unit 1 was operating at 100% power when a large break LOCA occurred. Given the following sequence of events: Time 1: Safety Injection occurs Time 2: Containment phase B isolation signal generated Time 3: FWST low level alarm occurs Time 4: Containment Sump Isolation valves opened

At what time did KC to the ND heat exchangers automatically open? A. B. Time 1 Time 2

C. D.

Time 3 Time 4

Distracter Analysis: A. B. C. D. Incorrect: Plausible: operator believes KC auto aligns on S signal Correct: KD will fail open on either phase B or S signal with low FWST level. Incorrect: Plausible: operator fails to realize the KC valves opened on Sp Incorrect: Plausible: operator believes valves are interlocked with aligning ND to the sumps Level: RO&SRO KA: 005 K1.10 (3.2/3.4) Lesson Plan Objective: ND SEQ 8 Source: New Level of knowledge: memory

Ques_751

For Official Use Only

Page 112

References: 1. Lesson plan ND page 18

For Official Use Only Catawba Nuclear Station RO Exam Question #67

Bank Question: 752 1 Pt(s)

Answer: B

Unit 2 is operating at 100% power. Given the following events and conditions: KC cooling is lost to an NCP. No operator action is taken.

Which one of the following conditions will first occur and require the NCP to be tripped? A. High # 1 seal leakoff discharge temperature. B. High motor bearing temperature.

C. High radial bearing temperature. D. High pump shaft vibration. Distracter Analysis: A. Incorrect: seal leakoff temperature is not cooled by KC Plausible: Loss of seal injection would cause the discharge temperature to increase. Correct: A loss of KC cooling to the NC pumps results in a gradual approach to an overheated condition prior to a possible shaft seizure. The duration of the heatup phase, estimated to be approximately ten minutes, provides a sufficient time for operator recognition and response. Additional time exists between exceeding the high bearing temperature limit and the conditions required for shaft seizure. NCP is tripped at 195 degrees (motor bearing) Incorrect: KC does not cool the radial bearing this is cooled by seal injection flow. Plausible: the operator may confuse seal injection with KC cooling. Incorrect: KC flow does not affect the pump vibration Plausible: high shaft vibration will occur if the motor bearing overheats but this will happen long after the high temperature alarm is received. Level: RO&SRO

B.

C.

D.

Ques_752

For Official Use Only

Page 113

KA: SYS 008K3.03(4.1/4.2) Lesson Plan Objective: KC SEQ 7, 14

For Official Use Only Catawba Nuclear Station RO Exam Question #67

Source: New Level of knowledge: comprehension References: 1. AP-21 page ?

Ques_752

For Official Use Only

Page 114

For Official Use Only

Catawba Nuclear Station RO Exam Question #68

Bank Question: 754 1 Pt(s)

Answer: A

Unit 1 is conducting a reactor startup. Given the following conditions and events: Steam dumps are in Auto in the Steam Pressure mode. The Train "A" P-12 solenoids lose power.

Which one of the following statements correctly describes the operation of the steam dumps? A. All the steam dumps will close and cannot be re-opened. B. All the steam dumps will close; the cooldown bank can be used by going to BYPASS. C. No effect, P-12 does not input to steam dump operation in the Steam Pressure mode. D. No effect, however, if Tave decreases below the P-12 setpoint, the dumps will not close. Distracter Analysis: A. Correct: The P-12 solenoids are in series and energized above P-12. If Tave decreases below the setpoint or the solenoids lose power, the valves close. However, without power, the solenoids cannot be re-energized. Incorrect: The solenoids cannot be bypassed. Plausible: Operator may believe BYPASS will allow the valves to be opened. Incorrect: P-12 solenoids will prevent the valves from opening Plausible: Operator may believe P-12 only affects Tave operation. Incorrect: P-12 solenoids will prevent the valves from opening Plausible: Operator may believe P-12 does not affect operation unless necessary below 553. Level: RO&SRO KA: 041 G2.1 (3.7/3.6) Lesson Plan Objective: IDE SEQ 8 Source: New

B.

C. D.

Ques_754

For Official Use Only

Page 115

For Official Use Only Catawba Nuclear Station RO Exam Question #68 Level of knowledge: comprehension References: 1. Lesson plan IDE page 6 and 27

Ques_754

For Official Use Only

Page 116

For Official Use Only

Catawba Nuclear Station RO Exam Question #69

Bank Question: 755

Answer: D

1 Pt(s)

Which of the following conditions will cause 2RN47A (RN SUPPLY XOVER ISOL) to automatically close? A. Phase B on either unit or Emergency low level in either pump house pit. Unit 2 Phase B or Emergency low level in pump house pit A. Emergency start of either Unit 2 emergency Diesel generator or Phase B on either unit. Unit 2 Phase B or Emergency low level in pump house pit B. Distracter Analysis: RN Supply Crossover Isolation Valves (1&2RN47A and 48B) The RN supply crossover isolation valves are normally open to supply cooling water to the non-essential header. Each valve closes upon a "Sp" Phase B Isolation signal from its respective unit. The RN supply crossover isolation valves are also equipped with safety related interlocks to close upon emergency low level in the pump house pits. Pit A Emergency Low Level will close: 1 and 2 RN48B Pit B Emergency Low Level will close: 1 and 2 RN47A A. Incorrect: Phase B on Unit 1 will not cause isolation Plausible: believes either unit or pit level isolates the non-essential header. Incorrect: emergency low level on pump house pit A will not cause isolation Plausible: partially correct if candidate believes 47A isolated due to pit A Incorrect: emergency DG start will not cause isolation Plausible: partially correct - if the candidate thinks that an emergency start of the DG should result in isolating the nonessential header Correct: Level: RO Only

B. C. D.

B.

C.

D.

Ques_755

For Official Use Only

Page 117

KA: 076 K4.06 (2.8/3.2)

For Official Use Only Catawba Nuclear Station RO Exam Question #69
Lesson Plan Objective: RN SEQ 12 Source: New Level of knowledge: memory References: 1. OP-CN-PSS-RN page 17

Ques_755

For Official Use Only

Page 118

For Official Use Only

Catawba Nuclear Station RO Exam Question #70

Bank Question: 756 1 Pt(s)

Answer: C

Unit 1 was operating at 100% power when a LOCA occurred. Given the following events and conditions: 0200 = LOCA occurs inside containment, reactor trip 0201 = loss of offsite power, 0202 Emergency diesels start and energize emergency buses 0203 = Safety Injection occurs 0204 = Containment pressure exceeds 1.2 psig 0210 = Containment pressure exceeds 3.0 psig 0215 = Instrument air decreases to 75 psig

At what point did Instrument Air to containment isolate? A . B . C . D . 0200 - 0202 0203 - 0204 0205 - 0210 0211- 0220 Distracter Analysis: A. B. Incorrect: instrument air isolation is not caused by the LOOP Plausible: operator believes the valves are closed on LOOP Incorrect: VI isolation is not caused by either the safety injection or the phase "A" isolation signal, St Plausible: Breathi ng air isolates on St Correct: VI isolates on a phase B containment isolation signal, Sp Incorrect: VI already isolated Plausible: If operator feels VI isolates on decreasing pressure (vice the cross tie opening) Level: RO Only KA: 078 K3.01 (3.1/3.4) Lesson Plan Objective: VI LPRO 25 Source: New Level of knowledge: comprehension

C. D.

Ques_756

For Official Use Only

Page 119

For Official Use Only Catawba Nuclear Station RO Exam Question #70 References: 1. Lesson plan VI page 9-10,34 2. OP-CN-ECCS-ISE page 18

Ques_756

For Official Use Only

Page 120

For Official Use Only

Catawba Nuclear Station RO Exam Question #71

1 Pt(s) Bank Question: 757 Answer: B

Units 1 and 2 are operating at 100% power with a normal service water lineup and RN pump 2A running. Given the following conditions and indications: Alarm RN PIT SWAP TO SNSWP (1AD-12 E/2) - LIT Alarm RN PIT A SCREEN HI DIP (1AD-12 B12) - LIT All RN pumps start. RN suction and discharge valves swap to the SNSWP.

Which of the following best describes the cause of this condition? A. B. C. D. The Lake Wylie dam failed. RN pump intake pit "A" screens are clogged. An RN pump intake pit level indicator failed low. There was a spurious safety injection actuation.

Distracter Analysis: A. B. C. D. Incorrect: low lake level would not actuate level differential alarm Plausible: event would cause the other conditions. Correct: Incorrect: It takes 2 of 3 instrument failures. Plausible: if the candidate does not know the coincidence logic. Incorrect: S/I would not cause level alarms. Plausible: S/I causes some of the other conditions. Level: RO&SRO KA: 000062AA2.02(2.9 / 3.6) Lesson Plan Objective: PSS-RN SEQ12 Source: New Level of knowledge: comprehension References: Ques_757

For Official Use Only

Page 121

1. OP-CN-PSS-RN pages 14, 32 2. OP/1/Q/6100/010M lAD-12 B/3 3. OP/1/A/6100/010M lAD-12 E/2

For Official Use Only Catawba Nuclear Station RO Exam Question #72

1 Pt(s) Bank Question: 758 Answer: C

Unit 2 was operating at 50% power. During surveillance testing of CA pump 2A, a large fire started in the CA pump room. Which one of the following best characterizes the plant fire system response to this event? A. An alarm will sound in the affected pump cubicle to warn personnel of the electric shock hazard of the pending sprinkler actuation in 1.5 minutes. An alarm will sound in the control room to notify operators to actuate the affected pump CO2 system after the personnel have been safely evacuated from the pump room. An alarm will sound in the affected pump cubicle to warn personnel of the asphyxiation hazard of the pending CO2 actuation in 6.5 minutes. An alarm will sound in the control room to notify operators to actuate the affected pump deluge system after the personnel have been safely evacuated from the pump room.

B.

C.

D.

Distracter Analysis: A. Incorrect: There is no time delay in sprinkler system actuation. Plausible: if the candidate does not understand the protection provided for this fire area. Incorrect: the system actuates automatically. Plausible: Some fire systems are manually actuated. Correct: Incorrect: There is no deluge protection for CA pumps. Plausible: MFPs have deluge protection. Level: RO Only KA: APE 067A10.02(2.5 / 3.3) Lesson Plan Objective: SS-RFY SEQ 36 Source: New Level of knowledge: memory

B. C. D.

Ques_758

For Official Use Only

Page 122

References: 1. OP-CN-SS-RFY pages 13, 21-22

For Official Use Only

Catawba Nuclear Station RO Exam Question #73

1 Pt(s) Bank Question: 763 Answer: C

Unit 2 is responding to a small break LOCA. The operators have entered FR-C.2 (Response to Degraded Core Cooling) due to failure of the NI and NV systems to inject. The note at the start of FR-C.2 states: Normal conditions for running NC pumps are desired, but NC pumps should not be tripped if normal conditions cannot be established or maintained. Which one of the following best describes the operator response and basis if the lower bearing temperature on all four NCP's is 300F and increasing? A. All four NCP's should be tripped to prevent imminent failure of the NCP seals. All four NCP's should remain in operation to assure rapid depressurization to accumulator injection pressure. All four NCP's should remain in operation to facilitate two-phase flow through the core. D. All four NCP's should be tripped to minimize coolant loss through the break.

B.

C.

Distracter Analysis: A. B. Incorrect: The pumps should not be tripped. Plausible: This is the basis for NCP trip criteria (>235F) Incorrect: wrong basis for action. Plausible: This is the logical consequence of this action, and solves the cause of the problem. Correct: Incorrect: The pumps should not be tripped. Plausible: This is the reason for tripping NCP's during a LOCA with NV or NI injecting. Level: RO Only KA: 000074EK3.08(4.1 / 4.2) Lesson Plan Objective: EP-FRC SEQ 4 Ques_763 For Official Use Only Page 123

C. D.

Source: New

For Official Use Only Catawba Nuclear Station RO Exam Question #73 Level of knowledge: memory References: I. OP-CN-EP-FRC page 5 2. ERG Basis Document FR-C.2 page 1 3. EP/1/A/5000/FR-C.2 page 2 4. Ep/I/A/5000/F-0 page 3

Ques_763

For Official Use Only

Page 124

For Official Use Only Catawba Nuclear Station RO Exam Question #74

1 Pt(s)
Bank Question: 764 Answer: D

Unit 2 was operating at 100% power. At 0200, NCP 2C tripped on an electrical fault. Reactor power is 96% and the reactor trip breakers are closed. The operators implement FR-S.1, (Response to Nuclear Power Generation/ATWS). Given the following plant indications: Indication 2NV-312A 2NV-314B 2NV- 252A 2NV- 253B 2NV-236B 2NV-188A 2NV-189B 2N1-9A 2N1-10B NV PMP 2A NV PMP 2B CHG Flow A OPEN OPEN CLSD CLSD CLSD OPEN OPEN CLSD CLSD ON OFF 78 B OPEN CLSD OPEN CLSD CLSD OPEN OPEN CLSD CLSD OFF ON 0 C CLSD OPEN CLSD OPEN CLSD CLSD OPEN OPEN CLSD OFF ON 25 D OPEN CLSD OPEN CLSD CLSD OPEN CLSD CLSD OPEN ON ON 140

Which one of the above columns contains the correct plant indications from which it can be concluded that NV system is effectively combating the ATWS event? A. B. C. D. Column A indications Column B indications Column C indications Column D indications

Distracter Analysis: A. Incorrect: No emergency boration from BAT or FWST 2NV-236B blocks suction from the BAT. Plausible: if the candidate is focused on normal NV line-up or doesn't know the emergency boration sources. Incorrect: No charging flow. For Official Use Only Page 125

B. Ques_764

Plausible: there's a flow path from the FWST and an NV pump running.

For Official Use Only Catawba Nuclear Station RO Exam Question #74 C. D. Incorrect: only 25 gpm (versus 30gpm). Plausible: there's a flow path from FWST through 2NI-9. Correct: - uses B train valves instead of A Level: RO&SRO KA: APE 024G4.1(4.3/4.6) Lesson Plan Objective: EP-FRS SEQ 5 Source: New Level of knowledge: comprehension References: 1. OP-CN-EP-FRS page 6 2. EP/1/A/5000/FR-S.1 pages 2-3

Ques_764

For Official Use Only

Page 126

For Official Use Only

Catawba Nuclear Station RO Exam Question #75

1 Pt(s) Bank Question: 766 Answer: B

Unit 2 was operating at 75% power with rod control in automatic. A Rod Control Urgent Failure alarm is received on 2AD-2. Given the following events and conditions: Control rod bank D No motion PR instruments decreasing Tave decreasing Tref / Tauct deviation increasing All control rod groups At programmed withdrawal All shutdown rod groups Fully withdrawn except one 2AD-2, D/9 "RPI at Bottom Rod Drop" LIT

Which of the following statements correctly describes the required Reactor Operator actions? A. Verify only one rod dropped and reactor power stabilizes; perform actions of AP/14 (Control Rod Misalignment) as directed by the SRO. Verify only one rod dropped or misaligned; switch rod control to manual; and perform actions of AP/14 as directed by the SRO. Verify only one rod dropped; ensure CRD Bank Select in Manual; and perform actions of AP/15 (Rod Control Malfunction) as directed by the SRO. D. Verify no rod movement; switch rod control to manual; and perform actions of AP/15 as directed by the SRO.

B.

C.

Distracter Analysis: A. Incorrect: Does not switch rod control to manual. Plausible: Reasonable actions if the candidate forgets the immediate actions. B. Correct: Immediate actions of AP/14, Case II. C. Incorrect: Does not verify another misalignment, and wrong AP. Plausible: If the candidate misses the misalignment verification, and/or chooses the wrong AP based on its title. D. Incorrect: Correct answer for the wrong AP. Plausible: If the candidate chooses AP/15 based on its title.

Ques_766

For Official Use Only

Page 127

Level: RO&SRO

For Official Use Only Catawba Nuclear Station RO Exam Question #75
KA: APE 003G2.4.1(4.3/4.6) Lesson Plan Objective: IRE SEQ 20 Source: New Level of knowledge: comprehension References: 1. OP-CN-IC-IRE pages 10, 13, 21 2. AP/1/A/5500/14 page 6

Ques_766

For Official Use Only

Page 128

For Official Use Only Catawba Nuclear Station RO Exam Question #76

Bank Question: 767

Answer: D

1 Pt(s)

Unit I is responding to a small-break loss of coolant accident inside containment. Given the following events and conditions: Operators have implemented E-0 (REACTOR TRIP), and E-1 (LOSS OF REACTOR OR SECONDARY COOLANT) through step 10. Bus 1ETB is de-energized NI pump 1A has failed. Containment hydrogen concentration is 7% The TSC has recommended purging containment to reduce hydrogen concentration to 3.5% before starting the recombiners.

Which one of the following statements correctly describes the method for performing this evolution to control the off-site dose? A. B. C. Containment air is exhausted to the auxiliary building where it is filtered prior to release to the unit vent stack. Containment air is exhausted to the containment air release system where it is filtered prior to release to the unit vent stack. Containment air is exhausted to the annulus where it is continuously recirculated through the annulus ventilation filters.

D.

Containment air is exhausted to the annulus where it is continuously recirculated and filtered prior to release to the unit vent stack.

Distracter Analysis: A. B. C. D.
Ques_767

Incorrect: Air is released to the annulus area. Plausible: This could be a way to filter the release if the air was vented to the auxiliary building. Incorrect: Air is released to the annulus area. Plausible: If the candidate confuses the VQ and VY systems. Incorrect: The air is released to the vent stack. Plausible: If the candidate does not remember that VE releases to the stack to maintain negative pressure in the annulus. Correct:
For Official Use Only Page 129

Level: RO&SRO KA: EPE 009G2.3.9(2.5/3.9)

For Official Use Only Catawba Nuclear Station RO Exam Question #76 Lesson Plan Objective: VX SEQ 4 Source: New Level of knowledge: comprehension References: 1. OP-CN-CNT-VE pages 5- 7 2. OP-CN-CNT-VX pages 6, 9-10 3. OP-CN-CNT-VQ pages 7, 9 4. OP-CN-PSS-VA page 23

Ques_767

For Official Use Only

Page 130

For Official Use Only

Catawba Nuclear Station RO Exam Question #77

Bank Question: 768 1 Pt(s)

Answer: B

Unit 1 is at 25% power when a loss of main feedwater occurs. Given the following events and conditions: The main turbine trips. CA pumps lA and 1B start automatically. CA flow to the S/Gs was throttled when the S/G A N/R level was greater than 11%. Control valve 1CA-62A to S/G A cannot be closed.

Which one of the following statements correctly describes the consequences of operator failure to respond to this malfunction? A. B. C. The reactor will be overcooled causing a criticality concern. SIG A will be overfilled causing a main steam pipe stress concern. The reactor will be overcooled causing a pressurized thermal shock concern. S/G A will be overfilled causing a feed ring thermal stress concern.

D.

Distracter Analysis: A. Incorrect: The reactor remains critical and maintains programmed Tave. Plausible: could be a concern if the reactor tripped. Correct: Incorrect: The reactor does not see a significant cooldown. Plausible: could be a concern if the reactor tripped. Incorrect: The feed rings are designed to withstand the shock of CA flow. Plausible: A possible concern if the candidate does not understand S/G design. Level: RO&SRO KA: APE 054AK3.03(3.8/4.1) Lesson Plan Objective: SM LPRO 25 Source: New

B. C. D.

Ques_768

For Official Use Only

Page 131

For Official Use Only Catawba Nuclear Station RO Exam Question #77 Level of knowledge: comprehension References: 1. OP-CN-STM-SM pages 16-17 2. AP/1/A/550/06 page 2 3. OP-CN-TA-PTS pages 18, 20, 22 4. OP-CN-CF-CA pages 10,11,15

Ques_768

For Official Use Only

Page 132

For Official Use Only

Catawba Nuclear Station RO Exam Question #78

1 Pt(s) Bank Question: 769 Answer: C

Unit 1 is recovering from a loss of secondary coolant accident. Safety injection initiated properly. A total loss of feedwater has caused the operators to implement FR-H.1, (Loss of Secondary Heat Sink). Given the following plant conditions: NCS Pressure NCS Temperature S/G 1A, 1B, 1C Pressure S/G 1A, 1B, 1C Level (WR) S/G 1D Pressure S/G 1D Level (WR) VI system pressure Containment pressure 2335psig 565F 1180psig 2% 100psig 35% lOpsig 3.4psig

Which one of the following actions is initially required to assure the maintenance of adequate core cooling? A. Depressurize S/G 1A, 1B, and 1C to allow feeding the S/G using the condensate system. Reset the CAPT and align it to feed S/Gis 1A, 1B and 1C. Open 1NC-32B (PZR PORK and 1NC-34A (PZR PORE) using nitrogen pressure.

B. C.

D.

Reset safety injection and containment phase "A" isolation signals to re-establish instrument air pressure to open 1NC-32B and 1NC-34A.

Distracter Analysis: A. Incorrect: Can't feed dry S/G's Plausible: if the candidate does not recognize dry SIG criteria met, this is one FR-Z.1 recovery method. Incorrect: Can't feed dry SIG's Plausible: if the candidate does not recognize dry S/G criteria met, this is one FR-Z.1 recovery method. Correct: Incorrect: Must reset S, to reopen VI valves. For Official Use Only Page 133

B.

C. D. Ques_769

Plausible: if the candidate thinks that VI is a phase "A" isolated system. Level: RO&SRO

For Official Use Only Catawba Nuclear Station RO Exam Question #78

KA: WE05AA1.1(4.1/4.0) Lesson Plan Objective: FRH LPRO 5 Source: New Level of knowledge: analysis References: 1. OP-CN-EP-FRH page 6

Ques_769

For Official Use Only

Page 134

For Official Use Only

Catawba Nuclear Station RO Exam Question #79

Bank Question: 770 1 Pt(s)

Answer: B

Unit 1 is starting-up in mode 1. Given the following conditions and events: Battery charger 1ECB is out of service. Battery charger 1ECS is supplying bus 1EDB. Annunciator IAD-11, H/3 (125VDC ESS PWR CHANNEL C TROUBLE) alarms due to the loss of power from battery charger 1 ECC. The battery charger lECC output breaker is faulted.

Which of the following describes operator action needed in response to these conditions? REFERENCES PROVIDED: - Tech Spec's 3.8.4, 3.8.7, 3.8.9 A. Switch inverter IEIC to the regulated power supply, and be in mode 3 within 6 hours. Cross-tie bus 1EDC to bus 1EDA, and fix one of the chargers within 10 days. Switch inverter IEIC to the regulated power supply, and fix one of

B.

C.

the chargers within 24 hours. D. Cross-tie bus lEDC to bus 1EDA, and fix one of the chargers or be in mode 3 within 6 hours.

Distracter Analysis: A. Incorrect: they would have 14 hours if the buses were not crosstied. Plausible: This is a possible action if the buses could not be cross-tied. Correct: Incorrect: They still have to cross-tie or shutdown within 14 hours.. Plausible: If the candidate focuses on inverter vs DC source TS. Incorrect: the busses can be cross-tied for 10 days. Plausible: If the candidate thinks more than one train is affected. Level: RO&SRO KA: APE 058AA1.01(3.4/3.5)

B. C. D.

Ques_770

For Official Use Only

Page 135

Lesson Plan Objective: EPL LPRO 9, 19 Source: New

For Official Use Only Catawba Nuclear Station RO Exam Question #79

Level of knowledge: memory References: 1. OP-CN-EL-EPL page 19 2.Tech Spec's 3.8.4, 3.8.7, 3.8.9 - PROVIDED

Ques_770

For Official Use Only

Page 136

For Official Use Only

Catawba Nuclear Station RO Exam Question #80

1 Pt(s) Bank Question: 771 Answer: D

Unit 1 is operating at full power and Unit 2 is refueling. Unit 1 is releasing a waste gas decay tank with significant packing leak on isolation valve 1WG-160, (WG Decay Tank Outlet to Unit Vent Control). Which one of the following statements correctly describes the automatic and manual actions required to assure that the release is contained and filtered? A. 1EMF-50 (Waste Gas Disch) automatically stops the auxiliary building ventilation unfiltered exhaust fans. 1EMF-41 (Aux Bldg Vent Hi Rad) automatically closes valve 1WG-160. The operators must manually stop the auxiliary building ventilation unfiltered exhaust fans. 1EMF-37 (Unit Vent Iodine) automatically stops the auxiliary building ventilation unfiltered exhaust fans. The operators must manually close 1WG-160. D. 1EMF-35 (Unit Vent Part Hi Rad) automatically closes valve 1WG160 and stops the auxiliary building ventilation unfiltered exhaust fans.

B.

C.

Distracter Analysis: A. B. Incorrect: EMF-50 does not secure the VA fans. Plausible: A reasonable response to a leak in the Aux Bldg. Incorrect: WG-160 does not automatically close Plausible: if the candidates do not recognize that this is not an automatic action. EMF-41 is an ARM that will alarm under these conditions. Incorrect: EMF-37 auto trips WG-160 closed. Plausible: partially correct EMF 37-stops the VA fans. Incorrect: EMF-35 can secure 1WG-160 and VA fans, but will not see an abnormal release in this case. Level: RO&SRO KA: APE 060AK3.02(3.3/3.3) Lesson Plan Objective: VA LPRO 9

C. D.

Ques_771

For Official Use Only

Page 137

Source: New

For Official Use Only Catawba Nuclear Station RO Exam Question #80
Level of knowledge: analysis References: 1. OP-CN-PSS-VA pages 8, 13, 14, 16, 23 2. OP-CN-WE-EMF pages 9, 10, 17-18

Ques_771

For Official Use Only

Page 138

For Official Use Only Catawba Nuclear Station RO Exam Question #81

1 Pt(s) Bank Question: 773 Answer: D

You are manually preparing a Red Tag Removal and Restoration (R&R). You notice a red stamp on the electrical one-line drawing you are using that reads 'Interim As Built' along with CNEE-05163 written in the space next to the stamp. What actions are required prior to using this drawing for the R & R? A. Note the modification number on the R & R Record Sheet for tracking purposes. Go to the WCC to verify that the modification changes do not affect the R & R. Notify the NSM coordinator that the drawing needs to be updated prior to hanging the tags.

B.

C.

D.

Go to the Control Room to verify that the modification changes do not affect the R & R. - - - - - - - - - - - - - - - - Distracter Analysis: A. Incorrect: the drawing accuracy must be verified Plausible: a reasonable answer if the notation were only administrative Incorrect: the WCC does not maintain as -built information for electrical drawings. Plausible: WCC does maintain flow diagrams. Incorrect: the tags are hung after verification of drawing accuracy Plausible: a reasonable action if the right answer is not known. Correct: Level: RO&SRO KA: G2.1.24(2.8/3.1) Lesson Plan Objective: ADM-NSO2 SEQ 2 Source: New Level of knowledge: memory

B.

C. D.

Ques_773

For Official Use Only

Page 139

References: 1. OMP 2.10 pages 3, 5-6

For Official Use Only Catawba Nuclear Station RO Exam Question #81
2. OP-CN-AD-NSO2 page 8

Ques_773

For Official Use Only

Page 140

For Official Use Only

Catawba Nuclear Station RO Exam Question #82

1 Pt(s) Bank Question: 774 Answer: C

Unit 2 is in a refueling outage with core alternations in progress. IAE is performing a calibration of source range detector N-31. Which of the following best describes the actions required when BDMS channel B fails low? REFERENCES PROVIDED: Tech Spec 3.9.2 A. Suspend core alterations until the RMW pump capacity is verified to be less than COLR limits. Core alterations may continue because one channel of BDMS and channel N-32 remain operable. Suspend core alterations and verify that unborated water sources are isolated.

B.

C.

D.

Core alterations may continue because channel N-32 remains operable and the RMW pump capacity was verified to be less than COLR limits during refueling prerequisites.

Distracter Analysis: A. B. Incorrect: the unborated sources must be verified closed. Plausible: this would be true if both SR instruments were in service. Incorrect: core alterations must be suspended, must have both SR's or both BDMS. Plausible: a reasonable approach considering many plants don't have to have BDMS. Correct: Incorrect: core alterations must be suspended. Plausible: an even more reasonable approach considering many plants don't have to have BDMS Level: RO Only KA: G2.2.22 (3.4 / 4.1) Lesson Plan Objective: FH-FHS SEQ 6 Source: New

C. D.

Ques_774

For Official Use Only

Page 141

Level of knowledge: memory

For Official Use Only Catawba Nuclear Station RO Exam Question #82 References: 1. OP-CN-FH-FHS page 21 2. Tech Spec 3.9.2

Ques_774

For Official Use Only

Page 142

For Official Use Only

Catawba Nuclear Station RO Exam Question #83

1 Pt(s) Bank Question: 775 Answer: B

An NCP motor has been removed from containment to the electrical maintenance shop for overhaul. Health Physics surveyed the motor with the following results: General radiation levels in the shop work area -- <0.1 mR/hr Radiation levels on contact with the motor -0.3 mR/hr Radiation level on contact with the pump shaft coupling -100 mR/hr External loose surface contamination -400 dpm/cm2 Internal loose surface contamination -750 dpm/cm2

Which of the following statements correctly describes the controls, if any, that are required to minimize the exposure to the workers performing the overhaul? A. A standing RWP is required because this is a special operation requiring decontamination. An RWP is required because the operation requires temporary shielding.

B.

C. D.

No RWP is required because the pump is not inside the RCA. A standing RWP is required because this is a special operation requiring temporary shielding.

Distracter Analysis: A. Incorrect: an RWP is required and decontamination is not an option. Plausible: if the candidate focuses on a blanket RWP for shop work or S for special. Correct: Incorrect: An RWP is required. Plausible: If the candidate assumes no controls are needed outside the radiological controlled area. Incorrect: An RWP is required. Plausible: if the candidate focuses on a blanket RWP for shop work or S for special.

B. C.

D.

Ques_775

For Official Use Only

Page 143

Level: RO&SRO KA: G2.3.10 (2.9/3.3)

For Official Use Only Catawba Nuclear Station RO Exam Question #83
Lesson Plan Objective: RAD-HP LPRO 9, 11 Source: New Level of knowledge: comprehension References: 1. OP-CN-RAD-HP pages 18, 25, 28

Ques_775

For Official Use Only

Page 144

For Official Use Only

Catawba Nuclear Station RO Exam Question #84

1 Pt(s) Bank Question: 776 Answer: C

Unit 1 was operating at 100% when a LOCA with loss of offsite power occurs. One EDG fails to start. The operators are entering EP/E-1, (Loss of Reactor or Secondary Coolant). Given the following critical safety function status indications: Core Cooling ORANGE Subcriticality GREEN Containment RED Inventory - GREEN Heat Sink RED Integrity GREEN

Which one of the following describes the highest priority problem, and the appropriate operator action? A. Core cooling; Transition to FR-C.2, (Response to Degraded Core Cooling). Core cooling; Transition to FR-C.1, (Response to Inadequate Core Cooling). Heat Sink; Transition to FR-H.1, (Response to Loss of Secondary Heat Sink). D. Containment; Transition to FR-Z.1, (Response to High Containment Pressure).

B.

C.

Distracter Analysis: A. Incorrect: heat sink is the highest priority RED Plausible: if the candidate picks the first item requiring transition from E-1. Incorrect: heat sink is the highest priority RED Plausible: if the candidate picks the first RED condition and does not recognize that the wrong procedure is listed. Correct: Incorrect: heat sink is the highest priority RED Plausible: if the candidate picks the first RED condition. For Official Use Only Page 145

B.

C. D.

Ques_776

Level: RO&SRO KA: G2.4.17 (3.1/3.8)

For Official Use Only Catawba Nuclear Station RO Exam Question #84

Lesson Plan Objective: EP-INTRO LPRO 1 Source: Bank Ques_776 Level of knowledge: memory References: 1. OP-CN-EP-INTRO page 6, 8 2. OMP 1-7 pages 9-10

Ques_776

For Official Use Only

Page 146

For Official Use Only Catawba Nuclear Station RO Exam Question #85

1 Pt(s)

Bank Question: 779

Answer: C

The crew is realigning a Control Bank "C", Group 1 rod in accordance with OP/1/A/6150/008, (Rod Control), Enclosure 4.6, (Rod Retrieval (>12 Steps Misaligned)). The RO has been directed to operate the lift coil disconnect switches in accordance with the enclosure. The RO reports that the lift coil disconnect switches for the unaffected rods in Control Bank "C" Group 1 are in the DISCONNECTED position and that the misaligned rod switch is in the CONNECTED position. Which of the following additional actions, if any, must be performed to correctly realign the Bank C control rod? A. Return the unaffected group I rods to CONNECTED and place the misaligned rod disconnect switch to DISCONNECTED. Place all the Control Bank A, B, D and Control Bank C group 2 rods in DISCONNECTED.

B.

C. D.

Place all Control Bank "C" Group 2 rods in DISCONNECTED. No additional actions necessary, continue with the misaligned rod alignment.

Distracter Analysis: The next step in the procedure reads: 2.10 Disconnect all lift coils in the affected bank, except for the affected rod, by placing the control rod disconnect switches in the "DISCONNECTED" position. A. Incorrect: This action would further exacerbate the misaligned rod Plausible: operator feels the action taken is backwards from what's required Incorrect: not correct to disconnect rods that are not part of bank C Plausible: operator feels all the control bank rods, except the misaligned one should be in disconnect. Correct: if group 2 is not disconnected then they will move when the misaligned rod is realigned. Plausible: all the control rods in the bank, group 1 and 2, except the

B.

C.

Ques_779

For Official Use Only

Page 147

D.

misaligned one should be in disconnect Incorrect: this will cause group 2 of bank C rods to move. Plausible: operator believes the actions taken are correct.

For Official Use Only Catawba Nuclear Station RO Exam Question #85

Level: RO&SRO KA: APE 005 EA1.2 (3.7/3.5) Lesson Plan Objective: IRE SEQ 10, 17 Source: New Level of knowledge: comprehension References: 1. OP-6150/008 2. Lesson IRE page 12

Ques_779

For Official Use Only

Page 148

For Official Use Only Catawba Nuclear Station RO Exam Question #86

Bank Question: 780 1 Pt(s)

Answer: A

Unit 1 was operating at 100% power when a reactor trip occurred. The crew is verifying natural circulation in ES-0.1 (Reactor Trip Response). Given the following events and conditions: NC WR Thot 580F, stable NC WR Tcold 555F, stable NC Pressure 1920 psig Core Exit Thermocouples 585F, stable Steam Generator pressure 1090 psig, stable Steam Generator narrow range level off scale low Steam Generator wide range level 70% CA flow 400 gpm per steam generator

Which one of the following statements correctly describes the status of natural circulation? REFERENCES PROVIDED: ES-0.1 End 3 page 21 A. B. C. D. All conditions are met; natural circulation is established. Natural circulation is not established; subcooling is insufficient. Natural circulation is not established; heat sink is insufficient. Natural circulation is not established; NC delta T is insufficient.

Distracter Analysis: Objective: Explain the bases of Enclosure 1 (Foldout Page) actions of EP/1/A/5000/ES-0.1 (Reactor Trip Response). A. B. C. Correct: Incorrect: subcooling is greater then 0 F Plausible: operator error determining subcooling I n c o r r ec t: he a t s i nk is s a t is fa c to ry ha ve S G WR l e ve l 7 0% w i t h 4 00 g pm C A fl ow Pl au sib l e: ope ra t o r be l i e ve s N R l e ve l re qui re d fo r he a t s i nk Incorrect: Plausible: operator looks for normal at power delta T Level: RO Only

D.

Ques_780

For Official Use Only

Page 149

For Official Use Only Catawba Nuclear Station RO Exam Question #86 KA: WE 09 EK2.1 (3.2/3.4) Lesson Plan Objective: EP1 SEQ 15 Source: New Level of knowledge: analysis References: 1. OP-CN-EP-EP1 pages 6-7 2. ES-0.1 page 21

Ques_780

For Official Use Only

Page 150

For Official Use Only

Catawba Nuclear Station RO Exam Question #87

1 Pt(s) Bank Question: 782 Answer: B

Which one of the following statements correctly describes the purpose of the time delay associated with the "KC HX A RN Outlet Flow-Lo" Annunciator on 1AD-12 following a safety injection signal? A. To allow sufficient time for the 1A RN pump to develop sufficient flow following a sequencer start. To allow sufficient time for 1RN-291 (KC HX lA OUTLET THROTTLE) to stroke open. To allow sufficient time for 1RN-287A (KC HX lA INLET ISOL) to stroke open. D. To allow sufficient time for the 1KC-C37A (TRAIN A MINIFLOW ISOL) to stroke open.

B.

C.

Distracter Analysis: A. B. Incorrect: the RN pump starts without a time delay Plausible: pumps start on Ss signal. Correct: The "KC HX A(B) RN Outlet Flow-Lo" Annunciator (OBJ #15,) is interlocked with an Ss signal such that the alarm will only be enabled while a Ss signal is present. It also provides a 72 second time delay after the Ss signal to allow the RN valve to stroke full open. Incorrect: These valves do not move. Plausible: If the candidate confuses the inlet isolation valves with the throttle valves Incorrect: These valves do not affect RN flow to the heat exchanger. Plausible: if the candidate confuses the miniflow isolation valves with the heat exchanger isolation valves some miniflow valves have automatic actions. Level: RO&SRO KA: APE 026 AA1.07 (2.9/3.0) Lesson Plan Objective: KC SEQ 15 Source: New Level of knowledge: memory

C.

D.

Ques_782

For Official Use Only

Page 151

References:

For Official Use Only Catawba Nuclear Station RO Exam Question #87
1. KC lesson page 8 of 26 2. OP/1/A/6100/010M 1AD-12 D/2

Ques_782

For Official Use Only

Page 152

For Official Use Only

Catawba Nuclear Station RO Exam Question #88

Bank Question: 783


1 Pt(s)

Answer: C

Unit 2 is operating at 100% power. Given the following events and conditions: Pressurizer pressure control is in automatic One set of backup Heaters is energized in "ON" Actual pressurizer pressure is 2270 psig

The pressurizer Master Pressure Controller malfunctions and the setpoint is step changed from 2235 psig to 2335 psig. Which one of the following statements correctly describes the initial automatic responses in the pressurizer Pressure Control System as a result of this failure? A. PORV NC-34A opens, Spray Valves open, and pressurizer Heaters deenergize B. PORV NC-32 and NC-36 open, Spray Valves open, and pressurizer Heaters de-energize C. Spray Valves close and pressurizer Heaters energize D. All PORVs remain closed, Spray Valves open and pressurizer

Heaters de-energize
. . . m . r . -

Distracter Analysis: A. Incorrect: Plausible: operator believes that the setpoint of 2335 psig will open the PORV Incorrect: Plausible: operator believes setpoint increase causes spray valves to open, and the other two PORV respond to a "high" pressure condition. Correct: Plausible: system responds to "low" pressure. Heaters on, spray valves that were open, will close. Incorrect: Plausible: Level: RO&SRO KA: APE 027 AK2.03 (2.6/2.8)

B.

C.

D.

Ques_783

For Official Use Only

Page 153

For Official Use Only Catawba Nuclear Station RO Exam Question #88

Lesson Plan Objective: IPE SEQ 8, 10 Source: New Level of knowledge: analysis References: 1. IPE pages 8, 14, 15

Ques_783

For Official Use Only

Page 154

For Official Use Only

Catawba Nuclear Station RO Exam Question #89

Bank Question: 786 1 Pt(s)

Answer: B

Which of the following conditions would be consistent with excessive number 2 seal leakage on an operating NCP?

A.

1AD-7, C/2, "NCP #1 SEAL LEAKOFF LO FLOW" and 1AD-7, B/3 "NCP #1 SEAL LO D/P" annunciators lit.
1AD-7, C/2, "NCP #1 SEAL LEAKOFF LO FLOW" annunciator lit. 1AD-7, C/1, "NCP #1 SEAL LEAKOFF HI FLOW" and 1AD-7, B/3, "NCP #1 SEAL LO DIP" annunciators lit. 1 AD-7, B/1 "NCP #1 SEAL OUTLET HI TEMP" annunciator lit.

B. C.

D.

Distracter Analysis:

A.

B.

C.

D.

Incorrect: C/2 cause is #1 seal damage or #2 seal failure, B/3 cause is loss of #1 seal. Plausible: operator believes that the failure of the number 2 seal will cause seal d/p to decrease. Correct: Plausible: Number 2 seal leakoff increasing will rob flow from the number 1 seal, leakoff will decrease. Incorrect: cause for C/1 is damaged or cocked #1 seal. Plausible: as stated previously, operator feels the failure will decrease number 1 dip. Incorrect: B/1 caused by insufficient cooling water. Plausible: operator feels the failure will cause a seal outlet temperature to increase. Level: RO&SRO KA: APE 015 AK2.07 (2.9/2.9) Lesson Plan Objective: NCP SEQ 12 Source: New Level of knowledge: comprehension References: 1. OP/1/B/6100/010H annunciators B/1, B/3, C/1, C,2

Ques_786

For Official Use Only

Page 155

For Official Use Only Catawba Nuclear Station RO Exam Question #89

Ques_786

For Official Use Only

Page 156

For Official Use Only Catawba Nuclear Station RO Exam Question #90

Bank Question: 788

Answer: D

1 Pt(s)

Unit 1 was operating at 100% power when a total loss of offsite and onsite AC electrical power occurred. Given the following events and conditions: The crew is performing the actions of ECA-0.0 (LOSS OF ALL AC POWER). Power has not been restored. The operator reports core exit thermocouples read 1200F and increasing.

Which one of the following statements correctly describes the actions the crew should take? A. B. C. Immediately go to FR-C.1 (RESPONSE TO INADEQUATE CORE COOLING). Remain in ECA-0.0 until after power is restored to at least one emergency bus then transition to FR-C.1 Complete ECA-0.0 and when directed to implement monitoring CSF status trees in the appropriate recovery procedure, verify a valid RED path exists and transition to FR-C.1. Immediately transition to EG/1/A/CSAM/SACRG1 (SEVERE ACCIDENT CONTROL ROOM GUIDELINE INITIAL RESPONSE).

D.

Distracter Analysis: Step 30, IF core exit temperatures greater than 1200 F AND increasing, THEN GO TO EG/1/A/CSAM/SACRG1 (Severe Accident Control Room Guideline Initial Response). A. Incorrect: must immediately transition to SACRG 1 Plausible: operator knows an immediate transition is necessary but believes C.1 is appropriate. B. Incorrect: must immediately transition to SACRG1 Plausible: operator believes one emergency bus must be restored prior to addressing the inadequate core cooling condition. C. Incorrect: must immediately transition to SACRG I Plausible: operator follows the normal rules of usage regarding Red and Orange paths in ECA-0.0 D. Correct: efforts to adequately cool the core have failed, SAMG usage is appropriate Level: RO&SRO

Ques_788

For Official Use Only

Page 157

For Official Use Only

Catawba Nuclear Station RO Exam Question #90

KA: APE 055 EA1.01 (3.7/3.9)

Lesson Plan Objective: CRG-1 SEQ 2 Source: New Level of knowledge: memory References: 1. ECA-0.0 page 23 step 30 2. OP-CN-EP-CRG-1 page 6

Ques_788

For Official Use Only

Page 158

For Official Use Only

Catawba Nuclear Station RO Exam Question #91

Bank Question: 789


1 Pt(s)

Answer: C

Unit 1 was shutdown for refueling in mode 6. IAE was calibrating set points on various area radiation monitors when the following occurred: 1WL-867A, (VUCDT CONT ISOL) closed 1WL-825A, (CONT SMP PMPS DISCH CONT ISOL) closed

Which one of the following EMFs could have correctly caused this action? A. B. C. D. 1EMF-49(L) (Liquid Waste Discharge Monitor) 1EMF-52 (Clean Area Floor Drain Monitor) IEMF-53(A) (Containment Hi Range Monitor) IEMF-54 (Unit Vent Hi Range Monitor)

Distracter Analysis: A. Incorrect: does not automatically isolate 1WL-867A and 1WL825A Plausible: 1EMF-49 provides automatic isolation signal to 1WL-124 to terminate a liquid release Incorrect: does not automatically isolate 1WL-867A and 1WL-825A Plausible: 1EMF-52 automatically diverts flow from turb bldg sump to ND and NS sump. Correct: Incorrect: does not automatically isolate 1WL-867A and 1WL-825A Plausible: Automatically trips sample pump supplying EMF-35, 36 and 37 Level: RO&SRO KA: SYS 072A4.01(3.0*/3.3) Lesson Plan Objective: WE-EMF LPRO 2 Source: New Level of knowledge: memory

B.

C. D.

Ques_789

For Official Use Only

Page 159

References:

For Official Use Only Catawba Nuclear Station RO Exam Question #91 1. OP-CN-WE-EMF page 11, 12 2. OAC alarm response

Ques_789

For Official Use Only

Page 160

For Official Use Only Catawba Nuclear Station RO Exam Question #92

Bank Question: 790

Answer: C

1 Pt(s)

Unit 1 is operating at 100% power and is preparing to discharge a waste monitor tank (WMT). Given the following events and conditions: IAE has been unable to adjust the 1EMF-49 (LIQUID WASTE DISCH) to the required trip 2 setpoint specified on the discharge permit. 1EMF-49 operates properly in all other calibration tests 1EMF-57, (MONITOR TANK BUILDING LIQUID DISCHARGE) is operable Secondary coolant activity is 0.005 micro curies/gram dose equivalent Iodine 131.

Which one of the following statements correctly describes the complete set of actions that must be completed before releasing contents of the WMT? REFERENCES PROVIDED: - SLC 16.11-2 A. B. C. The release may proceed for up to 30 days provided that the flow rate is estimated once every four hours. 1EMF-57 can be used to monitor the release path. The release may proceed for up to 30 days provided that the flow rate is estimated once every four hours. A dedicated operator must be stationed at IEMF-49 to monitor the release rates and ensure that the setpoint is not exceeded. Two independent analyses of the WMT contents must be performed prior to the start of the release. The discharge permit calculations and the release path valve lineup must be independently verified. D. The release may proceed for up to 30 days provided that grab samples are taken once per 24 hours. 1EMF-57 can be used to monitor the release path.

Distracter Analysis: A. Incorrect: need to IV the calculations and the release path. Plausible: Partially correct This is part of action statement D the candidate may mistakenly refer to 3.d (monitor tank building waste For Official Use Only Page 161

Ques_790

liquid effluent line) -which is the discharge line in use. lEMF-57 is used to monitor the release.

For Official Use Only Catawba Nuclear Station RO Exam Question #92
B. Incorrect: Cannot use a dedicated operator to monitor the release need to IV the calculations and the release path and two analyses of RMT are required. Plausible: This is part of action statement D the candidate may mistakenly refer to 3.d (monitor tank building waste liquid effluent line) -which is the discharge line in use. Correct: action statement C in SLC 16.11-2 Incorrect: Grab sample are not required - need to IV the calculations and the release path and two analyses of WMT are required. Plausible: First action is the corrective action for loss of 1EMF-31 in the same SLC table. If the candidate refers to the wrong corrective action, this is possible. This is action statement E of the SLC may select this statement if the candidate mistakenly references 1.b 1EMF-57 is used to monitor the release. Level: RO Only KA: SYS 068A2.04 (3.3 / 3.3) Lesson Plan Objective: WE-WL SEQ 16 Source: New Level of knowledge: comprehension References: 1. SLC 16.11-2 - PROVIDED

C. D.

Ques_790

For Official Use Only

Page 162

For Official Use Only

Catawba Nuclear Station RO Exam Question #93

Bank Question: 791 1 Pt(s)

Answer: B

Unit 1 was operating at 100% power when a reactor trip occurred on low pressurizer pressure. Given the following events and conditions: Main steamlines isolated Safety injection actuated NC temperature initially decreased to 400F then increased rapidly NC pressure initially decreased to 1700 psig then increased rapidly Pressurizer level initially decreased to 0% then increased rapidly Containment temperature is 100F Pressurizer tailpipe temperatures 120F Containment EMF trip 1 lights are dark All steam generator pressures approximately 700 psig and stable All steam generator narrow range levels are off scale low, wide range levels are increasing

Based on these indications, which of the following events has occurred? A. B. C. D. Pressurizer safety or PORV has failed open. Steam line break downstream of the MSIVs. Small break LOCA Steam line break upstream of the MSIVs.

Distracter Analysis: A. Incorrect: tailpipe temps are normal, as is containment parameters Plausible: candidate misinterprets the pressure and temperature response and holds to the increasing pressurizer level. Correct: when isolated, NC pressure and temperature will begin to increase. Incorrect: normal containment conditions Plausible: operator believes NI flow is causing the system to recover. Incorrect: system would not recover as described Plausible: candidate confuses upstream and downstream of the MSIVs Level: RO&SRO KA: APE 040 AA1.12 (4.2/4.2) Ques_791 For Official Use Only Page 163

B. C. D.

For Official Use Only Catawba Nuclear Station RO Exam Question #93
Lesson Plan Objective: EP1 SEQ 23 Source: New Level of knowledge: analysis References: 1. E-0, steps 24, 25, 26

Ques_791

For Official Use Only

Page 164

For Official Use Only

Catawba Nuclear Station RO Exam Question #94

Bank Question: 792 1 Pt(s)

Answer: C

Unit 1 is operating at 100% power. The spare battery charger (1 ECS) is being aligned to replace charger lECA. How would the operators know if 1ECS was being supplied by motor control center 1 EMXJ in this lineup? A. 1AD-11 II/1 (125 VDC ESS PWR CHANNEL A TROUBLE) will clear when the lEDA tie breaker to lEDC is closed. lAD-11 H/1 (125 VDC ESS PWR CHANNEL A TROUBLE) will alarm when the lEDS output breaker to 1EDA/C is closed. 1AD-11 1/6 (STBY CHARGER lECS INPUT/OUTPUT TRAINS XCONNECTED) will alarm when the 1EDS output breaker to IEDAIC is closed.

B.

C.

D.

1 SI-14 STANDBY CHARGER ECS TRAIN A BKRS CLOSED status light will light when the 1EDA tie breaker to lEDC is closed.

Distracter Analysis: A. Incorrect: the 125VDC trouble annunciator will light. Plausible: this would eliminate the CR alarm caused when the spare charger is in service. Incorrect: the trouble annunciator will alarm when the 1 EDA tie breaker is closed. Plausible: if the candidate confuses this annunciator with the crosstrain alarm. Correct: Incorrect: the status light for B train will light when lEMXJ power to 1 ECS is aligned. Plausible: if the candidate does not understand the power supply status light monitoring. Level: RO&SRO KA: G2.1.31(4.2 / 3.9) aware of the control room Lesson Plan Objective: EL-EPL SEQ 3

B.

C. D.

Ques_792

For Official Use Only

Page 165

Source: New

For Official Use Only Catawba Nuclear Station RO Exam Question #94 Level of knowledge: comprehension References: 1. OP-CN-EL-EPL pages 9, 15-16, 24 2. ARP lAD-11; 1/6 3. ARP 1AD-11; H/1

Ques_792

For Official Use Only

Page 166

For Official Use Only Catawba Nuclear Station RO Exam Question #95

Bank Question: 793

Answer: D

1 Pt(s)

Unit 2 is operating at 100% power with all rods out. An operator notices that one core exit thermocouple for quadrant II on the plasma display indicates 2200F. Which of the following correctly describes a reason for this thermocouple to be much higher than the other thermocouples? A. B. The thermocouple reference junction temperature has increased. The thermocouple reference junction temperature has decreased.

C. D.

The thermocouple measuring junction has an open circuit. The thermocouple measuring junction has shorted.

Distracter Analysis: The voltage across a thermocouple junction increases as the temperature of that junction increases. A shorted measuring junction will cause the temperature to fail high. A. Incorrect: the temperature measured is based on the difference on voltage between the reference junction (at 165 F) and the TIC. If the reference junction is heated above 165 F, then the voltage difference will decrease and the temperature signal will decrease. Plausible: if the candidate reverses the effects Incorrect: the temperature measured is based on the difference on voltage between the reference junction (at 165 F) and the TIC. If the reference junction is cooled below 165 F, then the voltage difference will increase and the temperature signal will increase. However, the increase of 1600 F is not possible because the reference junction temperature cannot be cooled enough. Plausible: the effect of a decrease in reference junction temperature will be to make the measured temperature increase. Incorrect: an open measuring junction causes the temperature indication to fail LOW not high. Plausible: if candidate believes that like an RTD and open causes a high reading For Official Use Only Page 167

B.

C.

Ques_793

D.

Correct: a shorted thermocouple causes the removal of the difference in EMF, the TC will read high. Level: RO&SRO KA: 017 A2.01 (3.1/3.5)

For Official Use Only Catawba Nuclear Station RO Exam Question #95

Lesson Plan Objective: IG SEQ 2 Source: New Level of knowledge: comprehension References: 1. OP-CN-SS-IG page 10 2. OP-CN-TA-AM page 26 3. OP-CN-PS-CCM page 15

Ques_793

For Official Use Only

Page 168

For Official Use Only

Catawba Nuclear Station RO Exam Question #96

1 Pt(s) Bank Question: 794 Answer: C

A large fire has enveloped Unit 1 B-train blackout switchgear room. As the fire brigade prepares to attack the fire with hoses, the control room isolates the B-train blackout bus electrically. Which of the following correctly describes the impact of these events and conditions on the plant's ability to combat this fire? A. The fire brigade will be able to spray down the fire because fire hoses can be pressurized from the A-train fire header. The fire brigade will be able to spray down the fire but with reduced fire main capacity. Only one of the fire pumps will be available from an operable power supply. The fire brigade will be able to spray down the fire. Two of the fire pumps will be available from an operable power supply. D. The fire brigade will not be able to spray down the fire. The blackout bus provides power to the fire pumps.

B.

C.

Distracter Analysis: A. Incorrect: there is only one fire main with redundant pumps. Plausible: if the candidate thinks that the fire main has train separation. Incorrect: there will be 2 pumps running, not only one. Plausible: if the candidate thinks both Unit 1 fire pumps are powered from the same bus. Correct: 3 fire pumps are powered from 1TC, 1FTB and 2FTA only 1FTB is deenergized. Incorrect: there will be water available. Plausible: if the candidate does not know that redundant blackout power supplies are available. Level: RO Only KA: SYS 086K3.01(2.7 / 3.2) Lesson Plan Objective: SS-RFY SEQ 2 Source: New

B.

C. D.

Ques_794

For Official Use Only

Page 169

Level of knowledge: comprehension

For Official Use Only Catawba Nuclear Station RO Exam Question #96

References: 1. OP-CN-SS-RFY page 8

- . .. .. ._. ,_ .. ... "

Ques_794

For Official Use Only

Page 170

For Official Use Only

Catawba Nuclear Station RO Exam Question #97

Bank Question: 795 1 Pt(s)

Answer: D

Unit 1 is operating at 100% power with rod control in manual. Given the following events and conditions: Maintenance is conducting a valve stroke test on 1NI-147B (NI PMP RECIRC TO FWST) with the plant at full power. The SRO directs the balance of plant (BOP) operator to operate INI14713 in support of the maintenance procedure.

While the valve is stroking (both indicating lights on), an annunciator IAD-7 F/3 (LETDN HA' OUTLET HI TEMP) alarms. Which of the following actions describes BOP's actions in response to this condition? A. The BOP should immediately ensure correct letdown and KC flow to the letdown heat exchanger. The BOP should ensure 1NI-147B is closed, and then ensure correct KC flow to the letdown heat exchanger. The BOP should immediately verify letdown heat exchanger outlet temperature and ensure correct letdown and KC flow. The BOP should ensure 1NI-147B is open and then verify letdown and KC flow to the letdown heat exchanger.

B.

C.

D.

Distracter Analysis: A. Incorrect: BOP must first address NI inoperability due to 147 not open. Plausible: These are the ARP immediate actions. Incorrect: INI-147 needs to be open to restore operability. Plausible: If the operator incorrectly assesses operability. Incorrect: BOP must first address NI inoperability due to INI-147 not open. Plausible: a more correct description of BOP response than A. Correct: Level: RO Only KA: G2.2.23(2.6 / 3.8) Lesson Plan Objective: ADM-TS SEQ 5

B. C.

D.

Ques_795

For Official Use Only

Page 171

For Official Use Only Catawba Nuclear Station RO Exam Question #97

Source: New Level of knowledge: comprehension References: 1. OP-CN-ADM-TS page 22 2. OMP 1.7 page 9 3. ARP 1AD-7; F/3

Ques_795

For Official Use Only

Page 172

For Official Use Only

Catawba Nuclear Station RO Exam Question #98

Bank Question: 796 1 Pt(s)

Answer: C

Unit 1 was operating at 100% power. Given the following events and conditions: 1 AD-6 F/10, (PRT HI TEMP) in alarm 1 AD-6 F/11 (PRT HI PRESS) in alarm Lower containment temperature = 124 F The NC system is at normal operating temperature Letdown is in service

Which one of the following statements correctly describes a condition that could cause these alarms? A. The PRT has heated up due to ambient containment temperature. 1 ND-14, ( ND SUCTION RELIEF VALVE) has lifted. 1 NV-15B, (LETDN CONT ISOL), has spuriously closed. The reactor vessel inner 0-ring has leaked. Distracter Analysis: A. Incorrect: Containment temperature is less than PRT temperature. Plausible: Heat up from containment ambient conditions can cause this to occur if containment temperature is high enough. Incorrect: the ND system is too low in temperature and isolated from the NC system, to cause this to occur even if the ND suction relief were to lift. Plausible: the ND suction relief line goes to the PRT. Correct: if 1NV-15B lifts, the letdown relief valve 1NV-14 will lift and relieve to the PRT. Incorrect: the reactor vessel inner 0-ring leaks to the NCDT. Plausible: if the candidate thinks that this leaks to the PRT Level: RO Only KA: SYS 007A1.03(2.6/2.7) Lesson Plan Objective: PS-NC LPRO3 Source: New

B. C. D.

B.

C. D.

Ques_796

For Official Use Only

Page 173

For Official Use Only Catawba Nuclear Station RO Exam Question #98 Level of knowledge: comprehension References: 1. OP-CN-PS-NC pages 12, 21-22, 25-26 2. OP-CN-PS-NV pages 11-13 3. OP-CN-PS-NC page 31 4. OP/1/B/6100/010G F/10, F/11

Ques_796

For Official Use Only

Page 174

For Official Use Only

Catawba Nuclear Station RO Exam Question #99

Bank Question: 797 1 Pt(s)

Answer: D

Unit 1 is in mode 6. Given the following events and conditions: Both trains of SSPS are in "TEST". Window A/2 annunciator is received on Panel 1RAD-1 "lEMF-39 CONTAINMENT GAS HI RAD" 1EMF-39 is the only monitor in alarm.

What one of the following statements correctly describes the automatic actions (if any) that should occur? A. B. C. lEMF-39 has no auto actions with both trains of SSPS in test. lEMF-39 will generate a containment evacuation alarm only. IEMF-39 will generate a containment evacuation alarm and generate a containment ventilation isolation (SH) signal. lEMF-39 will generate a containment evacuation alarm and isolate containment purge.

D.

Distracter Analysis: A. Incorrect: EMF-39 actions are not blocked by SSPS Plausible: candidate may confuse EMF 39 with either 38 or 40, which are blocked by both trains of SSPS. Additionally, the containment evacuation alarm is only blocked by P-6 Incorrect: will also isolate containment purge Plausible: This is true for EMF 38 and 40 Incorrect: will not generate containment ventilation isolation with SSPS in test. Plausible: This is true if one train of SSPS is not in test. Correct: EMF-39 will directly close down VP if it goes into high alarm and both trains of SSPS are in test. Level: RO&SRO KA: WE 016 EK2.1 (3.0/3.1) Lesson Plan Objective: VP SEQ 7 Source: New

B. C.

D.

Ques_797

For Official Use Only

Page 175

For Official Use Only Catawba Nuclear Station RO Exam Question #99

Level of knowledge: memory References: 1. OP-CN-OW1/B/6100/010X, annunciators A/1, A/2, A/3

Ques_797

For Official Use Only

Page 176

For Official Use Only

Catawba Nuclear Station RO Exam Question #100

Bank Question: 798


1 Pt(s)

Answer: A

Which of the following is the power supply to the unit 2 "A" Hydrogen Recombiner? A. B. 2EMXK 2SMXW

C. D.

2MXW 2EMXL

Distracter Analysis: This question tests the candidates' knowledge of the thumb rules for labeling power supplies as well as the type of power provided to the recombiners. 2EMXK is the ONLY train A emergency power supply in the list. A. B. Correct: A and B Hydrogen recombiners are from essential power supplies, (EMXK and EMXJ) Incorrect: Hydrogen recombiners are powered from EMXK Plausible: 2SMXW is a shared power supply - candidate may believe it's a shared power supply Incorrect: Hydrogen recombiners are powered from EMXK Plausible: 2MXW is a blackout power supply - candidate may believe it's a blackout power supply Incorrect: Hydrogen recombiners are powered from EMXK Plausible: 2EMXK is a train B power supply - candidate may confuse the A vs. B trains. Level: RO&SRO KA: 028 K2,01 (2.5/2.8) Lesson Plan Objective: none Source: New Level of knowledge: memory References:

C.

D.

Ques_798

For Official Use Only

Page 177

1. DBD Unit 2 System and Equipment Description - section 32.4 page 55

Potrebbero piacerti anche